SBI Clerk Prelims 2021 Complete Online Study Kit

You might also like

Download as pdf or txt
Download as pdf or txt
You are on page 1of 442

A Complete Online Study Kit For

SBI Clerk Exam (All in One E-Book)


INTRODUCTION

The present book has been specially published for the aspirants of the ‘SBI Clerk
Prelim Exam’. The book, conceived and prepared by experts, is highly recommended
to study and Sharpen your Problem-Solving Skills with thorough practice of numerous
questions provided in the book, and prepare yourself to face the exam with confidence,
successfully.

The E-Book comprises Specialised Study and Practice Material with Solved Previous
Paper to make you familiar with the exam pattern, the type of questions asked, and
their answers. Based on the Current Pattern of Exam, the book will prove very useful
for study, practice, and precious moments before the exam for reference and revision.

The Notes which are Provided by us are the best notes, After Preparing From these SBI
Clerk Study Material Book Notes, You can definitely achieve your goal.

For More Free PDF Visit: www.dreambiginstitution.com

By Dream Big Institution Team


(Deepali Jadhav)

Page 1 Follow us: Official Site, Telegram, Facebook, Instagram, Instamojo


INDEX

Sr No. Topic Page No.

1 How to Prepare for SBI Clerk 3

2 SBI Clerk Exam Pattern 3

3 Reasoning Ability Practice Questions 4

4 Quantitative Aptitude Practice 93


Questions

5 English Language Practice Questions 251

6 Previous Year Questions asked in SBI 426


Clerk

Page 2 Follow us: Official Site, Telegram, Facebook, Instagram, Instamojo


How to Prepare for SBI Clerk Exam
1. Understand the exam pattern and syllabus: Start your preparation by taking a proper glimpse
of exam pattern and syllabus. This step should not be skipped at any cost as it will make you aware
of the topics and the division of marks for each sections.
2. Make a strategy: Quant, Reasoning and English are common in both the prelims and mains exam
but GA and descriptive test is an added feature in the mains exam. So, read current affairs daily for
20-25 minutes and allot every other section 1-2 hours on daily basis. Also, you can start writing
essay as you will not have time to prepare for descriptive after the prelims exam. So you plan
should be a comprehensive target sheet giving importance to all the sections of prelims as well as
mains
3. Previous year’s paper hidden fact: Go through the previous year’s paper. Take note of the topics
that are reoccurring. Also, look for the scoring areas. Do not miss on solving previous year’s paper
because they are the mirror image of the upcoming exam. To check the trend, you can also check
papers of other exam having same exam pattern like IBPS Clerk, etc
4. Practice: Start your practice drill. Solve questions of all the important topics as per the trend and
previous year’s paper. You have time and you can do anything. Practice has no substitute and it
will just enhance your skills, with speed.
5. Revise: Revise whatever you are learning at regular intervals so that your mind do not flush out
the old things. After 20 days from now, make a revision plan as well in addition to your regular
strategy to retain everything you have read so far.

SBI Clerk Preliminary Exam Pattern:


This test is of 1-hour duration consisting of 3 Sections (with separate timings for each section) and the
pattern is as follows:

S.No. Name of Tests (Objective) No. of Questions Maximum Duration


Marks

1 English Language 30 30 20 minutes

2 Quantitative Aptitude 35 35 20 minutes

3 Reasoning Ability 35 35 20 minutes

Total 100 100 60 minutes

Page 3 Follow us: Official Site, Telegram, Facebook, Instagram, Instamojo


REASONING ABILITY SECTION
Reasoning Ability is one of the key part of all competitive examinations like SBI Clerk,
PO, IBPS PO, Clerk, RRB, RBI etc. The Reasoning as a subject and Questions are
considered as a scoring part of exam but only if you are well versed with practice and
tricks. These questions crucial in securing your rank.

TOPIC COVERD:
1) Puzzle
2) Alphanumeric Series
3) Direction Sense
4) Data Sufficiency
5) Inequality
6) Seating Arrangement
7) Syllogism
8) Blood Relation
9) Order & Ranking
10) Coding & Decoding
11) Machine Input Output

How to complete the Reasoning Section in the Given time?


 Do not try to target to attempt all the questions asked. This will add pressure to you.
 It's very important that you only attempt those questions whose topic you are well
aware of.
 The test is to score the maximum marks in less time. Do not spend more time on any
question no matter how well you know that topic.
 Do not feel panic if you have solved fewer questions because maybe the exam is
difficult.
 While practicing make sure to keep a timer as this will help you to fasten your speed.
 Attempt easy topics first like blood relation, inequality, syllogism, etc.
 Attempt the puzzle in the end no matter how good you are in it.
 Give as many mocks as you can.
 Analyze your mocks to make sure that you are correcting your mistakes.

Page 4 Follow us: Official Site, Telegram, Facebook, Instagram, Instamojo


PUZZLE
How to solve Reasoning Puzzles Quickly:

Puzzles are a curious thing; they solve so easily when solving them at home and almost always let us
down in exams. Why? Have you asked yourself?

It all happens because of tension. At home, we don’t take tension = puzzles to get solved correctly. In
the exam, we take a lot of tension = puzzles to get more puzzled!

So let us try not to take tension and learn how to solve puzzles in an easy manner.

1. Know your enemy – or in this case, the common puzzle structure.

Seating arrangements with Banks/Colour/Fruit/Language/State/car models etc. likes and dislikes


and options.
Or, Days and subjects with similar likes and dislikes the thing. Or, maybe months/floors/single
row/double row, etc. with multiple likes/dislike options.
The more complicated the better for them and worse for us!

2. Every battle has a plan – so should you!

I look at puzzles as a personal battle with the Gods of Luck, and I used to always lose – until the day
I planned my puzzle-solving quest.
Yes, dear readers, plan. How? Well, then read on …

3. Read once, start drawing –

A table or a circle or a row or multiple rows…whatever as per the given question.

Do not waste time reading the whole puzzle! In the two lines, you get to know if it’s a seating
arrangement in circle/square/row/multiple rows, etc., immediately draw the required figure.

4. Keep reading and arranging –

Once you have the basic structure of the arrangement, then keep reading and after every line, arrange
the information given in that sentence.
Go one sentence at one time. Arrange the information given in one sentence and only then move on to
the next one.

5. Use tables

To solve puzzles of days/subjects/floors/persons where there is no seating order.


This is a great way to solve puzzles – and you’ll see that your questions will start looking easy once
you have all the information nicely arranged in a tabulated manner.

6. Indirect clues

Page 5 Follow us: Official Site, Telegram, Facebook, Instagram, Instamojo


After arranging as per the direct information available in question, you’ll see that some are left un-
arranged.
These will require your logical reasoning abilities to be able to solve these indirect relations.
Everyone can do it. You can do it. But the difference between a successful candidate and the
unsuccessful one is – the presence of mind/clarity of thinking/ and keeping calm and composed.

PRACTICE QUESTIONS
(Day/ Month/ Year based puzzles.) C) October

Direction (Q1 – Q5): Study the D) June


following information carefully and E) December
answer the given questions:
Ten persons have marriage anniversary
in five different months viz. April, June, Q2. Who among the following has
August, October and December, but not marriage anniversary on 15th April?
necessarily in the same order. Their
marriage anniversary have on 2 A) G
different dates viz. 15 and 18. Each of B) Y
the ten person have anniversary on ten
C) H
different dates.
There are only two person’s marriage D) V
anniversaries before F’s marriage
E) None of these
anniversary. Two person’s anniversary
between the anniversary of F and H. E
and Z have anniversary in October. Only
Q3. Who among the following has
one person’s anniversary between the
marriage anniversary on 18th
anniversary of E and V. V’s anniversary
December?
is not in same month as of H. There are
same number of anniversaries between A) W
Z and G’s anniversary as between F and
B) Y
W’s anniversary. W’s anniversary is in a
month having 31 days. There is no one’s C) V
anniversary between the anniversaries
D) V
of X and K, whose anniversary is exactly
between the anniversaries of E and Y. X’s E) None of these
anniversary is not in same month as K’s
anniversary.
Q4. Which of the following Date W has
Q1. In which month X has marriage marriage anniversary?
anniversary? A) 18th December
A) August B) 18th December
B) April C) 18th December

Page 6 Follow us: Official Site, Telegram, Facebook, Instagram, Instamojo


D) 18th December anniversary is exactly between the
anniversary of E and Y. X’s anniversary
E) 18th December
is not in the same month as K’s
anniversary. W’s anniversary is in a
month having 31 days. So new
Q5. Four of the following five belong
arrangement will be-
to a group in a certain way, find which
of the one does not belong to that
group?
A) K
B) G
C) Z
D) W
E) Y

SOLUTION - Q(1 -5):


W’s anniversary is in a month having 31
days. There are same number of
There are only two person’s marriage
anniversaries between Z and G’s
anniversary before F’s marriage
anniversary as between F and W’s
anniversary. Two person’s anniversary
anniversary. By this condition case- 1 is
between the anniversary of F and H. E
cancelled. So final arrangement will be-
and Z have anniversary in October. Only
one person’s anniversary between the
anniversary of E and V. V’s anniversary
is not in same month as H. From these
conditions there are two possible cases-

Q1. Ans(D)

Q2. Ans(A)

Q3. Ans(C)
There is no one’s anniversary between
the anniversaries of X and K, whose

Page 7 Follow us: Official Site, Telegram, Facebook, Instagram, Instamojo


Q4. Ans(D) Q5. Ans(E)

(Order and Ranking Based Puzzles) e) None

Direction (1-5): Answer the questions 3) If weight of N and P are 72 and 60


based on the information given respectively then what is the possible
below. weight of C?

Seven persons A, B, C, M, N, P, Q and R a) 65


are different weight persons but
necessarily in the same order. M is b) 67
heavier than B, who is not lighter than N
and A. R is heavier than N but not lighter c) 58
than C. Number of person is heavier than
d) 61
N as same as lighter than N. B is lighter
than R, who is not the heaviest e) 70
person.Two person are heavier between
B and P, who is not heavier than A. 4)Four of them following are in the
same group, which is not belongs to
1) Who is third lightest person in the that group?
group?
a) RB
a) P
b) AP
b) A
c) PC
c) C
d) AR
d) N
e) NB
e) B
5) Which of the following pair is
2) How many person are weight lightest and heaviest person in the
between R and P? group?
a) Two a) MC
b) Four b) CR
c) One c) MA
d) Three d) MP

Page 8 Follow us: Official Site, Telegram, Facebook, Instagram, Instamojo


e) CB c) W

SOULUTION 1 to 5: d) T

1) B e) R

2) D 7)If height of U is 142 cm. what is sum


of the height of P and U?
3) C
a) 200
4) D
b) 310
5) A
c) 220
Solution(1-5):
d) Cannot be determined
Final arrangement as follows,
e) None of these
M>R>B>N>A>P>C
8)How many persons are shorter
Direction (6-10): Answer the than Q?
questions based on the information
given below. a) Five

Eight persons P, Q, R, S, T, U, V, W and X b) Four


are different height person but not
necessarily in the same order. R is taller c) One
than T and shorter than W, who is not
the tallest person. T is taller than U, who d) Three
is the shortest person. Number of
e) Two
person are taller than before V as same
as number of person are shorter than R. 9)Which of the following statement is
Sum of the height of P and V is 328 cm. true with respect
Fourth tallest person height is 168cm. S
is taller than P and V, who is 160 cm to Q?
height. V is taller than only two person.T
and U are not neighbours. a) Fourth shortest person

6)Who is fourth shortest person in b) Shorter than U


the group?
c) Possible height is 155 cm
a) U
d) Shortest person
b) P
e) All are false

Page 9 Follow us: Official Site, Telegram, Facebook, Instagram, Instamojo


10)Who is the tallest person in the 6) D
group?
7) B
a) V
8) C
b) R
9) C
c) T
10) E
d) U
Solution (6-10):
e) S
Final arrangement as follows,
SOLUTION 6-10
S > W > R > P (168)> T > V (160)> Q > U

(Floor/ lift based puzzles)

Direction (Q1 – Q5): Study the Q1. Which of the following movie J
information carefully and answer the likes?
question given below.
A) Raazi
Seven persons J, K, L, M, P, Q and R are
living on floors of a seven storey B) Raazi
building, but not necessarily in the same
order. The first floor is numbered as 1 C) Padman
and above it floor is numbered as 2 and
D) Padman
so on till the topmost floor is numbered
as 7. Each of them likes different movies E) None of these
viz. Newton, Raazi, Stree, Andhadhun,
Padman, Parmanu and Soorma, but not Q2. Who among the following likes
necessarily in the same order. Soorma movie?
The one who likes Soorma lives below J. A) R
R lives on the third floor. There are three
floors between J and the one who likes B) P
Newton. There are two floors between
the floors of L and P. The one who likes C) J
Parmanu lives on the ground floor. The
one who likes Padman lives just below P. D) M
L lives on an odd numbered floor. Q likes
Stree. There is only one floor between K E) None of these
and the one who likes Raazi.

Page 10 Follow us: Official Site, Telegram, Facebook, Instagram, Instamojo


Q3. On which of the following floor R lives on the third floor. The one who
lives the one who likes Raazi movie? likes Parmanu lives on the ground floor.
There are three floors between J and the
A) 7th one who likes Newton. The one who
likes Soorma lives below J. From these
B) 5th conditions there will be two possible
cases-
C) 5th

D) 4th

E) None of these

Q4. On which of the following floor


lives Q?
There are two floors between the floors
A) 5th of L and P. The one who likes Padman
lives just below P. L lives on an odd
B) 7th
numbered floor. From these conditions
C) 6th Case- 2 is cancelled. Also, we get one
more case in Case- 1. So new
D) 6th arrangement will be-

E) None of these

Q5. Four of the following five belong


to a group in a certain way, find which
of the one does not belong to that
group?

A) L

B) Q Q likes Stree. There is only one floor


between K and the one who likes Raazi.
C) K The one who likes Soorma lives below J.
From these conditions Case- 1a is
D) K

E) R

SOLUTION Q(1 -5):

Page 11 Follow us: Official Site, Telegram, Facebook, Instagram, Instamojo


cancelled. So final arrangement will be- Q1. Ans(B)

Q2. Ans(D)

Q3. Ans(D)

Q4. Ans(B)

Q5. Ans(C)

(Age Based Puzzle) C)H


(Direction 1 to 5) D)E
Eight persons A, B, C, D, E, F, G and H E)D
were born on the same day of the same
2. Who is the second eldest person?
month but in different years. All their
ages are to be considered from the base A)E
year i.e. 2019. Each of them likes
B)D
different colours- Pink, White, Blue,
Brown, Yellow, Black, Green and Orange. C)The one who is born in 1985
Age difference between B and F who D)The who likes Yellow Colour
likes yellow colour is 2 years and B is
E)The one whose age is 30 years.
older than F. E who is 28 years old likes
Blue colour. Person whose age is prime 3. What is the age of the person who
number likes green colour. A was born likes Orange colour?
in 1989 and likes Pink Colour. H was A)11
born in leap year and is youngest person
and does not like green or white colour B)16
and he is 11 years younger than C who C)14
likes Brown colour. G is 4 year older
than A and he does not like green or D)34
orange colour. The one who was born in E)37
2003 likes white colour. Age of D is not
less than G’s age and he is 26 year older 4. Apart from D whose age is also a
than H whose age is not less than 5 prime number?
years. A)F
1. Who likes Green Colour ? B)G
A)The one who is born in 2003 C)E
B)Whose age is 29 years D)H

Page 12 Follow us: Official Site, Telegram, Facebook, Instagram, Instamojo


E)B
5. Which of the following statement is
not correct according to the
arrangement?
A)F was born in 2005
B)E is 14 years older than F
C)The one who was born in 1997 likes
brown colour 1) Ans: E
D)The one who was born in 1982 likes 2) Ans: C
green colour
3) Ans: A
E)B is second youngest person in the
arrangement 4) Ans: D
5) Ans: E

SOLUTION (1 to 5)

(Box Based Puzzle) and is placed somewhere above the box


numbered four.
Directions (1 – 5): Study the following
information to answer the given There are three boxes between blue box
questions: and the one which contains Gulab
Jamun. The number written on Green
There are 8 boxes which are colored
box is odd and is placed above the box
with different colors – white, pink,
which contains Gulab Jamun.The box
green, blue, yellow, red, orange and
which contains Petha is placed
brown. Each of them contains a number
immediately above orange colored box,
written on them from 1 to 8 according to
which contains neither Gulab jamun nor
which they are placed one above the
Rasgulla. The yellow box does not
other. The box containing number 1 is at
contain Petha. There are three boxes
the lowest place. Each box has different
placed between orange and white boxes.
sweets namely – Rasgulla, Laddu, Peda,
The box which contains Burfi is placed
Nankhatai, Jalebi, Gulab Jamun, Petha
immediately above the one which
and Burfi but not necessarily in the same
contains Laddu, but not at the topmost
order.
position. Only one box is there between
Only one box is there between pink box red colored box and the one which
and the one which contains Jalebi. contains Peda. The number of boxes
Number 4 is written on the brown box. placed above red colored box is same as
The number on red colored box is odd the number of boxes placed between
blue and red colored boxes. Only one

Page 13 Follow us: Official Site, Telegram, Facebook, Instagram, Instamojo


box is there between the one which 4) Four of the following five are alike
contains Burfi and brown colored box. in a certain way and so form a group.
Two boxes are placed between green Which one of the following does not
colored box and the one which contains belong to the group?
Rasgulla.
A. orange box – Nankhatai
1) Which of the following Statements
B. green box – Gulab jamun
is true with respect to the given
information? C. red box – Jalebi
A) Orange colored box is placed D. brown box – Burfi
immediately above the one which
E. blue box – Peda
contains Rasgula
5) How many boxes are placed
B) Yellow colored box is placed
between green colored box and the
immediately above green box
one which contains Peda?
C) Three boxes are placed between red
A) None
colored box and the one which contains
Burfi. B) One
D) Blue box contains Jalebi. C) Two
E) Number 7 is written on green colored D) Three
box
E) More than three
2) Which box is placed exactly
SOLUTION (1 to 5)
between brown colored box and the
one which contains Laddu?
A) yellow and red colored boxes
B) orange and blue colored boxes
C) pink and white colored boxes
D) green and red colored boxes
E) yellow and white colored boxes
3) Which of the following sweets does
yellow box contain?
A) Burfi
B) Laddu
1) Ans: E
C) Peda
2) Ans: E
D) Jalebi
3) Ans: C
E) Petha

Page 14 Follow us: Official Site, Telegram, Facebook, Instagram, Instamojo


4) Ans: C 5) Ans: D

SEATING ARRANGEMENT
5 Golden Rules of Seating Arrangements:

Rule 1: Always start the arrangement with 100% fixed information

Example: 8 persons A,B,C,D,E,F,G,H are sitting around a circular table . A is sitting 3rd to the right of B,
C is sitting opposite to D who is 3rd left to the person opposite to A…………………… so on.
If the question is like this then one should start with A is sitting 3rd to the right of B. Because this is
only the confirmed information.

Note: If nothing is mention regarding the direction they are facing like facing towards center or facing
away from centre then by default take it as facing centre.

Rule 2:

For a closed figure (circle, square, rectangle , octagon etc..,) arrangement, Only persons sit
opposite to each other when there are even number of members.

In a given arrangement if four persons are given as in first diagram then two persons can be
accommodated exactly opposite to each other but not in case of figure (ii)

Rule-3: “And” vs “Who”

In a given question if it is given as

 In a circular arrangement, there are four persons A, B, C, D sitting in the following way. A
is right of B, who is left of C

Page 15 Follow us: Official Site, Telegram, Facebook, Instagram, Instamojo


In this case WHO refers to B: Thus A is placed right to B and B is placed left to C.

 In a circular arrangement, there are four persons A, B, C, D sits in the following way. A is
right of B and is left of C

In this case AND refers to A: Thus A is placed left to C as well as right to B.

Rule 4: Left vs Immediate left

Either in circular or linear arrangement unless it is mentioned in the question we cannot assume left
an immediate left.
Example: 5 persons standing in a row ,namely X,Y,Z,P,A. No one sits left to the vowel. Only 3 persons
are to the right of X. X sits immediate left of Y and left to Z. Z does not sit at extreme ends.

In the Above arrangement, X is immediate left to Y, whereas X is left to Z. No matter how many places
far from Z but it is left to Z.

Rule 5: Identifying the left and right in a given arrangement

For Circular/closed figure:

Page 16 Follow us: Official Site, Telegram, Facebook, Instagram, Instamojo


Whenever it is given towards right go for anti-clockwise direction.
Whenever it is given
Towards left go for clockwise rotation.
These are the important rules one should be aware of while solving seating arrangement questions. A
competitive exam needs speed, but not hurry.

Practice Questions
(Circular Seating Arrangement) same direction. The one who likes Tulip
is immediate to the right of one who is
Directions (1 – 5): Answer the
from Australia. G is adjacent to the one
questions on the basis of the
who likes Daffodil and the one from UK
information given below
does not like Tulip. The one from
Eight friends – A, B, C, D, E, F, G and H Singapore likes Hibiscus and is opposite
are sitting around a circular table. Four to C and he faces back of C. E who likes
of them are facing centre and rest are Daisy is third to the right of one from
facing away from centre. All 4 facing a Australia. F is sitting second to the left of
same direction are not adjacent to each G. E is sitting opposite the one who likes
other. They like different flowers – Lily, Daffodil. The one from USA faces inside
Jasmine, Rose, Lotus, Daffodil, Tulip, and is fourth to left of D who is from
Daisy, and Hibiscus, but not necessarily China. H is sitting to the immediate right
in the same order. They are from of the one who likes Jasmine.
different states – Canada, UK, Japan,
1) H likes which of the following
USA, China, Russia, Singapore and
flowers?
Australia, but not necessarily in the
same order. A) Daffodil
G is sitting second to the left of the one B) Lotus
who is from Australia. The one who likes
C) Hibiscus
Daffodil is opposite the one who is from
UK and they are not facing each other. C D) Tulip
is from Russia and likes Rose. The one
E) Cannot be determined
from Japan is second to the right of G. A
who likes Lotus is third to the right of B 2) Who is third to the left of F?
who is from Canada and both are facing

Page 17 Follow us: Official Site, Telegram, Facebook, Instagram, Instamojo


A) E B) E is sitting second to right of D.
B) The one who is from China C) H is sitting second to the left of E.
C) The one who is from Japan D) There are three persons between B
and H.
D) The one who likes Rose
E) None of the above is correct.
E) The one who likes Daffodil.
3) Which of the following pairs of
persons is facing the same direction? SOLUTION: (1 to 5)
A) C, G
B) A, D
C) A, G
D) F, H
E) None from above is facing same
direction
4) Which of the following triplet is
correct as per given arrangement?
A) H – Tulip – Singapore
B) F – Lily – UK
C) E – Lotus – Japan
D) G – Hibiscus – Singapore
1) Ans:D
E) G – Lotus – Singapore
2) Ans:B
5) Which of the following is correct
with respect to the given 3) Ans:C
arrangement? 4) Ans:D
A) G is facing outside. 5) Ans:C

(Linear Seating arrangement) same order. Some of them are facing


north and some of them are facing south.
Directions (1-4): Study the following
information carefully and answer the T sits third to the left of V. T sits at one
questions given below: of the ends. W sits fourth to the right of
Q. Q and V are not immediate
Eight persons P, Q, R, S, T, U, V and W are
neighbours. Q and V face same
sitting in a row but not necessarily in the
direction. Two persons sit between P

Page 18 Follow us: Official Site, Telegram, Facebook, Instagram, Instamojo


and R. S sits second to the left of P. S 4) Which of the following person sits
does not sit second from any of the ends. third to the right of U?
Immediate neighbours of V faces
A) Second to the left of Q
opposite direction. Immediate
neighbour of Q faces same direction. B) Immediate right of S
Person sits at end faces same direction.
C) Second to the left of W
Immediate neighbour of U faces same
direction. P and U faces opposite D) Third to the left of R
direction. Not more than four persons
E) None of these
facing north.
1) How many persons are facing
south? SOLUTION (1 to 4)
A) 4
B) 3
C) 2
D) 5
T sits third to the left of V. T sits at one
E) None of these of the ends.
2)If W is related to S, U is related to V,
in the same way,R is related to who
among the following?
A) Q
B) V
C) W
D) T W sits fourth to the right of Q. Q and V
E) P are not immediate neighbours. Q and V
face same direction. Two persons sit
3) Which of the following statement is between P and R.
true?
A) U sits third to the left of V
B) Two persons sit between Q and W
C) More than three persons sit between
R and P
D) As many persons sit to the right of T
is same as the left of R
E) None of the given statement is true

Page 19 Follow us: Official Site, Telegram, Facebook, Instagram, Instamojo


S sits second to the left of P. S does not Case 1 will be dropped because not
sit second from any of the ends. more than four persons facing north.
Immediate neighbours of V faces
opposite direction. Immediate 1) Answer: D
neighbour of Q faces same direction.
Person sits at end faces same direction. 2) Answer: E
P and U faces opposite direction.
Immediate neighbour of U faces same 3) Answer: D
direction. Not more than four members
facing north. 4) Answer: E

(Seating Arrangement based on Blood right of daughter of E. E is sitting to the


Relation) immediate left of sister of A.

Directions (1 – 5): Answer the 1) Who is the mother of H?


questions on the basis of the
information given below. A) F

A, B, C, D, E, F, G and H are sitting around B) G


a circle facing the centre but not
necessarily in the same order. Each of C) C
them has a relationship with A.
D) D
G is sitting second to the left of father of
A. F is immediate neighbor of A. D, E) None of these
mother of A is sitting opposite to the
sister of A. B is sitting to the immediate 2) Who is the grand-daughter of E?
right of wife of A. E who is a male is
sitting second to the right of mother of C. A) B
Brother of A is sitting third to right of B.
Daughter of A is sitting to third to right B) C
of sister of A. A is sitting second to the

Page 20 Follow us: Official Site, Telegram, Facebook, Instagram, Instamojo


C) D A) None

D) G B) One

E) None of these C) Two

3) Who is sitting second to the right of D) Three


F’ sister?
E) More than three
A) wife of A
5) What is the position of G’s
B) brother of C daughter with respect to D’s
daughter?
C) daughter of A
A) third to left
D) father of B
B) second to left
E) None of these
C) third to right
4) How many persons are sitting
between A’s wife and D’s husband D) second to right
when counted from right of A’s wife?
E) fourth to right
SOLUTION (1 to 5)

First use D, mother of A is sitting opposite to the sister of A. Daughter of A is sitting to


third to right of sister of A. E is sitting to the immediate left of sister of A.
After this
Use B is sitting to the immediate right of wife of A. Remember E is male. We get 3
arrangements as:

Next use: Brother of A is sitting third to right of B.

Page 21 Follow us: Official Site, Telegram, Facebook, Instagram, Instamojo


Next we have only 1 position left for placing A in all 3 arrangements as

Next, see In 3rd arrangement, A is sitting second to the right of daughter of E. But
here E is 2nd to right of his own daughter. Then E should be wife of W. But in this
arrangement, E and wife are different. SO cancel this arrangement.
Next, see in 2nd arrangement: we have E who is a male is sitting second to the right of
mother of C. But here E is second to right of brother who cannot be mother of C.
So now we are left with only 1st arrangement.
Now B or E can be father of A. G is sitting second to the left of father of A. So this gives
that E is father of A, thus B his son. Proceed further. We get

1) Answer: D 2) Answer: B 3) Answer: D 4) Answer: B 5) Answer: C

Page 22 Follow us: Official Site, Telegram, Facebook, Instagram, Instamojo


(Square Based Seating arrangement)
Directions: 1-5) Answer the questions 3) What is the sum of the ages of the
based on the information given persons sitting at the corners of the
below. table?
There are 7 persons P, Q, R, S, T, U and V a. 16
sitting around square table, where four
b. 21
of them were sitting in the middle of the
edges and three are sitting at the c. 29
corners. Each of them is of different ages
d. 31
among- 2, 3, 5, 8, 9, 10 and 12. Each of
them is facing towards the center of the e. Cannot be determined
table. T, who is sitting at the edge, is
4) Who among the following is
sitting second to the right of R, who is 8
youngest?
years old. U, who is the oldest person, is
sitting three places away from T and is a. Q
sitting at the corner. V, who is 1 year b. U
elder than S, is sitting opposite to S. P is
2 years elder than Q, and both of them c. T
are either sitting at the corner or at the d. P
edge. P and V are not immediate
neighbors. e. Cannot be determined
1) Who is sitting second to the right of 5) Who is sitting third to the left of R?
V? a. Q
a. Q b. U
b. U c. P
c. P d. S
d. S e. Cannot be determined
e. Cannot be determined SOLUTION 1-5
2) What is the sum of the ages of P 1. e
and Q?
2. b
a. 7 years
3. d
b. 8 years
4. c
c. 12 years
5. d
d. 14 years
Solutions 1-5
e. Cannot be determined

Page 23 Follow us: Official Site, Telegram, Facebook, Instagram, Instamojo


1. T is sitting at the edge and 2nd to the
right of R. S(9) P(5)/Q(3)

2. R is 8 years old. U, who is the oldest


person, is sitting three places away from
Q(3)/
T and is sitting at the corner. P(5)
T(2)

So, T is sitting at the edge which means


we have four possible places for T. U(12
V(10)
R(8)
)

(Rectangular Seating arrangement)


Directions: Read the given E) Rithwik
information carefully and answer the
questions given beside:
2) Tanish is related to Kiansh in
Eight students Aadvik, Tanish, Jiral, certain way, Bhavin is related to
Vaibhav, Yashith, Bhavin, Rithwik and Aadvik then in the same way Jiral is
Kiansh sit in a square table. Four related to which among the
persons sit in each corners of the table following?
and four persons sit in each sides of the
A) Rithwik
table but not necessarily in the same
order. All are facing outside. Aadvik is B) Tanish
immediate neighbor of Kiansh. Bhavin
C) Vaibhav
does not sit third to the left of Tanish
who doesn’t sit at the corner. Bhavin sits D) Bhavin
fourth to the right of Yashith. There are E) Aadvik
three persons sit between Tanish and
Jiral. Jiral sits third to the left of Vaibhav.
One person sits between Jiral and 3) Who among the following sits fifth
Kiansh. Rithwik and Kiansh are not to the right of Kiansh and immediate
immediate neighbors. right of Rithwik?
A) Aadvik and Vaibhav
1) Four of the following five are alike
in a certain way and hence form a B) Only Bhavin
group. Which of the following does C) Vaibhav and Yashith
not belong to the group?
D) Tanish and Yashith
A) Aadvik
E) Only Vaibhav
B) Vaibhav
C) Bhavin
D) Yashith

Page 24 Follow us: Official Site, Telegram, Facebook, Instagram, Instamojo


4) Which of the following person sits 5) Which of the following person sits
between Jiral and Kiansh? opposite to the one who sits second to
the right of Rithwik.
A) Tanish
A) Bhavin
B) Rithwik
B) Jiral
C) Aadvik
C) Tanish
D) None
D) Yashith
E) None of these.
E) None of these.

SOLUTION 1-5

1)E 2)A 3)E 4)C 5)B

(Triangular Based Seating between C and F and both are facing


Arrangement) same directions.E and B are facing
opposite directions. Only two persons sit
Direction (1-5): Study following
between the one who likes Rs.1 and the
information carefully and answer the
one who likes Rs.5 and both are facing
questions given below.
same directions. Product of the coin rate
Six persons-A, B, C, D, E and F are sitting of A and F is equal to the E’s coin rate. E
in a equated triangular table. Three of sits third to the right of F. F’s coin rate is
them sit at the corner of the table and five times of D’s coin rate. E’s coin rate is
three of them are sits at the middle of ten times of A’s coin rate. The one who
the side. Some of them facing center and likes Rs.1 facing opposite direction of
some of them are facing outward of the the one who likes Rs.100. The one who
table. Each of them likes different likes Rs.5 sits immediate left of the D.C
amount of coins, viz. Rs.1, Rs.2, Rs.5, sits third to the left of the one who likes
Rs.10, Rs.50 and Rs.100, but not Rs.2 and both are facing opposite
necessarily in the same order. directions. E faces center.

A is an immediate neighbour of E. The 1) Who among the following likes


one who likes Rs.2 sits at the one of the Rs.1?
corner of the table. Only oneperson sits

Page 25 Follow us: Official Site, Telegram, Facebook, Instagram, Instamojo


a) A e) None of these

b) F 5) Which of the following statements


is correct?
c) B
a) A likes Rs.5 and sits immediate right
d) C of E.

e) None of these b) B sits second to the right of D


2) Which of the following c) C likes Rs.50
combinations is correct?
d) Only one persons sit between E and A.
a) B-Rs.5
e) None of these
b) A-Rs.1
Directions(1-5):
c) E-Rs.10
 The one who likes Rs.2 sits at
d) C-Rs.50 the one of the corner of the
table.
e) None of these
 C sits third to the left of the one
3) What is the position the one who who likes Rs.2 and both are
likes Rs.10 with respect D? facing opposite directions.
 Only one person sits between C
a) Second to the right
and F and both are facing same
b) Third to the left directions.
 E sits third to the right of F.
c) Immediate right
 A is an immediate neighbour of
d) Second to the left E.
 E faces center.
e) None of these
 Now we have 4 cases.
4) How many persons are facing
outward of the table?

a) Two

b) Three

c) four

d) Five

Page 26 Follow us: Official Site, Telegram, Facebook, Instagram, Instamojo


 1*5=5, its not satisfied because
If A=Rs.1 means E is Rs.10.
 Lets take, A=Rs.5 and F=Rs.5
 5*5=25, not satisfied.
 Lets take, A=Rs.10 and F=Rs.5
 A*F=E
 10*5=50, not satisfied because
If A=Rs.10 means E is Rs.100
 Lets take, A=Rs.1 and F=Rs.10
 E=10A
 Product of the coin rate of A
and F is equal to the E’s coin  A*F=E
rate.  1*10=10, its satisfied all the
 F’s coin rate is five times of D’s conditions.
coin rate.  Lets take, A=Rs.5 and F=Rs.10
 E’s coin rate is ten times of A’s  A*F=E
coin rate.
 E=10A
 The one who likes Rs.5 sits
 5*10=50, its also satisfied all
immediate left of the D.
the conditions.
 From the above condition,
 Lets take, A=Rs.10 and F=Rs.5
 A*F=E
 A*F=E
 F=5D
 10*5=50 its not satisfied
 E=10A because If A=Rs.10 means E is
 Lets assume, Rs.100

 If D=Rs.1 means F is Rs.5.  From the above conditions A’s


rate is either Rs.5 or Rs.1.
 If D=Rs.2 means F is Rs.10
 And F is rate is Rs.10 and D is
 If D=Rs.10 means F is Rs 50. rate is Rs.2.
 If A=Rs.1 means E is Rs.10  The one who likes Rs.5 sits
 If A=Rs.5 means E is Rs.50 immediate left of the D, Hence A
is rate is Rs.5.
 If A=Rs.10 means E is Rs.100
 From the above condition Case
 A*F=E 2 and Case 3 was dropped.
 Lets take, A=Rs.1 and F=Rs.5

Page 27 Follow us: Official Site, Telegram, Facebook, Instagram, Instamojo


 So the final arrangement is..

 If A is Rs.5 means E is Rs. 50.


 E and B are facing opposite
directions.
 The one who likes Rs.1 facing
opposite direction of the one 1) Answer: C
who likes Rs.100.
2) Answer: E
 Only two persons sit between
the one who likes Rs.1 and the 3) Answer: C
one who likes Rs.5 and both are
facing same directions. 4) Answer: B
 From the above condition Case
5) Answer: B
4 was dropped.
(Pentagon Based seating immediate left of the one who is having
Arrangement) TVS.

Direction (1-5): Study the following


information carefully and answer the 1) Who among the following person
below questions. has a Bajaj bike?

Five persons- P, Q, R, S, and T are sitting A.the one who sits second to the left of Q
at the pentagonal shaped table at the
corners and facing inside. They have a B.S
different brand bike- KTM, Hero, TVS,
Bajaj, and Royal Enfield but not C.P
necessarily in the same order.
D.the one who sits immediate right of T
T is having Hero brand bike sits second
E.None of these
to the right of S. Neither S nor P hasa
TVS bike. R does not sit adjacent to S.
The one who has KTM sits adjacent to S. 2) What is the position of S with
Q does not sit adjacent to R. The one respect to the one who has TVS?
who is having Royal Enfield sits

Page 28 Follow us: Official Site, Telegram, Facebook, Instagram, Instamojo


A.Immediate right
5) Who among the following person
B.Third to the right sits third to the left of Q?
C.Second to the left A.The one who has Royal Enfield
D.Immediate left B.S

E.None of the above C.The one who has KTM

D.T
3) Who among the following person
are adjacent to each other? E.R

A.KTM,T Directions (1-5) :

B.Q,P

C.Bajaj, S

D.Hero, TVS

E.S, R

4) Which of the following


combination is true?

A.S-Hero

B.Q-Royal Enfield 1) Answer: D


C.P-KTM 2) Answer: D
D.T-Bajaj 3) Answer: D
E.R-TVS 4) Answer: C

5) Answer: E

Page 29 Follow us: Official Site, Telegram, Facebook, Instagram, Instamojo


Reasoning Inequality
Rules of Inequality

Signs used to denote Inequality Meaning


If a ≠ b, then ≠ denotes not equal to, i.e. a is not equal to b
If a ≤ b, then ‘≤’ denotes less than or equal to, i.e. a is less than
b or at most b
If a ≥ b, then ‘>’ denotes greater than or equal to, i.e. a is
greater than b or at least b
Strict Inequalities
If a <b, then ‘<’ denotes less than, i.e. a is less than b
If a > b, then ‘>’ denotes greater than, i.e. a is greater than b

Properties of Inequalities

Property ≥ ≤
Addition If a ≥ b, then a + c ≥ b + c If a ≤ b, then a + c ≤ b + c
Subtraction If a ≥ b, then a – c ≥ b – c If a ≤ b, then a – c ≤ b – c
Multiplication If a ≥ b, then ac ≥ bc, If a ≤ b, then ac ≤ bc,
where c>0 where c>0
Division If a ≥ b, then a/c ≥ b/c, If a ≤ b, then a/c ≤ b/c,
where c>0 where c>0
Transitivity If a ≥ b and b ≥ c then, a If a ≤ b and b ≤ c then, a
≥c ≤c
Inverse If a ≥ b, then -a ≤ -b, if If a ≤ b, then -a ≥ -b, if
a>0, b>0 a>0, b>0
·Additive
·Multiplicative If a ≥ b, then 1/a ≤ 1/b, If a ≤ b, then 1/a ≥ 1/b,
if a>0, b>0 if a>0, b>0

PRACTICE QUESTIONS
(Simple Inequality Type) C. W≤A≤L=R<U
D. U>C>=F≤H; W<F
1) In which of these expressions ‘U > E. U>T=O≥P; W<J=P
W’ be definitely false?
A. U>P≥Q=G≥R>W Answer – B. P<A≤U≤T;W≥O>T
B. P<A≤U≤T;W≥O>T

Page 30 Follow us: Official Site, Telegram, Facebook, Instagram, Instamojo


2) Which of the following symbols Answer – B. P<A≤S≤T;V≥O>T<B
should be placed in the blank spaces
respectively(in the same order from 6) Which of the following symbols
left to right) in order to complete the should be placed in the blank spaces
given expression in such a manner respectively(in the same order from
that both ‘N≥Q’ as well as ‘Q≤M’ left to right) in order to complete the
definitely holds true? M _ N _ P _ Q _ R given expression in such a manner
A. >, ≥, <, = that both ‘B>S’ as well as ‘E≤F’
B. >, >, ≥, < definitely holds true? B _ A _ S _ E _ D _
C. ≥, ≥, ≤,≤ F_G
D. ≥, =, ≥,< A. >, ≥, <, =, <, <
E. Other than those given as options B. >, >, ≥, <, >, =
C. ≥, ≥, ≥, ≤, >, >
Answer – D. ≥, =, ≥,< D. >, =, ≥, =, ≤, =
E. Other than those given as options
3) In Which of the following
expressions does the expression Answer – D. >, =, ≥, =, ≤, =
‘D=V’ to definitely hold true?
A. K ≥ D ≤ R = P < S ≤ V 7) In Which of the following
B. U ≥ V ≥ M = F ≤ A ≥ D expressions does the expression ‘L=T’
C. D ≥ C > Q ≥ B = N ≤ V and “E≥W” to definitely hold true?
D. G ≥ D = A < B ≤ S ≤ V A. E ≥ W ≤ R = P < S ≤ T
E. V ≥ E = G ≥ W = Y ≥ D B. U ≥ T ≥ M = W ≤ E ≥ L
C. L ≥ C > E ≥ W = N ≤ T
Answer – E. V ≥ E = G ≥ W = Y ≥ D D. E ≥ W = A < B ≤ S ≤ T
E. T ≥ E = G ≥ W = Y ≥ L
4) Which of the following expressions
is true if the expression Answer – E. T ≥ E = G ≥ W = Y ≥ L
P<T<=B>S>M>=A is definitely true?
A. A ≤ P 8) Which of the following expressions
B. S < P is true if the expression P<T<=Q>= R
C. M > P ≥ S>M>=W>A = R is definitely true?
D. A < B A. W ≤ P
E. T ≤ M B. S < P
C. M < R
Answer – D. A < B D. W > Q
E. T ≤ M
5) In which of these expressions ‘S >
V’ and ‘V > B’ be definitely false? Answer – C. M < R
A. S>P≥Q=G≥R>V>B
B. P<A≤S≤T;V≥O>T<B 9) In which of these expressions ‘P >
C. B>V≤A≤L=R<S R’ and ‘P = R’ be definitely true?
D. S>C>=F≤H; B>V<F A. S>P≥Q=G≥R>V
E. S>T=O≥P; B<V<J=P B. P<A≤S≤T<R;V≥O>T

Page 31 Follow us: Official Site, Telegram, Facebook, Instagram, Instamojo


C. V≤A≤L=R<S=P 11) Statements: Q ≥ R, P >R, P = O ≥ T,
D. P>S>C>=F≤H; V<F<R S ≥ U>Q
E. S>T=O≥P; V<J=P>R Conclusions:
I.S≥R II.P<T III.Q<T IV.U>R
Answer – A. S>P≥Q=G≥R>V A.Only I is true
B.Only II is true
10) In which of these expressions ‘T > C.Only III is true
P’ and ‘T = P’ be definitely false? D.Only IV is true
A. T≥S≥P≥Q=G≥R>V E.All are true
B. P<A≤S≤T;V≥O>T
C. V≤A≤L=R<S Answer –D.Only IV is true
D. S>C>=F≤H=P≤Q=T; V<F Explanation :
E. S>T=O≥P; V<J=P S≥U>Q≥R<P=O≥T

Answer – B. P<A≤S≤T;V≥O>T

(Coded Inequality Type)

Directions (12-20):- 13) statements:


‘P $ Q’ means ‘P is not smaller than Q’ D # R, R * K, K @ F, F $ J
‘P @ Q’ means ‘P is neither smaller Conclusions:
than nor equal to Q’ I.J # R
‘P # Q’ means ‘P is neither greater II.J # K
than nor equal to Q’ III.R # F
‘P & Q’ means ‘P is neither greater IV.K @ D
than nor smaller than Q’ a) Only I II and III are true
‘P * Q’ means ‘P is not greater than Q’ b) Only II III and IV are true
c) Only I III and IV are true
12) statements: d) All I II III and IV are true
H @ T, T # F, F& E, E * V e) None of these
Conclusions:
I.V $ F Answer
II.E @ T e) None of these
III.H @ V
IV.T # V 14) statements:
a) Only I II and III are true N & B, B $ W, W # H, H * M
b) Only I II and IV are true Conclusions:
c) Only II III and IV are true I.M @ W
d) Only I III and IV are true II.H @ N
e) All I II III and IV are true III.W & N
IV.W # N
Answer a)Only I is true
b) Only I II and IV are true b)Only III is true
c)Only IV is true

Page 32 Follow us: Official Site, Telegram, Facebook, Instagram, Instamojo


d)Only either III or IV are true Answer
e)Only either III or IV and I are true a)None is true

Answer
e)Only either III or IV and I are true 16) statements:
M $ K, K @ N, N * R, R # W
Conclusions:
15) statements: I.W @ K
R * D, D $ J, J # M, M @ K II.M $ R
Conclusions: III.K @ W
I.K # J IV.M @ N
II.D @ M a)Only I and II are true
III.R # M b)Only III and IV are true
IV.D @ K c)Only III or IV are true
a)None is true d)Only II III and IV are true
b)Only I is true e)None of these
c)Only II is true
d)Only III is true Answer
e)Only IV is true e)None of these

Directions:(17-20)
‘A % B’ means ‘A is not smaller than B’
‘A @ B’ means ‘A is neither smaller than nor equal to B’
‘A $ B’ means ‘A is neither greater than nor equal to B’
‘A * B’ means ‘A is neither greater than nor smaller than B’
‘A # B’ means ‘A is not greater than B’

17) statements:
F @ J, J # R, R * L, L % M 18) statements:
Conclusions: L # V, V $ E, E % U, U @ B
I.F $ R Conclusions:
II. M # R I.B $ E
III.M % J II.L $ E
a)None is true III.B * L
b)Only I is true a)Only I and II are true
c)Only II is true b)Only III is true
d)Only either II or III is true c)Only either I or II is true
e)All are true d)All are true
e)None of these
Answer
c)Only II is true
Page 33 Follow us: Official Site, Telegram, Facebook, Instagram, Instamojo
Answer
a)Only I and II are true 20) statements:
H % R, R @ W, W * F, J $ F
19) statements: Conclusions:
M $ T, T * R, R @ H, H # G I.H @ F
Conclusions: II.J $ W
I.M $ H III.F @ J
II.R @ G a)Only I and II are true
III.M # R b)Only II and III are true
a)Only I is true c)only III is true
b)Only II is true d)Only either I or III is true
c)Only III is true e)All are true
d)All are true
e)None is true Answer
b)Only II and III are true
Answer
e)None is true

(Some Important Inequality e. Either conclusion I or II is true.


Questions)
Direction: 1-5) In the following
2) Statements:
questions assuming the given
statements to be true, find which of T = V > W = M > R; X < G ≤ M
the conclusion among given
Conclusions:
conclusions is/are definitely true and
then give your answers accordingly. I. W > G
1) Statements: II. R ˃ X
H ≤ Q ≤ R = E; P ≥ B > H a. Only conclusion I is true.
Conclusions: b. Only conclusion II is true.
I. Q ≤ B c. Either conclusion I or II is true.
II. B ˃ P d. Both conclusions I and II are true.
a. Neither conclusion I nor II is true. e. Neither conclusion I nor II is true.
b. Both conclusions I and II are true.
c. Only conclusion I is true. 3) Statements: T < Q; R = S; Q >P ≥ R
d. Only conclusion II is true. Conclusions:

Page 34 Follow us: Official Site, Telegram, Facebook, Instagram, Instamojo


I. T < R Directions: 6-9) In the following
questions assuming the given
II. P = S
statements to be True, find which of
a. None is True the conclusion among given
conclusions is/are definitely true and
b. Both I and II are True
then give your answers accordingly.
c. Only II is True
6)Statement:
d. Only I is True
D ≤ R > E ≤ B; S ≤ M = E > D; G > B
e. Either I and II is True
Conclusion:
I. D > E
4) Statements: Y = X; Z < U < V; X > Z
II. B < R
Conclusions:
a. Only I is True
I. V > X
b. Only II is True
II. Y > U
c. Either I or II is True
a. None is True
d. Neither I nor II is True
b. Both I and II are True
e. Both I and II are True
c. Only II is True
d. Only I is True
7) Statement:
e. Either I and II is True
D ≤ R >E ≤ B; S ≤ M = E > D; G > B
Conclusion:
5) Statements: P ≥ Q ≥ R = S = T ≥ U ≤
I) S < B
V≤W=X
II) B = S
Conclusions:
a. Only I is True
I. W > S
b. Only II is True
II. X ≤ R
c. Either I or II is True
a. None is true
d. Neither I nor II is True
b. Both I and II are true
e. Both I and II are True
c. Only II is true.
d. Only I is true
8) Statement:
e. Either I or II is true
N = K ≥ L ≥ P < O < U ≥ R; P > F

Page 35 Follow us: Official Site, Telegram, Facebook, Instagram, Instamojo


Conclusion: II. S ≥ U
I) F ≥ R a. None is True
II) N > F b. Both I and II are True
a. Only I is True c. Only II is True
b. Only II is True d. Only I is True
c. Either I or II is True e. Either I or II is True
d. Neither I nor II is True
e. Both I and II are True 11) Statements: T ≥ M = K< B = G < P ≥
V > L; X > Z > T
Conclusions:
9) Statement:
I. X > P
Q > A ≥ Z ≤ X ≤ C; Z = H
II. P ≥ T
Conclusion:
a. Only II is True
1) Q > H
b. Only I is True
2) Z ≤ C
c. Both I and II are True
a. Only I is True
d. Either I or II is True
b. Only II is True
e. None is true
c. Either I or II is True
d. Neither I nor II is True
12) Statements: P < Q ≥ G; G ≥ I ≥ E; C
e. Both I and II are True
≤ P; C > U
Conclusions:
Directions: 10-12) In the following
I. U > I
questions assuming the given
statements to be True, find which of II. P ≤ E
the conclusion among given
a. Both I and II are True
conclusions is / are definitely true
and then give your answers b. Only II is True
accordingly.
c. Either I or II is True
10) Statements: H < Y < U ≥ Q = N > R;
d. Only I is True
S=T≥G=V>H
e. Neither I nor II is true
Conclusions:
I. U < R
Page 36 Follow us: Official Site, Telegram, Facebook, Instagram, Instamojo
SOLUTION 1-12 Solutions 3
1. a Given statements: T < Q; R = S; Q > P ≥ R
2. e On combining: Q > P ≥ R = S; T < Q
3. a Conclusions:
4. a I. T < R → False (as T < Q, Q > P and P ≥ R
→ there is no clear relation between T
5. a
and R)
II. P = S → False (as P ≥ R = S → P ≥ S)
Solutions 1
Hence, none of the given conclusion is
Given statements: H ≤ Q ≤ R = E; P ≥ B > true.
H
On combining the given statements: P ≥
Solutions 4
B>H≤Q≤R=E
Given statements: Y = X; Z < U < V; X > Z
Conclusions:
On combining: Y = X > Z < U < V
I. Q ≤ B → False (as B > H ≤ Q) → thus
relation between Q and B cannot be Conclusions:
determined.
I. V > X → False (asX > Z < U < V → thus
II. B ˃ P → False (as P ≥ B). clear relation between V and X cannot be
determined)
Hence, neither conclusion I nor II is true.
II. Y > U → False (Y = X, X > Z and Z < U
thus clear relation between Y and U
Solutions 2 cannot be determined)
Given statements: T = V > W = M > R; X < Hence, none of the two follows.
G≤M
On combining the given statements: T =
Solutions 5
V>W=M≥G>X
Statements: P ≥ Q ≥ R = S = T ≥ U ≤ V ≤ W
Conclusions:
=X
I. W > G → False (as W = M ≥ G)
Conclusions:
II. R > X → False (as R < M ≥ G < X) →
I. W > S → False (as clear relationship
thus relation between R and X cannot be
between W and S cannot be determined)
determined.
II.X ≤ R → False (as clear relationship
Hence, neither conclusion I nor II is true.
between X and R cannot be determined)

Page 37 Follow us: Official Site, Telegram, Facebook, Instagram, Instamojo


The relation between elements can't be Here, conclusions I and II form a
found so the correct option (a) none is complementory pair, thus, either I or II
true. is true.

Answers Solutions 8
6. d Given statement:
7. c N = K ≥ L ≥ P < O < U ≥ R; P > F
8. b Conclusion:
9. e I) F ≥ R → false (as F < P < O < U ≥ R →
clear relation
between F and R cannot be determined)
Solutions 6
II) N > F → true (as N = K ≥ L ≥ P > F → N
Given statement:
> F)
D ≤ R > E ≤ B; S ≤ M = E > D; G > B
Hence, only conclusion II is true.
Conclusion:
I. D > E → false (as D ≤ R > E → clear
Solutions 9
relation between D and E cannot be
determined.) Given the statement:
II. B < R → false (as R > E ≤ B → clear Q > A ≥ Z ≤ X ≤ C; Z = H
relation between B and R cannot be
On combining:
determined.)
Q>A≥Z=H≤X≤C
Hence, neither conclusion I nor II is true.
Conclusion:
I. Q > H → True (as Q > A ≥ Z = H)
Solutions 7
II. Z ≤ C → True (as Z ≤ X ≤ C)
Given statement:
Hence, both I and II are True.
D ≤ R > E ≤ B; S ≤ M = E > D; G > B
On combining:
Answers
S≤M=E≤B<G
10. a
Conclusion:
11. e
I) S < B → false (as S ≤ M = E ≤ B → S ≤ B)
12. e
II) B = S → false (as S ≤ M = E ≤ B → S ≤
B)
Page 38 Follow us: Official Site, Telegram, Facebook, Instagram, Instamojo
Solutions 10 On combining: X > Z > T≥ M = K< B = G <
P≥V>L
Given statements: H < Y < U ≥ Q = N > R;
S=T≥G=V>H Conclusions:
On combining: S = T ≥ G = V > H < Y < U ≥ I. X > P → False (as X > Z > T≥ M = K< B =
Q=N>R G < P)
Conclusions: II. P ≥ T → False (as T≥ M = K< B = G < P)
I. U < R → False (as U ≥ Q = N > R → U > Hence, none is true.
R)
II. S ≥ U → False (as S = T ≥ G = V > H < Y
Solution 12
<U→
Given statements: P < Q ≥ G; G ≥ I ≥ E; C
clear relation between S and U cannot be
≤ P; C > U
determined)
On combining: U<C ≤ P< Q ≥ G ≥ I ≥ E
Hence, none is true.
I. U > I → False (as U<C ≤ P< Q ≥ G ≥ I)
II. P ≤ E → False ( as P< Q ≥ G ≥ I ≥ E)
Solution 11
Given statements: T ≥ M = K< B = G < P ≥
V > L; X > Z > T

Direction Sense
How to Solve Direction Sense?

Directions questions asked in the exam are based on two principles

1. Distance
2. Direction
You might be asked to calculate the distance or the direction. The foremost thing that you are
supposed to do is draw a basic direction map.

Page 39 Follow us: Official Site, Telegram, Facebook, Instagram, Instamojo


PRACTICE QUESTIONS
1) Riya starts from her home and Answer –b) north east
walks 30 meter towards north. From Explanation :
here she took a right turn and walk
20 meters after that she took right
turn and walk 30 meter. At last she
took another left and walked 20
meter and stopped. How much
distance is riya from her home and in
which direction.
a) 30m east
b) 40m west 3) Rohan starts walking in north-east
c) 40m east direction and travels 30 meter. Now
d) 40m south he moves towards east and walks 20
e) None of these meter. Now he turns towards south
west and moves 30 meter and finally
walks towards west for 5 meter and
Answer –c) 40m east stopped. Find the distance of rohan
Explanation : from the starting point.
a) 10m
b) 15m
c) 20m
d) 25m
e) None of these

Answer –b) 15m


2) Village P is north to the village Q. Explanation :
Village S is east to the village Q and
village R is to the left of village P.
Then village S is in which direction
with respect to R.
a) South-west
b) north east
c) south east
d) north west
e) None of these 4) Neha starts running in west
direction and after some distance she
turns to her left and cover some
distance. After this she took a left
turn and then again left turn and run

Page 40 Follow us: Official Site, Telegram, Facebook, Instagram, Instamojo


some distance. After that she took a 6) A person starts walking from his
right turn and finally a left turn. Now home in east direction and after
in which direction she is running. walking 20 meter he took a left turn
a) north and walk 30 meters. Now he took a
b) south right turn and walks 10 meter to
c) east reach the bus stand. Find the distance
d) west between home and stand.
e) None of these a) 20√2
b) 30√2
c) 40√2
Answer –a) north d) 50√2
Explanation : e) None of these

Answer –b) 30√2


Explanation :

5) One morning akash starts walking


in the direction of sun. After walking
some distance he took a left turn then
a right turn and again a left turn and 7) Ram goes 10 m east, then take a
stopped. In which direction akash is right turn and walk 15m and again
facing? take a right turn and walk 20m.
a) north Calculate the approximate distance of
b) south ram from starting point and in which
c) east direction he is now from starting
d) west point.
e) None of these a) 19 and south west
b) 18 and south west
c) 19 and south east
Answer –a) north d) 18 and south east
Explanation : e) None of these

Page 41 Follow us: Official Site, Telegram, Facebook, Instagram, Instamojo


Answer –b) 18 and south west b) north
Explanation : c) east
d) west
e) None of these

Answer –a) South

8) A person starts walking in north


direction and takes turn of 45 degree
anti-clockwise and then a 90 degree
shift in clockwise direction and Explanation :
finally a 135 degree shift anti
clockwise direction. Now in which 10) A person starts walking in north
direction he is facing. direction and after travelling a
a) north distance of 20 meter takes a left turn
b) south and walks 25 meter. From here he
c) east took a right turn and walk 20 meter
d) west and finally took a right turn and
e) None of these walks 35 meter. How far is he from
the starting point and in which
direction?
Answer –d) west a) 15√85 m and North east
Explanation : b) 20√87 m and north east
c) 15√85 m and south east
d) 10√17 m and North east
e) None of these

Answer –d) 10√17 m and North east


Explanation :

9) Rohit and shobhit are two friends


talking with each other. The shadow
of Rohit falls on the right side of
Shobhit. If rohit is facing north then
the direction in which Shobhit is
facing. 11) A person starts walking in south
a) South direction and walks a distance of 7

Page 42 Follow us: Official Site, Telegram, Facebook, Instagram, Instamojo


meters. Now he tooks a left turn and Answer – a) north
walk 6m. Again he takes a left turn Explanation :
and walk 15m and reached a point P.
Find the distance between starting
point and P and in which direction is
the person from the initial point.
a) 10m, south east
b) 10m, north west
c) 10m, north east
d) 10m, south west
e) None of these
13) Two Person P and Q are
separated by a distance of 20 meter
Answer – c) 10m, north east in west –east direction respectively.
Explanation : Now P and Q start walking in north
and south direction respectively and
walked for 5 meter. Now P and Q took
a right turn and walked 10m each.
Now P and Q took left turn and after
walking 5 meter both of them
stopped. Find the distance between
them
a) 15
b) 25
c) 30
d) 35
e) None of these
12) Riya starts running in west
direction and after some distance she
Answer – e) None of these
turns to her left and cover some
Explanation :
distance. After this she took a left
turn and then again left turn and run
some distance. After that she took a
right turn and finally a left turn. Now
in which direction she is running.
a) north
b) south
c) east
d) west
e) None of these

Page 43 Follow us: Official Site, Telegram, Facebook, Instagram, Instamojo


direction and then 120 degree in
clockwise direction. In which
direction is he facing?
a) south east
b) north east
c) south west
d) north west
e) None of these

Answer – c) south west


20
Explanation :
meter.

14) P walks 8m to the south ,then he


turn to his left and walks 15m then he
turn to his right and walk 12m again
he turns to his right and walk 15m
and turn right and stopped how far
and in which direction from the
starting point
a) 10 north
b) 20 south 16) A postman was returning to the
c) 20 east post office which was in front of him
d) 20 west to the south. When the post office is
e) None of these 80m away from him, he turned to the
right and walks 50m to deliver the
last letter. He continues walks in the
Answer – b) 20 south same direction for 20 meter and then
Explanation : took a left turn and walks 80 meter.
How many meters was he away from
the post office
a) 30
b) 40
c) 60
d) 70
e) None of these

Answer – d) 70
Explanation :
15) A boy is facing south direction. He
turns 60 degree in anti-clockwise

Page 44 Follow us: Official Site, Telegram, Facebook, Instagram, Instamojo


a) 5√51
b) 5√53
c) 5√57
d) 5√59
e) None of these

Answer – b) 5√53
Explanation :

17) A person starts walking in north


to his house and walks 50m. Now he
took a right turn and walks 20m and
after that he took another right and
walks 20m. Now he is moving
towards his house. In which direction 19) A person starts walking in north
he is walking? direction and after walking some
a) south east distance he turns to 45 degree in
b) south west clockwise direction. After that he
c) north east turns 180 degree anti clockwise and
d) north west again 45 degree in clockwise
e) None of these direction. Now in which direction he
is walking
Answer – b) south west a) west
Explanation : b) east
c) north
d) south
e) None of these

Answer – a) west
Explanation :

18) A person starts walking in east


direction and walks 20m. After that
he turn to his right and walks 10m
and then turn to his left and walks
15m and reached at a point A. Find
the distance between A and initial
point

Page 45 Follow us: Official Site, Telegram, Facebook, Instagram, Instamojo


20) Anil starts walking in east
direction and after travelling some Answer – b) north
distance he took a right turn and then Explanation :
a left turn followed by another left
turn. Now he again took a right turn
and finally took a left turn. In which
direction is anil walking.
a) south
b) north
c) east
d) west
e) None of these

(Coded Based Distance & Direction) 2) How far and in which direction
does A with respect to E, if the
Directions (1-3): Study the following following expression is A#Z,
information carefully and answer the B&A, C@B, M$C, such that distance
questions given below: between E and Z is 1m more than the
distance between C and B, E is north
If, west of C?
‘P@Q’ means P is 2m north of Q. a) 4m, North East
‘P$Q’ means P is 10m south of Q. b) 5m, South West

‘P&Q’ means P is 5m east of Q. c) 5m, North West


‘P#Q’ means P is 4m west of Q. d) Cannot be determined
1) In which direction does T with e) None of these
respect to L, if the following
expression is ‘T&R@Z#N@L’? 3) If the following expression is
K$R&P@L#T and Q is north of K,L is
a) North West west of Q, then what is the distance
between Q and T?
b) South East
a) 2m
c) North East
b) 3m
d) South West
c) 5m
e) None of these

Page 46 Follow us: Official Site, Telegram, Facebook, Instagram, Instamojo


d) 1m

e) None of these

(SOLUTION 1 – 3)

1) Answer: C

3) Answer: D

2) Answer: B

Blood Relation
Rules of Blood Relation
Generation Male Female
Three generations above↑↑↑ Great grandfather Great grandmother
Maternal great grandfather Maternal great grandmother
Great grandfather-in-law Great grandmother-in-law
Two generations above ↑↑ Grandfather Grandmother
Maternal grandfather Maternal grandmother
Grandfather-in-law Grandmother-in-law
One generations above ↑ Father, Uncle, Mother, Aunt
Maternal uncle, Father-in- Maternal aunt, Mother-in-
law law
Current generation(Self) → Husband, Brother Wife, Sister
Cousin, Brother-in-law Cousin, Sister-in-law
One generation below ↓ Son Daughter
Nephew Niece
Son-in-law Daughter-in-law

Page 47 Follow us: Official Site, Telegram, Facebook, Instagram, Instamojo


Two generations below ↓↓ Grandson Grand daughter
Grandson-in-law Grand daughter-in-law
Three generations below ↓↓↓ Great grandson Great grand daughter
Great grandson-in-law Great grand daughter-in-law

Important Blood Relations

Father of grandfather Great Daughter of father or Sister


or grandmother grandfather mother
Mother of grandfather Great Son of second wife of Step brother
or grandmother grandmother father
Father of father or Grandfather Daughter of second Step sister
mother wife of father
Mother of father or Grandmother Son/daughter of Cousin
mother uncle/aunt
Wife of grandfather Grandmother Brother of husband or Brother-in-law
wife
Husband of Grandfather Sister of husband or Sister-in-law
grandmother wife
Father-in-law of Grandfather Husband of Brother-in-law
father/mother sister/sister-in-law
Mother-in-law of Grandmother Son of father Oneself/Brother
father/mother
Father’s Father Mother of Oneself/Wife
father/mother only son/daughter
Only daughter-in-law Mother Father of daughter/son Oneself/husband
of father’s
father/father’s mother
Husband of mother Father Son of son of Brother/Oneself
grandmother/grandfat /Cousin
her
Wife of father Mother Daughter of son of Cousin/Oneself
grandmother/grandfat /Sister
her
Second wife of father Step mother Son of brother or sister Nephew
Brother of father Uncle Daughter of Niece
brother/sister
Brother of mother Maternal Uncle Grandson of Son/Nephew

Page 48 Follow us: Official Site, Telegram, Facebook, Instagram, Instamojo


father/mother
Sister of father Aunt Granddaughter of Daughter or Niece
father/mother
Sister of mother Maternal Aunt Husband of daughter Son-in-law
Husband of aunt Uncle Wife of Sister-in-law
brother/brother-in-
law
Wife of uncle Aunt Wife of son Daughter-in-law
Son of Father/Uncle Son of son/Daughter Grandson
grandfather/grandmo
ther
Daughter of father-in- Mother/Aunt Daughter of Granddaughter
law/mother-in-law of son/Daughter
father
Father of Father-in-law Son’s/Daughter’s Great Grandson
wife/husband grandson
Mother of Mother-in-law Son’s/Daughter’s Great granddaughter
wife/husband granddaughter
Children of same Siblings
parents
Father’s/Mother’s Oneself
only son/daughter
Son of father or Brother
mother

Representation through diagram

⊕ Father-Son


⊕ Father-Daughter

ϴ
ϴ Mother-Son


ϴ Mother-Daughter

Page 49 Follow us: Official Site, Telegram, Facebook, Instagram, Instamojo


ϴ

Practice Questions
Direction (Q1 – Q2): Study the A) Cannot be determined
following information to answer the B) Sister-in-law
given questions. C) Mother
D) Aunt
Aman is the brother of Neeraj. Neeraj E) Mother-in-law
has only one daughter. Neeraj is the Q2. How is Gaurav related to Aman?
mother of Arun. Arun is the sister of A) Niece
Gaurav. Vivek is the father of Gaurav. B) Cannot be determined
Gaurav is married to Silpi. Babu is the C) Nephew
son of Silpi. D) Son
E) Daughter
Q1. How is Neeraj related to Silpi?
SOLUTION Q(1 -2):

Q1. Ans(E) Q2. Ans(C)

Direction (Q3 – Q5): Study the Q4. How is M related to L


following information carefully to A) Brother-in-law
answer the given questions: B) Sister-in-law
C) Brother
* Z is sister of J. K is brother of L D) Cannot determined
* K has only one son. M is the wife of K E) Sister
* M is the daughter of Z. J is brother- Q5. Following the instruction given in
in-law of X question and in question no. 30, Find
Q3. If V is the husband of Y and son of out how is K related to Y
M, then how is X related to V? A) Father
A) Paternal grandfather B) Father-in-law
B) Father C) Mother
C) Mother D) Maternal Uncle
D) Uncle E) None of these
E) Maternal grandfather
Page 50 Follow us: Official Site, Telegram, Facebook, Instagram, Instamojo
SOLUTION Q(3 – 5):

Q3. Ans(E) Q4. Ans(B) Q5. Ans(B)

Direction (Q6 – Q7): Study the B) Sister-in-law


following information to answer the C) Mother
given questions. D) Aunt
E) Mother-in-law
B is the brother of M. M has only one
daughter. M is the mother of X. X is Q7. How is C related to B?
the sister of C. O is the father of C. C is A) Niece
married to Z. A is the son of C. B) Cannot be determined
C) Nephew
Q6. How is M related to Z? D) Son
A) Cannot be determined E) Daughter
SOLUTION Q(6 – 7):

Q6. Ans(E) Q7. Ans(C)

Direction (Q8 – Q10): Study the of Gaurav


following information carefully to * Arun has only one son. Megha is the
answer the given questions: wife of Arun
* Megha is the daughter of Sani. Ram
* Sani is sister of Ram. Arun is brother
Page 51 Follow us: Official Site, Telegram, Facebook, Instagram, Instamojo
is brother-in-law of Vivek A) Brother-in-law
B) Sister-in-law
Q8. If Raghav is the husband of Kriti C) Brother
and son of Megha, then how is Vivek D) Cannot determined
related to Raghav? E) Sister
A) Paternal grandfather
B) Father Q10. How is Ram related to Megha?
C) Mother A) Uncle
D) Uncle B) Father
E) Maternal grandfather C) Aunt
D) Maternal Uncle
Q9. How is Gaurav related to Megha? E) None of these
SOLUTION Q(8 – 10):

Q8. Ans(E) Q9. Ans(D) Q10. Ans(D)

Directions (Q.11-13): C. Daughter


There are some people in a family. Jyoti D. Son
is the grandmother of Ananya. Hemant is E. Wife
the husband of Aanchal who is sister in
law of Aarushi. Arun is the maternal 12. If Aastha is the wife of Arun then
uncle of Ritu and Manoj. Meena is the how is Jyoti related to Aastha?
father of Seema. Arun is the uncle of A. Daughter in law
Nitu. Nitu is the granddaughter of Jyoti. B. Husband
Arun is the brother of Seema who is the C. Son
husband of Aarushi. Ananya and Nitu is D. Mother in law
the Daughter of Aarushi. Ritu & Manoj is E. Father in law
the son of Hemant.
13. Who is the father of Nitu?
11. How is Aarushi related to Meena? A. Aarushi
A. Sister in law B. Hemant
B. Daughter in law C. Seema
Page 52 Follow us: Official Site, Telegram, Facebook, Instagram, Instamojo
D. Meena E. Arun
SOLUTION (11 to 13)
11. Ans. B.

12. Ans. D.

13. Ans. C.

(Coded Blood Relation) P$Q%S#T*R#V*W


Directions (Q.1-2): 1). How is W related to Q?
$ means A is sister of B A. Son
@ means A is father of B B. Wife
% means A is son of B C. Uncle
* means A is mother of B D. Aunt
# means A is husband of B E. C. &. D.

Page 53 Follow us: Official Site, Telegram, Facebook, Instagram, Instamojo


Q%B#P$S#T@R C. Son
2). How is P related to R? D. Husband
A. Aunt E. Father
B. Nephew

SOLUTION 1 & 2
1. Ans. E.
2. Ans. A.

Alphanumeric Series
How to Solve the Alphanumeric Series:

 Here are some useful tips which you should go through in order to solve the series questions.
 Fasten your calculation so that you can easily solve the series.
 Learn the ranks of alphabets to quickly apply the operations on it.
 Be clear with the use of operations; “to the right/left of”, “in the middle of”, “to the right or left
of” and vice versa.
 Practice a lot of questions to master the topic

The series can be of following topics:

1. Numerical Series
2. Alphabet Series
3. Alphanumeric Series
4. Alphanumeric Symbol Sequence series

PRACTICE QUESTIONS
(Numerical Series) numbers thus formed will be
divisible by three?
Direction (1-5): The following
(a) None
questions are based on the five three
(b) One
– digit numbers given below:
(c) Two
547 247 465 742 343
(d) Three
Q1. If 3 is added to the second digit of
(e) None of these
each of the numbers how many
Page 54 Follow us: Official Site, Telegram, Facebook, Instagram, Instamojo
Q2. If all the digits in each of the (d) 742
numbers are arranged in descending (e) 343
order within the number, which of Q5. If in each number the first and the
the following will be the highest third digits are interchanged then
number in the new arrangement of which will be the highest number?
numbers? (a) 547
(a) 547 (b) 247
(b) 247 (c) 465
(c) 465 (d) 742
(d) 742 (e) 343
(e) 343
Q3. What will be the resultant
SOLUTION (1 -5)
number if the second digit of the
second lowest number is divided by Q1. Ans.(b)
the third digit of the highest number? Only 465 will be divisible by 3 when
(a) 2 added 3 to the second digit of each
(b) 3 number.
(c) 0 Q2. Ans.(a)
(d) 1 754 742 654 742 433
(e) 4
Q4. If 3 is added to the first digit and 1 Q3. Ans.(a)
is added to the last digit of each of the 4÷2=2
numbers then which of the following
numbers will be the second highest Q4. Ans.(a)
number? 848 548 766 1043 644
(a) 547
(b) 247
Q5. Ans.(a)
(c) 465
745 742 564 247 343

(Alphanumeric Symbol Sequence series)


Direction (Q1 – Q5): Study the following arrangement carefully and answer the
questions given below:

Q1. Which of the following is sixth to C) %


the left of the fifteenth from the left D) $
end of the given arrangement? E) Other than the given as option
A) 2 Q2. How many such numbers are
B) # there in the given arrangement each
Page 55 Follow us: Official Site, Telegram, Facebook, Instagram, Instamojo
of which is immediately preceded by series based on the above
a consonant but not immediately arrangement?
followed by a letter?
A) one A) 69H
B) none B) #D8
C) five C) 6#9
D) two D) #9D
E) three E) Other than the given options
Q3. How many such symbols are
there in the arrangement, each of Q5. Four of the following five are alike
which is immediately followed by a in a certain way based on their
letter but not immediately preceded positions in the given arrangement
by a number? and so from a group. Which is the one
A) One that does not belong to that group?
B) Three A) BE6
C) None B) @©U
D) Two C) 9DH
E) More than three D) 5V1
Q4. What should come in place of E) #ZG
question mark (?) in the following

SOLUTION (1 to 5)
Q1. Ans(C)

Q2. Ans(A)

Q3. Ans(b)

Q4. Ans(B)

Q5. Ans(E)

Directions (1-5): Study the following


(Alphanumeric Series) arrangement carefully and answer these
questions.

Page 56 Follow us: Official Site, Telegram, Facebook, Instagram, Instamojo


Z4 E J 5 X 2 B M Q 4 8 N I K 7 W F 6 1 D of which is immediately preceded by
9U2O3 a number and immediately followed
Q1. Four of the following five are alike by a consonant?
in a certain way based on their (a) One
positions in the above arrangement (b) Two
and so form a group. Which is the one (c) Three
that does not belong to that group? (d) Four
(a) 5XQ (e) More than four
(b) Q4K Q4. Which of the following is exactly
(c) 7WD in the middle between Q and 9 in the
(d) 612 above arrangement?
(e) 8N7 (a) I
(b) N
Q2. How many such numbers are (c) 7
there in the above arrangement, each (d) K
of which is immediately preceded by (e) None of these
a consonant? Q5. If all the vowels in the above
(a) None arrangement are dropped, which of
(b) One the following element will be the
(c) Two ninth to the left of D?
(d) Four (a) X
(e) More than four (b) A
(c) 4
Q3. How many such numbers are (d) J
there in the above arrangement, each (e) None of these

SOLUTION (1 to 5)
1) Ans.(e), 8N7
2) Ans.(e)
3) Ans.(b), 48N, 61D
4) Ans.(c), 7
5) Ans.(c), 4
Q1. How many C’s are there in above
arrangement which is immediately
(Alphabet Series) followed by vowel ?
(a) Three
Directions (1-4): These questions are (b) Two
based on the following alphabet (c ) One
series. (d) Five
AAFBBFUUABFFAUCBBBFFFCCCCACA (e ) Four
UUAABCCUFFCAUCCC

Page 57 Follow us: Official Site, Telegram, Facebook, Instagram, Instamojo


Q2. If all the F’s are deleted from the (c) C
above arrangement then which of the (d) A
following letter is 12th to right of 3rd (e) B
from left ?
(a) B Q4. How many B’s are there in above
(b) U arrangement which is immediately
(c) C preceded by vowel but not
(d) A immediately followed by consonant?
(e) None of these (a) Two
(b) One
Q3. Which of the following letter is (c) None
4th to the left of 15th from left end ? (d) Three
(a) F (e) More than three
(b) U

SOLUTION (1 to 4)
Q1. Ans.(E)
Four
Q2. Ans.(C)
12th to right of 3rd from left= 15th from left= C
Q3. Ans.(A)
4th to the left of 15th from left end = 15-4= 11th from left= F
Q4. Ans.(C)

Order and Ranking


Rules of Order and Ranking

1. To find the total number of the person sitting in a row or column:


→ When the rank of a person is given from both sides (i.e. from left and right both) of the row:
(r1 + r2) – 1 = Total

→ When the rank of two persons and number of persons sitting between them is given:
(r1 + r2) + Number of middle persons.

2. To find rank from either left/top or from right/bottom side:


(Total + 1) – Given rank = Required rank.

3. To find the number of persons sitting between any two persons:


For simple case:
Total – (Rank from left + Rank from Right).

For overlapping case:


(Rank from left + Rank from right)–(Total + 2).
Page 58 Follow us: Official Site, Telegram, Facebook, Instagram, Instamojo
4. To find the number of maximum and minimum persons in a row or column:
When (smaller rank – number of middle persons) ≥ 2 then

Simple case will give total number of maximum persons,


(r1 + r2) + Number of middle persons.

Overlapping case will give the total number of minimum persons,


(r1 + r2)–(Number of middle persons + 2).

When (smaller rank – number of middle) < 2 then


Only simple case is valid, so the total number of maximum/minimum persons is
(r1 + r2) + Number of middle persons.

Practice Questions B) Madhu


C) Jayant
Q1. M, T, B, D and R are five friends
D) Pranav
each of them having a different
E) Can’t be determined.
weight. R is heavier than M and T but
lighter than only B. M is heavier than
Q4. Raghav is twenty first from the
only T. Who among the following is at
left end in a row of 40 boys, and
the third position according to their
Prarthi is twenty first from the right
weights?
end in the same row. How many boys
A) R
are there between them in the row ?
B) M
A) One
C) D
B) Two
D) T
C) Three
E) None of these
D) Data inadequate
E) None of these
Q2. Among D, E, F, G and H, each
having a different height, F is taller
Q5. P is greater than R. Q is greater
than only D and G is shorter than E
than S, who is greater than T. Q is
and taller than H. Who among the
equal to R. than who is greatest?
following is the tallest?
A) P
A) E
B) Q
B) H
C) R
C) G
D) S
D) F
E) T
E) None of these
Q6. Among J, K, L, M and N, who is the
Q3. Jayant is taller than Rajeev but not
shortest? J is shorter than only N. K is
as tall as Madhu. Madhu is not as tall
taller than M and L. If they are
as Pranav. Who is the tallest among
arranged in ascending order of their
Rajeev, Madhu, Jayant and Pranav ?
A) Rajeev
Page 59 Follow us: Official Site, Telegram, Facebook, Instagram, Instamojo
heights from left to right then M is B) A
second from the left? C) Can’t be determine
A) J D) E
B) K E) D
C) L
D) M Q10. E is heavier than G and K but not
E) Can’t be determined. as much as F. L is only lighter than M.
The one who is second heaviest is
Q7. Each of the five friends Kamal, 56kg. The weight of third lightest is
Rahul, Suri, Kalia and Bhoopendra 47 kg. K is 10kg lighter than E. G is of
has a different age. Kamal is youger 18kg. What can be the weight of F?
than only Rahul. Suri is older than A) 58 kg
Kalia. Who is not the youngest. Who B) 70 kg
among the following is/are older than C) 50 kg
Suri? D) 45 kg
A) Only Kamal and Bhoopendra E) None of these
B) Only Kalia and Kamal
C) Only Bhoopendra (SOLUTION 1 to 10)
D) Only Bhoopendra and Kalia Q1. Ans(C)
E) None of these B>R>D>M>T
Q2. Ans(A)
Q8. On which day was Shilpi definitely E>G>H>F>D
born? Shilpi’s mother correctly Q3. Ans(D)
remembers that Shilpi was born Pranav > Madhu > Jayant > Rajeev
before Friday but after Monday. Q4. Ans(E)
Shilpi’s brother correctly remembers
that his sister was born before
Saturday but after Wednesday.
A) Monday
B) Tuesday
C) Thursday Q5. Ans(A)
D) Friday
E) Can’t be determined.
Q6. Ans(C)
Q9. Five people A, B, C, D, E lives on
five different floors from bottom to
Q7. Ans(E)
top. Two people live between B and C.
A lives immediately above C. D lives
on one of the floor above E. Then who Q8. Ans(C)
among the following lives on fourth Q9. Ans(C)
floor?
A) B

Page 60 Follow us: Official Site, Telegram, Facebook, Instagram, Instamojo


Q10. Ans(C)

(Reasoning Aptitude based Mix Order and Ranking Questions)


Q1. The positions of how many digits Q2. In a row where all are facing
in the number 2451479638 will north, Priya is 15th from the left end
remain same when the first half and and Garima is 19th from the right
the second half of the digits are end. They interchange their positions,
arranged in ascending order and Ram who sits 24th from the left
separately? end sits at the 5th place to the left of
Priya's new position. How many
A) Only one
persons were there in the row?
B) Two
A) 36
C) None
B) 42
D) Three
C) 47
E) None of these
D) 56
Ans: Correct Option: C
E) None of these
The given number:
2 4 5 1 4 7 9 6 3 8 Answer: Option: C
After arranging the first half and the Using the given information we can
second half of the digits of given number create a following figure:
in ascending order, we get new number
as:
1 2 4 4 5 3 6 7 8 9
Now,
2 4 5 1 4 7 9 6 3 8
1 2 4 4 5 3 6 7 8 9
Comparing both the numbers, we can
say that there is no number whose
Here the total number of persons in the
position remains unchanged.
queue = (*29 + 19 - 1) = 47
Hence, the correct answer is option C.
Note: 29 is derived by adding Ram's
position from left end of the row (24) and

Page 61 Follow us: Official Site, Telegram, Facebook, Instagram, Instamojo


his position with respect to Priya (5). Janhvi + 7)
Hence, the correct answer is option C = (8 + 1 + 3 + 1 + 3 + 1 + 7) = 24
Thus, the total number of people in the
queue was 24.
Q3. During a prize distribution
Hence, the correct answer is option D.
ceremony, Vikram was ninth from the
left while Janhvi was eighth from the
right in the front row. If Hariom was Q4. In a north facing row of NCC
thirteenth from the left and was Cadets, Trisha is 9th from the left end
exactly in the middle of Vikram and and Tina is 12th from the right end.
Janhvi in the same row then what was There are 5 cadets between Trisha
the total number of people in the and Tanya which is equal to the
front row? number of cadets between Tanya and
Tina. Find how many cadets are there
A) 18
in the row?
B) 19
A) 34
C) 21
B) 32
D) 24
C) 31
E) None of these
D) 33
Correct Option: D
E) Can't be determined
Here, we know that Vikram was ninth
from the left while Hariom was Correct Option: B
thirteenth from the left. So, we can say
that there were 3 persons between
Vikram and Hariom.
And, we also know that Hariom was
exactly in the middle of Vikram and
Janhvi so the number of persons
between Hariom and Janhvi will also be Adding all the persons in the above
3. image, we get:
At this point, using the given 8 + 1(Trisha) + 5 + 1(Tanya) + 5 +
information we can create a following 1(Tina) + 11 = 32
figure: Thus there are 32 cadets in the row.
Hence option B is the correct answer.
Q5. In a queue of students facing
north, Ayesha and Anisha are
standing at 10th and 8th position
from the left and right end
respectively. If another student Ariva
Now, total number of people in the who is 12th from the left end is
queue = (8 + Vikram + 3 + Hariom + 3 + exactly in between Ayesha and
Page 62 Follow us: Official Site, Telegram, Facebook, Instagram, Instamojo
Anisha then find the position of From the given image it is clear that
Ayesha from right end ? Ayesha is 12th from the right end.
A) 10th
B) 12th
C) 15th
D) 8th Position of Ayesha from right end = 7 +
1(Anisha) + 1 + 1(Ariva) +1 + 1 = 12
E) Can't be determined Hence option B is the correct answer.
Correct Option: B

Coding - Decoding
Rules of Coding Decoding

First of all we should know the positioning (in numerical value) and opposite of all
alphabets. The below given diagram explain itself:

Page 63 Follow us: Official Site, Telegram, Facebook, Instagram, Instamojo


Practice Questions
(Chinese Coding Decoding)

Directions (1-5): Study the information


and answer the following questions:
In a certain code language,
‘January December March June’ is
written as ‘ ol vh pe xv’
‘March Monday Tuesday January’ is
written as ‘ ty xv pe re’
‘December Monday March school’ is
written as ‘ vh ty xv tf’
Q1. What is the code of ‘January’ Q3. What is the code of ‘School
according to the given information? January’?
(a) vh (a) tf vh
(b) xv (b) vh re
(c) pe (c) ty pe
(d) ol (d) pe tf
(e) None of these (e) None of these
Answer & Explanation Answer & Explanation

Q2. What may be the code of ‘June Q4. What is the code of ‘School’?
Sunday March’? (a) re
(a) ol xv ui (b) vh
(b) ol pe bh (c) ty
(c) xv we nj (d) pe
(d) vh ol xv (e) tf
(e) ty ol xv Answer & Explanation
Answer & Explanation

Page 64 Follow us: Official Site, Telegram, Facebook, Instagram, Instamojo


Q6. What does the code ‘ze’ stand for
in the given code language?
(a) Rita
(b) is
(c) of
(d) good
(e) Can’t be determined
Answer & Explanation

Q5. ‘ol’ is the code of which word?


(a) Monday
(b) Tuesday
(c) June
(d) December
(e) January
Answer & Explanation

Q7. What is the code for ‘Rita’?


(a) kl
(b) mx
(c) mh
(d) ze
(e) Can’t be determined
Answer & Explanation
Directions (6-10): Study the following
information carefully and answer the
given questions:
In a certain code language,
‘she go markets Rita’ is written as ‘ma co
he mx,
‘Rita is a good girl’ is written as ‘mx mh
la sa ox’,
‘Mohan markets Rita’ is written as ‘mx
he kl’
‘go is girl of Mohan’ is written as ‘kl mh
co ze ox’.

Page 65 Follow us: Official Site, Telegram, Facebook, Instagram, Instamojo


Q8. In the given code language, which
of the following means ‘a good girl’?
(a) la sa mh
(b) sa la ox
(c) ox sa mh
(d) Either (a) or (b)
(e) mx mh la
Answer & Explanation

Q10. What is the code for ‘Mohan’ in


the given code language?
(a) ma
(b) she
(c) co
(d) kl
(e) mh
Answer & Explanation
Q9. What does the code ‘co’ stand for?
(a) go
(b) markets
(c) he
(d) Rita
(e) Either (a) or (c)
Answer & Explanation

(New Pattern Coding Decoding)


Directions (1-5): Study the “Career power is premier institute” is
information carefully and answer the coded as ‘ 9X12 4K10 1R4
questions given below. 9K14 16R18’
In a certain code language, “For all government exams” is coded
as ‘1U6 1Z6 9T20 4V10’
Page 66 Follow us: Official Site, Telegram, Facebook, Instagram, Instamojo
“An alumni company” is coded (b) 8H8 4G5 4K10
as ‘1Z4 9Z12 4X14’ (c) 16S14, 4T18, 1P21
“Associated with bankersadda” is coded (d) 6M10, 8O15, 64T17
as ‘25Z20 1D8 16Y22’ (e) None of these

Q1. What is the code for ‘difference’ in Directions (6-10): Study the following
the given code language? information to answer the given
(a) 6W20 questions.
(b) 16W20 In a certain code language,
(c) 8W10 ‘last time the battle’ is written as
(d) Can’t be determined ‘b%6i g%4v o#2c s&3t’
(e) None of these ‘time bat on final’ is written as
‘t%2m o#2c d#5a p&3u’
Q2. What is the possible for ‘4V8, ‘on last kingdom calling’ is written as
9Y14’ in the given code language? ‘n#6h m%2k t%2m g%4v’ and
(a) clear job ‘final king the on’ is written as
(b) make even ‘d#5a s&3t r%2x t%2m’
(c) difference love
(d) even between Q6.In the given code language, what
(e) None of these does the code ‘m%2k’ stand for?
(a) Either ‘last’ or ‘the’
Q3. What may be the possible word (b) battle
for ‘4O8 1G6 4G8’ in the given code (c) on
language? (d) Either ‘calling’ or ‘kingdom’
(a) making clear job (e) king
(b) love the team
(c) less place difference Q7. What is the code for ‘final’ in the
(d) working between team given code language?
(e) None of these (a) t%2m
(b) r%2x
Q4. What may be the possible code (c) s#4t
for ‘make emphasis’ in the given code (d) d#5a
language? (e) Other than those given as options
(a) 4N16 9V64
(b) 16N4 64V9 Q8. What is the code for ‘time’ in the
(c) 4N4, 16V4 given code language?
(d) 4N8, 9V16 (a) n#6h
(e) None of these. (b) o#2c
(c) g%4v
Q5. What is the code for ‘solve tough (d) p&3u
puzzle’ in the given code language? (e) r%2x
(a) 4H16 4G10 4K6

Page 67 Follow us: Official Site, Telegram, Facebook, Instagram, Instamojo


Q9. What is the code for ‘final bat’ in Q3. Ans.(b)
the given code language? Q4. Ans.(d)
(a) v#6e be Q5. Ans.(e)
(b) d#5a n#6h
(c) n#6h v&5e Directions (6-10):
(d) p&3u d#5a Word Code
(e) b%6i d#5a
Last g%4v
Q10. If ‘on the home’ is written as
Time o#2c
‘a$4j t%2m s&3t’ in the given code
language, then what is the code for The s&3t
‘home bat battle’?
(a) a$4j g%4v nh On t%2m
(b) b%6i p&3u a$4j
(c) b#6i d#5a a$4j Battle b%6i
(d) b%6i d#5a a$4j
Final d#5a
(e) b%6i d%5a a$4j
Bat p&3u
SOLUTION 1 to 10:
Directions (1-5): King r%2x
Logic:- First number is the square of the
total number of vowels in a word. Then Kingdom/ m%2k/n#6h
letter of the code is opposite of the first calling
letter of the word, and last number is the Q6. Ans.(d)
total numbers of letters in the word Q7. Ans.(d)
multiplied by 2. Q8. Ans.(b)
Q1. Ans.(b) Q9. Ans.(d)
Q2.Ans.(d) Q10. Ans.(b)

(Coding Decoding Questions asked in 1 Mark)


Study the following information carefully and answer the below questions.
1) If DEVIL is coded as ABSFI then 2) If WATER is coded as 12346 and
how “OTHER” is coded? STEAM is coded as 53427 then how
“MASTER” is coded?
a) LQFBO
a) 725346
b) LQEBO
b) 725364
c) LPEBQ
c) 732546
d) LQEAO
d) 724536
e) None of these

Page 68 Follow us: Official Site, Telegram, Facebook, Instagram, Instamojo


e) None of these c) 594798
3) If GEOMETRY is coded as d) 594789
VTPNGVSZ then what is the code of
e) None of these
“MOTION”?
7) If SMILE is coded as ELIMS then
a) LNUJML
what is the code of “FAITH”?
b) LMUJML
a) THIAF
c) LNUHML
b) HTIAF
d) LNUJNL
c) IAFTH
e) None of these
d) TIFAH
4) If THERMAL is coded as KZLSDGS
e) None of these
then what is the code of
“GEOGRAPHY”? 8) If BDFH is coded as YEUI then what
is the code of GRCI?
a) XGOZFSNDF
a) TUXJ
b) XGPZSFNDF
b) TSXJ
c) XGOZSFMDF
c) USYJ
d) XGOZSFNDF
d) TYXK
e) None of these
e) None of these
5) If MEMORY is coded as 383172 and
NATION is coded as 564915, then 9) If WAYIN is coded as TXVFK then
what is the code of ROTATION? what is the code of LCUNT?
a) 71464915 a) IZRKQ
b) 71464195 b) IYRJQ
c) 71465915 c) IYRKQ
d) 71416495 d) IZRJQ
e) None of these e) None of these
6) If FAITH is coded as 82731, HABIT SOLUTION:
is coded as 12573, and HEALTH is
1. Answer: B
coded as 192431, then what is the
code of BELIEF? Explanation:
a) 597498 DEVIL → ABSFI
b) 549798 D – 3 = A, E – 3=B, V – 3 = S, I – 3 = F, L –
3=I
Page 69 Follow us: Official Site, Telegram, Facebook, Instagram, Instamojo
Thus, OTHER → LQEBO 5.Answer: A
Explanation:
2. Answer: A We have:
Explanation: Letter M E O R Y N A T I
Letter W A T E R S M Code 3 8 1 7 2 5 6 4 9
Code 1 2 3 4 6 5 7 Thus, ROTATION → 71464915
Thus, MASTER → 725346
6.Answer: C
3.Answer: A Explanation:
Explanation: Letter F A I T H B E L
GEOMETRY →VTPNGVSZ Code 8 2 7 3 1 5 9 4
The paring of letters are taken from left Thus, BELIEF → 594798
to right and then for the first pair
reverse alphabetical letters are written
and interchange their position, again for 7.Answer: B
next pair immediate next letter of
Explanation:
English alphabetical series are taken and
the same sequence followed till the last SMILE → ELIMS
pair.
Letters of the word are reversed.
Thus, MOTION → LNUJML
Then, FAITH → HTIAF

4.Answer: D
8.Answer: B
Explanation:
Explanation:
THERMAL →KZLSDGS
BDFH → YEUI
The middle letter is replaced by the next
1st and 3rd letters (from the left end)
letter and the remaining letters are
are replaced by the opposite letter and
replaced by the previous letters in
2nd& 4th letters are replaced by the
alphabetical order and are interchanged
immediate next letter according to the
by 1st& 7th, 2nd& 6th, & 3rd and 5th
alphabetical series.
letter.
Thus, GRCI → TSXJ
Thus, GEOGRAPHY →XGOZSFNDF

9.Answer: A
Page 70 Follow us: Official Site, Telegram, Facebook, Instagram, Instamojo
Explanation: Thus, LCUNT → IZRKQ
WAYIN → TXVFK (For A, X and for C, Z are taking as third
preceding letter)
Each letter is replaced by the third
preceding letter in alphabetical series.

Data Sufficiency
Data sufficiency tricks
Step 1 – Examine the Question:
What is asked? Do we have to find a value or do we have to check a relationship?
Before looking at the two numbered statements, take twenty to thirty seconds to consider the
question by itself. Figure out what is being asked. There are usually 2 possibilities a specific number
may be sought (“What is the value of y?” "How many gallons of milk is in the tank?”), or a true/false
answer may be needed (“Is it true that a >7?” “Is n a prime number?”) Make sure you understand what
the question is asking.
Then consider what information would be needed to answer the question. This will depend on the
type of question, of course. If it is a geometry question, the information needed will be based on rules
you’ve learned about how one geometric fact can be deduced from another. For example, to determine
the area of a circle, you need to know its radius, its diameter, or its circumference. To determine the
length of the hypotenuse of a right triangle, you need to know the length of the other two sides.
On the other hand, if it is a percentage question, different rules will come into play. To determine what
percentage X is of Y, for example, you need to know the value of X and the value of Y. When a change
from one value to another is involved – the increase in value of an investment, for example – you need
to know both the old value and the percentage by which it has increased if you want to calculate the
new value.
As these examples suggest, the data sufficiency question format allows the test makers to measure
your knowledge of a wide array of mathematical topics.

Step 2 – Consider each statement individually


Having figured out the nature of the question and decided, in a general way, what information is
needed to answer it, look at each of the two numbered statements provided. Consider them one at a
time, without reference to each other.
First look at statement A. Does it provide, all by itself, enough information to answer the question? If
so, you’ve already narrowed the possible answer choices to just two: 1 and 4. If not, three answer
choices are possible: 2, 3 and 5.
Then look at statement B. Does it provide, all by itself, enough information to answer the question? If
so, only answers 2 and 4 are possible. If not, only answers 1, 3 and 5 are possible.
Having gotten this far, you may already be able to pick the right answer. If either statement by itself
provides enough information to answer the question, you can pick from answers 1, 2 and 4,
depending on which statement is sufficient or whether either statement will do.
If neither statement by itself is sufficient to answer the question, go on to the third stage:
Page 71 Follow us: Official Site, Telegram, Facebook, Instagram, Instamojo
Step 3 – Combine the two statements
Third, if necessary, combine the two statements. If neither of the statements by itself is sufficient to
answer the question, consider whether you can answer the question by combining the information
given in both statements. If so, the answer is 3; if not, the answer is 5.

PRACTICE QUESTIONS
Directions (Q1 – Q5): Each of the Q2. What does ‘$’ mean in a code
questions below consists of a language?
question and two statements I.‘5 $ # 3’ means ‘flowers are really
numbered I and II given below it. You good’.
have to decide whether the data II.‘7 # 3 5’ means ‘good flowers are
provided in the statements are available’.
sufficient to answer the question.
Read both the statements and give Q3. How is P related to J?
answer. I. M is the brother of P and T is the sister
(a) if the data in statement I alone are of P.
sufficient to answer the question, while II. P’s mother is married to J’s husband,
the data in statement II alone are not who has one son and two daughters.
sufficient to answer the question.
(b) if the data in statement II alone are Q4. How is ‘never’ written in a code
sufficient to answer the question, while language ?
the data in statement I alone are not I. ‘never ever go there’ is written as ‘na
sufficient to answer the question. ja ni ho’ in that code language.
(c) if the data either in statement I alone II.‘go there and come back’ is written as
or in statement II alone are sufficient to ‘ma ho sa ni da’in that code language.
answer the question.
(d) if the data even in both the statements Q5. Among M, P, K, J, T and W, who is
I and II together not sufficient to answer lighter than only the heaviest?
the question. I. P is heavier than M and T.
(e) if the data in both the statements I II. W is heavier than P but lighter than J,
and II together are necessary to answer who is not the heaviest.
the question.
Q1. How many students are there Directions (6 to 10): Each of the
between Suresh and Mohan in a row following questions below consists of
of 50 students ? a question and two statements
I. Suresh is twelfth from the left end and numbered I and II given below it. You
Mohan is seventeenth from the right have to decide whether the data
end. provided in the statements are
II. Suresh is six places away from Jayesh, sufficient to answer the question.
who is twentieth from the left end. Read both the statements and give
answer.

Page 72 Follow us: Official Site, Telegram, Facebook, Instagram, Instamojo


(a) If the data in statement I alone are I. Rudra correctly remembers that his
sufficient to answer the question, while exam is scheduled on a day after
the data in statement II alone are not Tuesday, but before Thursday of the
sufficient in answer the question. same week.
(b) If the data in statement II alone are II. Rudra’s father correctly remembers
sufficient to answer the question, while that Rudra’s exam is scheduled on the
the data in statement I alone are not third day of the week.
sufficient to answer the question.
(c) If the data in either in statement I 10. How may marks has Sujit scored
alone or in statement II alone are in the test?
sufficient to answer the question. I. Sujit scored two-digit marks. His
(d) If the data in both the statements I marks were less than or equal to 20.
and II together are not sufficient to II. Sujit scored more than 9 marks in the
answer the question. test.
(e) If the data in both the statements I
and II are together necessary to answer SOLUTION (1to 10)
the question. Q1. Ans.(a)
From statement I:
6. Who amongst P, Q, R, S and T is the 11 → Suresh 4 ← 21 → Mohan← 16
tallest? So, there are 21 students between
I. P is taller than Q. T is not the tallest. Suresh and Mohan.
II. R is taller than P. S is not the tallest. Using statement II:
Mohan’s name is not even mentioned in
7. In which direction is point E, with the given statement. So, we cannot find
reference to point S? the answer using statement II alone.
I. Point D is to east of point E. Point E is
to the south of point F. Q2. Ans.(e)
II. Point F is to the north-west to point S. Using either of the statements alone we
Point D is to the north of point S. cannot find the code for $, but using
both the statements together we can
8. In which month of the year did find that ‘#53′ is ‘good flowers are’. So,
Raman go abroad for a vacation? the code for $ is ‘really’.
I. Raman correctly remembers that he
went for a vacation in the first half of the Q3. Ans.(e)
year. Using statement, I:
II. Raman’s son correctly remembers We cannot find the relation using this
that they went for a vacation after 31st statement as J’s name is not even
March but before 1st May. mentioned in it.
Using statement II:
9. On which day of the same week is
Rudra’s exam scheduled (Monday
being the first day of the week)?

Page 73 Follow us: Official Site, Telegram, Facebook, Instagram, Instamojo


We can say that J is the mother of P but
cannot decide whether P is the son or
From I and II:
daughter of J.
Using both statements together:

Point E is to the north-west of Point S.

Q4. Ans.(d) Q8. Ans : (b)


Using either of the statements alone we From I: Possible months: January,
cannot find the code, and even by using February, March, April, May or June.
both the statements together we can From II: Raman’s son correctly
only find that ‘never ever’ is coded as remembers that his father went on
‘naja’ the code for ‘never’ cannot be vacation after 31st March but before 1st
uniquely determined even by using both May.
the statements together. So his father went on vacation in the
month of April.
Q5. Ans.(e) Hence only II is sufficient.
Using either of the statements alone we
Q9. Ans : (c)
cannot find the answer, but when we use
Sol. From I: The possible day of exam is
both the statements together we can
Wednesday.
find the relation in terms of weight, ie
From II: The third day of the week is
K>J>W>P>M,T
Wednesday.
Hence, either statement I alone or
Q6. Ans : (e)
statement II alone is sufficient.
From I: P> Q, but T is not the tallest.
From II: R> P, but S is not the tallest. Q10. Ans : (d)
From I and II: R > P > Q. Neither S From I: Two-digit marks is less than or
nor T can be the tallest. Hence R is the equal to 20.
tallest. Possible marks: 10, 11, 12, 13, 14, 15,
16, 17, 18, 19 and 20.
Q7. Ans : (e)
From II:Sujit scored more than 9 marks.
From II:

Page 74 Follow us: Official Site, Telegram, Facebook, Instagram, Instamojo


Possible marks: 10, 11, 12, 13, 14, 15, Hence, statement I and II together are
16, 17, 18, 19 and 20. not sufficient.

(Some other Questions for Practice)


1. What is marked price of the the E. Either Statement I or II.
product?
StatementI – After giving the 10%
3. Lina buy braslet. Find the iner
discount on mark price the
radius of braslet?
shopkeeper make a profit of 26% by
selling the item. Ratio of cost price StatementI – Braslet is made with
and mark price is 5:7. 269.5sqm metel.
Statement II – Difference of mark Statement II – length of outer
price and cost price is 40. Difference diameter is 28m.
of selling price and marked price is
A. Only Statement I alone.
14. Marked price is 40% of the cost
price. B.Only Statement II alone.
A. Only Statement I alone. C. Both Statements I and II together.
B.Only Statement II alone. D. Neither Statement I nor II is sufficient.
C. Both Statements I and II together. E. Either Statement I or II.
D. Neither Statement I nor II is sufficient.
E. Either Statement I or II. 4. Find the length of the train P?
StatementI – The train P cross the
train Q in 6 sec when train Q is not
2. How many tiles is needed to fit tiles
moving. Train Q cross train R in 68
in the rectangular floor of a room?
sec when both are moving same
StatementI – Ratio of length and direction, speed of train R is 25m/sec.
breadth of the room is 9:5 and
Statement II – Train P cross train R in
peremeter of the floor is 140 m.
64 sec when both are moving same
Statement II – Price of 100 tiles is direction. Length of train Q is 140m.
500. And total expenditure to fit the
A. Only Statement I alone.
floor is 12000.
B.Only Statement II alone.
A. Only Statement I alone.
C. Both Statements I and II together.
B.Only Statement II alone.
D. Neither Statement I nor II is sufficient.
C. Both Statements I and II together.
E. Either Statement I or II.
D. Neither Statement I nor II is sufficient.
Page 75 Follow us: Official Site, Telegram, Facebook, Instagram, Instamojo
5. How many days A, B and C can B.Only Statement II alone.
complete the work together?
C. Both Statements I and II together.
StatementI – Ratio of effiecency of A
D. Neither Statement I nor II is sufficient.
and B is 2:1. B and C can complte the
work in 12 days. E. Either Statement I or II.
Statement II – Ratio of effiecency of A 8. There is three number, first,
and C is 3:1. C can alone complte the second and third. What is the 45% of
work in 30 days. first number?
A. Only Statement I alone. StatementI – 6/11 of the first number
is equal to the 22% percent of second
B.Only Statement II alone.
number. second number is equal to
C. Both Statements I and II together. the ¼ of third number.
D. Neither Statement I nor II is sufficient. Statement II – First number is 92
more than the ¼ of the second
E. Either Statement I or II.
number. Difference of first and
second number is 458.
6. Is A is an even number? A. Only Statement I alone.
StatementI – 6A+3B is even number. B.Only Statement II alone.
Statement II – 3A+6B is even number. C. Both Statements I and II together.
A. Only Statement I alone. D. Neither Statement I nor II is sufficient.
B.Only Statement II alone. E. Either Statement I or II.
C. Both Statements I and II together.
D. Neither Statement I nor II is sufficient. 9. In a company there are two type of
employee in a company either 25
E. Either Statement I or II.
year or more than 25 year. Find how
many male are 25 year old?
7. Find the present age of B? StatementI – There is 45 % male
employee in the company. Out of the
StatementI – Age A 2years ago equal
total female employee 60% is 25
to the age of C 4 years ago. Age of C is
years age.
average age of A and B.
Statement II – Total employee of the
Statement II – ratio of present age of
company is 4800. Out of that 40% is
B and age of C after 2 years is 1:1. A is
above 25 years age.
2 years younger than C.
A. Only Statement I alone.
A. Only Statement I alone.
B.Only Statement II alone.
Page 76 Follow us: Official Site, Telegram, Facebook, Instagram, Instamojo
C. Both Statements I and II together. To calculate the iner radius we have to
use both Statement I and Statement II.
D. Neither Statement I nor II is sufficient.
E. Either Statement I or II.
Q4) Answer D
From two statements we can not
10. Find the rate of interest?
calculate the length of train R, speed of
StatementI – The simple interest train train Q so we cannot able to know
accrued of Rs.22500 at the end of four the length of train.because lots unknown
years is Rs 10800. value.
Statement II – The compound
interesest accrued on the Rs.22500 at
Q5) Answer c
certain time at certain rate is Rs.5724
From both statements we can calculate
A. Only Statement I alone.
the efficiency of A,B and C. And from
B.Only Statement II alone. statements 2 we know the days of C to
compllte the work. From that we can
C. Both Statements I and II together.
easily calculate total days to complte the
D. Neither Statement I nor II is sufficient. work by A,B and C.
E. Either Statement I or II.
Q6) Answer B
SOLUTION 1-10 From Statement I we cannot say about A
is even or not because we donot know B
Q1) Answer e
is even or not. But Statement II we can
From Statement I we can calculate the say A is even because 6B is always even
marked price. From Statement II we can and 3A have to be even.
calculate the marked price.Either
Statement I or II is sufficient.
Q7) Answer d
From both Statements I and II we cannot
Q2) Answer d
calculate the exact age of any one. So we
From both statements we can not cannot calculate the age of B.
calculate the area of each tiles so we
cannot calculate the numbers of tiles
required. Q8) Answer b
Statement I we cannot calculate the frist
number but from Statement II we can
Q3) Answer c
calculate the first number from the

Page 77 Follow us: Official Site, Telegram, Facebook, Instagram, Instamojo


realtion between ffirst and second
number.
Q10) Answer a
From statement I we cancaulate the rate
Q9) Answer C because we knoe the principle, time and
interest, but Statement II we donot
Statement I we canculate the total male
know the time so we cannot calculate
employee and total female employee
the the rate.
who are 25 years age in from of
percentage. And with the help Statement
II we can calculate the exact value.

Machine Input Output


All You Need to Know About Input Output in Reasoning

 The shortcut method seems tough, but it is truly an easy method.


 You will need to sit down with a pen and paper to understand this one.
 Once you grasp it, you can cut down your time spent on input-output questions to less than a
quarter of what you used to. Isn’t that amazing?
 If earlier you spent 8-10 minutes trying to solve 5 input output questions, you can now solve them
in 2 minutes!

What is Input Output?

Machine input Output is a question type, where the candidate is given some kind of word and number
arrangement. With each subsequent operation, the arrangement of the words and numbers changes.
These operations are performed until a final arrangement is reached or is performed in loop. The
student is required to identify the hidden pattern in the rearrangement and apply it to the machine
input output questions.

Tips & Tricks to Solve the Input-Output Reasoning Questions


 The first and the most important thing is to read the question carefully and analyse the steps
based on which the Output is given. Once you carefully see the steps. You shall be able to
apprehend the pattern that is being followed to get the Output.
 At times, just looking at Step 1 and step 2 only candidates shall be able to understand the pattern
followed.
 Use tabular form while solving the question as the length of the Input may be longer and may
make the solution even more confusing.

Page 78 Follow us: Official Site, Telegram, Facebook, Instagram, Instamojo


 Do not try solving this type of question verbally as you may miss a few terms and steps and end up
answering the questions wrongly.
 Once you understand the pattern that is being followed in the question, then try applying the same
to the Input given to be solved.

PRACTICE QUESTIONS
Directions (1 – 5): Answer the questions on the basis of the information
given below.
A number arrangement machine when given an input of words/numbers,
rearranges them following a particular rule in each step. The following is an
illustration of input and steps of rearrangement.

Input: 56 fooled 23 32 carry seemed 7 sunny 12 happy 15


Step 1: 7 56 fooled 23 32 carry seemed sunny 12 happy 15
Step 2: 7 seemed 56 fooled 23 32 carry sunny 12 happy 15
Step 3: 7 seemed 15 56 fooled 23 32 carry sunny 12 happy
Step 4: 7 seemed 15 sunny 56 fooled 23 32 carry 12 happy
Step 5: 7 seemed 15 sunny 23 56 fooled 32 carry 12 happy
Step 6: 7 seemed 15 sunny 23 fooled 56 32 carry 12 happy
Step 7: 7 seemed 15 sunny 23 fooled 12 56 32 carry happy
Step 8: 7 seemed 15 sunny 23 fooled 12 happy 56 32 carry
Step 9: 7 seemed 15 sunny 23 fooled 12 happy 32 56 carry
Step 10: 7 seemed 15 sunny 23 fooled 12 happy 32 carry 56

This is the final arrangement and step 10 is the last step for this input.

Input: suffix 27 42 breeze letter 5 53 20 called 28 community

1) How many elements are D) 53


between ‘suffix’ and ‘letter’ in E) called
step 5?
A) 2 3) Which step number is
B) 3 following step?
C) 5 Step: 5 breeze 27 suffix 53 called
D) 4 20 community 28 42 letter
E) None of these A) 6
B) 7
2) Which element is third to left C) 9
of sixth element from the right D) 8
end in step 7? E) There is no such step
A) 42
B) suffix 4) In step 6, ‘53’ is related to ‘42’
C) 27 in the same way as in step 2,
Page 79 Follow us: Official Site, Telegram, Facebook, Instagram, Instamojo
‘letter’ is related to ‘called’. Now
in step 4, ‘53’ is related to what Input: suffix 27 42 breeze letter 5 53
in the same way? 20 called 28 community
A) 27 Step 1: 5 suffix 27 42 breeze letter 53
B) 20 20 called 28 community
C) letter Step 2: 5 breeze suffix 27 42 letter 53
D) called 20 called 28 community
E) 28 Step 3: 5 breeze 27 suffix 42 letter 53
20 called 28 community
5) What are last two Step 4: 5 breeze 27 suffix 53 42 letter
numbers/words in step 7? 20 called 28 community
A) called, 20 Step 5: 5 breeze 27 suffix 53 called 42
B) 28, community letter 20 28 community
C) 5, breeze Step 6: 5 breeze 27 suffix 53 called 20
D) 42, letter 42 letter 28 community
E) letter, 28 Step 7: 5 breeze 27 suffix 53 called 20
community 42 letter 28
SOLUTION (1 to 5) Step 8: 5 breeze 27 suffix 53 called 20
Words arrangement: all words have community 28 42 letter
double letters. So double letters are Step 9: 5 breeze 27 suffix 53 called 20
used in ascending order in each community 28 letter 42
arrangement of words.
seemed sunny fooled happy carry
1) Ans:B
Number arrangement: first odd
2) Ans:C
numbers in ascending order, then
3) Ans:D
even numbers in ascending order.
4) Ans:B
5) Ans:E
(New Pattern Input Output Questions)

Direction 1-5: Read the given information carefully and answer the
questions given beside:

The first step has been obtained by multiplying the digits in input. No other step
uses multiplication concept. They are obtained by applying certain logic. Numbers
of step II have been obtained by using at least 1 digit of each number in step 1.
Each step is a resultant of previous step

Page 80 Follow us: Official Site, Telegram, Facebook, Instagram, Instamojo


Question 1: What is the B. 112
difference between the numbers C. 96
of Step II? D. 72
A. 6 E. None of these
B. 10
C. 12 Question 4: If the difference of
D. 8 the number of step III is
E. None of these multiplied by step IV, what will
be the resultant number?
Question 2: Which of the A. 2.5
following will be the double of B. 3
Last step? C. 9
A. 5 D. 1.25
B. 2.5 E. None of these
C. 7.5 Question 5: Which of the
D. 9 following is one of the numbers
E. None of these of any step?
A. 58
Question 3: What is the sum of B. 22
largest and the smallest number C. 30
of step I? D. 12
A. 106 E. None of these

Page 81 Follow us: Official Site, Telegram, Facebook, Instagram, Instamojo


Directions 6-10: Study the following information carefully and answer the
questions given below:

The first step has been obtained by multiplying the digits in input. The next steps
are not obtained the same way. They are obtained by applying certain logic.
Numbers of step II have been obtained by using at least 1 digit of each number in
step 1. Each step is a resultant of previous step.

Question 6: Which of the A. 0


following will be the last step? B. 1
A. 6 C. 2
B. 7.5 D. 3
C. 3.5 E. None of these
D. 5
E. None of these Question 9: Which of the
following is a number in step I?
Question 7: What is the sum of A. 64
the numbers of step III? B. 32
A. 9 C. 69
B. 12 D. 67
C. 18 E. None of these
D. 26
E. None of these Question 10: If the sum of
numbers of step III is multiplied
Question 8: If we do half of each by step IV, find the resultant
number in step II, what will be number.
the difference of those numbers? A. 31.5

Page 82 Follow us: Official Site, Telegram, Facebook, Instagram, Instamojo


B. 50 Correct Option: B
C. 26.4
22 is one of the numbers of any step.
D. 37.5
Option B, is hence the correct answer.
E. None of these
Question 6:
SOLUTION (1to 10) Correct Option: C
Question 1: Following the common explanation, we
can say that 3.5 is the last step. Hence,
Correct Option: D
the correct answer is option C.
Difference between the numbers of Step
Question 7:
II is 8. Option D, is hence the correct
answer. Correct Option: A
Question 2: The sum of the numbers of step III is 9.
Hence, the correct answer is option A.
Correct Option: B
Question 8:
2.5 is the double of Last step. Option B, is
hence the correct answer. Correct Option: D
Question 3: If we do half of each number in step II,
the difference of those numbers will be
Correct Option: A
2. Hence, the correct answer is option D.
106 is the sum of largest and the
Question 9:
smallest number of step I. Option A, is
hence the correct answer. Correct Option: D
Question 4: 67 is in step I. Hence, the correct answer
is option D.
Correct Option: D
Question 10:
1.25 is the resultant number. Option D,
is hence the correct answer. Correct Option: A The resultant number
will be 31.5. Hence, the correct answer is
Question 5:
option A.

SYLLOGISM
Some Common Rules in Syllogisms
Following are some common rules that you must know in order to solve Syllogism based problems:

 All+All will imply All.


 All+No will imply No.
 All+Some will imply No Conclusion.
 Some+All will imply Some.

Page 83 Follow us: Official Site, Telegram, Facebook, Instagram, Instamojo


 Some+No will imply Some Not.
 Some+Some will imply No Conclusion.

The most important are the possibility cases and you will need the below rules to attemt such
type of questions:

 If All A are B then Some B are Not A is a Possibility.


 If Some B are Not A then All A are B is a Possibility.
 If Some A are B then All A are B is a Possibility & All B are A is a Possibility.
The most important part about possibilities cases is that we have to create all possibilities to find
whether the given conclusion is possible or not. If the conclusion satisfies all the possibilities only
then it is assumed to be correct.

Tips and Tricks to Solve Syllogism based Questions


Following are some handy tips that you should follow to solve syllogism questions

 Go through all the statements one by one.


 Understand how you need to draw Venn Diagrams for each of these statements.
 Try to find out the pattern of the question.
 Understand how to analyses the conclusion for each statement..
 The key point you should remember is that you have to attempt the question in a sequential manner.
This was all about tips and tricks to solve Syllogism based questions. So try the above tips and tricks
on the syllogism based questions.

(Only a Few Type Syllogism)


1) Statements:
Only a few pink are public. (b) If only conclusion II follows.
All Metro are Pink. (c) If either conclusion I or II follows.
Only City are Metro. (d) If neither conclusion I nor II follows.
Conclusion (e) If both conclusions I and II follow.
I: All Public can never be Pink.
II: All City are Public. 3) Statements:
(a) If only conclusion I follows. No Abhi is Swati.
(b) If only conclusion II follows. Only a few Manali is Alpa.
(c) If either conclusion I or II follows. All Alpa is Abhi.
(d) If neither conclusion I nor II follows. Conclusion:
(e) If both conclusions I and II follow. I: Some Manali is Swati.
II: No Manali is Swati.
2) Statements: (a) If only conclusion I follows.
Few Bro is Sis. (b) If only conclusion II follows.
Only a few Wife is Son. (c) If either conclusion I or II follows.
No Son is Sis. (d) If neither conclusion I nor II follows.
Conclusion: (e) If both conclusions I and II follow.
I: Some Wife being Sis is a possibility.
II: All Bro being Sis is a possibility. 4) Statements:
(a) If only conclusion I follows. Some School are College.
Page 84 Follow us: Official Site, Telegram, Facebook, Instagram, Instamojo
Only a few College are University. Only a few School are College.
No University is Office. Only a few College are University.
Conclusions: No University is Office.
Conclusions:
1. All College being University is a
possibility. 1. Some School can be University.
2. Some College are not Office. 2. Some College can be Office.
(a) If only conclusion I follows. (a) If only conclusion I follows.
(b) If only conclusion II follows. (b) If only conclusion II follows.
(c) If either conclusion I or II follows. (c) If either conclusion I or II follows.
(d) If neither conclusion I nor II follows. (d) If neither conclusion I nor II follows.
(e) If both conclusions I and II follow. (e) If both conclusions I and II follow.

5) Statements: 8) Statements:
Some Tree are Leaf. Only Abhi is Alpa.
All Leaf are Green. Only a few Abhi is Ruchi.
Some Plant are not Green. Few Ruchi is Ram.
Conclusions: Conclusion:
I: Some Alpa is Ram.
1. Some Leaf are not Plant. II: Some Ruchi is Abhi.
2. Some Leaf can never be Tree. (a) If only conclusion I follows.
(a) If only conclusion I follows. (b) If only conclusion II follows.
(b) If only conclusion II follows. (c) If either conclusion I or II follows.
(c) If either conclusion I or II follows. (d) If neither conclusion I nor II follows.
(d) If neither conclusion I nor II follows. (e) If both conclusions I and II follow.
(e) If both conclusions I and II follow.
9) Statements:
6) Statements: Some Clock are Time.
Some Swati are Simi. Only a few Time is Watch.
Only a few Komal are Simi. No Watch is Clock.
All Komal are Sonal. Conclusions:
Conclusions:
1. All Watch being Time is a
1. All Simi being Komal is a possibility.
possibility. 2. All Time are Watch is a
2. Some Sonal are Swati. possibility.
(a) If only conclusion I follows. (a) If only conclusion I follows.
(b) If only conclusion II follows. (b) If only conclusion II follows.
(c) If either conclusion I or II follows. (c) If either conclusion I or II follows.
(d) If neither conclusion I nor II follows. (d) If neither conclusion I nor II follows.
(e) If both conclusions I and II follow. (e) If both conclusions I and II follow.

7) Statements:
Page 85 Follow us: Official Site, Telegram, Facebook, Instagram, Instamojo
10) Statements: Conclusion:
All Reebok are Raymond. I Some classroom are music
Some Raymond are Puma. II No music is classroom
Only a few Puma are Titan. (a) If only conclusion I follows.
Conclusions: (b) If only conclusion II follows.
(c) If either conclusion I or II follows.
1. Some Titan can never be (d) If neither conclusion I nor II follows.
Reebok. (e) If both conclusions I and II follow.
2. Some Titan can be Reebok.
(a) If only conclusion I follows. 14) Statements:
(b) If only conclusion II follows. Only a few blue is purple.
(c) If either conclusion I or II follows. All purple is black.
(d) If neither conclusion I nor II follows. Some black is pink.
(e) If both conclusions I and II follow. Conclusions:

11) Statements: 1. All black is blue is a possibility.


Only a few Book are Paper. 2. Some purple is pink.
No Paper are Pen. (a) If only conclusion I follows.
Conclusion: (b) If only conclusion II follows.
I: Some Book are Pen. (c) If either conclusion I or II follows.
II: All Book can never be Pen. (d) If neither conclusion I nor II follows.
(a) If only conclusion I follows. (e) If both conclusions I and II follow.
(b) If only conclusion II follows.
(c) If either conclusion I or II follows. 15) Statements:
(d) If neither conclusion I nor II follows. Some steam are cloud.
(e) If both conclusions I and II follow. Only a few cloud is liquid.
All liquid are bottle.
12) Statements: Conclusions:
Only Book are Pencil.
No Book are Cat. 1. All liquid are steam.
Conclusion: 2. At least some bottle are cloud.
I: Some Book are Cat. 3. Some bottle being steam is a
II: Some Pencil can be Cat possibility.
(a) If only conclusion I follows. (a)Only I follows
(b) If only conclusion II follows. (b) Only II follows
(c) If either conclusion I or II follows. (c) Only III follows
(d) If neither conclusion I nor II follows. (d) II and III follow
(e) If both conclusions I and II follow. (e) None of the above

13)Statements: 16) Statements:


Only a few classroom are smart . All rain are water.
Some smart are music. Some water is nature.

Page 86 Follow us: Official Site, Telegram, Facebook, Instagram, Instamojo


No nature is bad. (e) None follows
Conclusions:
19) Statements:
1. At least some bad is rain. All waste is box.
2. Few nature being rain is a Some box is oval.
possibility. No oval is an green
3. No bad is water. Conclusions:
(a) Only I follows
(b) Only II follows 1. Only a few waste is oval.
(c) Only III follows 2. Some green being box is a
(d) Either I or III follows possibility.
(e) None of these (a) Either I or II follows.
(b) Both I and II follow.
17) Statements: (c) Only I follows.
All tomato are milk. (d) Neither I nor II follows.
Only a few milk is pure. (e) Only II follows.
All Coffee are Milk.
Conclusions: 20) Statements:
Only a few mails are gmail.
1. Few Coffee are pure. Some gmail are google.
2. Only coffee is tomato is a Conclusion
possibility.
3. All pure is milk is a possibility. 1. Some mail are google
(a) I and II follow 2. No google is mail
(b) I and III follow (a) If only conclusion I follows.
(c) II and III follow (b) If only conclusion II follows.
(d) All follow (c) If either conclusion I or II follows.
(e) None of these (d) If neither conclusion I nor II follows.
(e) If both conclusions I and II follow.
18) Statements:
All mango are orange. 21) Statements:
Only a few orange is banana. All Duck is Bird.
some banana are apples. All Bird is Sparrow.
Conclusions: Only a few Crow are Sparrow.
Conclusions:
1. Some apples are not orange.
2. All banana are mango. 1. All Crow are Bird.
3. All orange are banana is a 2. Some Duck are Sparrow.
possibility. (a) If only conclusion I follows.
(a) Only I follows (b) If only conclusion II follows.
(b) I and II follow (c) If either conclusion I or II follows.
(c) I and III follow (d) If neither conclusion I nor II follows.
(d) Only III follows (e) If both conclusions I and II follow.
Page 87 Follow us: Official Site, Telegram, Facebook, Instagram, Instamojo
(e) If both conclusions I and II follow.
22) Statements:
Only a few Swati are Manali. 25) Statements:
Only a Few Manali are Vani. Only a few laptops are tablets.
All Vani are Roma. All tablets are mobiles.
Conclusions: Some mobiles are headphones.
Conclusions:
1. Some Swati can be Vani.
2. All Manali being Vani is a 1. All headphones being tablet is a
possibility. possibility.
(a) If only conclusion I follows. 2. All tablets are laptops
(b) If only conclusion II follows. (a) If only conclusion I follows.
(c) If either conclusion I or II follows. (b) If only conclusion II follows.
(d) If neither conclusion I nor II follows. (c) If either conclusion I or II follows.
(e) If both conclusions I and II follow. (d) If neither conclusion I nor II follows.
(e) If both conclusions I and II follow.
23) Statements:
Some Dhiraj are Aakash. 26) Statements:
No Aakash is Sarin. All bottles are cups.
Only a few Sarin are Poker. Only a few cups are plates.
Conclusions: Some plates are jugs.
Conclusion:
1. All Sarin are Poker.
2. Some Dhiraj are not Sarin. 1. Some jugs are not cups
(a) If only conclusion I follows. 2. All plates are bottles.
(b) If only conclusion II follows. (a) If only conclusion I follows.
(c) If either conclusion I or II follows. (b) If only conclusion II follows.
(d) If neither conclusion I nor II follows. (c) If either conclusion I or II follows.
(e) If both conclusions I and II follow. (d) If neither conclusion I nor II follows.
(e) If both conclusions I and II follow.
24) Statements:
Only a few Fox are Box. 27) Statements:
Only a few Box are Desk. Only a few TV are Cinema.
No Desk is Help. No Cinema is Movie.
Conclusions: All Movie are Paint.
Conclusions:
1. All Fox being Desk is a
possibility. 1. All TV are Cinema is a
2. Some Box are not Help. possibility.
(a) If only conclusion I follows. 2. Some Paint can never be
(b) If only conclusion II follows. Cinema.
(c) If either conclusion I or II follows. (a) If only conclusion I follows.
(d) If neither conclusion I nor II follows. (b) If only conclusion II follows.
Page 88 Follow us: Official Site, Telegram, Facebook, Instagram, Instamojo
(c) If either conclusion I or II follows. 2. Some rude persons may not be
(d) If neither conclusion I nor II follows. teachers.
(e) If both conclusions I and II follow. (a) If only conclusion I follows.
(b) If only conclusion II follows.
28) Statement- (c) If either conclusion I or II follows.
Only a few tests are exams (d) If neither conclusion I nor II follows.
Only a few exams are papers (e) If both conclusions I and II follow.
all papers are interviews
Conclusion- 31) Statements:
Only a few S are C.
1. All test being exam is a Only a few C are U.
possibility No U is O.
2. No interview is a test Conclusions:
(a) If only conclusion I follows.
(b) If only conclusion II follows. 1. Some S can be U.
(c) If either conclusion I or II follows. 2. Some C can be O.
(d) If neither conclusion I nor II follows. (a) If only conclusion I follows.
(e) If both conclusions I and II follow. (b) If only conclusion II follows.
29)Statements: (c) If either conclusion I or II follows.
A few of my friends are teachers. (d) If neither conclusion I nor II follows.
Teachers are rude. (e) If both conclusions I and II follow
Some rude persons are good at heart. Ans: E
Conclusions:

1. A few of my friends are good at


heart.
2. Some rude persons may not be
32) Statements:
teachers.
All R are M.
(a) If only conclusion I follows.
Only a few P are T.
(b) If only conclusion II follows.
Some M are P.
(c) If either conclusion I or II follows.
Conclusions:
(d) If neither conclusion I nor II follows.
(e) If both conclusions I and II follow. 1. Some T can never be R.
2. Some T can be R.
30)Statements: (a) If only conclusion I follows.
A few of my friends are teachers. (b) If only conclusion II follows.
Teachers are rude. (c) If either conclusion I or II follows.
Some rude persons are good at heart. (d) If neither conclusion I nor II follows.
Conclusions: (e) If both conclusions I and II follow
Ans: B
1. A few of my friends are good at
heart.

Page 89 Follow us: Official Site, Telegram, Facebook, Instagram, Instamojo


34). Statements:
Only a few wells are mats.
All pillows are mats.
Conclusions:
33). Statements:
Only a few lamps are bottles. 1. At least some pillows are wells.
No bottle is ship. 2. All wells can never be pillow.
Conclusions: Ans:B

1. Some ships are definitely not


lamps.
2. All lamps can never be ships.
Ans: B

(Possibility Syllogism)
Directions (1-5): Each questions below
contain 2 conclusions followed by
statements. Find from which of the
statements given, both the conclusions
given follow.

1. Conclusions:
I. All phones are laptops is a
possibility.
II. No phone is tablet.
STATEMENTS: 2. Conclusions:
A) All phones are tabs. Some tabs are I. All arm are eye is a possibility.
laptops. Some tabs are tablets. II. Some ear are not arm.
B) Some phones are tabs. No tab is STATEMENTS:
laptop. All laptops are tablets. A) No ear is nose. Some nose are arm.
C) All phones are tabs. Some laptops No nose is eye.
are tabs. No tablet is tab. B) Some ear are nose. All nose are eye.
D) No phone is tab. All laptops are No arm is nose.
tabs. No tablet is phone C) All ear are nose. All nose are arm.
E) None No nose is eye.
Answer: Option C D) Some nose are ear. All arm are
nose. No ear is eye.
E) None

Page 90 Follow us: Official Site, Telegram, Facebook, Instagram, Instamojo


Answer: Option B II. All red are green is a possibility.
STATEMENTS:
For conclusion I: A) Some red are blue. No white is blue.
All green are white.
B) Some red are blue. Some blue are
white. No blue is green.
C) All red are blue. Some red are
white. No white is green.
D) Some red are blue. All white are
blue. No white is green
E) None
In B option, since All ears are arms is
not a possibility, so II concl. also
follows Answer: Option D

3. Conclusions:
I. All euro are rupee is a possibility.
II. All rupee are yen is a possibility.
STATEMENTS:
A) All rupee are dollar. No dollar is
yen. Some dollar are euro.
B) No rupee is dollar. Some dollar are
yen. No yen is euro.
C) Some rupee are dollar. No yen is
dollar. Some euro are yen.
5. Conclusions:
D) No rupee is dollar. All dollar are
I. All parrots are sparrows is a
yen. Some dollar are yen
possibility.
E) None
II. Some sparrows are pigeons.
STATEMENTS:
Answer: Option B A) All eagles are sparrows. Some
pigeons are parrots. No parrot is
sparrow.
B) All eagles are sparrows. Some
eagles are pigeons. No pigeon is
parrot.
C) Some eagles are sparrows. Some
eagles are parrots. No parrot is pigeon.
D) Some eagles are sparrows. Some
eagle is parrot. No pigeons are eagles.
E) None
4. Conclusions:
I. All green are blue is a possibility.
Answer: Option B

Page 91 Follow us: Official Site, Telegram, Facebook, Instagram, Instamojo


QUANTITATIVE APTITUDE SECTION
Quantitative aptitude is an important section for Banking exams Like IBPS, SBI, IBPS RRB, SEBI, and
other exams. In this space, we will be providing you everything that you need to know to ace this
section. This will be beneficial for you if you are appearing for any of these exams
TOPIC COVERD
 Simplification
 Ratio and proportion
 Percentage
 Average
 Partnership
 Quadratic equation
 Profit and Loss
 Time and work
 Pipes and cisterns
 Boats and stream
 Problems on age
 Problems on train
 Mixture of allegation
 Time, speed & distance
 Mensuration
 Data interpretation
 Simple and Compound interest
 Permutation,
 Combination & Probability

Tips to Prepare Quantitative Aptitude


 Follow these guidelines to prepare quantitative aptitude for bank exams and other competitive
exams:
 Improve your calculation speed by doing calculations manually, learning tables till 30, square
roots and cube roots.
 Practice arithmetic portion thoroughly and learn all important formulas to solve arithmetic word
problems.
 Learn BODMAS and basic mathematical tips for calculation.

Page 92 Follow us: Official Site, Telegram, Facebook, Instagram, Instamojo


 Read the instructions of every question very carefully.
 It is important to practice with a time bound approach as time management is a must for
competitive exams.
 Practice with latest pattern based questions on bankersadda.com Also check previous years’
papers to get an idea about the level and types of questions in trend.

Simplification & Approximation


Rules of Simplification

Order of operations - BODMAS rule what is BODMAS rule?

BODMAS rule defines the correct sequence in which operations are to be performed in a given
mathematical expression to find its value.

We are providing you some tips which can help you to ace this topic:

In BODMAS,
B = Bracket
O = Order (Powers, Square Roots, etc.)
DM = Division and Multiplication (left-to-right)
AS = Addition and Subtraction (left-to-right)

 Learn Tables up to 25
 Learn Cubes up to 25
 Learn Squares up to 25
 Learn Fractions up to 25

Solve the simplification by using VBODMAS rule i.e


V → Vinculum
B → Remove Brackets – in the order ( ) , { }, [ ]
O → Of
D → Division
M → Multiplication
A → Addition
S → Subtraction

In some countries, the acronym PEMDAS is used instead of BODMAS. PEMDAS stands for
"Parentheses, Exponents, Multiplication, Division, Addition, and Subtraction". Some other variations
used to represent the same concept are BIDMAS, ERDMAS, PERDMAS and BPODMAS.

Order of operations

While simplifying an expression, the following order must be followed.

 Do operations in brackets first, strictly in the order (), {} and []


Page 93 Follow us: Official Site, Telegram, Facebook, Instagram, Instamojo
 Evaluate exponents (powers, roots, etc.)
 Perform division and multiplication, working from left to right. (division and multiplication
rank equally and done left to right).
 Perform addition and subtraction, working from left to right. (addition and subtraction rank
equally and done left to right).

Practise Questions
(Simplification Questions)
a. 12
1. 1035 ÷ [(3/4) of (71 + 65) – b. 4
15 ¾] = ? c. 9
d. 15
a. 12 e. None of these
b. 24 Answer: b
c. 18 Explanation:
d. 26 √((27 /5 )×(x/15)) =0.9+0.3
e. None of these (3/5) √ x = 1.2
Answer: a √ x = 6/3 =2
Explanation: X= 4
1035 ÷ [(3/4)*136 – 63/4] = x
X = 1035 ÷ [102 – (63/4)] 4. 5 ¾ – 4 1/8 + 7 ½ – 3 3/8 = ?
X = 1035 ÷ [(408 – 63)/4]
X = 1035 ÷ (345/4) a. 4 3/7
X = 1035 *4/345 b. 6 5/6
X = 12 c. 7 ½
d. 5 ¾
2. (7 × 7)3 ÷ (49 × 7)3 × (2401)2 = 7? e. None of these
Answer :d
a. 7 Explanation:
b. 5 5 ¾ – 4 1/8 + 7 ½ – 3 3/8 = x
c. 6 X = (5 – 4 + 7 – 3) (3/4 – 1/8 + ½ – 3/8)
d. 8 X = 5 [(6 – 1 + 4 – 3)/8]
e. None of these X = 5 (6/8) = 5 (3/4)
Answer: b
Explanation: 5. 363 × 40961/2 × 72 × 18 ÷ (93 × 722)
(7 × 7)3 ÷ (49 × 7)3 × (2401)2 = 7x = 4?
(72)3 ÷ (72 × 7)3 × (74)2 = 7x
76 ÷ 79 × 78 = 7x a. 8
76 – 9 + 8 = 7 x b. 12
75 = 7 x c. 7
X=5 d. 5
e. None of these
3. √((27 ÷ 5 ×?) ÷ 15) = 5.4 ÷ 6 +0.3 Answer :d
Page 94 Follow us: Official Site, Telegram, Facebook, Instagram, Instamojo
Explanation: c. 1960
(36*36*36*64*72*18)/(9*9*9*72*72) = d. 1890
4x e. 2370
4*4*43 = 4x Answer :d
45 = 4 x Explanation:
X=5 (3/7)*x – (2/5)*(13/16)*2160 =
(12/100)*900
6. (2 7/9) ÷ (5 1/2) × (693/350) = (3x/7) – 702 = 108
(1/3) of (5/7) of (147/175) of (?) (3x/7) = 108 + 702
(3x/7) = 810
a. 12 X = (810*7/3) = 1890
b. 26
c. 5 9. 12 ¾ % of 2400 + 23 ½ % of 800 +
d. 18 452 = 1125 ÷ 25 × 4 + ?
e. 9
Answer :c a. 658
Explanation: b. 532
(25/9)*(2/11)*(693/350) = c. 776
(1/3)*(5/7)*(147/175)*x d. 614
1 = (x/5) e. 590
X=5 Answer :c
7. (913 + 329 + 522 + 343) ÷ (18 + 24 Explanation:
– ? + 18) = 43 (51/400)*2400 + (47/200)*800 + 2025
= (1125/25) + 4x
a. 23 306 + 818 + 2025 – 45 = 4x
b. 11 3104 = 4x
c. 7 X = 3104/4 = 776
d. 29
e. 15 10. ∛157464 + √8836 – 45 % of 800
Answer :b =? + (1188 ÷ 9) – 362
Explanation:
(913 + 329 + 522 + 343) ÷ (18 + 24 – x + a. 952
18) = 43 b. 816
2107 ÷ (60 –x) = 43 c. 878
(2107/43) = 60 – x d. 924
49 = 60 – x e. None of these
X = 60 – 49 = 11 Answer :a
Explanation:
8. (3/7) of ? – (2/5) of (13/16) of 54 + 94 – (45/100)*800 = x + (1188/9)
2160 = 12 % of 900 – 1296
54 + 94 – 360 + 1296 = x + 132
a. 2150 54 + 94 – 360 + 1296 – 132 = x
b. 1540
Page 95 Follow us: Official Site, Telegram, Facebook, Instagram, Instamojo
X = 952 e. 61
Answer :a
11. (4/5) of 11325 + 56 % of 1750 = Explanation:
(?)2 + 436 1.8 × 625 + 5.6 ÷ 0.7 + 415 × 11.8 = (?)2 –
211
a. 116 1125 + 8 + 4897 = x2 – 211
b. 72 6241 = x2
c. 98 X = 79
d. 84
e. 102 14. 18 ¼ % of 17200 + 33 2/3 % of
Answer :c 1860 + 37 2/3 % of 19680 = ?
Explanation:
(4/5)*11325 + (56/100)*1750 = x2 + a. 11178
436 b. 13456
9060 + 980 – 436 = x2 c. 9824
10040 – 436 = x2 d. 10242
9604 = x2 e. 8454
X = 98 Answer :a
Explanation:
12. (4/5) of 11325 + 56 % of 1750 = (73/400)*17200 + (101/300)*1860 +
(?)2 + 436 (113/300)*19680= x
X = 3139 + 626.2 + 7412.8
a. 116 X = 11178
b. 72
c. 98 15. 586 × 9234 ÷ 48 % of 950 = 36 ×
d. 84 750 ÷ 4 + ? + 4500
e. 102
Answer :c a. 25
Explanation: b. 728
(4/5)*11325 + (56/100)*1750 = x2 + c. 5
436 d. 75
9060 + 980 – 436 = x2 e. 474
10040 – 436 = x2 Answer :c
9604 = x2 Explanation:
X = 98 586 × [9234 ÷ ((48/100)*950)] = 36 ×
(750/4) + x + 4500
13. 1.8 × 625 + 5.6 ÷ 0.7 + 415 × 11.8 = 586*(9234/456) = 6750 + x + 4500
(?)2 – 211 11866.5 – 11250
X = 616.5
a. 79
b. 85 16. 1323 ÷ (27 % of 700) = 35 × 7 3/5
c. 92 of (? ÷ 190)
d. 69
Page 96 Follow us: Official Site, Telegram, Facebook, Instagram, Instamojo
a. 7 (32450/11) + (4/9)*11520 +
b. 9 (48/100)*7500 = x2 – 430
c. 4 2950 + 5120 + 3600 + 430 = x2
d. 5 X2 = 12100
e. 8 X = 110
Answer :d
Explanation: 19. √13225 + ∛205379 + 52 % of 700
1323 ÷ [(27/100)*700] = 35 × = 408 + ?
(38/5)*(x/190)
1323 ÷ 189 = 35 × (38/5)*(x/190) a. 156
(7*5*190)/(35*38) = x b. 218
X=5 c. 189
d. 130
17. (28 × 9 + 54 × 3 + 12 × 11) ÷ (142 – e. None of these
√961 + 17) = ? Answer :d
Explanation:
a. 6 √13225 + ∛205379 + 52 % of 700 = 408
b. 3 +x
c. 5 115 + 59 + (52/100)*700 – 408 = x
d. 8 115 + 59 + 364 – 408 = x
e. 7 X = 130
Answer :b
Explanation: 20. 492 ÷ 14 × 8 + 582 = 4 × ? – 15 % of
(28 × 9 + 54 × 3 + 12 × 11) ÷ (142 – √961 2480
+ 17) = x
X = (252 + 162 + 132) ÷ (196 – 31 + 17) a. 1489
X = 546/182 b. 1277
X=3 c. 1645
d. 1823
18. 32450 ÷ 11 + (4/9) of 11520 + 48 e. None of these
% of 7500 = ?2 – 430 Answer :b
Explanation:
a. 150 (49*49*8)/14 + 582 = 4x –
b. 178 (15/100)*2480
c. 110 1372 + 3364 = 4x – 372
d. 92 1372 + 3364 + 372 = 4x
e. None of these 5108 = 4x
Answer :c X = 5108/4 = 1277
Explanation:

(Approximation)

Page 97 Follow us: Official Site, Telegram, Facebook, Instagram, Instamojo


1. 324.99 of 48.07% =? 5)3590
1) 162
2)156 7. 72.35 × 190 + 26 × 13.5 – 110 ×
3) 174 24.5 =?
4) 138 1)11490
5) 144 2)11370
3)11580
4)11290
1)281 5)11670
2)291
8. 49.8% of 579.88 – 529.78% of 540
3)271
+ 4451.887 =?
4)251
1)1880
5)261
2) 1460
3. 38.93 × 112.67 + 223.854 × 17.893 3)1360
=? 4)1580
1)8340 5) 1790
2)8430
9. (42.42)² + (39.549)² – (11.764)² –
3)8450
(45.47)² =?
4)8440
1) 1010
5)8450
2)1195
4. 945.11 ÷ 34.98 × 20.054 =? 3)2155
1)413 4)2210
2)527 5)2020
3) 365
10. 32.23% of 624.699 + 49.876% of
4)619
650.112 + 2574.98 =?
5) 567
1)4200
5. (18.79)² × √730 =? 2)3100
1)9570 3)2700
2)9750 4)1750
3)9700 5)5100
4)9500
5)10500
1)6
6. (16.072)² + (22.202)² + (35.986)² + 2)7
(38.8761)² =? 3)8
1)3560 4)9
2)3650 5)5
3)3750
4)3570
Page 98 Follow us: Official Site, Telegram, Facebook, Instagram, Instamojo
12. 487.532 + 2849.029 – 675.48 = 14. 61.33% of 700 – 450.17 = ? – 85%
743.095 + ? of 1020
1)1900 a) 890
2)1800 2) 820
3)1820 3)910
4)1900 4) 830
5)1920 5) 1040

13. (630 +?) x 26 = 2860 15. √6578 × √4330÷ √730 = ?


1)120 1) 171
2)260 2)151
3)520 3)191
4)315 4)161
5)290 5)201

SOLUTION (1 to 10)
Ans 1. 2 ? = 3557
325 × (48/100) =?
? = 156 Ans 7. 4
72 × 190 + 26 × 14 – 110 × 25 =?
Ans 2. 1 13680 + 364 – 2750 =?
1845 × (48/315) =? ? = 11294
? = 281.14
? = 281 approx Ans 8. 1
290 – 2862 + 4452 =?
Ans 3. 4 ? = 1880
39 × 113 + 224 × 18 =?
4407 + 4032 =? Ans 9. 2
? = 8439 (42)² + (40)² – (12)² – (45)² =?
Ans 4. 5 1764 + 1600 – 144 – 2025 =?
945 ÷ 35 × 21 =? 3364 – 2169 =?
27 × 21 =? ?= 1195
? = 567
Ans 10. 2
Ans 5. 2 32% of 625 + 50% of 650 + 2575 =?
(19)² × √730 =? 200 + 325 + 2575 =?
361 × 27 =? ? = 3100
? = 9747
Ans 11. 2
Ans 6. 1 (425 ÷ 12.5 +?³) ÷ 30 = 12(1/2)
(16)² + (22)² + (36)² + (39)² =? (34 + ?³) ÷ 30 = (25/2)
256 + 484 + 1296 + 1521 =? 34 + ?³ = 375

Page 99 Follow us: Official Site, Telegram, Facebook, Instagram, Instamojo


?³ = 341 ? = 520
? = 7 approx
Ans 14. 1
Ans 12. 5 61% of 700 – 450 =? – 85% of 1020
488 + 2849 – 675 = 743 +? 427 – 450 =? – 867
? = 1919 ? = 890

Ans 13. 3 Ans 15. 5


(630 +?) = 2860/26 (81 x 67)/27 =?
(630 +?) = 110 ? = 201

Number Series
Rules of Number Series

There are various tricks to solve the number of series problems provided by Mathematics. Check
these tricks below –

 If all the numbers are prime, even or odd.


 If all the numbers have particular divisibility.
 If all the numbers are perfect squares or cubes.
 If all the numbers succeed by a certain number of additions or subtractions or multiplications
or divisions, or by adding their cubes and squares.

1. Series consisting of Perfect Squares –

A series based on Perfect Squares is mostly based in a specific order of the perfect squares of
numbers, and in this type of series one of the numbers is generally missing which you have to find.

The formula is used to solve the number series problems is Xn= n2.

2. Perfect Cube Series –

In this type, the series consists of the cubes of numbers which are in same order and you have to find
out the missing or odd cube number.

The formula is used to solve the number series questions is Xn= n3.

3. Rational Number Series -

These are the numbers that can be written as a fraction or quotient in which both numerator and
denominator are integers.

Page 100 Follow us: Official Site, Telegram, Facebook, Instagram, Instamojo
4. Arithmetic Series –

It is a mathematical sequence there is a fixed difference between the numbers. The next terms are
obtained either by adding a fixed number or by subtracting it.

The formula used to solve the number series problems is Xn = x1 + (n – 1)d

5. Geometric Series -

It is a sequence in which each term of series is obtained by a fixed number multiplying or dividing the
preceding number.

6. Arithmetic-Geometric Series –

As the name suggests - a peculiar combination of Arithmetic and Geometric series forms the
Arithmetic –Geometric series. An important property of this series is that in the Geometric Sequence
there are differences of consecutive terms.

7. Geometrico- Arithmetic Series –

It is the reverse of the Arithmetic –Geometric series. In this series, the differences of the suggestive
terms are in the Arithmetic Series.

Practice Questions
(Missing Number Series)
What should come at the place of question mark (?) in the following number
series?
Ques 1. 26, 12, 10, 16, ? Ques 3. 122, 213, 340, 509, 726, ?
A. 942
A. 50
B. 997
B. 52
C. 919
C. 53
D. 950
D. 56
E. 922
E. 57
Ques 4. 25, 37.5, 56.25, ?, 12.5625
Ques 2. 82, 107, 207, 432, 832, ?
A. 72.375
A. 1550
B. 84.375
B. 1450
C. 92.255
C. 1457
D. 59.750
D. 1542
E. 69.450
E. 1478
Ques 5. 10, 50, 94, 142, 194, ?
A. 250
Page 101 Follow us: Official Site, Telegram, Facebook, Instagram, Instamojo
B. 270 E. 20
C. 260
D. 240 Ques. 11. 26, 34, ?, 50, -6, 66
E. 220
A. 12
Ques. 6. 3420, 1660, 780, 340, ?, 10 B. 10
C. 22
A. 135
D. 8
B. 120
E. 9
C. 75
D. 185
E. 90 Ques. 12. 52, ?, 105, 149, 206, 277
Ques. 7. 50, ?, 150, 240, 375, 577.5 A. 75
A. 90 B. 84
B. 60 C. 22
C. 80 D. 18
D. 75 E. 73
E. 95
Ques. 13. 13860, 6930, ?, 462, 66, 6
Ques. 8. 2, 10, 30, 68, ? A. 2272
A. 160 B. 2414
B. 125 C. 2310
C. 130 D. 2388
D. 135 E. 2339
E. None of these
Ques. 14. 16, 22, 42, 84, 156, ?
A. 255
Ques. 9. 4, 5, 18, 81, ?, 2065 B. 250
A. 272 C. 280
B. 414 D. 266
C. 492 E. 215
D. 388
E. 339
Ques. 15. 1448, 1446, 720, 236, ?, 5
A. 62
Ques. 10. 20, 25, 28, ?, 44, 43 B. 75
A. 31 C. 55
B. 23 D. 28
C. 12 E. 39
D. 32

Page 102 Follow us: Official Site, Telegram, Facebook, Instagram, Instamojo
SOLUTION (1 to 15)
Answer 1: D
Solution

Answer 2: C
Solution

Answer 3: B
Solution

Answer 4: B Solution

Answer 5: A

Page 103 Follow us: Official Site, Telegram, Facebook, Instagram, Instamojo
Solution

Answer 6: B
Solution:

Answer 7: A
Solution:

Answer 8: C
Solution:

Answer 9: D

Page 104 Follow us: Official Site, Telegram, Facebook, Instagram, Instamojo
Solution:

Answer 10: A
Solution:

Answer 11: B
Solution:

Answer 12: E
Solution:

Answer 13: C

Page 105 Follow us: Official Site, Telegram, Facebook, Instagram, Instamojo
Solution:

Answer 14: D
Solution:

Answer 15: C
Solution:

(Wrong Number Series) A.1575


Direction (1-50). Find out the wrong B.68
number in the following number
C.135
series.
D.70
1. 2.5, 5, 25, 200, 2201, 30800
E.5395
A.2.5
3. 643, 768, 805, 1147, 1211, 1940
B.200
A.805
C.2201
B.643
D.25
C.768
E.5
D.1211
2. 70, 68, 135, 395, 1575, 7869

Page 106 Follow us: Official Site, Telegram, Facebook, Instagram, Instamojo
E.1940 B.4
4. 8, 5, 6, 11, 21, 53.5 C.10
A.53.5 D.17
B.11 E.33
C.21 9. 2, 22, 47, 77, 115
D.5 A.115
E.6 B.77
5. 339, 341, 345, 353, 369, 400 C.47
A.400 D.22
B.341 E.2
C.345 10. 0.5, 2, 4, 7.5, 9.5, 13
D.353 A.3
E.339 B.14
6. 5, 20, 50, 112, 230, 470 C.10.5
A.112 D.5
B.20 E.7.5
C.50 11. 513, 520, 530, 544, 561, 580
D.230 A.513
E.470 B.530
7. 20, 25, 21, 27, 24, 29 C.520
A.20 D.544
B.21 E.561
C.27 12. 305, 336, 373, 422, 494, 616
D.24 A.305
E.29 B.422
8. 3,4,10,17,33,58 C.373
A.58 D.494

Page 107 Follow us: Official Site, Telegram, Facebook, Instagram, Instamojo
E.616 17. 256, 240, 218, 179, 130, 66
13. 15, 14, 25, 75, 296, 1475 A.218
A.14 B.66
B.1475 C.179
C.25 D.240
D.296 E.130
E.15 18. 32, 39, 56, 63, 80, 99
14. 4, 13, 41, 125, 389, 1183 A.39
A.13 B.56
B.389 C.99
C.41 D.80
D.1183 E.63
E.125 19. 24, 47, 78, 117, 164, 220
15. 105, 116, 131, 149, 167, 190 A.164
A.131 B.78
B.190 C.220
C.149 D.117
D.167 E.47
E.105 20. 13, 26, 78, 312, 1560, 9380
16. 32, 52, 92, 172, 332, 650 A.13
A.332 B.26
B.92 C.312
C.650 D.9380
D.172 E.1560
E.33

Solutions and explanations:

Page 108 Follow us: Official Site, Telegram, Facebook, Instagram, Instamojo
1. Answer: C 5. Answer: A
2.5 x 2 = 5 339 + = 341
5 x 5 = 25 341 + = 345
25 x 8 = 200 345 + = 353
200 x 11 = 2200 353 + = 369
2200 x 14 = 30800 369 + = 401

2. Answer: C 6. Answer: A
70 x 1 - 2 =68 5*2 + 10 = 20
68 x 2 – 3 = 133 20*2 +10 =50
133 x 3 – 4 = 395 50*2 +10 = 110
395 x 4 – 5 = 1575 110*2+10 = 230
1575 x 5 -6 = 7869 230*2+10 = 470

3. Answer: A 7. Answer: B
643 + = 768 20+5=25
768 + = 804 25-3 = 22
804 + = 1147 22+5 = 27
1147 + = 1211 27 – 3 = 24
1211 + = 1940 24 + 5 = 29

4. Answer: B 8. Answer: C
8 x 0.5 +1 = 5 3+ =4
5x1+1=6 4+ =8
6 x 1.5 + 1 = 10 8+ = 17
10 x 2 + 1 = 21 17 + = 33
21 x 2.5 +1 = 53.5 33 + = 58

Page 109 Follow us: Official Site, Telegram, Facebook, Instagram, Instamojo
9. Answer: A 75 x 4 - 4 = 296
2 22 47 77 112 296 x 5 -5 = 1475
20 25 30 35
14. Answer: E
10. Answer: E 4 x 3 + 1 = 13
0.5 + 1.5 = 2 13 x 3 +2 = 41
2 +2 = 4 41 x 3 +4 = 127
4+ 2.5 =6.5 127 x 3 + 8 = 389
6.5 + 3 = 9.5 389 x 3 + 16 = 1183
9.5 + 3.5 = 13
15. Answer: C
11. Answer: B 105 + 11 = 116
513 + 7 = 520 116 +15 = 131
520 + 11 = 531 131 + 17 = 148
531 + 13= 544 148 + 19 = 167
544 + 17= 561 169 + 21 = 190
561 + 19= 580
16. Answer: C
12. Answer: D 32 + 20 = 52
305 336 373 422 495 616 52 + 40 = 92
31 37 49 73 121 92 + 80 = 172
6 12 24 48 172 + 160 = 332
332 + 320 = 652
13. Answer: C
15 x 1-1 = 14 17. Answer: A
14 x 2 – 2 = 26 256 – = 240
26 x 3 – 3 = 75 240 – = 215

Page 110 Follow us: Official Site, Telegram, Facebook, Instagram, Instamojo
215 – = 179 – 2 = 47
179 – = 130 – 3 = 78
130 – = 66 – 4 = 117
– 5 = 164
18. Answer: B – 6 = 219
32 + 7 = 39
39 + 11 = 50 20. Answer: D
50 + 13 = 63 13 * 2 = 26
63 + 17 = 80 26 * 3 = 78
80 + 19 = 99 78 * 4 = 312
312 * 5 = 1560
19. Answer: C 1560 * 6 = 9360
– 1 = 24

Quadratic Equation/Inequality
Rules of Inequality (Quadratic Equation)

 Quadratic Equation in Standard Form: ax2 + bx + c = 0


 Quadratic Equations can be factored
 Quadratic Formula: x = −b ± √(b2 − 4ac)2a
 When the Discriminant (b2−4ac) is:
o positive, there are 2 real solutions
o zero, there is one real solution
o negative, there are 2 complex solutions

Practice Questions
Directions (Q. 1 – 5): In the following questions, two equations I and II are given.
You have to solve both the equations and give Answer as,

a) If x > y

b) If x ≥ y

Page 111 Follow us: Official Site, Telegram, Facebook, Instagram, Instamojo
c) If x < y

d) If x ≤ y

e) If x = y or the relation cannot be established

1) I) x2 – 3x – 28 = 0 3) I) 2x – 3y = -6

II) y2 – y – 72 = 0 II) 3x + 4y = 25

2) I) 3x2 + 19x + 28 = 0 4) I) 12x2– 37x + 21 = 0

II) 2y2 + 13y + 21 = 0 II) 15y2 + 54y + 27 = 0

5) I) x2 + 3√7 x – 70 = 0

II) y2 + 2√3 y – 105 = 0

SOLUTION: (1 to 5)
1) Answer: e) Can’t be determined

I) x2 – 3x – 28 = 0 2) Answer: e)

(x + 4) (x – 7) = 0 I) 3x2 + 19x + 28 = 0

X = -4, 7 3x2 + 12x + 7x + 28 = 0

II) y2 – y – 72 = 0 3x (x + 4) + 7 (x + 4) = 0

(y + 8) (y – 9) = 0 (3x + 7) (x + 4) = 0

Y = -8, 9 x = -7/3, -4 = -2.33, – 4

Page 112 Follow us: Official Site, Telegram, Facebook, Instagram, Instamojo
II) 2y2 + 13y + 21 = 0 (4x – 3) (3x – 7) = 0

2y2 + 6y + 7y + 21 = 0 x = ¾, 7/3 = 0.75, 2.33

2y (y + 3) + 7 (y + 3) = 0 II) 15y2 + 54y + 27 = 0

(2y + 7) (y + 3) = 0 15y2 + 45y+ 9y + 27 = 0

y = -7/2, -3 = -3.5, -3 15y (y + 3) + 9 (y + 3) = 0

Can’t be determined (15y + 9) (y + 3) = 0

y = -9/15, -3 = -3/5, -3

3) Answer: c) x>y

2x-3y = -6 –> (1) 5) Answer: e)

3x + 4y = 25 –> (2) I) x2 + 3√7 x – 70 = 0

By solving the equation (1) and (2), x2 + 5√7 x – 2√7 x – 70 = 0

x = 3, y = 4 (x + 5√7) (x – 2√7) = 0

x<y x = 2√7, – 5√7

II) y2 + 2√3 y – 105 = 0

4) Answer: a) y2 + 7√3 y – 5√3 y – 105 = 0

I) 12x2 – 37x + 21 = 0 (y + 7√3) (y – 5√3) = 0

12x2 – 28x – 9x + 21 = 0 y = 5√3, – 7√3

4x (3x – 7) – 3 (3x – 7) = 0 Can’t be determined

(Quantity based Quadratic Equation)


11) The ratio of the present age of 12 years younger than Rahim.
Bala to that of Arnav is 3 : 11. Arnav is Rahim’s age after 7 years will be 85

Page 113 Follow us: Official Site, Telegram, Facebook, Instagram, Instamojo
years. D. Quantity I ≤ Quantity II
Quantity I: The present age of Bala’s Explanation:
father, who is 25 years older 80/100 * x = 140/100 * y
than Bala x = 7/4y
Quantity II: Rahim’s present age x + y = 440
A. Quantity I > Quantity II 7/4 y + y = 440
B. Quantity I < Quantity II y = 160 ; x = 280
C. Quantity I ≥ Quantity II
D. Quantity I ≤ Quantity II
E. Quantity I = Quantity II or relation
cannot be established 13) Ravi, Hari and Sanjay are three
typists, who working simultaneously,
Answer: can type 228 pages in four hours. In
B. Quantity I < Quantity II one hour, Sanjay can type as many
Explanation: pages more than Hari as Hari can type
11x = 85 – 7 – 12 more than Ravi. During a period of
x=6 five hours, Sanjay can type as many
Present age of Bala = 18 passages as Ravi can, during seven
Present age of Bala’s father = 18 + 25 = hours.
43; Rahim’s present age = 78 Quantity I: Number of pages typed by
Ravi
Quantity II: Number of pages typed by
Hari
12) Mr. Ramesh bought two watches A. Quantity I > Quantity II
which together cost him Rs.440. He B. Quantity I < Quantity II
sold one of the watches at a loss of C. Quantity I ≥ Quantity II
20% and the other one at a gain of D. Quantity I ≤ Quantity II
40%. The selling price of both E. Quantity I = Quantity II or relation
watches are same. cannot be established
Quantity I: SP and CP one of the
watches sold at a loss of 20% Answer:
Quantity II: SP and CP one of the B. Quantity I < Quantity II
watches sold at a profit of 40% Explanation:
A. Quantity I > Quantity II Let Ravi, Hari and Sanjay can type x, y,
B. Quantity I < Quantity II and z pages respectively in 1 h.
C. Quantity I ≥ Quantity II Therefore, they together can type 4(x + y
D. Quantity I ≤ Quantity II + z) pages in 4 h
E. Quantity I = Quantity II or relation ∴ 4(x + y + z) = 228
cannot be established ⇒ x + y + z = 57 …..(i)
Also, z – y = y – x
Answer:
Page 114 Follow us: Official Site, Telegram, Facebook, Instagram, Instamojo
i.e., 2y = x + z ……(ii) Answer:
5z = 7x ……(iii) D. Quantity I ≤ Quantity II
From Eqs. (i) and (ii), we get Explanation:
3y = 57 x² – 26x + 168 = 0
⇒ y = 19 x = 12, 14
From Eq. (ii), x + z = 38 y² – 29y + 210 = 0
x = 16 and z = 22 y = 14, 15

14) The length of a rectangle wall is 16) Quantity I: x² – 21x + 110 = 0


3/2 times of its height. The area of the Quantity II: y² – 18x + 80 = 0
wall is 600m². A. Quantity I > Quantity II
Quantity I: Height of the wall B. Quantity I < Quantity II
Quantity II: Length of the wall C. Quantity I ≥ Quantity II
A. Quantity I > Quantity II D. Quantity I ≤ Quantity II
B. Quantity I < Quantity II E. Quantity I = Quantity II or relation
C. Quantity I ≥ Quantity II cannot be established
D. Quantity I ≤ Quantity II
E. Quantity I = Quantity II or relation Answer:
cannot be established C. Quantity I ≥ Quantity II
Explanation:
Answer: x² – 21x + 110 = 0
A. Quantity I > Quantity II x = 10 11
Explanation: y² – 18y + 80 = 0
length = 3x y = 10 8
height = 2x
Area of the wall = 3x * 2x = 6x² = 600
Length = 30 & Height = 20
17) A Cistern has an inlet pipe and
outlet pipe. The inlet pipe fills the
cistern completely in 1 hour 20
15) Quantity I: x² – 26x + 168 = 0 minutes when the outlet pipe is
Quantity II: y² – 29y + 210 = 0 plugged. The outlet pipe empties the
A. Quantity I > Quantity II tank completely in 6 hours when the
B. Quantity I < Quantity II inlet pipe is plugged.
C. Quantity I ≥ Quantity II Quantity I: X = Inlet Pipe Efficiency
D. Quantity I ≤ Quantity II Quantity II: Y = Outlet Pipe Efficiency
E. Quantity I = Quantity II or relation A. Quantity I > Quantity II
cannot be established B. Quantity I < Quantity II
C. Quantity I ≥ Quantity II
Page 115 Follow us: Official Site, Telegram, Facebook, Instagram, Instamojo
D. Quantity I ≤ Quantity II Quantity I: Probability that at least
E. Quantity I = Quantity II or relation one is Black.
cannot be established Quantity II: Probability that all is
Black.
Answer: A. Quantity I > Quantity II
A. Quantity I > Quantity II B. Quantity I < Quantity II
Explanation: C. Quantity I ≥ Quantity II
Inlet pipe Efficiency = 100/(8/6) = 75% D. Quantity I ≤ Quantity II
Outlet pipe Efficiency = 100/(6) = E. Quantity I = Quantity II or relation
16.66% cannot be established

Answer:
18) Out of 14 applicants for a job, A. Quantity I > Quantity II
there are 6 women and 8 men. It is Explanation:
desired to select 2 persons for the Total Balls = 15
job. Probability = 11c4/15c4 = 22/91
Quantity I: Probability of selecting no One is black = 1 – 22/91 = 69/91
woman
Quantity II: Probability of selecting at
least one woman
A. Quantity I > Quantity II 20) Two pipes A and B can fill a tank
B. Quantity I < Quantity II in 12 hours and 18 hours
C. Quantity I ≥ Quantity II respectively. The pipes are opened
D. Quantity I ≤ Quantity II simultaneously and it is found that
E. Quantity I = Quantity II or relation due to leakage in the bottom of the
cannot be established tank it took 48 minutes excess time to
Answer: fill the cistern.
B. Quantity I < Quantity II Quantity I: Due to leakage, time taken
Explanation: to fill the tank
Man only = 8C2 = 14
Quantity II: Time taken to empty the
Probability of selecting no woman =
full cistern
14/91
A. Quantity I > Quantity II
Probability of selecting at least one
B. Quantity I < Quantity II
woman = 1 – 14/91 = 77/91
C. Quantity I ≥ Quantity II
D. Quantity I ≤ Quantity II
E. Quantity I = Quantity II or relation
19) A basket contains 6 White 4 Black cannot be established
2 Pink and 3 Green balls. If four balls
are picked at random, Answer:

Page 116 Follow us: Official Site, Telegram, Facebook, Instagram, Instamojo
B. Quantity I < Quantity II = 7 hours 12 min + 48 min = 8 hours
Explanation: Work done by two pipes and leak in 1
Work done by the two pipes in 1 hour = hour = 1/8.
(1/12)+(1/18) = (15/108). Work done by the leak in 1 hour
Time taken by these pipes to fill the tank =(15/108)-(1/8)=(1/72).
= (108/15)hrs = 7 hours 12 min. Leak will empty the full cistern in 72
Due to leakage, time taken to fill the tank hours.

Average & Ages


Formulas & Shortcuts of Average & Ages

Average

Average refers to the sum of numbers divided by n. Also called the mean average.

Sums of data divided by the number of items in the data will give the mean average. The mean average
is used quite regularly to determine final math marks over a term or semester. Averages are often
used in sports: batting averages which means number of hits to number of times at bat. Gas mileage is
determined by using averages.

Hence, average =

Ages

Important Formulas on "Problems on Ages" :

1. If the current age is x, then n times the age is nx.

2. If the current age is x, then age n years later/hence = x + n.

3. If the current age is x, then age n years ago = x - n.

4. The ages in a ratio a : b will be ax and bx.

5. If the current age is x, then 1 of the age is x .


n n
Practice Questions
(Average Problems)
1. Average weight of the class is 28 kg increased by 3 kg when the teacher is
and the average weight of class is included. If the number of students in

Page 117 Follow us: Official Site, Telegram, Facebook, Instagram, Instamojo
the class is 27, then find the weight of B.20
teacher? C.30
A.108 kg D.35
B.112 kg E.40
C.116 kg
D.104 kg 5. The ratio of the number of shirts to
E.None of these the number of Saree and T-shirt
together of 1:4. The average cost of
2. The average age of A, B and C is 57 Shirt is Rs.200 and the average cost of
years. If D includes in the group, then Saree is Rs.220 and the average cost
the average age becomes 52.5 years. of T-shirt is Rs.300. If the total
If the average age of A, B and D is 55.5 revenue collected by shopkeeper is
years, then what is the average age of Rs.48000 and the total number of
C and D? shirts, sarees and T-shirt together is
A.41.25 years 200, then find the total cost of T-
B.43.50 years shirt?
C.45.51 years A.Rs.18000
D.47.83 years B.Rs.24000
E.49.20 years C.Rs.27000
D.Rs.15000
3.The average weight of all the E.Rs.9000
students in the class is 40 kg. If the
average weight of 40 students is 36 6. The average age of Anil, Bala and
kg and the average weight of the Nirmala is 35 years while the average
remaining students in the class is 48 of these three along with Dinesh and
kg. How many students are there in Kani is 41 years. If Kani is 20 years
the class? elder than Dinesh, then what is the
A.50 present age of Dinesh?
B.60 A.30 years
C.70 B.20 years
D.80 C.40 years
E.90 D.60 years
E.50 years
4. The average weight of the boys and
girls in the class is 24 kg and 50 kg 7.The total number of student takes
respectively. Total weight of the class admission in a coaching class on 4
is 1470 kg and the number of boys weeks is 364. Average number of
and girls in the class is x and (x – 15) student takes admission in first week
respectively. Find the value of x? is 12 and average number of students
A.25 take admission in third week is 2 less
Page 118 Follow us: Official Site, Telegram, Facebook, Instagram, Instamojo
than that of second week. Find total 10. The average weight of 6 persons
number of students takes admission is 60 kg. If the weight of lightest
in second week if average number of person in the group is 10 kg, then
student takes admission in fourth what is the average weight of the first
week is 10? five heaviest persons in the group?
A.98 A.60 kg
B.128 B.65 kg
C.84 C.70 kg
D.112 D.75 kg
E.None of these E.None of these

8. The average number of students in 11. The average weight of 26 students


class A, B, C and D is 52. Ratio of the in the class is 39 kg. If 9 new students
number of students in class A, B and C joined the class, then the average
is 5:4:3 and the number of students in weight of the class is increased by one
D is 6(2/3)% more than the number kg. Find the average weight of new
of students in A. Find the number of students of the class?
students in C? A.40.56 kg
A.24 B.42.89 kg
B.30 C.44.44 kg
C.36 D.46.92 kg
D.42 E.48.23 kg
E.45
12. The average weight of the class is
9. Average number of toys sold on 36 kg. If the teacher weight is added,
Tuesday, Thursday and Sunday is 320 then the average weight is increased
while average number of toys sold for by 2 kg and the total number of
the whole week except Friday and students in the class is 22 and the
Saturday is 282. If the number of toys ratio of the weight of teacher to HM is
sold on Monday is 80% of the number 41:45. Find the weight of HM?
of toys sold on Wednesday, then find A.90 kg
the number of toys sold on B.135 kg
Wednesday. C.45 kg
A.120 D.120 kg
B.134 E.None of these
C.140
D.250 13. S1 is a series of 6 consecutive odd
E.None of these numbers whose average is 26 and the
S2 series is a series of 6 consecutive
even numbers. If the fourth term of
Page 119 Follow us: Official Site, Telegram, Facebook, Instagram, Instamojo
S2 is equal to the half of the sum of A.76
the third and last term of S1, then B.48
what is the average of S2? C.55
A.23 D.62
B.25 E.None of these
C.27
D.29 17. The average age of 45 boys of a
E.21 class is 20 years, if the age of teacher
is included, the average age becomes
14. The average weight of 20 students 21 years, then find the age of teacher?
increases from 20kg to 25kg, when P, A.56 years
Q and R join them. The weight of P, Q B.66 years
and R are in the ratio 5: 3: 6. Find the C.65 years
weight difference between Q and R. D.55 years
A.35kg E.58 years
B.36.5kg
C.37kg 18. The ratio of the salary of P and Q
D.37.5kg is 9: 13 and each of them saves
E.None of these Rs.2000. What is the average of the
salary of P and Q, if the ratio of the
15. The average price of four cars is expenses of P and Q is 17: 29?
Rs.12.5 Lakhs. The average price of A.Rs.6600
two costliest cars and the least priced B.Rs.7200
car is 13 lakhs. Find the price of C.Rs.7680
second least priced car. D.Rs.8900
A.12 Lakhs E.None of these
B.11 Lakhs
C.15 Lakhs 19. 4 years ago, average age of 6
D.13 Lakhs employees in the company in is x
E.16 Lakhs years. A new employee joined the
company, now the average age of the
16. Average marks of Aman, Suman company is x years. If the present age
and Raman is 75. The marks of Aman of new employee is 22 years, then
is 25 less than Pawan and 10 more find the value of x?
than Suman. If Pawan scored 57 A.22
marks more B.24
than the average scores of Aman, C.36
Suman and Raman, then find the D.40
average of the scores of Suman and E.46
Raman?
Page 120 Follow us: Official Site, Telegram, Facebook, Instagram, Instamojo
20. The average weight of 26 students 4.: C
in the class is 39 kg. If 9 new students 24 * x + 50 * (x – 15) = 1470
joined the class, then the average 24x + 50x – 750 = 1470
weight of the class is increased by one x = 30
kg. Find the average weight of new
students of the class? 5.Answer Answer: A
A.40.56 kg Number of shirts = 1/5 * 200 = 40
B.42.89 kg Number of T-shirts and sarees = 200 –
C.44.44 kg 40 = 160
D.46.92 kg Total cost of Shirts = 40 * 200 = 8000
E.48.23 kg (160 – x) * 220 + x * 300 = 48000 – 8000
35200-220x+300x=40000
ANSWER 1-20 x = 60
1.Answer: B Required total = 60 * 300 = 18000
28 * 27 + x = 28 * 31
X = 868 - 756 6. Answer: C
x = 112 kg A + B + N = 35 * 3 = 105
A + B + N + D + K = 41 * 5 = 205
2.Answer: A D + K = 205 – 105 = 100 ---(1)
A + B + C = 57 * 3 = 171 K – D = 20 ---(2)
A + B + C + D = 52.5 * 4 = 210 From (1) and (2)
D = 210 – 171 = 39 2K = 120
A + B + D = 55.5 * 3 = 166.5 K = 60
A + B = 166.5 – 39 = 127.5 D = 60 – 20 = 40 years
C = 171 – 127.5 = 43.5
C + D = (39 + 43.5)/2 7. Answer: D
= 82.5/2 Let average number of students takes
= 41.25 admission in third
week = a
3. Answer: B Average number of students takes
Total number of students = x admission in second
Total weight of the class = 40 * x week = a + 2
Total weight of 40 students = 40 * 36 = According to question
1440 12 + a + a + 2 + 10 = 364/7
Total weight of remaining students = (x 2a = 52 – 24
– 40) * 48 a = 28/2 = 14
= 48x – 1920 Total number of students takes
40x = 1440 + 48x – 1920 admission in second
x = 60 week = 7 x (14 + 2) = 112

Page 121 Follow us: Official Site, Telegram, Facebook, Instagram, Instamojo
8. Answer: C (x + x + 2 + x + 4 + x + 6 + x + 8 + x +
A + B + C + D = 52 * 4 = 208 10)/6 = 26
D = 5x * (320/300) = 16x/3 6x = 126
5x + 4x + 3x + 16x/3 = 208 x = 21
Fourth term of S2 = 1/2 * (21 + 4 + 21 +
9. Answer: D 10)
Number of toys sold on Tuesday, = 28
Thursday and Sunday Average of S2 = (22 + 24 + 26 + 28 + 30
= 320 * 3 = 960 + 32)/6
Number of toys sold on Monday, = 27
Tuesday, Wednesday,
Thursday and Sunday = 282 * 5 = 1410 14. Answer: D
Number of toys sold on Monday and Average weight increases by 5
Wednesday = 1410 Total increase in weight = 3 * 20 + 23 * 5
– 960 = 450 = 175kg
Number of toys sold on Wednesday = x P: Q: R = 5: 3: 6
=> 80% of x + x = 450 => 5x + 3x + 6x = 175
=> x = 250 => x = 12.5
R – Q = 6x – 3x = 3x = 37.5kg
10. Answer: C
Total weight of the group = 60 * 6 = 360 15. Answer: B
Required average = (360 – 10)/5 = Let the cars be A, B, C, D in descending
350/5 = 70 kg order of price.
A being costliest car and D be the least
11. Answer: B priced car.
Initial weight of the class = 39 * 26 = Given average price of cars is 12.5 Lakhs
1014 A+B+C+D )/4 = 12.5
New weight of the class = (26 + 9) * 40 = A+B+C+D = 50 lakhs---(i)
1400 Average price of first two costliest cars
Required average = (1400 – 1014)/9 and the least
= 386/9 priced car = 13 lakhs.
= 42.89 kg (A+B+D)/3 = 13
A+B+D = 39 lakhs---(ii)
12. Answer: A (ii) in (i)
Weight of teacher = 38 * 23 - 36 * 22 = 39 + C = 50
874-792= 82 kg C = 11 Lakhs.
Weight of HM = 82 * 45/41 = 90 kg
16. Answer: E
13. Answer: C Let the score of Pawan is = x
S1 = x, x + 2, x + 4, x + 6, x + 8 and x + 10 So, the score of Aman is = x – 25
Page 122 Follow us: Official Site, Telegram, Facebook, Instagram, Instamojo
And the score of Suman is = x – 35
Then, according to the question, 18. Answer: A
= x = 75 + 57 Salary of P and Q be 9x and 13x.
= x = 132 (9x – 2000): (13x – 2000) = 17: 29
So, the score of Pawan is = 132 261x – 58000 = 221x - 34000
Score of Aman is = 132 – 25 = 107 => x = 600
Score of Suman is = 132 – 35 = 97 Required average = (9x + 13x)/2 =
Now, total marks of Aman, Suman and Rs.6600
Raman is = 3 ×
75 => 225 19. Answer: E
Score of Raman is = 225 – (107 + 97) = (x + 4) *6 + 22 = 7x
21 6x + 24 + 22 = 7x
So, the average score of Suman and x = 46
Raman is = (97 +
21)/2 20. Answer: B
= 118/2 => 59 Initial weight of the class = 39 * 26 =
1014
17. Answer: B New weight of the class = (26 + 9) * 40 =
Age of 45 boys = 45*20 1400
= 900 Required average = (1400 – 1014)/9=
when age of teacher included, = 46*21 = 386/9
966 = 42.89 kg
Hence teacher age = 966-900= 66

(Problems on Ages)
1). In a family, a couple has a E. None of these
daughter and son. The age of the
2). 5 years ago, the average age of
mother is three times that of his son
Chinnu and Nani is 23. At present, the
and the age of the daughter is 50% of
ratio between their ages is 15:13.
his father. The husband is 9 years
Find the Chinnu age after 10 years?
younger to her wife and the sister is
seven years older than his brother. A.50 years
Find the age of the daughter?
B.45 years
A.26 years
C.60 years
B.24 years
D.40 years
C.30 years
E.35 years
D.21 years

Page 123 Follow us: Official Site, Telegram, Facebook, Instagram, Instamojo
4). Ratio of the ages of Priya and is 42 years, then find the present age
Queen is 6:5 and the ratio of the ages of Rahul?
of Reshma and Shabhana is 5:6. If
A.50 years
Shabhana is 8 years elder than Queen
and Reshma is 3 years elder than B.45 years
Priya, then find the sum of the ages of
C.40 years
Priya, queen, Reshma and Shabhana?
D.55 years
A.66 years
E.60 years
B.44 years
C.55 years
7. The ratio of the ages of Anil and
D.33 years
Sunil, 6 years ago was 8:5
E. None of these respectively. After 2 years, the age of
Sunil will be 25% less than that of
Anil. What is the present age of Anil?
5. The present age of a father is 100% A.20 years
more than the present age of his son.
10 years ago, the age of father is B.21 years
200% more than as that of his son. C.22 years
What will be the ratio of the ages of
father and son after 15 years from D.23 years
now? E. None of these
A.9:5
B.11:7 8. The ratio of the present age of A
C.10:7 and B is 1:3 and after 16 years the
ratio of the ages of A to B becomes
D.8:3 3:5. What is the ratio of the ages of A
E.12:7 to B 4 years ago?
A.2:3

6. Present age of Pooja is 40% more B.4:5


than the present age of Sharmi and C.1:4
the present age of Ram is 25% more
than the age of Pooja 6 years ago. If D.2:5
the present age of Rahul is 22(2/9)% E. None of these
more than the present age of Ram
and the age of Sharmi after 12 years
Page 124 Follow us: Official Site, Telegram, Facebook, Instagram, Instamojo
9. Average ages of Anil, Mathavi and B.2:3:4
Merlin is 26 years. If the present age
C.3:4:6
of Mathavi is two times of the age of
Merlin 6 years ago and the present D.5:6:7
age of A is 12 years more than
E. None of these
Mathavi, then what is the present age
of Merlin?
A.12 years 12. Four years hence, Mala will be 5
times of her daughter’s age and four
B.18 years
years ago, the ratio of the age of Mala
C.15 years and her daughters is 6:1. What is the
present age of Mala’s daughter?
D.22 years
A.32
E.24 years
B.36
C.48
10. The sum of the present ages of
mother and daughter is 51 years. Six D.28
years ago, mother’s age is 12 times of
E.24
the age of daughter and the ratio of
the ages of mother to father is 14:15,
then find the difference between the
13. The average ages of Amala and
age of Daughter and father.
Vimala is 28 years. If the ratio of the
A.30 years ages of Amala 12 years hence and the
age of Vimala 8 years ago is 3:2, then
B.32 years
find the present age of Amala?
C.33 years
A.22
D.27 years
B.20
E.36 years
C.26
D.28
11. Yuvi’s present age is 33(1/3)% of
E.24
the sum of the present age of Som and
Ramya. If the present age of Som is
25% more than the present age of
14. If four years ago, the age of
Ramya, then what is the ratio of the
Punitha is half of that of Santhosh and
present age of Yuvi, Ramya and Som?
after 4 years, the age of Punitha is
A.1:2:3 40% less than that of Santhosh, then
Page 125 Follow us: Official Site, Telegram, Facebook, Instagram, Instamojo
what is the ratio of the present age of 17. Ratio of the ages of A and B is 4:5.
Santhosh and Punitha? Average age of A, B and C is 47 years.
After 12 years the age of C is 72 years.
A.10:7
What is the difference between the
B.9:5 ages of A and B?
C.11:8 A.9 years
D.12:5 B.11 years
E. None of these C.12 years
D.15 years
15. Ratio of the ages of A and B is 3:4 E. None of these
and the ratio of the ages of B and C is
8:9. If C is 6 years younger than D and
D’s age after 10 years is 34 years, 18. Present age of Arun is 10% more
then what is the age of A after 8 than the present age of Ashwin. Arun
years? is 4 years elder than Arjun. Anish will
be 3 years elder than Arjun. Find the
A.14
present age of Anish, if the average
B.19 present age of all four person
together is 31 years.
C.20
A.30
D.23
B.32
E. None of these
C.34
16. The age of Uday 8 years ago is
25% more than the age of Tarun at D.36
that time. Ratio of the ages of Tarun
E. None of these
to Renu is 6:7. 10 years hence the
difference between the age of Uday
and Tarun is 7 years, find the age of
19. Ratio of the ages of Sureka and
Renu after 6 years?
Francis 8 years ago is 7:6. Sureka’s
A.42 years age 6 years ago is 25% less than the
age of Francis after 8 years. If the
B.48 years
average age of Sureka, Francis and
C.54 years Dev is 32 years, find the present age
of Dev?
D.36 years
A.22 years
E.60 years

Page 126 Follow us: Official Site, Telegram, Facebook, Instagram, Instamojo
B.24 years So, age of daughter = 60/2 = 30 years
C.26 years
D.28 years 2. Answer: D
E.20 years 5 years ago, their average age is 23
therefore total age = 23*2 = 46
20. 2 years ago, Age of Naren was four present age of Chinnu and Nani =
times the age of Sam 5 years ago. If 46+5+5 = 56
the present age of Naren is 150%
Chinnu age = 15/28*56 = 30
more than the present age of Sam,
then what is the age of Sam? after 10 years = 30+10 = 40 years
A.6
B.8 3. Answer: E
C.10 Ratio of the ages of A, B and C = 35:40:48
D.12 35x + 40x + 48x + D = 40 * 4
E. None of these We can’t find the answer

SOLUTION 1-20 4. Answer: C


1. Answer: C Priya = 6x
Let the age of the father=x years. Queen = 5x
So, The age of mother is = (x + 9) years Reshma = 5y
The age of daughter is = x/2years Shabhana = 6y
The age of son is = (x/2 – 7) years 6y – 5x = 8 ------(1)
Now according to the question, 5y – 6x = 3 -----(2)
= 3 (x/2 – 7) = (x + 9) y=3
= (3x – 42)/2 = x + 9 x=2
= 3x – 42 = 2x + 18 Required total = (11 * 3) + (11 * 2)
= 3x – 2x = 42 + 18 = 55 years
= x = 60 years

Page 127 Follow us: Official Site, Telegram, Facebook, Instagram, Instamojo
5. Answer: B Present age of A = 8
Present age of son = x Present age of B = 3 * 8 = 24
Present age of father = 200/100 * x = 2x Required ratio = 4:20 = 1:5
(x – 10)/(2x – 10) = 100/300
2x – 10 = 3x – 30 9. Answer: B
x = 20 Anil + Mathavi + Merlin = 26 * 3 = 78
Present age of father = 2 * 20 = 40 Mathavi = 2 * (Merlin - 6)
Required ratio = (40 + 15):(20 + 15) Anil – Mathavi = 12
= 55:35 Anil = 12 + 2Merlin - 12
= 11:7 Merlin + 2Merlin – 12 + 2Merlin = 78
5 Merlin = 90
6. Answer: D Merlin = 18 years
Sharmi = 42 – 12 = 30 years
Present age of Pooja = 30 * 140/100 = 10. Answer: E
42
M + D = 51-----(1)
Ram = 125/100 * (42 – 6) = 45
M – 6 = 12 * (D – 6)
Rahul = 1100/900 * 45 = 55 years
M – 6 = 12D – 72
12D – M = 66-----(2)
7. Answer: C
13D = 117
6 years ago, Anil = 8x and Sunil = 5x
D=9
(5x + 8) = (100 – 25)/100 * (8x + 8)=> x
M = 51 – 9 = 42
=2
F = 42 * 15/14 = 45
Present age of Anil = 8x + 6 = 22 years
Difference = 45 – 9 = 36 years

8. Answer: E
11. Answer: E
x + 16/3x + 16 = 3/5
Som = 125/100 * Ramya
9x + 48 = 5x + 80
Som/Ramya = 5/4
x=8
Page 128 Follow us: Official Site, Telegram, Facebook, Instagram, Instamojo
(5K + 4K) * 100/300 = Yuvi (S + 4) * 60/100 = (P + 4)
Yuvi = 3k 3S + 12 = 5P + 20
Required ratio = 3K:4K:5K 3S – 5P = 8 -----(2)
= 3:4:5 From (1) and (2)
S = 36
12. Answer: B P = 20
M + 4 = 5(D + 4) Required ratio = 36:20
M + 4 = 5D + 20 = 9:5
M – 5D = 16 -----(1)
(M – 4)/(D - 4) = 6/1 15. Answer: C
6D – 24 = M – 4 D’s present age = 34 – 10 = 24
6D – M = 20 -------(2) C = 24 – 6 = 18
D = 36 B = 18 * 8/9 = 16
A = 3/4 * 16 = 12
13. Answer: E A’s age after 8 years = 12 + 8 = 20
Amala + Vimala = 28 * 2 = 56------(1)
(Amala + 12)/(Vimala – 8) = 3/2 16. Answer: B
3Vimala – 24 = 2Amala + 24 Tarun’s age 8 years ago = 4x
3Vimala – 2Amala = 48-----(2) Uday’s age 8 years ago = 4x * 125/100 =
5x
From (1) and (2)
(5x + 8 + 10) – (4x + 8 + 10) = 7
Vimala = 32
x=7
Amala = 56 – 32 = 24
Present age of Tarun = 4 * 7 + 8 = 36
Renu age after 6 years = (36 * 7)/6 + 6 =
14. Answer: B
48 years
P – 4 = 1/2 * (S – 4)
2P – 8 = S – 4
17. Answer: A
2P – S = 4 ----(1)
A + B + C = 141
Page 129 Follow us: Official Site, Telegram, Facebook, Instagram, Instamojo
C’s present age = 72 – 12 = 60 7x + 8 – 6 = 75/100 * (6x + 8 + 8)
A + B = 141 – 60 = 81 28x + 8 = 18x + 48
Required difference = 81 * 1/9 = 9 years x=4
Sureka = 7 * 4 + 8 = 36
18. Answer: B Francis = 6 * 4 + 8 = 32
Age of Ashwin = x Sureka + Francis + Dev = 32 * 3 = 96
Arun = 1.1x Dev = 96 – 36 – 32 = 28
Arjun = 1.1x – 4
Anish = 1.1x – 4 + 3 = 1.1x – 1 20. Answer: D
x + 1.1x + 1.1x – 4 + 1.1x – 1 = 31 * 4 Present age of Sam = x
=> x = 30 => Age of Sam 6 years ago = x – 5
Anish = 1.1x – 1 = 32 Age of Naren 2 years ago = 4x – 20
Present age of Naren = 4x – 20 + 2 = 4x –
18
19. Answer: D
4x – 18 = 250/100 * x
S = 7x + 8
=> x = 12 = Age of Sam
F = 6x + 8

Profit & Loss


Formulas & Shortcuts of Profit & Loss

Formulas

 Cost price (C.P.): This is the price at which an article is purchased.


 Selling price (S.P.): This is the price at which an article is sold.
 Profit or Gain: If the selling price is more than the cost price, the difference between them is the
profit incurred.

Formula: Profit or Gain = S.P. – C.P.

 Loss: If the selling price is less than the cost price, the difference between them is the loss
incurred.

Formula: Loss = Cost price (C.P.) – Selling Price (S.P.)


Page 130 Follow us: Official Site, Telegram, Facebook, Instagram, Instamojo
 Profit or Loss is always calculated on the cost price.
 Marked price: This is the price marked as the selling price on an article, also known as the
listed price.
 Discount or Rebate: This is the reduction in price offered on the marked or listed price.

Below is the list of some basic formulas used in solving questions on profit and loss:

 Gain % = (Gain / CP) * 100


 Loss % = (Loss / CP) * 100
 SP = [(100 + Gain%) / 100] * CP
 SP = [(100 – Loss %) / 100]*CP

The above two formulas can be stated as,

If an article is sold at a gain of 10%, then SP = 110% of CP.

If an article is sold at a loss of 10%, then SP = 90% of CP.

 CP = [100 / (100 + Gain%)] * SP


 CP = [100 / (100 – Loss%)] * SP

Practice Questions
1. A shopkeeper gives two successive
discounts of 10% and 20% on the 3. The shopkeeper sold the watch at the
marked price of shirt. If he received the profit of 20% and the cost price of watch
profit is 1/10 of the total discount, then is Rs.3600. He earns x% profit on
find the approximate profit percentage? pendrive costing Rs.3000. If the overall
A.2% profit on selling both watch and
B.4% pendrive is 25%, then find the value of
C.6% x?
D.8% A.24%
E.10% B.27%
C.29%
2. The marked price of the bulb is 60% D.31%
more than its cost price. A shopkeeper E.33%
offers a discount of x% on marked price
of the bulb while he gets the profit of 4. Sum of the selling price of mobile and
36%. Find the value of x? watch is Rs.9000. Ratio of the discount
A.10 offered by shopkeeper on mobile and
B.15 watch is 1:3. If the sum of the marked
C.18 price of mobile and watch is Rs.15000,
D.12 then find the discount offered by
E.20 shopkeeper on watch?
Page 131 Follow us: Official Site, Telegram, Facebook, Instagram, Instamojo
A.Rs.4500 8. Ratio of the marked to cost price of
B.Rs.4200 the Plane is 6:5 and the ratio of the
C.Rs.3600 marked to selling price of the plane is
D.Rs.4800 12:11. What is the profit percentage of
E.Rs.3300 the plane?
A.8%
5 . Ragu, Ragava and Rishi started the B.10%
business and invested their money in C.6%
the ratio of 1/12:1/15:1/9 and for time D.15%
period in the ratio of 1/8:1/6:1/4. If E.12%
Rishi’s share out of profit at the end of
year is Rs.4800, then what is the total 9. The shopkeeper offers a discount of
profit of the business? 20% on marked price of laptop and the
A.Rs.8220 ratio of the marked price of the laptop to
B.Rs.8320 mobile is 3:1. If the shopkeeper offers a
C.Rs.8420 discount of 10% on marked price of
D.Rs.8520 mobile while he gets the profit of 20%
E.Rs.8820 and the selling price of the laptop is
Rs.14400, then find the cost price of the
6. What will be the percentage profit mobile?
after selling the Perfume at Rs.x if there A.Rs.4000
is a loss of 20% when the perfume is B.Rs.4500
sold at two-third of x? C.Rs.5000
A.20% D.Rs.6000
B.25% E.Rs.3000
C.30%
D.40% 10. A seller makes 10% profit when he
E.35% gives 12% discount on marked price of
his pencil box. After sometimes, he
7. Ram and Janu sold their TV at begins to provide 10% discount on its
Rs.14000 each, but Ram faced a loss of marked price. Now what is his
10%, while Janu gained 30%.What is the percentage of profit?
ratio of the cost price of the TV sold by A.12%
Ram to that of Janu? B.12.5%
A.12: 7 C.13%
B.13: 9 D.13.5%
C.14: 3 E.None of these
D.19: 7
E.None of these 11. Dress A is sold at 10% discount and
earned a profit of Rs.525. Dress B is sold
Page 132 Follow us: Official Site, Telegram, Facebook, Instagram, Instamojo
at 20% profit for Rs.840. Find the incurred on selling the same article for
marked price of Dress A, if Cost price of Rs.1700, then find the selling price of the
dress A is 5% more than the cost price of article earn a profit of 20%?
Dress B. A.Rs.2540
A.Rs.1400 B.Rs.2580
B.Rs.1700 C.Rs.2590
C.Rs.2100 D.Rs.2500
D.Rs.2900 E.Rs.2520
E.None of these
15. A toy was sold by the seller after
12. A shopkeeper earned a profit equal giving a discount of 24% for Rs.114.
to cost price of 3 items while selling 18 What is the cost price of the toy if the
items at actual profit. If each item is ratio of marked price to cost price is 5:
marked 25% above its actual cost and 7?
Rs.40 discount is given while selling A.Rs.200
then what is the difference between Cost B.Rs.205
pirce and profit. C.Rs.208
A.Rs.720 D.Rs.210
B.Rs.240 E.None of these
C.Rs.400
D.Rs.360 16. The marked price of cycle and Watch
E.Rs.480 is Rs.2800 and Rs.3000 respectively. If
the shopkeeper allows the discount on
13. The selling price of the Laptop is marked price of the cycle is Rs.800
40% more than the selling price of the which is 80% of the discount of watch,
mobile and the selling price of the watch then what is the difference between the
is 40% less than the selling price of the selling price of watch and cycle?
mobile. If the selling price of the laptop A.Rs.500
is Rs.4000 more than the selling price of B.Rs.550
the mobile, then what is the average of C.Rs.600
the selling price of the watch and D.Rs.450
mobile? E.None of these
A.Rs.2000
B.Rs.4000 17. The marked price of the Doll is 25%
C.Rs.6000 more than its cost price. The shopkeeper
D.Rs.5000 offers the discount of Rs.360 on marked
E.None of these price of doll. If the doll sold for Rs.200
more while he gets the profit of 15%,
14. If the profit earned on selling an then find the marked price of the doll?
article for Rs.2400 is 75% of the loss A.Rs.2000
Page 133 Follow us: Official Site, Telegram, Facebook, Instagram, Instamojo
B.Rs.4000 D.Rs.2500
C.Rs.3500 E.Rs.2520
D.Rs.2500
E.Rs.3000 21. In the Diwali sale, Dinesh expects a
profit of 35% on the cost price of his
18. The marked price of the table is 25% fruits. What will be his profit, if he sells
more than the cost price of the table and the fruits worth Rs.4050 in a week?
the shopkeeper offers two successive A.Rs.1050
discounts of 20% and 10% B.Rs.2050
respectively on marked price of the C.Rs.3000
table. If the shopkeeper gets the loss of D.Rs.2800
Rs.1200, then what is the selling price of E.None of these
the table?
A.Rs.10800 22. If a toy is sold at 10% discount, then
B.Rs.9600 the amount gained is Rs.80, while if the
C.Rs.11400 discount given was 5%, then the amount
D.Rs.13500 gained is 100. What is the marked price
E.None of these of the toy?
A.Rs.200
19. The marked price of a laptop is B.Rs.400
Rs.16000 and the shopkeeper bought a C.Rs.500
laptop at the rate of Rs.12500. If the D.Rs.700
shopkeeper offers a discount of 10% on E.None of these
marked price of the laptop, then what is
the percentage of profit earned by 23. Renuka purchased a laptop for
shopkeeper? Rs.10200 from flipkart. If the flipkart
A.14.5% sold the laptop at the profit of 20% and
B.15.2% the flipkart also offers 20% discount on
C.16.8% marked price of the laptop, then what is
D.17.4% the difference between the cost and
E.18.9% marked price of the laptop?
A.Rs.4250
20. If the profit earned on selling an B.Rs.4450
article for Rs.2400 is 75% of the loss C.Rs.4860
incurred on selling the same article for D.Rs.4550
Rs.1700, then find the selling price of the E.None of these
article earn a profit of 20%?
A.Rs.2540 24. In the Diwali sale, Dinesh expects a
B.Rs.2580 profit of 35% on the cost price of his
C.Rs.2590
Page 134 Follow us: Official Site, Telegram, Facebook, Instagram, Instamojo
fruits. What will be his profit, if he sells x = 930/3000 * 100
the fruits worth Rs.4050 in a week? = 31%
A.Rs.1050
B.Rs.2050 4. Answer: A
C.Rs.3000 Discount of mobile and watch = 15000 –
D.Rs.2800 9000 = 6000
E.None of these 3x + x = 6000
x = 1500
25. A toy was sold by the seller after Discount of watch = 1500 * 3 = 4500
giving a discount of 24% for Rs.114.
What is the cost price of the toy if the 5. Answer: D
ratio of marked price to cost price is 5: Ratio of the investment of Ragu, Ragava
7? and Rishi =
A.Rs.200 1/12:1/15:1/9
B.Rs.205 = 15:12:20
C.Rs.208 Ratio of the time period of Ragu, Ragava
D.Rs.210 and Rishi =
E.None of these 1/8:1/6:1/4
= 3:4:6
ANSWER 1-25: Ratio of the profit share of Ragu, Ragava,
1. Answer: B Rishi =
MP of shirt = x 15*3:12*4:20*6
SP of shirt = x * 90/100 * 80/100 = 0.72x = 45:48:120
Discount = x – 0.72x = 0.28x = 15:16:40
Profit = 0.28x/10 = 0.028x Total profit of the business = 71/40 *
CP of shirt = 0.72x – 0.028x = 0.692x 4800
Required percentage = 0.028x/0.692x * = Rs.8520
100 = 4% x * 120/100 = 930
x = 775
2. Answer: B
CP of bulb = 100a 6. Answer: A
MP of bulb = 100a * 160/100 = 160a Original SP = x
160a * (100 – x)/100 = 100a * 136/100 New SP = 2/3 * x
x = 15 20/100 = (CP – 2/3 * x)/CP
CP/5 = CP – 2/3 * x
3. Answer: D CP = 5CP – 10x/3
Overall profit = 25/100 * (3600 + 3000) 3CP = 15CP – 10x
= 1650 10x= 12CP
Profit on Watch = 3600 * 20/100 = 720 x/CP = 6/5
Profit on Pendrive = 1650 – 720 = 930
Page 135 Follow us: Official Site, Telegram, Facebook, Instagram, Instamojo
Profit percentage = 6k – 5k/5k * 100= Profit earned while selling 18 items =
20% cost price of 3 items
18(SP – CP ) = 3CP
7. Answer: B 18SP – 18CP = 3CP
CP of TV of Ram = x 18SP = 21CP
CP of TV of Janu = y Profit percentage =16.67%
x * 90/100 = y * 130/100 Let cost price of a product =6x
=>x: y = 13: 9 Marked price =125% of 6x =7.5x
Selling price = 116.67% of 6x =7x
8. Answer: B Discount offered =Rs.40
MP = 6x 7.5x – 7x =40
CP = 5x 0.5x =40
SP = 11/12 * 6x = 5.5x X=80
Required percentage = (5.5x – 5x)/5x * Difference between CP and profit = 6x-x
100 = 10% =5x =Rs.400

9. Answer: B 13. Answer: E


MP of laptop = 100/80 * 14400 = 18000 Let SP of mobile = 500x
MP of mobile = 18000 * 1/3 = 6000 SP of laptop = 500x * 140/100 = 700x
SP of mobile = 6000 * 90/100 = 5400 SP of watch = 500x * 60/100 = 300x
CP of mobile= 5400 * 100/120 = 4500 700x – 500x = 4000
x = 20
10. Answer: B SP of watch = 300 * 20 = Rs.6000
Let marked price = Rs.100 SP of mobile = 500 * 20 = Rs.10000
SP = 100 * 88/100 = Rs.88 Average = (6000 + 10000)/2 = 16000/2
CP = 88/110 * 100 = Rs.80 = 8000
New selling price = 100 * 90/100 =
Rs.90 14. Answer: E
New profit = 90 – 80 = Rs.10 2400 – CP = (CP – 1700) * 75/100
Profit % = 10/80 * 100 = 12.5% 9600 – 4CP = 3CP – 5100
7CP = 14700
11. Answer: A CP = 2100
MP of Dress A = x Required SP = 2100 * 120/100 =
CP of Dress B = 840/1.2 = Rs.700 Rs.2520
CP of Dress A = 1.05 * 700 = Rs.735
=> 735 + 525 = (100 – 10)% of x 15. Answer: D
=> x = Rs.1400 Let x = Marked price of the toy
(100 – 24)% of x = 114
12. Answer: C => x = 150

Page 136 Follow us: Official Site, Telegram, Facebook, Instagram, Instamojo
Required Cost price = 7/5 * 150 =
Rs.210 20. Answer: E
2400 – CP = (CP – 1700) * 75/100
16. Answer: E 9600 – 4CP = 3CP – 5100
MP of cycle = Rs.2800 7CP = 14700
MP of Watch = Rs.3000 CP = 2100
SP of Cycle = 2800 – 800 = Rs.2000 Required SP = 2100 * 120/100 =
Discount of watch = 800 * 100/80 = Rs.2520
Rs.1000
SP of watch = 3000 – 1000 = Rs.2000 21. Answer: A
Difference = 2000 – 2000 = 0 CP = (100/135) * 4050 = Rs.3000
Profit = 4050 – 3000 = Rs.1050
17. Answer: A
CP of doll = 4x 22. Answer: B
MP of doll = 4x * 125/100 = 5x Marked price of the toy = x
Initial SP of doll = 5x – 360 (100 – 10)/100 * x – 80 = (100 – 5)/100
New sp of doll = 5x – 360 + 200 = 5x – * x – 100
160 => x = Rs.400
(5x – 160) * 100/115 = 4x
500x - 16000 = 460x 23. Answer: A
x = 400 SP of laptop =Rs. 10200
MP of Doll = 400 * 5 = 2000 CP = 100/120 * 10200 = Rs. 8500
MP * 80/100 = Rs. 10200
18. Answer: A MP = Rs. 12750
CP of table = 4x Difference = 12750 – 8500 = Rs. 4250
MP of table = 4x * 125/100 = 5x
5x * 80/100 * 90/100 = 4x – 1200 24. Answer: A
1200 = 0.4x CP = (100/135) * 4050 = Rs.3000
x = 3000 Profit = 4050 – 3000 = Rs.1050
CP of table = 4 * 3000 = 12000
SP of table = 12000 – 1200 = 10800 25. Answer: D
Let x = Marked price of the toy
19. Answer: B (100 – 24)% of x = 114
CP = Rs.12500 => x = 150
MP = 16000 Required Cost price = 7/5 * 150 =
SP = 16000 * 90/100 = 14400 Rs.210
Profit = (14400 – 12500)/12500 * 100 =
15.2%

Page 137 Follow us: Official Site, Telegram, Facebook, Instagram, Instamojo
Simple Interest & Compound Interest
Formulas & Shortcuts of SI & CI
Interest Formulas for SI and CI

The Interest formulas are given as,

Formulas for Interests (Simple and Compound)


SI Formula S.I. = Principal × Rate × Time
CI Formula C.I. = Principal (1 + Rate)Time – Principal

Formula for Continuous Compound Interest


A = P × ert

Where,

 A = Amount of money after a certain amount of time


 P = Principle or the amount of money you start with
 e = Napier’s number, which is approximately 2.7183
 r = Interest rate and is always represented as a decimal
 t = Amount of time in years

Daily Compound Interest Formula

The interest calculated on the primary principal and also on the accumulated interest of previous
periods of a deposit or loan is called Compound Interest. In much simpler terms, Compound interest is
the “interest on interest”. This interest usually makes a deposit or loan grow at a faster rate when
compared with simple interest.

The amount of interest computed on an account such as a savings account or a checking account on a
monthly basis or daily basis is known as the compound interest. A Certain amount is added to the
principal amount, the money added on this basis is known as the interest amount.

Daily Compound Interest Formula


= Principal (1+Rate/365) 365∗Time – Principal

Monthly Compound Interest Formula

Compound interest is an interest of interest to the principal sum of a loan or deposit. The concept of
compound interest is the interest adding back to the principal sum so that interest is earned during
the next compounding period.

Monthly Compound Interest = Principal (1+Rate/12) 12∗Time – Principal

Page 138 Follow us: Official Site, Telegram, Facebook, Instagram, Instamojo
Practice Questions

(Simple Interest)
a. 1500
1) Simple interest for the sum of b. 1700
Rs.1500 is Rs.50 in 4 years and c. 1575
Rs.80 in 8 years. Find the rate of d. 1650
SI? e. None of these
5). Sachin would have paid Rs. 5280
a. 0.5% at the end of 4 years, for a sum of
b. 1% money borrowed, at rate of 8% p.a.
c. 1.5% S.I. If he wants to repay his loan a
d. 2% year before its due, then what is the
e. None of these amount paid by him?
2) A sum of money amounts to Rs
19600 after 10 years and Rs 24010 a. Rs. 5060
after 16 years at the same rate of b. Rs. 4960
simple interest. The rate of interest c. Rs. 4760
per annum is: d. Rs. 4670
e. None of these
a. 4 % 6). Veena obtained an amount of Rs.
b. 3 % 8376 as simple interest on a certain
c. 5 % amount at 8 p.c.p.a. after 6 years.
d. 6 % What is the amount invested by
e. None of these Veena?
3) A sum of Rs. 2250 was lent partly
at 7 % and 10% p.a. simple interest. a. Rs. 17180
The total interest received after 3 b. Rs. 18110
years was Rs.540. The ratio of the c. Rs. 16660
money lent at 7% to that lent at 10% d. Rs. 17450
is? e. None of these
7) A sum of Rs. 6200 is divided into 2
a. 3:2 parts, one at 10% and another one at
b. 2:1 6%. If the total interest received is
c. 1:2 Rs.424. Find the money lent at 6 %
d. 2:3 interest rate.
e. None of these
4) A sum was put at S.I at a certain a. Rs. 3000
rate for 8 years. Had it been put at 4% b. Rs. 4500
higher rate, it would have fetched Rs. c. Rs. 3400
480 more. What was the Sum? d. Rs. 4900
Page 139 Follow us: Official Site, Telegram, Facebook, Instagram, Instamojo
e. None of these a. 600
8) Rs.1200 becomes Rs.1434 in 3 yrs b. 450
at a certain rate of S.I. If the rate of c. 500
interest is increased by 4%, then the d. 300
amount will be? e. None of these

a. Rs1245 10) At simple interest, a sum becomes


b. Rs.1432 three times in 20 years. Find the time
c. Rs.1578 in which the sum will be double at the
d. Rs.1365 same rate of interest?
e. None of these
9) The S.I on a sum of money will be a. 8 years
Rs. 300 after 5 yrs. If the principal is b. 10 years
tripled after 2 ½ yrs, what will be the c. 12 years
total interest at the end of the d. 14 years
5th year? e. None of these

SOLUTION (1 to 10)

1). Answer: a R=6%


S.I = (PRT / 100)
According to the question, = [(1500 × R 3). Answer: b
× 8) / 100] – [(1500 × R × 4) / 100] = 80 let the sum lent at 7% be Rs.x and that
– 50 (12000R – 6000R) / 100 = 30 lent at 10% be Rs.(2250-x).
6000R / 100 = 30 Then, Interest on x at 7% for 3 years +
R = 30/60 = ½ = 0.5% interest on (2250-x) at 10% for 3 years
= 450
2). Answer: d x*7*3/100 + (2250-x)*10*3/100 =540
We can get SI of 6 years = 24010 – 21x/100 + (2250-x)30 /100 =540
19600 = 4410 21x/100 + 67500-30x/100 =540
SI for 10 years = (4410/6)*10 21x+67500-30x = 54000
= 7350 [so that we can get principal 67500-9x =54000
amount after deducting SI] 9x =13500
Principal = 19600 – 7350 X=1500
= 12250 Required ratio = x : (2250-x)
SI = PTR/100 =1500:750
R = SI *100 /PT =2:1
R= 7350*100 / 12250*10
R = 735000/ 122500 4). Answer: a

Page 140 Follow us: Official Site, Telegram, Facebook, Instagram, Instamojo
At 4% more rate, the increase in S.I for 8
years = Rs.480 8). Answer: c
So, at 4% more rate, the increase in SI S.I = Rs(1434 – 1200)
for 1 year = 480/8 = Rs.60/- =Rs. 234
i.e. Rs.60 is 4% of the invested sum R = SI *100 / P*N
So, 1% of the invested sum = 60/4 R = 234 *100/1200*3
Therefore, the invested sum = 60 × = 13/2 %
100/4 New rate = 13/2 + 4 = 21/2 %
=Rs. 1500 New S.I = Rs. (1200 * 3 * 21 /200)
(or) =Rs. 378
(P*8*(r+4))/100 – (P*8*r)/100 = 480 New amount = Rs. (1200 + 378)
(8P/100)(r+4-r) =480 =Rs.1578
P = 480*100/32 = Rs. 1500
9). Answer: a
5). Answer: b Let the sum be Rs.x
P + (P × (8 / 100) × 4) = 5280 Now S.I = Rs.300
(33 / 25)P = 5280 T = 5years
P = Rs. 4000 R = 100*SI / P*T
Amount payable after 3 years = 100*300 / x*5
= 4000 + [4000 × (8 / 100) × 3] = Rs. = (6000/x) %
4960 S.I for first 2 ½ years = Rs. (x*5*6000 /
2*100*x)
6). Answer: d =Rs.150
S.I = (P × r × t) / 100 S.I for last 2 ½ years = Rs. (3x*5*6000 /
8736 = (P × 6 × 8) / 100 = (8736 × 100) 2*x*100)
/ (6 × 8) =Rs.450
= Rs. 17450 Therefore total interest = Rs. (450+150)
= Rs. 600
7). Answer: d
Let the sum lent at 32% be x and 10). Answer: b
The sum lent at be (6200 – x) Let Sum = P, T= 20 yrs;
(x*1*10/100) + ((6200-x)*1*6/100) = ∴ S.I = 3P – P = 2P
424 Formula for S.I = (PRT) / 100
10x/100 + 37200 – 6x/100 = 424 2P = (P × R × 20) / 100
4x + 37,200 = 42400 R = 10%
X = 5200/4 = 1300 To find Time (T),
Money lent at 10% = Rs. 1300 ∴ S.I = 2P – P = P
Money lent at 6% = Rs. (6200 – 1300) = P = (P × 10 × T) / 100
Rs. 4900 Hence, T = 10 yrs.

Page 141 Follow us: Official Site, Telegram, Facebook, Instagram, Instamojo
(Compound Interest) amount at the end of the period of
1) The compound interest on a investment?
certain sum at 20 % per annum for 3
years is Rs. 1456. The simple interest a. Rs. 28920
on the same sum for double the time b. Rs. 32140
at half the rate percent per annum is: c. Rs. 38180
d. Rs. 41160
a. Rs. 1500 e. None of these
b. Rs. 1200 5) The difference between simple and
c. Rs. 1300 compound interests together
d. Rs. 1400 annually on a certain sum for 2 year
e. None of these at 5% per annum is Rs 4. Find the
2) What total will be amount to Rs. sum?
51516 at compound interest in 2
years, the rate of interest for 1st& 2nd a. Rs. 2400
year being 6% and 8% respectively? b. Rs. 1600
c. Rs. 1250
a. Rs. 35000 d. Rs. 1800
b. Rs. 40000 e. None of these
c. Rs. 38000 6) If Rs.18000 amounts to 22579.20
d. Rs. 45000 in 2 yrs compounded annually, then
e. None of these find the rate of interest per annum?
3) Kumar wants to take Rs. 45000 at
rate of interest 4% p.a. at S.I and lend a. 8 %
the same amount at C.I at same rate of b. 10 %
interest for two years. What would be c. 14 %
his profit in the above transaction? d. 12 %
e. None of these
a. Rs. 72 7) A certain national bank offers 20
b. Rs. 68 % interest rate compounded
c. Rs. 54 annually. Selvi deposits Rs. 35000
d. Rs. 83 every year in his account. If she does
e. None of these not withdraw any amount, then how
4) A sum of money for the first two much balances will his account show
years at a rate of interest is 7% p.a., after four year?
for the next two years it is 8% p.a.
and 9% p.a. for the period exceeding a. Rs. 225456
four year; all at simple interest. If a b. Rs. 198734
person earns an interest of Rs. 15180 c. Rs. 267348
by the end of the 8 years, what is the d. Rs. 174562
e. None of these
Page 142 Follow us: Official Site, Telegram, Facebook, Instagram, Instamojo
8) Find the CI on 6 months on c. 2500
Rs.12350 at 12% per annum d. 2800
compounded quarterly. e. None of these

a. 756.114 10) Aswin invested Rs.4000 in a fixed


b. 752.115 deposit scheme for 2 years at CI rate
c. 751.674 10% per annum. How much amount
d. 753.118 will Aswin get on maturity of the fixed
e. None of these deposit?
9) The sum that amounts to Rs. 2704
in 2 yrs at 4% per annum CI a. 4730
compounded annually is, b. 4690
c. 4840
a. 2200 d. 4880
b. 3200 e. None of these

SOLUTION (1 to 10) Amount of money Kumar taken at S.I at


1) Answer: b 4% p.a. for two years = Rs.45000
Let the sum be Rs. P. He lend the same amount for C.I at 4%
[P(1+R/100)3-P]=1456 [P(1+20/100)3- p.a. for two years
P]=1456 [P(12/10)3-P]=1456 Kumar’s income = C.I – S.I
P[(12/10)3-1]=1456 = p [1 + r/ 100]n – p – [pnr/100]
P[(1728/1000)-1]=1456 = p {[1 + r/ 100]2 – 1} – [pnr/100
P[728/1000]=1456 = 45000{[1+ (4/100)]2-1}-
P=1456000/728 =2000 [45000*2*4/100]
SI=PNR/100 = 45000{[104/100]2-1}- 3600
=2000*6*10/100 = 45000{[26/25)2 – 1} – 3600
= 1200 = 45000[676/625 – 1} – 3600
= 3672 – 3600
2) Answer:d = Rs.72
Let Rs. P be the required sum.
51516 = P (1 + 6/100) (1 + 8/100) 4) Answer:c
51516 =P (106/100) (108/100) Let the principal be x
51516*(100/106)*(100/108) = P According to the question,
P= 45000 X*7*2/100 +X*8*2/100 +X*9*4/100 =
Hence the required amount is Rs. 45000 15180
14X/100 + 16X/100+36X/100 = 15180
3) Answer:a 66X/100 = 15180
66X = 1518000

Page 143 Follow us: Official Site, Telegram, Facebook, Instagram, Instamojo
X =23000
Total amount end of the period = 15180 8) Answer:b
+ 23000 Given P = Rs.12350; n=6 months = 2
= Rs. 38180 quarters
R=3% per quarter
5) Answer:b Amount = Rs [p(1+R/100)n]
The difference between S.I and C.I =Rs. [12350(1+3/100)2]
together on a certain sum for 2 years is, =Rs. [12350(103/100)2]
Diff = Sum*(r/100)2 =Rs. [12350*103/100*103/100]
4 = Sum*(5/100)2 =Rs. (12350*1.03*1.03)
4*25/(100*100) = Sum =Rs.13102.115
Sum = Rs. 1600 Compound interest = amount – principal
=Rs. (13102.115 – 12350)
6) Answer:d =Rs. 752.115
Given P = Rs.18000 (or)
Amount = 22579.20 12350*(3/100) = Rs. 370.5
n = 2years 12720.5*(3/100) = Rs. 381.615
Let the rate be R% per annum. Then C.I = 370.5 + 381.615 = Rs. 752.115
[18000(1+R/100)2] = 22579.20
(1+R/100)2 = 18000 / 22579.20 9) Answer:c
(1+R/100)2 = (28/25)2 Amount = P(1+r/100)n
1+R/100 = 28/25 2704 = P(1+4/100)2
(100+R)/100 = 28/25 P = Rs [2704/ (1+4/100)2]
100+R = 2800/25 P =Rs. (2704 / (104/100)2]
100+R = 112 P =Rs. (2704 *(100/104) * (100/104)]
R = 12 % P =Rs. (27040000/10816)
P =Rs. 2500
7) Answer:a
For the 1st year interest will be, 10) Answer:c
= > 35000*(20/100) = Rs. 7000 Amount = P (1+R/100)n
35000+7000+35000 = 77000 =4000(1+10/100)2
= > 77000*(20/100) = 15400 =4000(110/100)2
77000+ 15400+ 35000 = 127400 =4000*110/100*110/100
= > 127400*(20/100) = 25480 =40*11*11
127400 + 25480 + 35000 = 187880 =Rs.4840
= > 187880*(20/100) = 37576 Hence the required answer is Rs. 4840
187880 + 37576 = Rs. 225456

(Simple Interest & Compound Interest Mix)

Page 144 Follow us: Official Site, Telegram, Facebook, Instagram, Instamojo
1. An amount Rs. P was lent at r% per when invested in a compound interest
annum simple interest. After 3 years, scheme. What is the total amount will
total amount becomes Rs.4600 and after get if the sum invested in the same
5 years, total amount becomes Rs.5000. scheme for 2 years?
What is P? A.Rs.3630
A.Rs.2000 B.Rs.3640
B.Rs.3000 C.Rs.3650
C.Rs.4000 D.Rs.3880
D.Rs.5000 E.None of these
E. None of these
5. Shon invested totally Rs.6000 in two
2. Jenifer invested Rs. x in simple schemes A and B. Scheme A offers
interest scheme at the rate of 20% per simple interest at the rate of 15% per
annum for 6 years and after 6 years, she annum while scheme B offers 10% per
received the interest amount is Rs.5880. annum compound interest, compounded
Find the amount received when Rs. x annually. If the amount received by Shon
invested at 10% per annum after 2 years is Rs.7440, find the sum
compounded interest for 3 years? invested in scheme A?
A.Rs.6521.9 A.Rs.1000
B.Rs.7114.8 B.Rs.2000
C.Rs.8245.13 C.Rs.3000
D.Rs.6935.8 D.Rs.4000
E.Rs.7682.9 E.Rs.3200

3. Meena deposited Rs.7800 in 6. Babu deposited Rs.(x + 400) at 10%


compound interest scheme at the rate of per annum simple interest and earned
20% per annum for 2 years. Vinitha Rs.480 as interest after 3 years. Find the
deposited Rs.4800 in simple interest interest earned by him if he deposited
scheme at the rate of 15% per annum Rs.3x at 10% per annum for 3 years at
for 5 years. What is the difference compound interest.
between the interest earned by Meena A.Rs.1191.6
and Vinitha? B.Rs.2401
A.Rs.155 C.Rs.3412.4
B.Rs.158 D.Rs.4101
C.Rs.164 E.None of these
D.Rs.148
E.Rs.168 7. Renu invests Rs.x in simple interest at
the rate of 15% per annum for 3 years.
4. A sum of money becomes Rs.3300 Rama invests Rs.y in simple interest
after one year and Rs.3993 after 3 years, scheme at the rate of 15% per annum
Page 145 Follow us: Official Site, Telegram, Facebook, Instagram, Instamojo
for 3 years. If the sum of the value of x A.24 years
and y is Rs.11000 and the difference B.30 years
between the interest received by Renu C.28 years
and Rama is Rs.1800, then find the value D.32 years
of y? E.None of these
A.Rs.7500
B.Rs.6000 11. Deepika and Dinesh had sums in the
C.Rs.5100 ratio of 13:15. Deepika invested her sum
D.Rs.7000 at 15% per annum simple interest for 5
E.Rs.6800 years and Dinesh invested his sum at
10% per annum simple interest for 4
8. Meena borrows Rs.x from Tina at 20% years. If the sum of the amounts
per annum at compound interest for 3 received by Deepika and Dinesh is
years. If Meena returned Rs.3660 at the Rs.18375, then find the sum had by
end of second year and she Dinesh.
returned Rs.4999.68 at end of third year A.Rs.6000
and cleared all her debt, find the value of B.Rs.4800
x? C.Rs.5400
A.Rs.5345 D.Rs.4500
B.Rs.5245 E.None of these
C.Rs.5235
D.Rs.5435 12. A certain sum of money amounts to
E.Rs.5425 Rs.6300 in 2 years and to Rs.8550 in 4.5
years. Find the rate of simple interest.
9. The simple interest on Rs. P at 15% A.10%
p.a for 2 years is Rs.300 more than the B.15%
simple interest on Rs.(P + 500) at 12% C.18%
for 2 years. What is the D.20%
interest if (2P + 500) is lent for 2 years E.22%
at 12% SI rate?
A.Rs.1750 13. Certain sum triples itself in x years
B.Rs.2350 when invested in simple interest scheme
C.Rs.3480 at the rate of 20% per annum. Find the
D.Rs.4810 value of x?
E.None of these A.5
B.8
10. A sum of money invested doubles C.6
itself in 4 years. In how many years will D.10
it become nine times itself at the same E.12
rate?
Page 146 Follow us: Official Site, Telegram, Facebook, Instagram, Instamojo
14. Meena invested Rs.8400 in simple 17. The ratio of the amounts on the same
interest scheme at the rate of x% per sum for 11 years to 9 years is 36:25
annum for 4 years and after 4 years she when invested to earn compound
received the interest from the scheme is interest. What is the rate of interest per
Rs.3360. If Beela invested Rs.9600 in annum?
compound interest scheme at the rate of A.15%
2x% per annum for 3 years, then what is B.10%
the total amount received by Beela after C.20%
3 years? D.18%
A.Rs.16783.8 E.None of these
B.Rs.16456.8
C.Rs.16965.8 18. Mini invests Rs.21000 at 10% per
D.Rs.16588.8 annum compound interest scheme A
E.None of these and he also invests Rs.16000 at 15% per
annum simple interest scheme
15. Amala has Rs.x. She invests 60% of B. What is the difference between the
this amount in simple interest scheme at interest received from scheme A and B
the rate of 15% per annum and rest in after 2 years?
compound interest scheme at the rate of A.Rs.310
20% per annum. After 2 years she B.Rs.340
received the total interest from both C.Rs.390
schemes is Rs.2848, find the value of x? D.Rs.370
A.Rs.3000 E.None of these
B.Rs.5000
C.Rs.7000 19. Natchathira invests Rs.3000 in
D.Rs.6000 simple interest scheme at the rate of
E.Rs.8000 22% per annum for x years. After x
years she received the total amount is
16. A sum of Rs.4600 becomes Rs.6670 Rs.4980. If Nandhini invests Rs.8000 in
after 3 years at simple interest. If the compound interest scheme at 20% per
same sum invests on compound interest annum for x years, then find the interest
at the same rate of interest for 2 years, received by Nandhini?
then find the compound interest A.Rs.5824
received by after 2 years? B.Rs.6036
A.Rs.1683.5 C.Rs.5952
B.Rs.1483.5 D.Rs.5734
C.Rs.1883.5 E.Rs.6126
D.Rs.1683.5
E.None of these 20. Sharmi invested half of her PF
amount in scheme A at 10% p.a simple
Page 147 Follow us: Official Site, Telegram, Facebook, Instagram, Instamojo
interest for 2 years and another half of A.Rs.5200
the amount in scheme B at 15% B.Rs.5400
compounded annually for 2 years. If C.Rs.5800
Sharmi received Rs.661.5 less than D.Rs.5900
scheme A when compared to scheme B, E.None of these
then find the amount invested by her in
scheme B.

ANSWER 1-20: 5. Answer: B


1. Answer: C Amount invested in scheme B = x
Interest earned in 2 years = 5000 – 4600 Amount invested in scheme A = 6000 - x
= 400 SI = (6000 – x) * 15 * 2/100 = 1800 –
Interest earned in 1 year = 200 0.3x
Interest earned in 3 years = 200 * 3 = CA = x * (1 + 10/100)2 = 1.21x
600 (6000 – x) + 1800 – 0.3x + 1.21x = 7440
x = 4600 – 600 = Rs.4000 x = 4000
Sum invested by Scheme A = 6000 –
2. Answer: A 4000 = 2000
5880 = x * 20 * 6/100
x = 4900 6. Answer: A
CA = 4900 * (1 + 10/100)3 SI = Pnr/100
= Rs.6521.9 => (x + 400) * 3 * 10/100 = 480 => x =
1200
3. Answer: E Interest earned = P[1 + r/100]n– P
CI received by Meena = 7800 * (1 + = 3x[1 + 10/100]3– 3x
20/100)2 – 7800= 3432 =3600[1 + 10/100]3– 3600
SI received by Vinitha = 4800 * 15 * = Rs.1191.6
5/100= 3600
Required difference = 3600 – 3432 = 7. Answer: A
168 y = 11000 - x
SI received by Renu = x * 15 * 3/100 =
4. Answer: A 0.45x
CA = P * (1 + R/100)n SI received by Rama = (11000 – x) * 15 *
3300 = P * (1 + R/100)1------(1) 3/100
3993 = P * (1 + R/100)3------(2) = 4950 – 0.45x
From (1) * (2) 4950 – 0.45x – 0.45x = 1800
P2 x = 3500
* (1 + R/100)4 = 3300 * 3993 y = 11000 – 3500 = 7500
P * (1 + R/100)2 = 3630
8. Answer: D
Page 148 Follow us: Official Site, Telegram, Facebook, Instagram, Instamojo
120/100 * (1.44x – 3660) = 4999.68 3P – P = P * x * 20/100
1.728x – 4392 = 4999.68 x = 10
x = 5435
14. Answer: D
9. Answer: C 3360 = 8400 * 4 * x/100
P * 15% * 2 = (P + 500) * 12% * 2 + 300 x = 10%
30%P=24%P+120+300 CA = 9600 * (1 + 20/100)3
6%P=420 = Rs.16588.8
=> P = Rs.7000
Required interest = (2P + 500) * 2 * 15. Explanation
12/100 Answer: E
=14500*2*12/100 SI = x * 60/100 * 2 * 15/100 = 0.18x
= Rs.3480 CI = x * 40/100 * (1 + 20/100)2– x *
40/100
10. Answer: D = 0.176x
2P – P = P * 4 * R/100 0.18x + 0.176x = 2848
R = 25% x = 8000
9P = P + (P * 25 * N/100)
N = 32 16. Answer: B
CI = P * (1 + R/100)n– P
11. Answer: E SI = P * N * R/100
Amount received by Deepika = 13x + 6670 – 4600 = 4600 * R * 3/100
13x * 15 * 5/100 R = 15%
= 22.75x CI = 4600 * (1 + 15/100)2– 4600
Amount received by Dinesh = 15x + 15x CI = Rs.1483.5
* 4 * 10/100
= 21x 17. Answer: C
22.75x + 21x = 18375 CA = P * (1 + R/100)n
x = 420 CA for 11 years = P * (1 + R/100)11-----
Dinesh sum = 420 * 15 = Rs.6300 (1)
CA for 9 years = P * (1 + R/100)9------(2)
12. Answer: D From (1) and (2)
SI for (4.5 – 2) years = 8550 – 6300 = (1 + R/100)2 = 36/25
2250 (1 + R/100) = 6/5
SI for 2 years = 2250 * 2/2.5 = 1800 100 + R = 120
Sum = 6300 – 1800 = 4500 R = 20%
1800 = 4500 * 2 * R/100
R = 20% 18. Answer: C
CI = 21000 * (1 + 10/100)2– 21000
13. Answer: D = Rs.4410
Page 149 Follow us: Official Site, Telegram, Facebook, Instagram, Instamojo
SI = 16000 * 15 * 2/100 = 4800 = 5824
Difference = 4800 – 4410 = Rs.390
20. Answer: B
19. Answer: A Amount invested in scheme A or B be P.
SI = P * N * R/100 P[(1 + r/100)n – 1] – Pnr/100 = 661.5
CI = P * (1 + R/100)n– P =>P[(1 + 15/100)2– 1] – P * 2 * 10/100 =
4980 – 3000 = 3000 * 22 * x/100 661.5
x=3 => P = Rs.5400
CI = 8000 * (1 + 20/100)3– 8000

Ratio & Proportion (Partnership)


Rules & Formulas of Ratio & Proportion

Ratio Formula

Assume that, we have two quantities or two numbers or two entities and we have to
find the ratio of these two, then the formula for ratio is defined as;

a: b = a/b

where a and b could be any two quantities.

Here, “a” is called the first term or antecedent, and “b” is called the second term
or consequent.

Example: In ratio 4:9, is represented by 4/9, where 4 is antecedent and 9 is consequent.

If we multiply and divide each term of ratio by the same number (non-zero), it doesn’t
affect the ratio.

Example: 4:9 = 8:18 = 12:27

Proportion Formula

Now, let us assume that, in proportion, the two ratios are a:b & c:d. The two
terms ‘b’ and ‘c’ are called ‘means or mean term,’ whereas the terms ‘a’ and ‘d’ are
known as ‘extremes or extreme terms.’

Page 150 Follow us: Official Site, Telegram, Facebook, Instagram, Instamojo
a/b = c/d or a : b :: c :d

Let us consider one more example of a number of students in a classroom. Our first
ratio of the number of girls to boys is 3:5 and that of the other is 4:8, then the
proportion can be written as

3 : 5 :: 4 : 8 or 3/5 = 4/8

Here 3 & 8 are the extremes, while 5 & 4 are the means.

Note: The ratio value does not affect when the same non-zero number is multiplied or
divided on each term.

Fourth, Third and Mean Proportional

If a : b = c : d, then:

 d is called the fourth proportional to a, b, c.


 c is called the third proportion to a and b.
 Mean proportional between a and b is √(ab).

Comparison of Ratios

If (a:b)>(c:d) = (a/b>c/d)

The compounded ratio of the ratios: (a : b), (c : d), (e : f) is (ace : bdf).

Duplicate Ratios

If a:b is a ratio, then:

 a2:b2 is a duplicate ratio


 √a:√b is sub-duplicate ratio
 a3:b3 is triplicate ratio

Componendo and Dividendo

If a:b::c:d, then [(a+b):(a-b)::(c+d):(c-d)]

Page 151 Follow us: Official Site, Telegram, Facebook, Instagram, Instamojo
Ratio and Proportion Tricks

Let us learn here some rules and tricks to solve problems based on ratio and
proportion topic.

 If u/v = x/y, then uy = vx


 If u/v = x/y, then u/x = v/y
 If u/v = x/y, then v/u = y/x
 If u/v = x/y, then (u+v)/v = (x+y)/y
 If u/v = x/y, then (u-v)/v = (x-y)/y
 If u/v = x/y, then (u+v)/ (u-v) = (x+y)/(x-y), which is known as componendo -
Dividendo Rule
 If u/v = v/x, then u/x = u2/v2
 If u/v = x/y, then u = x and v =y
 If a/(b+c) = b/(c+a) = c/(a+b) and a+b+ c ≠0, then a =b = c

Practice Questions
(Ratio & Proportion) B.Rs 78
SET-1
C.Rs 56
1) The ratio of the number of male D.Rs 64
and female in a committee is 5:6. If
the percentage increase in the E.None of these
number of male and female be 12% 3) A box contains the coins of 5p, 10p
and 10% respectively, what will be and 25p are in the ratio 3:3:5. If there
the new ratio? is Rs. 34 in all, how many 10p coins
A.33:28 are there?

B.28:33 A.40

C.27:34 B.60

D.34:27 C.80

E.None of these D.100

2) A manager divided Rs 234 into E.None of these


three workers P, Q and R such that 4 4) The wages of Naveena and Menaka
times P’s share is equal to 6 times Q’s are in the ratio ratio 6:5. If the wages
share which is equal to 3 times R’s of each is increased by Rs.6000, the
share. How much did P get? new ratio becomes 38:35. What is
A.Rs 82 Menaka’s present salary?

Page 152 Follow us: Official Site, Telegram, Facebook, Instagram, Instamojo
A. 4000 A. A.150 runs
B. 5500 B. B.300 runs
C. 5000 C. C.250 runs
D. 4500 D. D.200 runs
E. None of these E. E.None of these
5) Ratio of the salary of X & Y is 5:8. If 8. The cost of 8A is equal to the
the salary of X increase by 40% and cost of 50B. The cost of 19C is
those of Y decrease by 15%, the new 456. The cost of B is twice the
ratio of their salary becomes 10:9. cost of 2C. What is the total
What is X’s salary? cost of 3A and 4B together?
A.24000
A. 2184
B.25000
B. 2168
C.28000
C. 2243
D.data inadequate
D. 2264
E.None of these
E. None of these
6) The ratio of the number of UG
students and PG students in a college 9. Divide Rs. 370 into three
is 5:4. If 40% of the UG students and parts such that second part is
50 % of the PG students are 1/4 of the third part and the
scholarship holders. What percentage ratio between the first and
of the students does not get the the third part is 3 : 5. Find the
scholarship? amounts of these three parts
A.54.6% respectively.

B.55.2% A. 50, 120, 200


C.55.5% B. 120, 50, 200
D.54.3% C. 200, 120, 50
E.None of these D. 200, 100, 50
7) Karan, Hari and Kowshik play E. None of these
cricket. The runs got by Karan to Hari
and Hari to Kowshik are in the ratio 10. Two numbers are in the
of 5:3. They get altogether 588 runs. ratio of (1 ½): (2 2/3). When
How many runs did Karan get? each of these is increased by
Page 153 Follow us: Official Site, Telegram, Facebook, Instagram, Instamojo
15, the ratio changes to 1 2/3: B. 27
2 ½. The larger of the
C. 36
numbers is,
D. 64
A. 48
E. 56

Answers 1-10:
1) Answer: B
Originally, let the number of male and
female in a committee be 5x and 6x
respectively.
Their increased number is (112% of 5x)
and (110% of 6x).
P:Q:R=3:2:4
(112/100)*5 and (110/100)*6
The sum of the total Salary,
=5.6 : 6.6
3x+2x+4x=234
=56 : 66
9x=234
=28 : 33
X=234/9=26
So, the required ratio = 28 : 33
Hence p gets =3x=3*26= Rs 78

2) Answer: B
3) Answer: B
4 times P’s share = 6 times Q’s share = 3
times R’s share Let the no. of 5p, 10p, 25 p coins be 3x,
3x & 5x respectively.
4*P = 6*Q
Then sum of their values = Rs.
P/Q = 3/2 [(5*3x/100) + (10*3x /100) +
6*Q = 3*R (25*5x/100)]

Q/R = ½ =Rs. [(15x/100) + (30x /100) +


(125x/100)]
=Rs. (15x+30x+125x / 100)
=Rs. (170x /100)
Therefore 170x / 100 = 34

Page 154 Follow us: Official Site, Telegram, Facebook, Instagram, Instamojo
X=34 *100/170 Therefore 7x : 74x/5 = 10:9
X=20 7x*5/74x =10:9
Hence the no. of 10p coins = 3x We can’t find x, so the given data is
inadequate
=3*20
=60
6) Answer: C
Let UG students be = 5x and
4) Answer: D
PG students be = 4x respectively
Let the original salary of Naveena and
Menaka be 6x and 5x respectively No. of those who do not get scholarship,
Then, (6x+6000) / (5x+6000) = 38/35 = 60% of 5x + 50% of 4x
35(6x+6000) = 38(5x +6000) = (60/100*5x) + (50/100*4x)
210x +210000 = 190x + 228000 = 300x/100+ 200x /100
210x – 190x = 228000 – 210000 = 3x + 2x
20x = 18000 = 5x
X=900 Required percentage = (5x/9x) *100
Menaka’s present salary = 5x = 55.5%
= 5*900 = Rs. 4500
7) Answer: B
5) Answer: D Karan : Hari =5:3
Let the original salary of X & Y be 5x & Hari : Kowshik =5:3
8x respectively
Karan : Hari : Kowshik = 25:15:9
New salary of X = 40% of 5x
Therefore, the runs made by Karan,
=140/100 * 5x
=25/49*588
=700x /100
=25*12=300 runs
= 7x
New salary of Y = 85% of 8x
8) Answer: A
=185 /100 *8x
8A= 50B=> A/B=50/8
= 74x/5
Page 155 Follow us: Official Site, Telegram, Facebook, Instagram, Instamojo
A : B= 50:8=>25:4 x = 10 × 4
19C = 456 x = 40
=>C=24 Therefore,
B=2*2C=4*24=96 First part = 3x =3 × 40=120
4’S=96=>1’S=24 Second part = 5x/4 =5 × 40/4=50
A=24*25=600 Third part = 5x =5 × 40=200
3A +4B=3*600+4*96=2184
10) Answer: A
9) Answer: B Two numbers are in the ratio of,
Let the first and the third parts be 3x (1 ½): (2 2/3) =>3/2: 8/3 =>9:16 (9x,
and 5x. 16x)
Second part = 1/4 of third part = (1/4) × 1 2/3: 1 ½ =>5/3:5/2 =>2:3
5x == 5x/4
9x+15/16x+15 =2/3
Therefore, 3x + (5x/4) + 5x = 370
27x+45 = 32x+30
(12x + 5x + 20x)/4 = 370
5x= 15
37x/4 = 370
X=3
x = (370 × 4)/37
The larger of the numbers is, 16x= 48

SET 2
1).Chintu, Pintu and Mintu are three e) 8 years
friends. Chintu is twice as old as Pintu
2).Find the ratio of speed of A,B and C if
and Mintu is as old as Cintu and Pintu
for every 11 steps taken by A,B takes 12
together. Two years later, the ratio of
steps and C takes 14 steps. Also 12 steps
age of Chintu and Mintu would be 7:10
of A are equal to 14 steps of B and 16
what was the age of Pintu five years
steps of C.
ago?
a) 154:144:147
a) 2 years
b) 132:168:224
b) 4 years
c) 25:24:23
c) 5 years
d) None f these
d) 1 years
Page 156 Follow us: Official Site, Telegram, Facebook, Instagram, Instamojo
e) 161:123:113 contribution made by each student of
class I and II is 19:17. If the total
3).The price of a ring varies jointly, with
contribution made by the all the
the cube root of number of diamonds
students of both the class is Rs.29200,
and weight of gold used in it. The price
then find the total contribution made by
of the ring was “10,000. When it had 64
class II students only.
diamonds and 15 gm. of gold. Find the
no. of diamonds if the price of ring a) Rs.10950
was~` 10,500 and weight of gold was 21
b) Rs.13789
gm.
c) Rs.10200
a) 3
d) Rs.13272
b) 9
e) None of these
c) 27
6).A donkey moves at a speed of 8
d) 81
kmph, when no load is put on him.
e) 16 Reduction in the speed of donkey varies
directly to the square root of the kgs of
4).There are two containers A and B.
load put on him. When only 4 kgs of load
Container A is filled with 30 liters of
is put the speed of the donkey becomes
kerosene while container B is filled with
6 kmph. Find the minimum load that can
30 liters of water. Ten liters of kerosene
be put on the donkey with which it
is taken from container A and put in
cannot move.
container B and then twelve liters of the
mixture (of kerosene and water) is a) 64 kg
taken from container B and put in
b) 63.9 kg
container A. Find the ratio of kerosene in
A and B respectively. c) 65.2 kg
a) 23:9 d) Cannot be determined
b) 21:5 e) None of these
c) 23:7 7). A container has a mixture of
kerosene and castor oil in the ratio of
d) 1:3
7:5 and another container contains
e) None of these kerosene and castor oil in the ratio of
5:3. Find the proportion in which the
5).In a school, students of class I and
mixtures from two containers should be
class II are going for a picnic to
mixed so that the resultant mixture has
Surajkund and Badkal lake respectively.
ratio of kerosene and castor oil of 3:2.
The ratio f number of students in class I
and II is 5:3. Also the ratio of the a) 2:3
Page 157 Follow us: Official Site, Telegram, Facebook, Instagram, Instamojo
b) 3:2 9).There are three vessels 1, 2 and 3.
The ratio of the total capacity of vessels
c) 4:1
1, 2 and 3 is 5:4:3 respectively. All the
d) 5:2 vessels are full of mixture of sugar syrup
and water. In vessel 1, ratio of sugar
e) None of these
syrup to water is 2:3. Similar ratio in
8).The no. of pens in three different case of vessel 2 and vessel 3 is 5:4 and
pencil boxes in the ratio of 1:2:3. Find 1:3 respectively. The mixture of all the
the ratio in which the number of pens in three vessels is emptied into one bigger
the first and the second boxes must be vessel. What is the resulting ratio of
increased so that the new ratio becomes sugar syrup and water?
3:2:1.
a) 17:25
a) 1:3
b) 179:253
b) 2:1
c) 253:179
c) 2:3
d) 233:169
d) 3:4
e) None of these
e) None of these

Answer 1-9:
1) d), 2) a), 3) c), 4) c), 5) c), 6) a), 7) b), 8) b), 9) b).

Solutions: distance are directly proportional to


each other. When time is constant.
1).Let Pintu’s present age =x, Chintu =
2x and Mintu = x+2x =3x A B C
Two years later [2x+2/3x+2] = (7/10) (x/12)×11 (x/14)×12 (x/16)×14
20x+20=21x+14 = 6=x Ratio is (11/12): (12/14): (14/16)
Pintu’s present age =6 L.C.M of 12, 14, 16 is 336
Five years ago, his age was 6-5= 1 year =308:288:294 = 154:144:147
Answer: d) [What is x? Since distance covered by A
in 12 steps is equal to distance covered
by B in 14 steps and distance covered by
2.Ratio of speed is same as ratio of C in 16 steps, hence let this distance =x.
distance covered by them as speed and Distance covered in 1 step by A, B and
Page 158 Follow us: Official Site, Telegram, Facebook, Instagram, Instamojo
C= (x/12), (x/14) and (x/16) 20 0 10 30
respectively ]
+3 +9-3 -9
Answer: a)
23 9 -7 21
Ratio of Kerosene in A to B = 23:7
3. Let price =P, number of diamonds = n
Answer: c)
and weight of gold = g
When p=10000, n=64 and g=15, k=?
5.Let the no. of students in class II be 5x
P= kg3√n => 10000 = k(15) (3√64)
and 3x respectively and contribution
K=(10000/15×4)=(500/3) made by each student of class I and class
II be 19y and 17y respectively.
P=10500, g=21, k=(500/3), n=?
Hence, ratio of total contribution of class
P=(500/3) g3√n =>
I and class II =5x×19y: 3x×17y = 95:51
10500=(500/3).213√n
Total contribution made by students of
3√n=10500×(3/500)×(1/21)=3=>n=27
class II (51/(95+51))×29200=10200
Answer: c)
Answer: c)

4.Container A Container B
6.Speed of the donkey, Without any load
Kerosene water Kerosene water = 8 kmph. With 4 kgs of load, speed
becomes 6 kmph, hence speed is
30 0 0 30
reduced by 2 kmph.
-10 –+10 –
Reduction in speed varies directly with
20 0 10 30 the square root of the load.
When 10 litres of kerosene is taken out Hence (8-6) = k√4=k±1
from A and put in B, then kerosene in A=
The donkey cannot move at zero speed.
20 litres. Mixture in B= 40 litres in which
i.e. when his speed reduced by 8 kmph.
ratio of Kerosene to water =10:30 = 1:3,
So reduction in speed= 8 kmph
when 12 litres of mixture is taken out
from B, and put in A, then out of 12 litres 8=1√l=>√l=8 and l=64, when √l=-8,
l=64
12×(1/4)=3 litres is kerosene and
remaining 9 litres is water. At 64 kg the donkey will stop.
Container A Container B Answer: a)
Kerosene water Kerosene water

Page 159 Follow us: Official Site, Telegram, Facebook, Instagram, Instamojo
7.(2) Let quantity of mixture taken from i.e (y/3), (2y/3), y
first be x and second be y.
The required number of pens is 3y, 2y
Amount of kerosene oil in the resultant and y in
mixture (x+y) is
Increase in 1st box=3y-(y/3)=(8/3)y&
(7/12)x+(5/8)y=(3/5)(x+y)
Increase in 2nd box = 2y-(2y/3)=(4/3)y
(7/12)x-(3/5)x=(3/5)y-(5/8)y-
Ratio of increase = (8y/3)L4/3)y=2:1
(1/60)x=-
(1/40)y=>(x/y)=(6/4)=(3/2)=3:2 Answer: b)
Answer: b)
9.Method-1: Let the total mixture in
vessel 1,2 and 3 be 5 litres and 3 litres
8.let the no. of pens in 1st, 2nd and 3rd
respectively. So, quantity of water in
pencil box be x, 2x and 3x respectively
(5+4+3)=12 litres of mixture is
and let the required no. be 3y, 2y and y.
=(3/5)×5+(4/9) ×4+(3/4) ×3
The quantity f pens in the third pencil
box would remain the same, hence =3+(16/9)+(9/4)=[(108+64+81)/36]=(
253/36)
3x=y or x =(y/3)
Quantity of sugar syrup is
Quantity of pens in the boxes originally
is x, 2x and 3x 12-(253/36)=(179/36)
When x=(y/3), hence quantity is Ratio of sugar syrup to water = 179:253
x=(y/3), 2x=(2y/3), 3x=y
Answer: b)

SET-3
Q1. A vessel of 120 litres is filled with E) 3 : 7
milk and water. 80% of milk and 40%
Ans(A)
of water is taken out of the vessel. It is
found that the vessel is vacated by Let the initial amount of milk=x
65%. What is the ratio of milk to ∴initial amount of water=120-x
water? According to question
80% of x+40% of (120-x)=65% of 120
A) 5 : 3
x=75
B) 6 : 5 Ratio= 75 : 45 = 5 : 3
C) 3 : 5
D) 4 : 3
Page 160 Follow us: Official Site, Telegram, Facebook, Instagram, Instamojo
Q2. The ratio of Radha’s and Ruchi’s E) None of these
ages is 9 : 4. If the difference between
the present age of Radha and the age
Ans(C)
of Ruchi 5 year hence is 5 then what is
the sum of the present ages of Radha Let Radha’s age=9x
and Ruchi? Ruchi’s age=4x
According to question
A) 18 years
9x-(4x+5)=5
B) 16 years 9x-4x=10
x=2
C) 26 years
Radha’s age=9×2=18
D) 32 years Ruchi’s age=4×2=8
Sum=26 years

(Partnership Problems)

SET-1
Q1. P and Q started a business by A’s share and C share in the profit will
investing Rs. 45,000 and Rs.54,000 be :
respectively. After four months R A) Rs 800
joined the business with a capital of B) Rs 600
Rs. 30,000. After two more months Q C) Rs 1200
left the business with his capital. At D) Rs 1,800
the end of the year P got a share of Rs. E) none of these
13,500 in the profit. What is the total
profit earned? Q3. A starts business with a capital of
A) Rs 26800 Rs1200. B and C join with some
B) Rs 27600 investments after 3 and 6 months,
C) Rs 28600 respectively. If at the end of a year,
D) Rs 29200 the profit is divided in the ratio 2 : 3 :
E) None of these 5 respectively, what is B’s investment
in the business?
Q2. A, B and C jointly start a business A) 2400 Rs.
A puts in Rs 15,000 for 8 months B B) 1800 Rs.
puts in Rs 12,000 for 9 months and C C) 3600 Rs
puts in Rs 8,000, for the whole year. D) 6000- Rs.
In the end of the year there is a profit E) 5500 Rs.
of Rs 10,800. The difference between

Page 161 Follow us: Official Site, Telegram, Facebook, Instagram, Instamojo
Q4. A, B and C enter into a
partnership. A invests Rs. 8000 for Q7. P and Q started a business by
the whole year, B puts in Rs. 12000 at investing Rs. 15,000 and Rs.18,000
the first and increasing to Rs. 16000 respectively. After four months R
at the end of 4 months, whilst C puts joined the business with a capital of
in at first Rs. 16000 but withdraw Rs. Rs. 10,000. After two more months Q
4000 at the end of 9 months. Find the left the business with his capital. At
profit of A at the end of year, if the the end of the year P got a share of Rs.
total profit is Rs. 22600. 4,500 in the profit. What is the total
A) Rs. 4800 profit earned?
B) Rs. 4600 A) Rs 6800
C) Rs. 4750 B) Rs 7600
D) Rs. 4300 C) Rs 8600
E) none of these D) Rs 9200
E) none of these
Q5. P and Q entered into partnership
investing Rs 12000 and Rs 16000 Q8. X, Y and Z enter into partnership.
respectively. After 8 months, R also X invests 1/4 part of total capital for
joins the business with a capital of Rs one-fourth of the time. Y contributes
15000. The share of R in a profit of Rs one fifth of the capital for half of the
45600 after 2 years will be time. Z contributes the remaining
A) 240000 Rs. capital for the whole time. How
B) 16000 Rs. should they divided a profit of Rs.
C) 12000 Rs. 1140?
D) 114000 Rs. A) Rs. 100, Rs. 160, Rs. 880
E) None of these B) Rs. 120, Rs. 140, Rs. 880
C) Rs. 120, Rs. 150, Rs. 840
Q6. A and B entered into partnership D) Rs. 140, Rs. 170, Rs. 830
with Rs 700 and Rs 600 respectively. E) None of these
After 3 months A withdrew 2/7 of his
stock but after 3 months more he put Q9. A and B started a business with
back 3/5 of what he had withdrawn. the investments in the ratio of 5 : 3
The profits at the end of the year are respectively. After 6 months from the
Rs 726, how much of this should A start of the business, C joined them
receive? and the respective ratio between the
A) Rs 633 investments of B and C was 2 : 3. If the
B) Rs 336 annual profit earned by them was Rs.
C) Rs 663 12300, what was the difference
D) Rs 366 between B’s share and C’s share in the
E) None of these profit?
Page 162 Follow us: Official Site, Telegram, Facebook, Instagram, Instamojo
A) Rs. 900 337600 while Gaurav withdraws Rs.
B) Rs. 800 45600. The ratio of investments then
C) Rs. 600 changes to 24 : 59 : 167. How much
D) Rs. 400 did Gaurav invest initially?
E) Rs. 700 A) Rs. 45600
Q10. Gaurav, Vivek and Neeraj enter B) Rs. 96000
into partnership by making C) Rs. 141600
investments in the ratio 3 : 5 : 7. After D) Rs. 156000
a year, Neeraj invests another Rs. E) None of these

SOLUTION (1 to 10)

Q1. Ans(B)

Q2. Ans(A)

Q3. Ans(A)

Page 163 Follow us: Official Site, Telegram, Facebook, Instagram, Instamojo
Q4. Ans(A)

Q5. Ans(C)

Q6. Ans(D)

Q7. Ans(D)

Page 164 Follow us: Official Site, Telegram, Facebook, Instagram, Instamojo
Q8. Ans(A)

Q9. Ans(A)

Q10. Ans(C)

Let the initial investments of gaurav, vivek, neeraj be Rs. 3x, 5x and 7x respectively.
Then,

Page 165 Follow us: Official Site, Telegram, Facebook, Instagram, Instamojo
SET-2
1. Radhika started a workshop with an . R had invested Rs. 1560 for 8
investment of Rs.40,000. She months .How much did P and Q
invested additional amount of contribute ?
Rs.10,000 every year. After two A) Rs.720, Rs.1280
years her sister Rama joined her B) Rs.650, Rs.1100
with an amount of C) Rs.758, Rs.1500
Rs.85,000.Therefore,Rama did not D) Rs.800, Rs.1720
invest any additional amount. On E) Rs.870, Rs.1750
completion of 4 years from the
Answer: Option A
opening of workshop they earned an
Solution:
amount of Rs.1,95,000. What will be
P : Q : R
Radhika’s share in the earning ?
[(960*3)/4] : [(960*8)/6] : [(156
A) Rs.2,20,000
0*8)13] =720 : 1280 : 960
B) Rs.1,10,000
C) Rs.2,45,000 3. There are two persons invests
D) Rs.3,35,120 Rs.1,15,000 and Rs.1,00,000 resp. in
E) Rs.1,01,5000 a project and agree that 50% of the
profit should be divided equally
Answer: Option B
between them and the remaining is
Solution:
to be treated as interest on the
Investment of Radhika = Rs. 40,000
capital. If first person get Rs.500
+Rs. 50,000 +Rs. 60,000+Rs. 70,000
more than the second persom .What
= Rs. 2,20,000
is the total profit made in the
Investment of Rama = 85,000 * 2 =
business ?
Rs. 1,70,000
A) Rs.1780.22
Ratio = 22 : 17
B) Rs.15445.12
Radhika’s share = (22/39 )*1,95,000
C) Rs.21245
= Rs.1,10,000
D) Rs.14333.33
2. P ,Q and R are partners in a business E) Rs.14758.41
.P whose money has been used for 4
Answer: Option D
months , claims (1/8) of the profit ,
Solution:
Q whose money has been used for 6
Ratio of the profit = 23:20
months ,claims at (1/3) of the profit
Therefore,
Page 166 Follow us: Official Site, Telegram, Facebook, Instagram, Instamojo
500*(100/50)*[(23+20)/(23-20)] = Answer: Option E
14333.33 Solution:
Tarun’s contribution in the business =
4. Ramesh, Suresh and Mahesh started
1-[(1/2)+(1/3)] = 1/6
a business with the investment in
Prabhu’s share : Sunny’s share :
the ratio 5:8:10 resp. After 1 year
Tarun’s share
Mahesh withdraw 50% of his capital
(1/4)*(1/2):(1/2)*(1/3):(1/6)*1
and Ramesh increased his capital by
3:4:4
80% of his investment . After 2 years
Tarun’s share in profit =
in what ratio should the earned
(4/11)*1,21,000 = Rs.44,000
profit be distributed among Ramesh
,Suresh and Mahesh? 6. A ,B and C sharing profits in the ratio
A) 15:10:17 3:2:2 . B retired from the company
B) 14:16:15 and A and C decide to share profits
C) 11:15:19 in the ratio 3:2.What is the gaining
D) 8:13:7 ratio?
E) 10:15:14 A) 5:3
B) 4:5
Answer: Option B
C) 3:2
Solution:
D) 2:1
Ramesh :Suresh:Mahesh
E) 3:5
[(5x*12)+(9x*12)]:(8x*24):[(10x*12
)+(3x*12)] =14:16:15 Answer: Option C
Solution:
5. Prabhu initiated his business with
Gaining ratio = [(3/5)-
(1/2) of the total capital for (1/4)th
(3/7)] : [(2/5)-(2/7)] = 3 : 2
of the time . His brother Sunny
invests (1/3) of the capital for 7. Rs.61,105 ,is divided between Ram
(1/2)th of the time and Prabhu’s and Raman in the ratio 3 : 8 .What is
friend Tarun invests the remaining the difference between thrice the
capital for the whole time. Find the share of Ram and twice the share of
share of Tarun in the total profit of Raman?
Rs. 1,21,000. A) Rs.52,500
A) Rs.20,000 B) Rs.42,000
B) Rs.24,500 C) Rs.35,720
C) Rs.50,000 D) Rs.38,885
D) Rs.33,420 E) Rs.47,200
E) Rs.44,000
Answer: Option D
Solution:
Required difference = [(8/11)*2 –

Page 167 Follow us: Official Site, Telegram, Facebook, Instagram, Instamojo
(3/11)*3 ] * 61,105 ,what is the total annual profit?
= (7/11)*61,105 =Rs. 38,885 A) Rs.7000
B) Rs. 8400
8. Mr. X started a business investing
C) Rs.8000
Rs.25,000 in 1996.In 1997 he
D) Rs.9000
invested an additional amount of Rs.
E) RS.8500
10,000 and Mr. Y joined him with an
amount of Rs.35,000.In 1998 , Mr.X Answer: Option B
invested another additional amount Solution:
of Rs.10,000 and Mr. Z joined them A, B and C’s equivalent capitals
with an amount of Rs. 35,000 .Find A : B : C
the share of Mr. Y in the profit of Rs. (30,000*12) : (20,000*6) : (50,00
1,50,000 earned at the end of 3 years 0*3)
from the start of the business in 12 : 4 : 5
1996? C’s profit = 5x/12 = 2000
A) Rs.50,000 => x = 8,400
B) Rs.80,000
10. Riya started a project by
C) Rs.70,000
investing Rs. 60,000. 6 months later
D) Rs.40,000
her sister Riama joins her by
E) Rs.60,000
investing Rs. 1,00,000.At the end of 1
Answer: Option A year from the commencement of the
Solution: business ,they earn a profit of
Mr. X : Mr. Y : Mr. Z Rs.27,038.Find Riama’s share in the
[(25000*3)+(10000*2)+(10000*1)] profit?
: (35000*2) : (35,000*1) A) Rs.14,150
105000 : 70000 : 35000 B) Rs.13,100
3 : 2 : 1 C) Rs.13,560
Mr. Y’s share in the profit = D) Rs.12,290
(2/6)*1,50,000 = Rs. 50,000 E) Rs.12,510
9. A starts a business with an initial Answer: Option D
investment of Rs. 30,000. After 6 Solution:
months , B enters into the Riya and Raima’s capital in the ratio
partnership with an investment of = (60,000*12) : (1,00,000*6)
Rs. 20,000.Again after 3 months C =5:6
enters with an investment of Therefore ,Raima’s share in the
Rs.50,000.If C receives Rs. 2000 in profit = (5/11)*27,038 = 12,290
the profit at the end of the year

Page 168 Follow us: Official Site, Telegram, Facebook, Instagram, Instamojo
Time & Work
Formulas & Shortcut of Time & Work

 Work Done = Time Taken × Rate of Work


 Rate of Work = 1 / Time Taken
 Time Taken = 1 / Rate of Work
 If a piece of work is done in x number of days, then the work done in one day = 1/x
 Total Wok Done = Number of Days × Efficiency
 Efficiency and Time are inversely proportional to each other
 X:y is the ratio of the number of men which are required to complete a piece of work, then the
ratio of the time taken by them to complete the work will be y:x
 If x number of people can do W1 work, in D1 days, working T1 hours each day and the number
of people can do W2 work, in D2 days, working T2 hours each day, then the relation between
them will be

Practice Questions
SET-1
1) A piece of work has to be when Ajay and Sneha complete the
completed in 60 days, a number of work individually. Prabha and Sneha
men are employed but it is found that started the work and left after 3 days.
only half of the work is done in 40 The number of days taken by Ajay to
days, then an additional 30 men were finish the remaining work is?
joined to complete the work on time. a) 12 days
Initially how many men are there to b) 9 ¾ days
work? c) 10 5/6 days
a) 30 men d) 11 2/5 days
b) 26 men e) None of these
c) 24 men
d) 34 men 3) Ragu and Rajesh can separately do
e) None of these a piece of work in 12 and 15 days
respectively. They worked together
2) The ratio of efficiency of Ajay and for 5 days, after which Rajesh was
Sneha is 6: 5. The ratio of number of replaced by Rohit. If the work was
days taken by Prabha to Sneha is 3: 2. finished in next 2 days, then the
Ajay takes 3 days less than Sneha,

Page 169 Follow us: Official Site, Telegram, Facebook, Instagram, Instamojo
number of days in which Rohit alone d) 20 days
could do the work? e) None of these
a) 20 days
b) 24 days 7) P takes 8 days to complete 2/3 of a
c) 32 days work, Q takes 3 days to complete 1/7
d) 28 days of the same work and R takes 8 days
e) None of these to complete 4/5 of the same work. If
they work for 3 days together then Q
4) A and B undertake to complete a and R leaves the work. Find the
piece of work for Rs. 3240. A can do it number of days P will take to
in 12 days, B can do it in 18 days and complete the remaining work?
with the help of C they complete the a) 117/28 days
work in 6 days. Find the share of C? b) 95/23 days
a) Rs. 620 c) 156/25 days
b) Rs. 480 d) 129/35 days
c) Rs. 500 e) None of these
d) Rs. 540
e) None of these 8) 20 men can complete a piece of
work in 16 days. After 5 days from
5) A and B can do a piece of work in the start of the work, some men left. If
10 and 15 days respectively. They the remaining work was completed
began the work together but A leaves by the remaining men 18(1/3) days,
after some days and B completed the how many men left after 5 days from
remaining work in 8 days. Number of the start of the work?
days after which A left the job? a) 10 men
a) 2 4/5 days b) 9 men
b) 4 ½ days c) 8 men
c) 3 ¾ days d) 6 men
d) 5 1/3 days e) None of these
e) None of these 9) 20 men can complete a piece of
work in 25 days. They started the
6) P, Q and R can complete the whole work and after 5 days, 5 more men
work in 20 days. P starts the work joined them. In how many days will
and works for ‘x’ days while Q and R the work be completed?
complete the remaining 2/5 of the a) 21 days
work in 14 days then find the value of b) 19 days
x? c) 16 days
a) 24 days d) 24 days
b) 28 days e) None of these
c) 32 days
Page 170 Follow us: Official Site, Telegram, Facebook, Instagram, Instamojo
10) Q can do a piece of work in 14 a) 5 3/7 days
days. Q is 50 percent more efficient b) 3 2/3 days
than P. In how many days half the c) 4 ¾ days
work is completed when both are d) 4 1/5 days
working simultaneously? e) None of these

SOLUTION (1 to 10)

1) Answer: a) = > Ajay = 15 days, Sneha = 18 days,


Let initially the no of men be x, Prabha = 27 days
A piece of work has to be completed in Work done by Prabha and Sneha in one
60 days day,
According to the question, = > (1/18) + (1/27) = 45/(18*27) =
Men days work 5/54
X 40 (1/2) Prabha and Sneha’s two day work
(x + 30) 20 (1/2) = > (5/54)*3 = 5/18
Work = men * days Rest of the work = 13/18
= > 40x/(1/2) = (x + 30)*20/(1/2) The number of days taken by Ajay to
= > 40x = (x + 30)*20 finish the remaining work is,
= > 40x = 20x + 600 Number of days = (13/18)*15 = 65/6 =
= > 20x = 600 10 5/6 days
= > x = 30 men
3) Answer: b)
2) Answer: c) Ragu and Rajesh worked together
The ratio of efficiency of Ajay and Sneha 1/12 + 1/15 = (12 + 15)/(12*15) = 3/20
= 6: 5 Ragu and Rajesh’s 5 days work =
The ratio of number of days taken by (3/20)*5 = (3/4)
Ajay and Sneha = 5: 6 Remaining work 1/4 done by Ragu and
The ratio of number of days taken by Rohit
Prabha and Sneha = 3: 2 Ragu and Rohit finished it in 4 days
Ratio of number of days taken by Ajay: (1/4)*(Ragu + Rohit)’s whole work = 2
Sneha: Prabha = 5: 6: 9 (Ragu + Rohit)’s whole work = 8
According to the question, Rohit’s one day work = (1/8) – (1/12) =
= > Sneha – Ajay = 8 days 1/24
= > 6’s – 5’s = 3 Rohit alone can complete the work in 24
= > 1’s = 3 days
Number of days taken to finish the
whole work, 4) Answer: d)
1/12 + 1/18 + 1/C = 1/6

Page 171 Follow us: Official Site, Telegram, Facebook, Instagram, Instamojo
1/C = (1/6) – (1/12 + 1/18) Work done in 3 days= 97*3= 291 units
(1/C) = (1/6) – (5/36) = 1/36 Time required by P to complete the
We get, C = 36 days remaining work
Now efficiency of A, B and C are in the = > (420-291)/35 = 129/35 days
ratio of = 1/12: 1/18: 1/36 = 3: 2: 1
6’s = 3240 8) Answer: c)
1’s = 540 Total units of work= 20*16= 320 units
The share of C = Rs. 540 Work done in 5 days= 20*5= 100 units
Remaining work= 320 -100= 220 units
5) Answer: a) Let the number of men left after 5 days
(1/10 + 1/15)*x + 8/15 = 1 be x
(1/10 + 1/15)*x = 1 – (8/15) So,
(1/10 + 1/15)*x = 7/15 20-x men completed the remaining 220
x/6 = 7/15 units in 55/3 days
x = 14/5 So,
X = 2 4/5 days (20-x)*55/3= 220
1100- 55x= 660
6) Answer: b) 55x= 440
Time required by Q and R to complete X= 8 men
the whole work
= 5*14/2= 35 days 9) Answer: a)
Time taken by P, Q and R= 20 days Total work = 20*25 = 500 work
Total units of work= 140 units 5 days work = 20*5 = 100 work
Q and R one day work= 4 units Remaining work = 500 – 100 = 400
P, Q and R one day work= 7 units According to the question,
P’s one day work= 7-4= 3 units = > 400/25 = 16 days
Units of work done by P = 3*140/5= 84 The total no of days = 16 + 5 = 21 days
units The work gets completed in 21 days
Required value of x = 84/3 = 28 days
10) Answer: d)
7) Answer: d) Efficiency ratio of Q: P = 150: 100 = 3: 2
Time taken by P= 8*3/2= 12 days Days ratio of Q: P = 2: 3
Q= 7*3= 21 days P can do a piece of work in 14 days
R= 8*5/4= 10 days 2’s = 14 = > 1’s = 7
Total units of work = 420 units So P can complete the work in 21 days
P’s one day work= 35 units (1/14 + 1/21)*x = ½
Q’s one day work= 20 units [35/(14*21)]*x = 1/2
R’s one day work= 42 units X = 21/5 = 4 1/5 days

Page 172 Follow us: Official Site, Telegram, Facebook, Instagram, Instamojo
SET-2
1.A alone completes the work in 16 days started the work and after 75% of the
and the efficiency of A is 25% more than work is completed A and B together left
B. In how many days they can complete the work, remaining work done by C
90% of the work together? alone, in how many days the whole work
A.8 days will be completed?
B.10 days A.8 days
C.6 days B.6.5 days
D.12 days C.7 days
E.9 days D.7.5 days
E.9 days
2. 20 men in 6 hours complete the work
in 3 days and 25 women in 6 hours 5. A, B and C alone can complete the
complete the same work in 4 days. In work in 21 days, 24 days and 28 days
how many days 4 men and 20 women respectively. If A, B and C together
are working together can complete the started the work, after 3 days A left the
work in one hour? work and 6 days before completion of
A.18.5 the work B also left the work. In how
B.20.5 many days is the work is completed?
C.22.5 A.10(4/13) days
D.24.5 B.11(4/13) days
E.25.5 C.12(4/13) days
D.13(4/13) days
3. Amirtha alone complete 75% of work E.14(4/13) days
in 15 days and the efficiency of Amirtha
is 20% more than Maklin. If Amirtha, 6. Anju, Banu and Kavin complete a
Maklin and Selvi together can complete work in 5 days for which they are paid a
the 75% of work in 6 days, In how many sum of Rs.1200. What is the daily wage
days Selvi alone complete the 80% of of Kavin, if the efficiency of Anju, Banu
work? and Kavin are in the ratio 3: 4: 5?
A.20 days A.Rs.50
B.30 days B.Rs.100
C.24 days C.Rs.150
D.28 days D.Rs.200
E.32 days E.None of these

4. A alone completes the work in 12 days 7. A and B alone can do a work in 10 and
and the ratio of the efficiency of C is 50% 15 days respectively. If both of them
more than B. Ratio of the efficiency of A started the work together and after 4
and B is 5:4. If A and B together can
Page 173 Follow us: Official Site, Telegram, Facebook, Instagram, Instamojo
days, A left the work. Then in how many alternatively, Swetha started the work.
days will B do the remaining work? After 12 days Chaitra and Anu together
A.5 can complete the work in 2 days. In how
B.6 many days Anu alone complete the
C.7 whole work?
D.8 A.9(8/9) days
E.None of these B.7(8/9) days
C.10(8/9) days
8. P and Q alone can paint a wall in 10 D.8(8/9) days
days and 12 days respectively. Both P E.None of these
and Q together started painting the wall,
but after 5 days, P left the work and the 10. X takes 16 days to complete 4/5th of
remaining work is completed by Q with a work, Y takes 12 days to complete
40% of his actual efficiency. Find the 3/7th of the same work and Z takes 20
time taken to complete the remaining days to complete 5/9th of the same
work. work. If they work together for 5 days
A.1 1/24 days and then X and Z leaves the work, then
B.4 days find the number of days to complete the
C.3 days whole work?
D.5 1/41 days A.17 1/9 days
E.None of these B.16 2/3 days
C.15 3/5 days
9. Swetha can complete the work in 16 D.12 1/9 days
days and Chaitra can complete the work E.None of these
in 20 days. Chaitra and Swetha worked

ANSWER:1-10
1. Explanation 2. Explanation
Answer: A Answer: C
A alone complete the work = 16 days 20m * 6 * 3 = 25w * 6 * 4
B alone complete the work = 16 * 3m = 5w
125/100 =20 days
20w = 12m
A and B together can complete the
4men + 20 women = 4 men + 12 men =
whole work = 1/16 + 1/20
16 men
= 9/80
20 * 6 * 3 = 16 * 1 * ?
Required time = 80/9 * 90/100 = 8 days
? = 22.5
Page 174 Follow us: Official Site, Telegram, Facebook, Instagram, Instamojo
3. Explanation 5. Explanation
Answer: C Answer: E
Amirtha alone complete the work = 3/21 + (x – 6)/24 + x/28 = 1
100/75 * 15 = 20 days
24 + 7x – 42 + 6x = 168
Maklin alone complete the work = 20 *
13x = 186
6/5 = 24 days
x = 14(4/13) days
Amirtha, Maklin and Selvi together can
complete the whole work = 100/75 * 6 =
8 days
6. Explanation
Selvi alone complete the work = 1/8 –
Answer: B
1/20 – 1/24
Wage per day = 1200/5 = Rs.240
= (15 – 6 – 5)/120
Work done by Anju, Banu and Kavin are
= 4/120
3x, 4x and 5x
= 1/30
Daily wage of Kavin = 5x/12x * 240=
Selvi alone complete 80% of work = Rs.100
80/100 * 30 = 24 days

7. Explanation
4. Explanation
Answer: A
Answer: D
Total work = 30 units
B alone complete the work = 5/4 * 12 =
A = 30/10 = 3 units
15 days
B = 30/15 = 2 units
C alone complete the work = 100/150 *
15 = 10 days Work done in 5 days = 4 * 5 = 20
A and B together can complete the 75% Remaining work = 30 – 20 = 10 units
of the work =
Time taken by B to complete the work =
(1/12 + 1/15) * 75/100 = 5 days 10/2 = 5 days
C alone complete 25% of the work = 10 *
25/100 = 2.5 days
8. Explanation
Required time = 5 + 2.5 = 7.5 days
Answer: A
Total work = 60

Page 175 Follow us: Official Site, Telegram, Facebook, Instagram, Instamojo
P’s efficiency = 60/10 = 6
Q’s efficiency = 60/12 = 5 10. Explanation
(P + Q) in 5 days = 5 * (6 + 5) = 55 Answer: A
Remaining work = 60 – 55 = 5 Time taken by X to complete work is =
16 × 5/4 => 20 days
Time taken by Q = 5/(40/100 * 12) =
50/48 = 1 1/24 days Time taken by Y to complete work is =
12 × 7/3 = 28 days
Time taken by Z to complete work is =
9. Explanation
20 × 9/5 = 36 days
Answer: D
Total number of work is = 1260 (LCM of
LCM of 20, 16 = 80 20, 28 and 36)
Chaitra done the work in one day = One day work of X = 1260/20 = 63 units
80/20 = 4 units
One day work of Y = 1260/28 = 45 units
Swetha done the work in one day =
One day work of Z = 1260/36 = 35 units
80/16 = 5 units
Work done in 5 days together = (63 + 45
Chaitra and swetha together done the
+ 35) × 5= 143 × 5 => 715 units
work in 12 days = 9 * 6 = 54
Remaining work is = 1260 – 715 =>545
Remaining work = 80 – 54 = 26 units
units
Chaitra alone done the work in 2 days =
Time taken by Y alone to complete
8 units
remaining work = 545/45 =>12 1/9
Remaining work = 26 – 8 = 18 units days
Anu alone complete 18 units of work in Thus, the total time taken is = 5 + 12 1/9
2 days => 17 1/9 days
Required time = (80/18) * 2 = 8(8/9)
days

Time, Speed & Distance


Formulas & Shortcuts of Time, Speed & Distance

1. Speed, Time and Distance:


Speed = Distance ,Time = Distance ,Distance = (Speed x Time).

Page 176 Follow us: Official Site, Telegram, Facebook, Instagram, Instamojo
Time Speed

2. km/hr to m/sec conversion:


5
x km/hr = x x m/sec.
18

3. m/sec to km/hr conversion:


18
x m/sec = x x km/hr.
5

4. If the ratio of the speeds of A and B is a : b, then the ratio of the


11
the times taken by then to cover the same distance is : or b : a.
ab

5. Suppose a man covers a certain distance at x km/hr and an equal distance at y km/hr.
Then,
2xy
the average speed during the whole journey is km/hr.
x+

Practice Questions
(Time, Speed & Distance Problem) respectively. At what distance from
Q1. The speed of two trains are in the Arun should Arun and Twinkle
ratio x : y. They are moving in the interchange their positions so that
opposite directions on parallel they meet exactly at the centre if they
tracks. The first train crossed a start from opposite ends?
telegraph pole in ‘a’ seconds where as A) 33.33m
the second train crossed the same B) 25m
telegraph pole in ‘b’ second. Find the C) 40 m
time taken by the train to cross each D) 27m
other completely. E) None of these
A)
B) Q3. Shatabdi express leaves lucknow
at 8 : 30 A.M. and expects to reach a
C) place 300 km from Lucknow at 12 :
D) 30 P.M. At 10 : 30 A.M. it was found
E) that it has covered only 40% of the
distance. By how much it has to
Q2. Arun and Twinkle can dig a increase the speed, in order to keep
trench 100 m long in 10 and 20 hours up his schedule?

Page 177 Follow us: Official Site, Telegram, Facebook, Instagram, Instamojo
A) 45 km/hr A) 380 km
B) 40 km/hr B) 320 km
C) 30 km/hr C) 240 km
D) 50 km/hr D) 330 km
E) None of these E) None of these

Q4. A person sets to cover a distance Q7. Two candles of same height are
of 12 km in 45 min. If he covers 3/4 of lighted at the same time. The first is
the distance in 2/3 rd time. What to be consumed in 5 hours and
should be his speed to cover the seconds in 4 hours. Assuming that
remaining distance in the remaining they burn at constant rates, in how
time? many hours their height will be in
A) 16 km/hr ratio 3 : 2 ?
B) 18 km/h
C) 12 km/h A)
D) 14 km/h
E) None of these B)
C)
Q5. From 2 places 60 km apart, A and
B start towards each other at the D)
same time and meet each other after
E)
12 hours. Had A travelled with 1/3 of
his speed and B travelled with 350% Q8. From 2 places 60 km apart, A and
of his speed, they would have met B start towards each other at the
after 7.5 hours. The speed of A is same time and meet each other after
A) 4 km/h
6 hours. Had A travelled with 2/3 of
B) 3 km/h his speed and B travelled with double
C) 1 km/h of his speed, they would have met
D) 2 km/h after 5 hours. The speed of A is
E) None of these
A) 4 km/h
B) 6 km/h
Q6. The distance between two C) 10 km/h
stations A and B is 900 km. A train D) 12 km/h
starts from A and moves towards B at E) None of these
an average speed of 30 km/hr.
Another train starts from B, 20 Q9. Ram and Shyam are travelling
minutes earlier than the train at A, from point A to B, which are 60 km
and moves towards A at an average apart. Travelling at a certain speed
speed of 40 km/hr. How far from A
ram takes one hour more than Shyam
will the two trains meet ?
to reach point B. If Ram doubles his
Page 178 Follow us: Official Site, Telegram, Facebook, Instagram, Instamojo
speed he will take 30 minutes less later than the second left point B, and
than Shyam to reach point B. At what it turned out on their meeting that he
speed was Ram driving from point A had travelled 12 km less than the
to B ? second tourist. After their meeting,
A) 15 kmph they kept cycling with the same
B) 35 kmph speed, and the first tourist arrived at
C) 30 kmph B 8 hours later and the second
D) 25 kmph arrived at A 9 hours later. Find the
E) 20 kmph speed of the faster tourist.
A) 4 km/h
Q10. Two Indian tourists in the US B) 6 km/h
cycled towards each other, one from C) 9 km/h
point A and the other from point B. D) 2 km/h
The first tourist left point A 6 hrs E) None of these

SOLUTION (1 to 10)

Q1. Ans(C)

Q2. Ans(A)

To meet at centre, they should interchange at half the time that they take together to
dig a trench.

Page 179 Follow us: Official Site, Telegram, Facebook, Instagram, Instamojo
Q3. Ans(C)

Q4. Ans(C)

Q5. Ans(B)

Q6. Ans(A)

Page 180 Follow us: Official Site, Telegram, Facebook, Instagram, Instamojo
Q7. Ans(A)

Q8. Ans(B)

Q9. Ans(D)

Page 181 Follow us: Official Site, Telegram, Facebook, Instagram, Instamojo
Q10. Ans(B)

From figure, clearly A is faster as it takes (t + 2) hrs. while B takes (t + 9) hrs.

Hence, speed of faster tourist is 6 kmph.

(Distance Problem) speed of 40 km/hr. How far from A


Q1. A 240 m long train takes 40 s will the two trains meet ?
longer to cross a platform twice its A) 380 km
length than the time it takes to cross B) 320 km
a pole at the same speed. What is the C) 240 km
speed of the train? D) 330 km
A) 6 m/s E) None of these
B) 24 m/s
C) 48 m/s Q3. Two trains were running in
D) 12 m/s opposite directions at the speed of 48
E) None of these kmph and 24 kmph respectively. If
the faster train passed a man sitting
Q2. The distance between two in the slower train in 9.5 seconds,
stations A and B is 900 km. A train what is the length of the faster train?
starts from A and moves towards B at A) 180m
an average speed of 30 km/hr. B) 190m
Another train starts from B, 20 C) 210m
minutes earlier than the train at A, D) 240m
and moves towards A at an average E) None of these
Page 182 Follow us: Official Site, Telegram, Facebook, Instagram, Instamojo
Q4. The length of train A is twice that Q7. Two candles of same height are
of train B and speed of train A is half lighted at the same time. The first is
of that of train B. If train A crosses a to be consumed in 5 hours and
man in 4 sec. then find how long will seconds in 4 hours. Assuming that
train B take to cross train A if they go they burn at constant rates, in how
in same direction. many hours their height will be in
A) 3s ratio 3 : 2 ?
B) 4s
C) 5s A)
D) 6s
E) None of these B)
C)
Q5. From 2 places 60 km apart, A and
B start towards each other at the D)
same time and meet each other after E)
12 hours. Had A travelled with 1/3 of
his speed and B travelled with 350% Q8. From 2 places 60 km apart, A and
of his speed, they would have met B start towards each other at the
after 7.5 hours. The speed of A is same time and meet each other after
A) 4 km/h
6 hours. Had A travelled with 2/3 of
B) 3 km/h
his speed and B travelled with double
C) 1 km/h of his speed, they would have met
D) 2 km/h after 5 hours. The speed of A is
E) None of these A) 4 km/h
B) 6 km/h
Q6. The distance between two C) 10 km/h
stations A and B is 900 km. A train D) 12 km/h
starts from A and moves towards B at E) None of these
an average speed of 30 km/hr.
Another train starts from B, 20
Q9. Ram and Shyam are travelling
minutes earlier than the train at A, from point A to B, which are 60 km
and moves towards A at an average apart. Travelling at a certain speed
speed of 40 km/hr. How far from A ram takes one hour more than Shyam
will the two trains meet ?
to reach point B. If Ram doubles his
A) 380 km speed he will take 30 minutes less
B) 320 km than Shyam to reach point B. At what
C) 240 km speed was Ram driving from point A
D) 330 km
to B ?
E) None of these
A) 15 kmph
Page 183 Follow us: Official Site, Telegram, Facebook, Instagram, Instamojo
B) 35 kmph second tourist. After their meeting,
C) 30 kmph they kept cycling with the same
D) 25 kmph speed, and the first tourist arrived at
E) 20 kmph B 8 hours later and the second
arrived at A 9 hours later. Find the
Q10. Two Indian tourists in the US speed of the faster tourist.
cycled towards each other, one from A) 4 km/h
point A and the other from point B. B) 6 km/h
The first tourist left point A 6 hrs C) 9 km/h
later than the second left point B, and D) 2 km/h
it turned out on their meeting that he E) None of these
had travelled 12 km less than the

SOLUTION (1 to 10)

Q1. Ans(D)

Q2. Ans(A)

Q3. Ans(B)

Page 184 Follow us: Official Site, Telegram, Facebook, Instagram, Instamojo
Q4. Ans(D)

Q5. Ans(B)

Q6. Ans(A)

Q7. Ans(A)

Page 185 Follow us: Official Site, Telegram, Facebook, Instagram, Instamojo
Q8. Ans(B)

Q9. Ans(D)

Q10. Ans(B)

From figure, clearly A is faster as it takes (t + 2) hrs. while B takes (t + 9) hrs.

Page 186 Follow us: Official Site, Telegram, Facebook, Instagram, Instamojo
Hence, speed of faster tourist is 6 kmph.

(Boat & Stream Problem) travel from B to C upstream. If the


Q1. P, Q and R are the three towns on distance between A and B is 4 km
a river which flows uniformly. Q is more than the distance between B
equidistant from P and R. I row from and C, what is the distance between A
P to Q and back in 10 hours and I can and B?
row downstream from P to R in 4 A) 112 km
hours. Compare the speed of my boat B) 140 km
in still water with that of the river. C) 56 km
A) 5 : 3 D) 84 km
B) 4 : 3 E) 28 km
C) 6 : 5
D) 7 : 3 Q4. A man can row 4 km/hr in still
E) None of these water and he finds that it takes him
twice as long to row up as to row
Q2. A river is flowing with a steady down the river. Find the rate of
speed of 4 km/h. One rows his boat stream.
downstream in the river and then A) 2.5 km/hr
returns by rowing upstream in the B) 2 km/hr
same river. When he returns to the C) 1 km/hr
starting point, the total distance D) 1.5 km/hr
covered by him is 42 km. If the return E) None of these
journey takes 2 h more than his
onward journey, then the speed of his Q5. A boat covered a certain distance
rowing in still water must be travelling downstream in 45 minutes,
A) 12 km/h while it came back to the starting
B) 10 km/h point in one hour 15 minutes. The
C) 9 km/h speed of the stream is 2 kmph. What
D) 8 km/h is the speed of the boat in still water?
E) None of these A) 8 kmph
B) 9 kmph
Q3. The speed of a boat in still water C) 10 kmph
is 24 kmph and the speed of the D) 11 kmph
stream is 4 kmph. The time taken by E) None of these
the boat to travel from A to B
downstream is 36 minutes less than Q6. A boat running downstream
the time taken by the same boat to covers a distance of 10 km in 2 hr.

Page 187 Follow us: Official Site, Telegram, Facebook, Instagram, Instamojo
While coming back stream the boat B) 4 hours
takes 5 h to cover the same distance. C) 6 hours
If the speed of the current is 1.5 D) 10 hours
km/hr, what is the speed of the boat E) 8 hours
in still water?
A) 5.125 km/h Q9. Speed of a boat is 25 km per hour
B) 5.125 km/h in still water and the speed of the
C) 5.3 km/h stream is 5 km per hour. If the boat
D) 4.3 km/h takes 15 hrs to go to a place and come
E) None of these back, the distance of the place is
A) 180 km
Q7. The ratio of speed of boat in still B) 160 km
water to the speed of stream is 5 : 3. A C) 120 km
boat takes total 12 hours to go 48 km D) 100 km
in upstream and same distance in E) 200 km
downstream. Find the speed of boat is
still water. Q10. Time taken by boat to cover 48
A) 12.5 km/hr km in upstream is 200% of the time
B) 10 km/hr taken by boat to cover 48 km in
C) 15 km/hr downstream. If sum of time taken by
D) 8 km/hr boat to cover 48 km distance in
E) 6 km/hr upstream and same distance in
downstream is 9 hours, then find
Q8. Sum of speed of boat in speed of boat?
downstream and in upstream is 24 A) 8 km/hr
km/hr while speed of boat in still B) 10 km/hr
water is 200% more than speed of C) 12 km/hr
stream, Time taken to cover 96 km D) 4 km/hr
downstream E) 6 km/hr
A) 2 hours

SOLUTION (1 to 10)

Q1.Ans(A)

Page 188 Follow us: Official Site, Telegram, Facebook, Instagram, Instamojo
Q2. Ans(B)

Q3. Ans(C)

Q4. Ans(E)

Page 189 Follow us: Official Site, Telegram, Facebook, Instagram, Instamojo
Q5. Ans(A)

Q6. Ans(B)

Q7. Ans(A)

Page 190 Follow us: Official Site, Telegram, Facebook, Instagram, Instamojo
Q8. Ans(C)

Required time = 96/16 = 6 hours

Q9. Ans(A)

Let total distance be D km

D/25 + 5 + D/25 – 5 = 15

D/30 + D/20 = 15

2D + 3D = 900

D = 180 km

Q10. Ans(C)

Page 191 Follow us: Official Site, Telegram, Facebook, Instagram, Instamojo
Mensuration
Formulas & Shortcuts of Mensuration
Geometrical Area Formulas Variables
Shapes
Area of Square Area = a x a = a2 a = Length of each side of the square
Area of Rectangle Area = l x b l = Length of the rectangle
b = Breadth of the rectangle
Area of Triangle Area = (b x h)/2 b = Length of the base of the triangle
h = Height of the triangle
Area of Circle πr2 π = 22/7 = 3.14159 (approx)
r = Radius of the circle
Area of Area = b x h b = Base of the parallelogram
Parallelogram h = Height of the parallelogram
Area of Trapezium Area = {(a + b) x a + b= Sum of the lengths of the two parallel sides of the
h}/2 trapezoid, i.e. Base 1 and Base 2
h = Perpendicular distance between the two parallel sides
Area of Rhombus Area = (p x q)/2 p = Length of 1st diagonal
q = Length of second diagonal
pq = Product of the two diagonals

Page 192 Follow us: Official Site, Telegram, Facebook, Instagram, Instamojo
Area of Ellipse Area = πab a = Major radius
b = Minor radius

Perimeter Formulas Of Common Geometrical Figures

Geometrical Shapes Perimeter Formulas Variables


Perimeter of Square Perimeter = 4a a = Length of each side of the square
Perimeter of Rectangle Perimeter = 2 (l + b) l = Length of the rectangle
b = Breadth of the rectangle
Perimeter of Triangle Perimeter = a + b + c a, b and c are the lengths of the three sides of the
triangle
Perimeter of Circle Perimeter = πd = 2πr π = 22/7 = 3.14159 (approx)
r = Radius of the circle
d = Diameter of the circle
Perimeter of Perimeter = 2 (l + b) l = Length of the parallelogram
Parallelogram b = Breadth of the parallelogram
Perimeter of Trapezium Perimeter = a + b + c a, b, c, d are the lengths of the four sides of the
+d trapezoid
Perimeter of Rhombus Perimeter = 4a a = Length of each side of the rhombus
Perimeter of Kite Perimeter= 2a + 2b a = Length of each side of the first pair
b = Length of each side of the second pair

Additional Area And Parameter Formulas

Area of a Triangle of Given Area = √ [s (s – a) (s – s = Semi-perimeter of the triangle = (a + b


Sides – a, b, c b) (s – c)] + c)/2
Area of an Isosceles Triangle Area = (base x a = Length of each of the equal sides of the
height)/2 isosceles triangle
base = b b = Length of the base
height = √(a2 − b2/4)
Perimeter of an Isosceles Perimeter = 2a + b a = Length of each of the equal sides of the
Triangle isosceles triangle
b = Length of the base
Area of an Equilateral Area = (√3 x a2)/4 a = Length of each side of the equilateral
Triangle triangle
Perimeter of an Equilateral Perimeter = 3a a = Length of each side of the equilateral
Triangle triangle

Page 193 Follow us: Official Site, Telegram, Facebook, Instagram, Instamojo
Perimeter of a Semi-Circle Perimeter = πr + d = r = Radius of the circle
3πr d = 2r = Diameter of the circle
Area of a Semi-Circle Area = (πr2) x (1/2) r = Radius of the circle

Practice Questions

SET-1

1).The total area of a Square and a. 9856


rectangle is 496 Sq cm. The side of the b. 10548
square is 14 cm. What is the sum of c. 12563
the perimeter of square and d. 11907
rectangle, if length of the rectangle is e. None of these
25 cm?
4) The circumference of a circle is
a. 120 cm equal to the perimeter of a square
b. 140 cm whose area is 4356 sq cm. What is
c. 130 cm the area of the circle?
d. 160 cm
e. None of these a. 4678 Sq cm
b. 5544 Sq cm
2) The perimeter of the rectangular c. 5126 Sq cm
field is 2 times the perimeter of d. 4282 Sq cm
square field. If the side of the e. None of these
square field is 18 m and the
breadth of the rectangular field is 5) Find the volume of a cylinder
45 m, then find the area of the whose radius is one – third of the
rectangular field? radius of a circle having area 1386
cm2. Height of the cylinder is
a. 1215 Sq m double its radius?
b. 1336 Sq m
c. 1050 Sq m a. 2584 cm3
d. 1152 Sq m b. 3562 cm3
e. None of these c. 2156 cm3
d. 3245 cm3
3) The cost of fencing a square plot at e. None of these
the rate of Rs. 15 per metre is Rs.
3780. What will be the cost of 6) The side of the equilateral triangle
flooring the plot at the rate of Rs. 3 is equal to the diameter of the
per square meter? circle. The area of the equilateral
Page 194 Follow us: Official Site, Telegram, Facebook, Instagram, Instamojo
triangle is 196√3 Sq cm. Find the a. 7680 Sq m
circumference of the circle? b. 5572 Sq m
c. 6128 Sq m
a. 92 cm d. 7054 Sq m
b. 88 cm e. None of these
c. 74 cm
d. 66 cm 9) The radius of the circle is equal to
e. None of these two-fifth of the side of the square.
The area of the circle is 616 sq cm.
7) If the breadth of a parallelogram is Find the perimeter of a square?
increased by 30% while the height
of the parallelogram is decreased a. 152 cm
by 20% then find percentage b. 166 cm
change in area of the c. 140 cm
parallelogram? d. 178 cm
e. None of these
a. 8 % increased
b. 5 % decreased 10) A Circular path is surrounding
c. 9 % decreased the circular plot is being graveled
d. 4 % increased at a total cost of Rs. 3080 at Rs. 4
e. None of these per square meter. Find the width
of the path, if the radius of the
8) The circumference of a circle is circle is 14 m?
half of the perimeter of a
rectangle. The area of the circle is a. 12 meter
2464 sq. m. What is the area of the b. 7 meter
rectangle if the breadth of the c. 15 meter
rectangle is 80 m? d. 20 meter
e. None of these

SOLUTION (1 to 10)

1.Answer c 25b = 496 – 196

a2 + lb = 496 25b = 300

142 + 25b = 496 B = 12 cm

196 + 25b = 496 Perimeter of square = 4a = 4*14 = 56 cm

Page 195 Follow us: Official Site, Telegram, Facebook, Instagram, Instamojo
Perimeter of rectangle = 2*(l + b) = The circumference of a circle =
2*(25 + 12) Perimeter of a Square

= > 2 * 37 = 74 cm Area of Square = a2 = 4356

Required sum = 56 + 74 = 130 cm Side (a) = 66 cm

Perimeter of square = 4a = 66*4 = 264


cm
2.Answer a
According to the question,
Side of square (a) = 18 m
2πr = 264
Perimeter of rectangle = 2*4a = 2*4*18 =
144 2*(22/7)*r = 264

= > 2(l + 45) = 144 r = 42

= > l + 45 = 72 Area of the circle= πr2 = (22/7)*42*42 =


5544 Sq cm
= > l = 72 – 45 = 27 m

Area of rectangle = lb = 45*27 = 1215 Sq


m. 5.Answer c

πr2 = 1386

3.Answer d => (22/7)* r2 = 1386

Perimeter of square = 3780/15 = 252 => r2 = 1386 x (7/22)

4a = 252 => r2 = 441

Side (a) = 252/4 = 63 => r = 21 cm

Area of square = a2 = 632 = 3969 Radius of the cylinder = 21*(1/3) = 7 cm

Cost of flooring the plot = 3979*3 = Rs. Height of the cylinder = 7*2 = 14 cm
11907
Volume of the cylinder = πr2h =
4.Answer b (22/7)*7*7*14 = 2156 cm3
Page 196 Follow us: Official Site, Telegram, Facebook, Instagram, Instamojo
6.Answer b Area of a circle= πr2

The area of the equilateral triangle = 2464 = 22r2/7


196√3 Sq cm
2464*(7/22) = r2
The area of the equilateral triangle =
(√3/4)*a2 r2 = 784

(√3/4)*a2 = 196√3 r = 28 m

a2 = 196*4 Circumference = 2*22/7 * 28 = 176 sq.


m
Side (a) = 14*2 = 28 cm
Perimeter of the rectangle = 2*176 =
The diameter of the circle = 28 cm 352 sq. m

Radius (r) = 14 cm 352 = 2(l + 80)

Circumference of the circle = 2πr = 176 = l + 80


2*(22/7)*14 = 88 cm
l = 176 – 80
7.Answer d
l = 96
Let the breadth and height of the
parallelogram is 10 cm and 10cm, Area of the rectangle = 96*80 = 7680 sq.
m
Normal area= 10*10= 100
9.Answer c
New length= 10*130/100= 13
Radius = (2/5)*side
New height= 10*80/100= 8
Area of circle = πr 2 = 616
New area= 13*8= 104
(22/7)*r2 = 616
Required percentage = [(104 –
100)/100]*100 = 4 % increased r2 = 616*(7/22) = 196

Radius (r) = 14 cm

8.Answer a Side = Radius*(5/2) = 14*(5/2) = 35 cm

Page 197 Follow us: Official Site, Telegram, Facebook, Instagram, Instamojo
Perimeter of the square = 4a = > (22/7) [(14+x)2 – 142]

= > 35*4 = > (22/7) [196 + 28x + x2 – 196]

= > 140 cm = > (22/7) [28x + x2]

(22/7) [28x + x2] = 770

10.Answer b 28x + x2 = 770*(7/22)

Radius of the circular plot = 14 m 28x + x2 = 245

Area of the circular path = 3080/4 = 770 X2 + 28x – 245 = 0

Area of the path = π(r + x)2 – πr2 (Here x (x+35) (x – 7) =0


is the width of the path)
X = 7 meter

SET- 2
(Mensuration 2D 3D Based If the side of the square field
Questions) is 18 m and the breadth of the
rectangular field is 45 m,
1. The total area of a Square and then find the area of the
rectangle is 496 Sq cm. The rectangular field?
side of the square is 14 cm.
What is the sum of the a. 1215 Sq m
perimeter of square and b. 1336 Sq m
rectangle, if length of the c. 1050 Sq m
rectangle is 25 cm? d. 1152 Sq m
e. None of these
a. 120 cm
b. 140 cm 3. The cost of fencing a square
c. 130 cm plot at the rate of Rs. 15 per
d. 160 cm metre is Rs. 3780. What will
e. None of these be the cost of flooring the plot
at the rate of Rs. 3 per square
2. The perimeter of the meter?
rectangular field is 2 times
the perimeter of square field. a. 9856

Page 198 Follow us: Official Site, Telegram, Facebook, Instagram, Instamojo
b. 10548 e. None of these
c. 12563
d. 11907 7. If the breadth of a
e. None of these parallelogram is increased by
30% while the height of the
4. The circumference of a circle parallelogram is decreased
is equal to the perimeter of a by 20% then find percentage
square whose area is 4356 sq change in area of the
cm. What is the area of the parallelogram?
circle?
a. 8 % increased
a. 4678 Sq cm b. 5 % decreased
b. 5544 Sq cm c. 9 % decreased
c. 5126 Sq cm d. 4 % increased
d. 4282 Sq cm e. None of these
e. None of these
8. The circumference of a circle
5. Find the volume of a cylinder is half of the perimeter of a
whose radius is one – third of rectangle. The area of the
the radius of a circle having circle is 2464 sq. m. What is
area 1386 cm2. Height of the the area of the rectangle if the
cylinder is double its radius? breadth of the rectangle is 80
m?
a. 2584 cm3
b. 3562 cm3 a. 7680 Sq m
c. 2156 cm3 b. 5572 Sq m
d. 3245 cm3 c. 6128 Sq m
e. None of these d. 7054 Sq m
e. None of these
6. The side of the equilateral
triangle is equal to the 9. The radius of the circle is
diameter of the circle. The equal to two-fifth of the side
area of the equilateral of the square. The area of the
triangle is 196√3 Sq cm. Find circle is 616 sq cm. Find the
the circumference of the perimeter of a square?
circle?
a. 152 cm
a. 92 cm b. 166 cm
b. 88 cm c. 140 cm
c. 74 cm d. 178 cm
d. 66 cm e. None of these
Page 199 Follow us: Official Site, Telegram, Facebook, Instagram, Instamojo
10. A Circular path is a. 12 meter
surrounding the circular plot b. 7 meter
is being graveled at a total c. 15 meter
cost of Rs. 3080 at Rs. 4 per d. 20 meter
square meter. Find the width e. None of these
of the path, if the radius of
the circle is 14 m?
Answers: 1-10
1.Answer c 4.Answer b
a2 + lb = 496 The circumference of a circle =
142 + 25b = 496 Perimeter of a Square
196 + 25b = 496 Area of Square = a2 = 4356
25b = 496 – 196 Side (a) = 66 cm
25b = 300 Perimeter of square = 4a = 66*4 = 264
B = 12 cm cm
Perimeter of square = 4a = 4*14 = 56 cm According to the question,
Perimeter of rectangle = 2*(l + b) = 2πr = 264
2*(25 + 12) 2*(22/7)*r = 264
= > 2 * 37 = 74 cm r = 42
Required sum = 56 + 74 = 130 cm Area of the circle= πr2 = (22/7)*42*42 =
5544 Sq cm
2.Answer a
Side of square (a) = 18 m 5.Answer c
Perimeter of rectangle = 2*4a = 2*4*18 = πr2 = 1386
144 => (22/7)* r2 = 1386
= > 2(l + 45) = 144 => r2 = 1386 x (7/22)
= > l + 45 = 72 => r2 = 441
= > l = 72 – 45 = 27 m => r = 21 cm
Area of rectangle = lb = 45*27 = 1215 Sq Radius of the cylinder = 21*(1/3) = 7 cm
m. Height of the cylinder = 7*2 = 14 cm
Volume of the cylinder = πr2h =
3.Answer d (22/7)*7*7*14 = 2156 cm3
Perimeter of square = 3780/15 = 252
4a = 252 6.Answer b
Side (a) = 252/4 = 63 The area of the equilateral triangle =
Area of square = a2 = 632 = 3969 196√3 Sq cm
Cost of flooring the plot = 3979*3 = Rs. The area of the equilateral triangle =
11907 (√3/4)*a2
(√3/4)*a2 = 196√3

Page 200 Follow us: Official Site, Telegram, Facebook, Instagram, Instamojo
a2 = 196*4 l = 96
Side (a) = 14*2 = 28 cm Area of the rectangle = 96*80 = 7680 sq.
The diameter of the circle = 28 cm m
Radius (r) = 14 cm
Circumference of the circle = 2πr = 9.Answer c
2*(22/7)*14 = 88 cm Radius = (2/5)*side
Area of circle = πr 2 = 616
7.Answer d (22/7)*r2 = 616
Let the breadth and height of the r2 = 616*(7/22) = 196
parallelogram is 10 cm and 10cm, Radius (r) = 14 cm
Normal area= 10*10= 100 Side = Radius*(5/2) = 14*(5/2) = 35 cm
New length= 10*130/100= 13 Perimeter of the square = 4a
New height= 10*80/100= 8 = > 35*4
New area= 13*8= 104 = > 140 cm
Required percentage = [(104 –
100)/100]*100 = 4 % increased 10.Answer b
Radius of the circular plot = 14 m
8.Answer a Area of the circular path = 3080/4 = 770
Area of a circle= πr2 Area of the path = π(r + x)2 – πr2 (Here x
2464 = 22r2/7 is the width of the path)
2464*(7/22) = r2 = > (22/7) [(14+x)2 – 142]
r2 = 784 = > (22/7) [196 + 28x + x2 – 196]
r = 28 m = > (22/7) [28x + x2]
Circumference = 2*22/7 * 28 = 176 sq. (22/7) [28x + x2] = 770
m 28x + x2 = 770*(7/22)
Perimeter of the rectangle = 2*176 = 28x + x2 = 245
352 sq. m X2 + 28x – 245 = 0
352 = 2(l + 80) (x+35) (x – 7) =0
176 = l + 80 X = 7 meter
l = 176 – 80

Data Sufficiency (Quant)


Steps to Solve Data Sufficiency

Page 201 Follow us: Official Site, Telegram, Facebook, Instagram, Instamojo
1. It is better that you read the data carefully and comprehend it. One of the most important
things you need to know in data sufficiency is that never assume any extra information that is
not given in the question.
2. Now, combine the data that is available to you in the question and see whether you can arrive
at any solution or not. It is not necessary to solve the question, just get a hint whether the
question is solvable or not!
3. If you are not able to get the solution from statement 1 then move on to the statement 2 and
check the same for statement 2. Remember not to use the data available in statement 1.
4. If you are not able to get any solution from either of the statements that combine the data
available in both the statements and try and find the solution.
5. Choose the correct answer.

Practice Questions
SET-1

Directions (1-4): In each of the 2. What is the length of the train?


following questions, a question is Statement I: If train can cross a
followed by three statements platform in 35 second and a pole in
numbered I, II and III. Read all the 15 sec.
statements and answer accordingly. Statement II: If the speed of train is
1. What is the principal amount? 70 kmph
Statement I: The difference between Statement III: If the length of
Compound Interest and Simple platform is 400 m
Interest for 2 years is 300 A) I and III together
Statement II: If the Compound B) I alone
Interest for 3 years at the rate of C) I and II together
10% is 3310 D) II and III together
Statement III: If the sum becomes E) Even all I, II, and III are not
double in 32 years at Compound sufficient
Interest?
A) Only I Answer
B) Only II Option C
C) II and III together Solution:
D) Only I or II From I and II: speed = 70 km/hr and
E) II and I or III crosses the pole in 15 sec, so length
= speed*time
Answer
Option B 3. What will be the increase percent in
Solution: volume?
From II, using the formula of CI, Statement I: An ice cube is dip into
amount can be found. 10 litre water
Statement II: If radius of a cylinder
Page 202 Follow us: Official Site, Telegram, Facebook, Instagram, Instamojo
increases by 20% and height A) I and II Together
increases by 10% B) II and III Together
Statement III: If the height of ice C) Only III
cube is 10 cm. D) All I, II and III Together
A) All I, II and III E) None of these
B) II alone and I and III together
C) I and II together Answer
D) II and III alone Option C
E) II alone Solution:
Using III.
Answer Gain=20%=1/5
Option B CP : SP=5:6— (i)
Solution: Discount=20%=1/5
II: MP:SP=5:4 —-(ii)
Volume = π*r*r*h hence
So by successive formula, the CP:SP:MP= 10:12:15
increase in volume can be found hence (15-10)/10*100
First: r*r
So 20 + 20 + (20)(20)/100 = 44% Directions (5-10): In each of the
Next: 44 + 10 + (44)*(10)/100 – following questions, a question is
final increase in volume followed by two statements
From I and III: numbered I and II. Read both the
volume of given shape (cylinder, statements and answer accordingly.
cube, cone etc) = 10 l (initial 5. What is the ratio of milk and water
volume). And ice cube of height 10 in the mixture?
cm is dip into it. Statement I: If a trader earns a
So after finding new volume or profit of 20%after adding water in
increase in volume after the ice cube milk and sold it at Cost Price.
is dipped, % increase in volume can Statement II: If a trader earns some
be found. profit after selling milk at Rs
25/Litre by adding water in it.
4. Find the increase percent in Cost A) Only I
Price to make Marked Price if B) Only II
Statement I: A trader gain 25% on C) Either I or II
Cost Price D) Neither I nor II
Statement II: A trader gain 20% on E) Both I and II
Selling Price
Statement III: A trader gain 20% on Answer
Cost Price after giving a discount of
20% on Marked Price
Page 203 Follow us: Official Site, Telegram, Facebook, Instagram, Instamojo
Option A Answer
Solution: From I. Option D
Water:Milk=20:100 Solution:
We cannot find the age of teacher
6. What is the age of B? with the given data.
Statement I: The ratio of present
age of A and B is 3:5 and 4 years 8. What of the distance travelled?
before A was 8 years younger than B Statement I: If a person covers a
Statement II: 3 years ago the ratio distance in 12 hour, covering the
of ages of A and B was 4:7 and 3 first half at 20 kmph and the second
years hence the ratio will become half at 30 kmph
2:3 Statement II: If a person covers a
A) Only I distance at 30 kmph. If her decreases
B) Only II his speed to 25 kmph he takes two
C) Either I or II more hour to reach the destination
D) Neither I nor II A) Only I
E) Both I and II B) Only II
C) Either I or II
Answer D) Neither I nor II
Option C E) Both I and II
Solution:
From I: Answer
A/B = 3/5 Option C
(A-4) = (B-4) – 8 or A = B – 8 Solution:
From II also, we get 2 equations. Using I. Find the avg speed =
2*x*y/(x+y) multiply it by 12 to get
7. What is the age of teacher? distance
Statement I: In a class of 20 Using II. Time =D/S=> 2= x/25 –
students the average age of students x/30 where x is the distance
is 21 and it increases to x if the age
of teacher is included. 9. What will be the profit of A?
Statement II: In a class of 10 Statement I: A and B invested Rs
students the average age of students 6000 and Rs 7000 respectively. A
and teacher is 22 years increases his investment by 50%
A) Only I after 6 months and their annual
B) Only II profit is Rs 3800
C) Either I or II Statement II: A and B invested Rs
D) Neither I nor II 10,000 and Rs 12,000 respectively .
E) Both I and II If B doubles his investment after
some months and their annual profit
Page 204 Follow us: Official Site, Telegram, Facebook, Instagram, Instamojo
is Rs 2,000 Statement I: If A and B do a work
A) Only I together for 10 days and B left the
B) Only II work and A do the remaining work
C) Either I or II in 20 days.
D) Neither I nor II Statement II: If A and B can do the
E) Both I and II work in 15 days
A) Only I
Answer B) Only II
Option A C) Either I or II
Solution: D) Neither I nor II
In II. definite time is not known as E) Both I and II
we cannot get the ratio in which
profit is divided. Answer
In I. A=6000*6 + 9000*6=54000 Option E
B=7000*12=84, we can find the Solution:
profit now From I. 30/A + 10/B=1
From II. 1/A + 1/B=1/15. We can get
10. Find in how many days A alone A and B using both
can do the work?

SET-2
1. What is marked price of the the StatementI – Ratio of length and
product? breadth of the room is 9:5 and
StatementI – After giving the 10% peremeter of the floor is 140 m.
discount on mark price the shopkeeper Statement II – Price of 100 tiles is 500.
make a profit of 26% by selling the item. And total expenditure to fit the floor is
Ratio of cost price and mark price is 5:7. 12000.
Statement II – Difference of mark price A. Only Statement I alone.
and cost price is 40. Difference of selling B.Only Statement II alone.
price and marked price is 14. Marked C. Both Statements I and II together.
price is 40% of the cost price. D. Neither Statement I nor II is sufficient.
A. Only Statement I alone. E. Either Statement I or II.
B.Only Statement II alone.
C. Both Statements I and II together. 3. Lina buy braslet. Find the iner
D. Neither Statement I nor II is sufficient. radius of braslet?
E. Either Statement I or II. StatementI – Braslet is made with
269.5sqm metel.
2. How many tiles is needed to fit tiles Statement II – length of outer diameter
in the rectangular floor of a room? is 28m.
Page 205 Follow us: Official Site, Telegram, Facebook, Instagram, Instamojo
A. Only Statement I alone. C. Both Statements I and II together.
B.Only Statement II alone. D. Neither Statement I nor II is sufficient.
C. Both Statements I and II together. E. Either Statement I or II.
D. Neither Statement I nor II is sufficient.
E. Either Statement I or II. 7. Find the present age of B?
StatementI – Age A 2years ago equal to
4. Find the length of the train P? the age of C 4 years ago. Age of C is
StatementI – The train P cross the train average age of A and B.
Q in 6 sec when train Q is not moving. Statement II – ratio of present age of B
Train Q cross train R in 68 sec when and age of C after 2 years is 1:1. A is 2
both are moving same direction, speed years younger than C.
of train R is 25m/sec. A. Only Statement I alone.
Statement II – Train P cross train R in B.Only Statement II alone.
64 sec when both are moving same C. Both Statements I and II together.
direction. Length of train Q is 140m. D. Neither Statement I nor II is sufficient.
A. Only Statement I alone. E. Either Statement I or II.
B.Only Statement II alone.
C. Both Statements I and II together. 8. There is three number, first,
D. Neither Statement I nor II is sufficient. second and third. What is the 45% of
E. Either Statement I or II. first number?
StatementI – 6/11 of the first number is
5. How many days A, B and C can equal to the 22% percent of second
complete the work together? number.second number is equal to the
StatementI – Ratio of effiecency of A ¼ of third number.
and B is 2:1. B and C can complte the Statement II – First number is 92 more
work in 12 days. than the ¼ of the second number.
Statement II – Ratio of effiecency of A Difference of first and second number is
and C is 3:1. C can alone complte the 458.
work in 30 days. A. Only Statement I alone.
A. Only Statement I alone. B.Only Statement II alone.
B.Only Statement II alone. C. Both Statements I and II together.
C. Both Statements I and II together. D. Neither Statement I nor II is sufficient.
D. Neither Statement I nor II is sufficient. E. Either Statement I or II.
E. Either Statement I or II.
9. In a company there are two type of
6. Is A is an even number? employee in a company either 25
StatementI – 6A+3B is even number. year or more than 25 year. Find how
Statement II – 3A+6B is even number. many male are 25 year old?
A. Only Statement I alone. StatementI – There is 45 % male
B.Only Statement II alone. employee in the company. Out of the
Page 206 Follow us: Official Site, Telegram, Facebook, Instagram, Instamojo
total female employee 60% is 25 years
age. 12. What are maximum marks of the
Statement II – Total employee of the examination?
company is 4800. Out of that 40% is StatementI – Avearge marks of three
above 25 years age. students is 643. First student get 56%
A. Only Statement I alone. second student gets 634 marks.
B.Only Statement II alone. Statement II – Third students gets 92%
C. Both Statements I and II together. marks in the exam which is 171 marks
D. Neither Statement I nor II is sufficient. less than Second student. Second
E. Either Statement I or II. student gets 144 more marks than First
student.
10. Find the rate of interest? A. Only Statement I alone.
StatementI – The simple interest B.Only Statement II alone.
accrued of Rs.22500 at the end of four C. Both Statements I and II together.
years is Rs 10800. D. Neither Statement I nor II is sufficient.
Statement II – The compound E. Either Statement I or II
interesest accrued on the Rs.22500 at
certain time at certain rate is Rs.5724 13. What is the present age of D?
A. Only Statement I alone. StatementI – Ratio between present age
B.Only Statement II alone. of M and D is 5:13. M is 9 year younger
C. Both Statements I and II together. than P.
D. Neither Statement I nor II is sufficient. Statement II – P’s age after 9 year will
E. Either Statement I or II. be 33 years. The difference between D’s
age and M’s age is same as the present
11. What was annual income of M? age of P.
Statement I – R’s monthly income is A. Only Statement I alone.
three times V’s monthly income.V’c B.Only Statement II alone.
monthly income is 15% more than C’s C. Both Statements I and II together.
monthly income. C’s monthly income is D. Neither Statement I nor II is sufficient.
32000. E. Either Statement I or II.
Statement II – V’s monthly income is Rs
4800 more than the monthly income of 14. What is the perimeter of the
C’s. Total income of V and C is Rs.68800. rectangle?
V’s monthly income is 1/3 of M’s StatementI – Area of the rectangle is
monthly income. seventh fourth area of square whose
A. Only Statement I alone. side is 7 more than the breadth of
B.Only Statement II alone. Rectangle .
C. Both Statements I and II together. Statement II – Breadth of rectangle
D. Neither Statement I nor II is sufficient. equal to the radius of circle whose
E. Either Statement I or II.
Page 207 Follow us: Official Site, Telegram, Facebook, Instagram, Instamojo
perimeter is 176cm and ratio of length Statement II – The selling price of the
and bradth of rectangle is 9:7. car is double of cost price
A. Only Statement I alone. A. Only Statement I alone.
B.Only Statement II alone. B.Only Statement II alone.
C. Both Statements I and II together. C. Both Statements I and II together.
D. Neither Statement I nor II is sufficient. D. Neither Statement I nor II is sufficient.
E. Either Statement I or II. E. Either Statement I or II.

15. If taps A, B and C are open 18. How many students from AEC
simultaneously, how long will it take College got placement?
to fill the tank completely where C is StatementI – 75% student from the XYZ
out let pipe? college got placement. Number of
StatementI – When pipe A and B is open student who got placement from AEC is
together the tank fill in 2.4 hr. 120% percent of number of students
Statement II – When pipe c is open it who got placement from college XYZ.
can empity the full tank in 3 hr. Statement II – Ratio of Number of
A. Only Statement I alone. students of college AEC and XYZ is 4:3.
B.Only Statement II alone. Difference of number of student in this
C. Both Statements I and II together. college is 400.
D. Neither Statement I nor II is sufficient. A. Only Statement I alone.
E. Either Statement I or II. B.Only Statement II alone.
C. Both Statements I and II together.
16. What is M’s share of profit? D. Neither Statement I nor II is sufficient.
StatementI – M started the business E. Either Statement I or II.
with investment Rs.29500. S joined with
him after 4 months. 19. What is average of a,b,c,d and e?
Statement II – S investment 33500 and StatementI – a+e=50, c:d=6:7, average
after 3 month he spend 1500 more. of a and c is equal to b.
Where M invest 3000 after 6 month Statement II – d+e=58 and b+d=50.
from the beginning. b:e=11:15.
A. Only Statement I alone. A. Only Statement I alone.
B.Only Statement II alone. B.Only Statement II alone.
C. Both Statements I and II together. C. Both Statements I and II together.
D. Neither Statement I nor II is sufficient. D. Neither Statement I nor II is sufficient.
E. Either Statement I or II. E. Either Statement I or II.

17. What is the percent profit 20. What is the ratio of length and
earened byselling a car? breadth of recangle?
StatementI – The amount of profit StatementI – area of a rectangle is twice
earned on selling the car was 320000. area of circle. Radius of a circle is equal
Page 208 Follow us: Official Site, Telegram, Facebook, Instagram, Instamojo
to the side of a square whose area is A. Only Statement I alone.
196sqm. B.Only Statement II alone.
Statement II – perimeter of the C. Both Statements I and II together.
rectangle is 28m more than the D. Neither Statement I nor II is sufficient.
circumference of the circle. E. Either Statement I or II.
ANSWER 1-20:
Q1) Answer e From Statement I we cannot say about A
From Statement I we can calculate the is even or not because we donot know B
marked price. is even or not. But Statement II we can
From Statement II we can calculate the say A is even because 6B is always even
marked price. and 3A have to be even.
Either Statement I or II is sufficient.
Q7) Answer d
Q2) Answer d From both Statements I and II we cannot
From both statements we can not calculate the exact age of any one. So we
calculate the area of each tiles so we cannot calculate the age of B.
cannot calculate the numbers of tiles
required. Q8) Answer b
Statement I we cannot calculate the frist
Q3) Answer c number but from
To calculate the iner radius we have to Statement II we can calculate the first
use both Statement I and Statement II. number from the realtion between ffirst
and second number.
Q4) Answer D
From two statements we can not Q9) Answer C
calculate the length of train R, speed of Statement I we canculate the total male
train train Q so we cannot able to know employee and total female employee
the length of train.because lots unknown who are 25 years age in from of
value. percentage. And with the help Statement
II we can calculate the exact value.
Q5) Answer c
From both statements we can calculate Q10) Answer a
the efficiency of A,B and C. And from From statement I we cancaulate the rate
statements 2 we know the days of C to because we knoe the principle, time and
compllte the work. From that we can interest, but Statement II we donot
easily calculate total days to complte the know the time so we cannot calculate
work by A,B and C. the the rate.

Q6) Answer B Q11) Answer c

Page 209 Follow us: Official Site, Telegram, Facebook, Instagram, Instamojo
From Statement I we can calculate the
monthly income C,V and R but we don,t Q16) Answer c
know about the montly income of M but From both Statements I and II we
with help Statement II we can calculate calculate the investment of each one but
monthly income M and from that annual we don’t the total profit so we cannot
income. calculate the share of profit of M.

Q12) Answer c Q17) Answer b


From each Statement I or II. We cannot From Statement II we can easily get the
calculate the maximum marks becuse profit percentage.
we don’t know the any one percentage
and marks both. But with both Q18) Answer c
Statement I and II we get marks third From Statement I we get the percentage
student and also gets third student of number of studentgot placement but
percentage, from that we can calculate we don’t know exact number because
the maximum marks.. number of student is unknown from
Statement II we get the number of
Q13) Answer c student , and from that we cancaulte the
To calculate the age of D we need to both number of student gets placement.
statements.
Q19) Answer c
Q14) Answer b From both statements we can calculate
From Statement II we can calculate the the exact value and from that we
length and breadth of rectangle and canculate average.
from that perimeter can easily calculate.
Q20) Answer c
Q15) Answer c If we calculate the the ratio of length and
Each Statement I or II we can get the breadth than we have to use both
answer but if we use both Statement I Statements.
and II than we can calculate the answer.

Permutation & Combination


Formulas & Shortcuts of Permutation & Combination

Page 210 Follow us: Official Site, Telegram, Facebook, Instagram, Instamojo
Formulas

There are many formulas involved in permutation and combination concept. The two key formulas
are:

Permutation Formula

A permutation is the choice of r things from a set of n things without replacement and where the order
matters.

nPr = (n!) / (n-r)!

Combination Formula

A combination is the choice of r things from a set of n things without replacement and where order
does not matter.

Practice Questions

SET-1
Q1. In how many different ways can E) None of these
the letters of the word “CHARGES” be Q3. In how many different ways can
arranged in such a way that the the letters of the word “CANDIDATE”
vowels always come together? be arranged in such a way that the
A) 1440 vowels always come together?
B) 720 A) 4320
C) 360 B) 1440
D) 240 C) 720
E) None of these D) 840
Q2. In how many different ways can E) None of these
the letters of the word “COMPLAINT” Q4. In how many different ways can
be arranged in such a way that the the letters of the word “RADIUS” be
vowels occupy only the odd arranged in such a way that the
positions? vowels occupy only the odd
A) 14400 positions?
B) 43200 A) 72
C) 1440 B) 144
D) 5420 C) 532
Page 211 Follow us: Official Site, Telegram, Facebook, Instagram, Instamojo
D) 36 D) 21400
E) None of these E) None of these
Q5. In how many different ways can Q8. In how many different ways can
the letters of the word ‘LEADING’ be the letters of the word ‘SERVING’ be
arranged in such a way that the arranged?
vowels always come together? A) 5040
A) 360 B) 720
B) 480 C) 120
C) 720 D) 40320
D) 5040 E) None of these
E) None of these Q9. In how many different ways can
Q6. In how many different ways can any 4 letters of the word ‘WORKING’
the letters of the word be arranged?
‘CORPORATION’ be arranged so that A) 5040
the vowels always come together? B) 840
A) 810 C) 24
B) 1440 D) 120
C) 2880 E) None of these
D) 50400 Q10. In how many different ways can
E) 5760 the letters of the word
Q7. Out of 7 consonants and 4 ‘ARRANGEMENT’ be arranged?
vowels, how many words of 3 A) 2494800
consonants and 2 vowels can be B) 4989600
formed? C) 831600
A) 210 D) Can’t be determined
B) 1050 E) None of these
C) 25200

SOLUTION (1 to 10)

Q1. Ans(A)

6! ×2!=1440

Q2. Ans(B)
Q3. Ans(A)

Page 212 Follow us: Official Site, Telegram, Facebook, Instagram, Instamojo
Q8. Ans(A)
Q4. Ans(D)

Q9. Ans(B)
Q5. Ans(C)
Since all letters are unique, choosing any
5! × 3! = 720 four letters will lead us to 7×6×5×4 =
840 permutations.
Q6. Ans(D)
Q10. Ans(A)

Q7. Ans(C)

SET-2

Question 1: In how many different ways a) 6! × 5!


can the letters of the word ‘THERAPY’ be b) 6! × 4!
arranged so that the vowels never come c) 6! × 3!
together? d) 6! × 2!
a) 720 e) None of these
b) 1440 Question 3: What is the number of
c) 5040 words formed from the letters of the
d) 3600 word ‘JOKE’ So that the vowels and
e) 4800 consonants alternate?
Question 2: If 6 boys and 6 girls have to a) 4
sit in a round circular music chair. So, b) 8
that there is a girl between every 2 boys. c) 12
Find the number of ways they can sit? d) 18

Page 213 Follow us: Official Site, Telegram, Facebook, Instagram, Instamojo
e) None of these Question 8: If three marbles are picked
Question 4: In how many different ways at random what is the probability that
can 5 men and 3 women be seated in a either all are green or all are red ?
row such that no two women are next to a) 744
each other? b) 712
a) 12200 c) 512
b) 14400 d) 144
c) 15600 e) None of these
d) 16400 Instructions
e) None of the above Study the given information carefully
Question 5: In how many different ways and answer the question that follow:
can the letter of the word ‘SIMPLE’ be A committee of five members is to be
arranged ? formed out of 3 trainees 4 professors
a) 520 and 6 research associates In how many
b) 120 different ways can this be done if____
c) 5040 Question 9: The committee should have
d) 270 all 4 professors and 1 research associate
e) None of these or all 3 trainees and 2 professors ?
Question 6: In how many different ways a) 12
can the letters of the word ‘SECOND’ be b) 13
arranged ? c) 24
a) 720 d) 52
b) 120 e) None of these
c) 5040 Question 10: The committee should
d) 270 have 2 trainees and 3 research
e) None of these associates ?
Question 7: Let x=123456….424344. a) 15
What is the remainder when x is divided b) 45
by 45? c) 60
a) 0 d) 9
b) 9 e) None of these
c) 4 Question 11: A die is thrown twice.
d) none What is the probability of getting a sum
Instructions 7 from both the throws?
Study the given information carefully a) 518
and answer the question that follow: b) 118
A basket contains 4 red 5 blue and 3 c) 19
green marbles. d) 16
e) 536

Page 214 Follow us: Official Site, Telegram, Facebook, Instagram, Instamojo
Question 12: In how many different Question 14: In how many different
ways can the letters of the word STRESS ways can the letters of the word DAILY
be arranged ? be arranged ?
a) 360 a) 60
b) 240 b) 48
c) 720 c) 160
d) 120 d) 120
e) None of these e) None of these
Question 13: In how many different Question 15: In how many different
ways can the letters of the word ways can the letters of the word ‘PARTY’
BAKERY be arranged ? be arranged ?
a) 2400 a) 120
b) 2005 b) 2005
c) 720 c) 2400
d) 5040 d) 720
e) None of these e) None of these

Answers & Solutions: = 7! – (6!2!)


= 3600
1) Answer (D)
2) Answer (A)
Number of ways of arranging seven
letters = 7! Circular permutation = n! (n – 1)!
Let us consider the two vowels as a
group ∴ Number of ways = 6! (6 – 1)! = 6! × 5!
Now the remaining five letters and the
group of two vowels = 6 3) Answer (B)
These six letters can be arranged in 6!2!
ways( 2! is the number of ways the two
Word name: ‘JOKE’ Vowels: O, E
vowels can be arranged among
Consonants: J, K
themselves)
∴ Possible arrangement beginning with
The number of ways of arranging seven
consonant: JOKE, KOJE, JEKO, KEJO = 4
letters such that no two vowels come
Numbers
together
beginning with vowel: OJEK, OKEJ, EJOK,
= Number of ways of arranging seven
EKOJ = 4 Numbers
letters – Number of ways of arranging
Required number = 4+4 = 8 numbers
the letters with the two vowels being
together
4) Answer (B)
Page 215 Follow us: Official Site, Telegram, Facebook, Instagram, Instamojo
No two women would be seated next to unit digit has to be 0 or 5.
each other if they sit between men. So, Since the unit digit is 4, the remainder
first the men can be arranged in 5! ways. obtained when x is divided by 5 is 4
There are six spots between the men. Let us check the divisibility of x by 9:
Number of ways to arrange the 3 women
in those six spots is P63 ways. So, total Sum from 1 to 9 = 45. Sum from 10 to 19
number of ways = 120*120 = 14400 = 45+10 =55. Sum from 20 to 29 =65.
ways. Sum from 30 to 39 = 75. Sum from 40 to
44 = 30. Therefore total sum of the digits
5) Answer (E) of the number = 270. Therefore, the
number is divisible by 9.
If there are n different letter in a word
then n! different words can be formed. Using Chinese Remainder theorem,
5p+4=9q
In SIMPLE there are 6 different letters When p = 1, both LHS = RHS
and hence 6! new words can be formed Hence when p = 1, 5p+4=9
by different arangement. So 9 is the remainder when x is divided
by 45.
Hence, 720 new words can be formed.
8) Answer (D)
6) Answer (A)
Number of ways of selecting 3 green
If a word has n different letters in it, marbles out of 3 green marbles =3C3=1
then they can be arranged in n! different
ways. Number of ways of selecting 3 red
marbles out of 4 red marbles = 4C3=4
Here SECOND has 6 different letters and
they can be arranged in 6! ways i.e 720 Total ways = 1+4 = 5
different ways.
Probability =512C3=1/44
7) Answer (B)
9) Answer (A)
To check if a number is divisible by 45,
we have to check the divisibility of the 4 professors and 1 resarch associate can
number by 5 and 9. be selected in 4C4∗6C1=6 ways
x = 123456789………424344
Let us check the divisibility of x by 5: Similarly 3 trainees and 2 professors
For a number to be divisible by 5, it’s would be selected in 3C3∗4C2 ways = 6

Page 216 Follow us: Official Site, Telegram, Facebook, Instagram, Instamojo
Total ways = 6+6 = 12 13) Answer (C)

10) Answer (C) As there are 6 different letters in the


word “Bakery” hence, total number of
We have to select 2 trainees out of 3 ways of arranging it will be = 6! = 720
trainees and 3 resarch associates out of
6 resarch associates. 14) Answer (D)

Required number of If a word has n different letters then


ways = 3C2∗6C3=60 ways they can be arranged in n! different
ways.
11) Answer (D) DAILY has 5 different letters which can
bve arranged in 5! different ways.
The number 7 can be obtained in 6 5!=5*4*3*2*1=120
ways. (6,1), (1,6), (5,2), (2,5), (4,3), (3,4). Therefore, option D is answer.

Required probability = 6/36 = 1/6 15) Answer (A)

12) Answer (D) The word “Party” can be arranged in 5!


ways as there are all different letters in
the word.
There are 6 letters in total of which
three are repeated.
Hence, answer will be = 120
Hence, total number of arrangements
= 6!3! = 120

Probability

Page 217 Follow us: Official Site, Telegram, Facebook, Instagram, Instamojo
Important Probability Formulas List

1. Probability Problems on Dice


This type of Probability Questions is asked on the basis of rolling dice with six sided dots – 1,2,3,4,5
and 6.
When one dice is rolled, the number of possibilities is 6.
When the two dice are rolled together, the number of possibilities is 6*6 = 36.
To check the outcomes of two dice, below we have shared the sample space of Probability.
1 2 3 4 6 6
1 (1,1) (1,2) (1,3) (1,4) (1,5) (1,6)
2 (2,1) (2,2) (2,3) (2,4) (2,5) (2,6)
3 (3,1) (3,2) (3,3) (3,4) (3,5) (3,6)
4 (4,1) (4,2) (4,3) (4,4) (4,5) (4,6)
5 (5,1) (5,2) (5,3) (5,4) (5,5) (5,6)
6 (6,1) (6,2) (6,3) (6,4) (6,5) (6,6)

Here in the table, the outcomes (1,1), (2,2), (3,3), (4,4), (5,5) and (6,6) are known as doublets and the
pair (1,2) and (2,1) are different outcomes.

2. Probability Problems on Coins


When we throw a coin in the air there are two conditions that can occur. Whether it lands with head
or with a tail.
Most of the time questions are asked on Coins problem.
Probability questions on coins can be asked in three different types –

Page 218 Follow us: Official Site, Telegram, Facebook, Instagram, Instamojo
When only One Coin Flipped –
In this case, total events will be 2 and Probability P(E) is = 1/2.

When Two Coins are Flipped –


The sample space will be – {(H,T), (T,H), (H,H), (T,T)}
If the question is asked “What is the probability that both the coins shows head?”, The possibility of
two heads will be decided as P(H,H) = n(E)/n(S)
Putting the values in formula, we can say that the probability when both the coins show head is 1/4.

When Three Coins are Flipped –


This time the sample space will be changed as - {(H,H,H), ( T,T,T), (H,H,T), ( H,T,H), (T,H,H), (H,T,T),
(T,H,T) ,(T ,T, H)}
Here, we see the probability that both the coins show tails will be P(E) is = 3/8.

3.Probability Problems on Cards


Questions are also asked on the basis of a well-shuffled deck of 52 Playing Cards Probability.
Here you can clear all the concepts to attempt the Playing Cards Probability Questions.
The deck of 52 playing cards is divided into 4 suits of 13 cards each i.e. –
Spades (Black Cards)
Hearts (Red Cards)
Diamonds (Red Cards)
Clubs (Black Cards)

Practice Problem
SET-1
1. There are 100 tickets in a box Prob. = 1/2 × 1/2 × 1/2
numbered 1 to 100. 3 tickets are Case 2: even, odd, even
drawn at one by one. Find the Prob. = 1/2 × 1/2 × 1/2
probability that the sum of number Case 3: odd, even, even
on the tickets is odd. Prob. = 1/2 × 1/2 × 1/2
A) 2/7 Case 4: odd, odd, odd
B) 1/2 Prob. = 1/2 × 1/2 × 1/2
C) 1/3 Add all the cases, required prob. =
D) 2/5 1/2
E) 3/7
2. There are 4 green and 5 red balls in
View Answer first bag. And 3 green and 5 red balls
Option B in second bag. One ball is drawn
Solution: from each bag. What is the
There will be 4 cases probability that one ball will be
Case 1: even, even, odd green and other red?
Page 219 Follow us: Official Site, Telegram, Facebook, Instagram, Instamojo
A) 85/216 E) 5/12
B) 34/75
C) 95/216 View Answer
D) 35/72 Option B
E) 13/36 Solution:
2 men means other 2 woman and
View Answer children
Option D So prob. = 3C2 × 7C2 /10C4 = 3/10
Solution:
Case 1:first green, second red 5. Tickets numbered 1 to 120 are in a
Prob. = 4/9 × 5/8 = 20/72 bag. What is the probability that the
Case 2:first red, second green ticket drawn has a number which is
Prob. = 5/9 × 3/8 = 15/72 a multiple of 3 or 5?
Add the two cases A) 8/15
3. A bag contains 2 red, 4 blue, 2 white B) 5/16
and 4 black balls. 4 balls are drawn C) 7/15
at random, find the probability that D) 3/10
at least one ball is black. E) 13/21
A) 85/99
B) 81/93 View Answer
C) 83/99 Option C
D) 82/93 Solution:
E) 84/99 Multiples of 3 up to 120 = 120/3 =
40
View Answer Multiples of 5 up to 120 = 120/5 =
Option A 24 (take only whole number before
Solution: the decimal part)
Prob. (At least 1 black) = 1 – Prob. Multiple of 15 (3×5) up to 120 =
(None black) 120/15 = 8
So Prob. (At least 1 black) = 1 – So total such numbers are = 40 + 24
(8C4/12C4) = 1 – 14/99 – 8 = 56
So required probability = 56/120 =
4. Four persons are chosen at random 7/15
from a group of 3 men, 3 women and
4 children. What is the probability 6. There are 2 people who are going to
that exactly 2 of them will be men? take part in race. The probability
A) 1/9 that the first one will win is 2/7 and
B) 3/10 that of other winning is 3/5. What is
C) 4/15 the probability that one of them will
D) 1/10 win?
Page 220 Follow us: Official Site, Telegram, Facebook, Instagram, Instamojo
A) 14/35 B) 7/15
B) 21/35 C) 6/13
C) 17/35 D) 5/14
D) 19/35 E) 7/13
E) 16/35
View Answer
View Answer Option D
Option D Solution:
Solution: Case 1: 1 boy and 4 girls
Prob. of 1st winning = 2/7, so not Prob. = 5C1 × 4C4/9C5 = 5/146
winning = 1 – 2/7 = 5/7 Case 2: 2 boys and 3 girls
Prob. of 2nd winning = 3/5, so not Prob. = 5C2 × 4C3/9C5 = 40/126
winning = 1 – 3/5 = 2/5 Add the two cases = 45/126 = 5/14
So required prob. = 2/7 * 2/5 + 3/5 *
5/7 = 19/35 9. A bucket contains 2 red balls, 4 blue
balls, and 6 white balls. Two balls
7. Two cards are drawn at random are drawn at random. What is the
from a pack of 52 cards. What is the probability that they are not of same
probability that both the cards color?
drawn are face card (Jack, Queen and A) 5/11
King)? B) 14/33
A) 11/221 C) 2/5
B) 14/121 D) 6/11
C) 18/221 E) 2/3
D) 15/121
E) 14/221 View Answer
Option E
View Answer Solution:
Option A Three cases
Solution: Case 1: one red, 1 blue
There are 52 cards, out of which Prob = 2C1 × 4C1 / 12C2 = 4/33
there are 12 face cards. Case 2: one red, 1 white
So probability of 2 face cards Prob = 2C1 × 6C1 / 12C2 = 2/11
= 12C2/52C2 = 11/221 Case 3: one white, 1 blue
8. A committee of 5 people is to be Prob = 6C1 × 4C1 / 12C2 = 4/11
formed from among 4 girls and 5 Add all cases
boys. What is the probability that the
committee will have less number of 10. A bag contains 5 blue balls, 4
boys than girls? black balls and 3 red balls. Six balls
A) 7/12 are drawn at random. What is the
Page 221 Follow us: Official Site, Telegram, Facebook, Instagram, Instamojo
probability that there are equal E) 15/77
numbers of balls of each color?
A) 11/77 View Answer
B) 21/77 Option E
C) 22/79 Solution:
D) 13/57 5C2× 4C2× 3C2/ 12C6

SET-2
Q1. A bag contains 5 red balls and 7 C) 2/11
blue balls. Two balls are drawn at D) 3/11
random without replacement, and E) None of these
then find the probability of that one Q5. A basket contains 5 black and 8
is red and other is blue. yellow balls. Four balls are drawn at
A) 33/65 random and not replaced. What is
B) 35/66 the probability that they are of
C) 37/66 different colours alternatively.
D) 41/65 A) 56/429
E) None of these B) 57/429
Q2. A bag contains 3 red balls and 8 C) 61/429
blacks ball and another bag contains D) 68/429
5 red balls and 7 blacks balls, one E) None of these
ball is drawn at random from either Q6. A bag contains 6 red balls and 8
of the bag, find the probability that green balls. Two balls are drawn at
the ball is red. random one after one with
A) 93/264 replacement. What is the probability
B) 95/264 that Both the balls are green
C) 91/264 A) 13/49
D) 97/264 B) 15/49
E) None of these C) 16/49
Q3. 12 persons are seated at a circular D) 17/49
table. Find the probability that 3 E) None of these
particular persons always seated Q7. Find the probability that in a
together. leap year, the number of Mondays is
A) 9/55 53?
B) 7/55 A) 1/7
C) 4/55 B) 2/7
D) 3/55 C) 3/7
E) None of these D) 4/7
Q4. P and Q are two friends standing E) None of these
in a circular arrangement with 10 Q8. A box contains 4 red, 5 black and
more people. Find the probability 6 green balls. 3 balls are drawn at
that exactly 3 persons are seated random. What is the probability that
between P and Q. all the balls are of same colour?
A) 5/11 A) 33/455
B) 4/11 B) 34/455
Page 222 Follow us: Official Site, Telegram, Facebook, Instagram, Instamojo
C) 44/455 E) None of these
D) 47/455 Q10. From a pack of 52 cards, 2
E) None of these cards are drawn at random. What is
Q9. A box contains 6 blue, 5 green the probability that both cards are
and 4 red balls. Two balls are drawn black card or heart card?
at random. What is the probability A) 31/102
that there is no red ball? B) 21/73
A) 3/30 C) 1/5
B) 11/21 D) 17/100
C) 5/18 E) 3/10
D) 11/23
ANSWER 1-10

Q1. Ans(B)

Q2. Ans(C)

Q3. Ans(D)

Taking 3 particular persons as single entity, we have 12 – 3 + 1 = 10 persons to be


seated.

Page 223 Follow us: Official Site, Telegram, Facebook, Instagram, Instamojo
Q4. Ans(C)

Q5. Ans(A)

Q6. Ans(C)

Q7. Ans(B)

Q8. Ans(B)

Page 224 Follow us: Official Site, Telegram, Facebook, Instagram, Instamojo
Q9. Ans(B)

Q10. Ans(A)

Data Interpretation
Tips-Tricks of Data Interpretation

1. Memorize some basic maths to improve calculation ability: For solving, Data Interpretation
problems in less time. At first, you need to self-assess your mathematical knowledge whether you
have the following things in your mind.

 Mathematical tables up to 20.

Page 225 Follow us: Official Site, Telegram, Facebook, Instagram, Instamojo
 Squares of numbers 12 to 302.
 Cubes of numbers 13 to 20
 Powers of 2 up to 6.
 Powers of 3 up to 5.
 Fractional values 1/2, 1/3 ……… 1/20.

2. Learn some fast calculation tips to save time

 Multiplication tricks
 Approximations
 Addition Tricks
 Division Tricks
 Percentages
 Averages
 Squares & Cubes, Square roots, and Cube roots
 Comparison of fractions

3. Improve your mental calculation ability. Which comes by practising more model question
papers.
4. Use approximate values while solving DI problems: In DI, for some questions, we don’t need to
get exact answers. For such questions use approx. values to solve. This approach saves time.

5. Use previous calculations if needed again: While solving some questions, we require previously
solved answers. For those questions, use previous answers and no need to calculate the same sums
again and again. Reuse previous calculations at required places

6. Skip time-consuming questions and do at the end. Don’t waste time with lengthy
calculations: According to our above analysis, some questions need lengthy calculations. First check,
whether you can able to solve such questions using shortcut methods, maths tricks and by following
calculation tips in less time.

Trick: Need to know which question to pick and which question to skip and also where we have to
use a pen and where we have to mental calculations and where to use approximations while solving
the questions.

7. Get answers by watching the diagram: For some questions, we can get answers by seeing the
diagram itself carefully. No need to use pen and paper in solving that sort of question. Avoid
paperwork for simple calculations.

8. Important Topics to concentrate on Data Analysis and Interpretation: Data Analysis and
Interpretation majorly covers questions from these 3 areas of quantitative aptitude.

 Ratios: The ratio is nothing but comparing two same values in different niches.
 Percentages: The percentage is nothing but finding some value out of 100.
 Averages: Averages are nothing but additions and division of given numbers

Page 226 Follow us: Official Site, Telegram, Facebook, Instagram, Instamojo
So, speed up your calculations and focus more on these topics in learning smart techniques, shortcuts,
formulas, etc.,

9. Follow the Right Approach while Practice:

Speed along with accuracy plays the main role in one’s success in bank exams. Speed improves mainly
through the right approach of practice. we have to analyze our performance in every go and make
improvements in weak areas to have a better performance next time.

10. Use Data Interpretation Shortcut Tricks to save Time

With the analysis of recent competitive exams – ibps po, sbi po, sbi clerk, ibps clerk, RBI exams, RRB
exams. DI questions will be asking based on aptitude topics also –

 Time and work


 Time & distance
 Profit & loss
 Averages
 Percentages
 Pipes & cisterns
 Ratio & proportion

Study aptitude formulas and shortcuts on the above topics and speed maths trick to solve data
interpretation questions quicker.

Data Interpretation Formulas

We use some aptitude formulas in solving DI questions. Most Common questions are asking in this
format from Percentages

1) X is what % of y that means x/y * 100.

Eg: 72 is what % of 360 = [72/360]×100 =20%

2) What percentage is x of y? I.e. x/y * 100.

3) X is what percent more than y = x-y/y * 100

4) X is what percent less than y = y-x/y * 100

5) Percentage change between two particular values = [final – initial/ initial] * 100

Memorize some fractional values and percentage values.

 1/2 = 50%  1/5 = 20%


 1/3 = 33.33%  1/6 =16.66%
 ¼ = 25%  1/7 = 14.28%
Page 227 Follow us: Official Site, Telegram, Facebook, Instagram, Instamojo
 1/8 = 12.5%  30% =3/10
 1/9 = 11.11%  40% =2/5
 1/10 = 10%  50% =1/2
 1/11 =9.09 %  55%=11/20
 1/12 = 8.33%  60% = 3/5
 1/13 = 7.69%  70% = 7/10
 1/14 = 7%  75% = 3/4
 1/15 = 6.66%  80% = 4/5
 1/16 = 6.25%  90% = 9/10
 1/17 = 5.88%  100%=1
 1/18 =5.55%  6 ¼ % = 1/6
 1/19 = 5.26%  12 ½ % = 1/8
 1/20= 5%  16 2/3 % = 1/6
 5%= 1/20 = 0.05  33 1/3 % = 1/3
 10% =1/10 =0.1  66 2/3 % =2/3
 15% =3/20  125% =5/4
 20% = 1/5  150% = 3/2
 25% = ¼

Practice Questions
(TABULAR DI)
Questions (1-5): Following table shows the total number of students appeared from
five different institutes, ratio of boys and girls among those appeared students,
percentage of passed students and number of passed girls among them.

1. What is the average number of E) None


Girls appeared in the examination
from all six cities?
View Answer
A) 1855
B) 1960 Option B
C) 1720 Solution:
D) 1880 Appeared Girls A
Page 228 Follow us: Official Site, Telegram, Facebook, Instagram, Instamojo
9 == 4500
4 ? = 2000
3. What is the total number of failed
Appeared Girls B
students in the examination from
5 == 5000
all six cities together?
2 ? = 2000
A) 7250
Appeared Girls C
B) 6800
12 == 3600
C) 7570
5 ? = 1500
D) 7360
Appeared Girls D
E) None
11 == 4400
View Answer
7 ? = 2800
Appeared Girls E Option C
13 == 3900 Solution:
5 ? = 1500 A: (100-55)% of 4500= 2025
So, B: (100-60)% of 5000=2000
Average=(2000+2000+1500+2800+ C: (100-70)% of 3600=1080
1500)/5 D: (100-75)% of 4400=1100
=1960. E: (100-65)% of 3900=1365
Total
=2025+2000+1080+1100+1365=75
2. The total number of girls passed 70.
from Institute C is what
percentage of the total number of
girls appeared from City C? 4. The total number of girls passed
A) 88% in the examination is
B) 75% approximately what percentage of
C) 82% the total number of girls appeared
D) 72% in the examination, taking all
E) None cities together?
A) 75.7%
B) 78%
View Answer
C) 69.5%
Option A D) 65%
Solution: E) none
From Ques 1 we know that
Appeared Girls C
View Answer
12 == 3600
5 ? = 1500 Option A
Then 1500 = 100 Solution:
1320 ? ==> 88%. From Ques 1 we know that
Page 229 Follow us: Official Site, Telegram, Facebook, Instagram, Instamojo
Total girls A) 1750
appeared=2000+2000+1500+2800+ B) 1600
1500=9800. C) 1560
Total girls D) 1700
passed=1250+1300+1320+2100+14 E) None
50=7420. View Answer
9800 == 100
Option D
7420 ? ==>75.7%.
Solution:
Total no of passed students in city B
5. The total number of boys passed 60% of 5000=3000.
from City B is what percentage No of girls passed in city B =1300
more than the total number of Then No of boys passed in city
B=3000-1300=1700.
girls passed from that city?

(BAR GRAPH)
Directions (1 – 5): Study the following bar graph and answer the questions that
follow.
Votes are polled in 10 constituencies of Haryana. After the polls, some votes were
declared invalid.
The bar chart shows the % of votes received by the winning and the losing candidates
(Suppose there are only 2 candidates) out of the “valid votes”, which is “total votes”
polled minus the “invalid votes”. The one which got the highest number of votes from
the “valid votes” was declared the winner.

Page 230 Follow us: Official Site, Telegram, Facebook, Instagram, Instamojo
The total number of invalid votes in each constituency – Karnal, Faridabad, Gurgaon,
Hisar, Sirsa, Bhiwani, Ambala, Rohtak, Panipat and Kurukshetra are 3800, 2000, 11400,
0, 2700, 150, 4200, 360, 320, 6800 respectively.

1. What is the total number of voters Same as with other constituencies.


in the given 10 constituencies of So
Haryana? Total number of votes polled in
A) 217000 Karnal = (3800/19) * 100 = 20,000
B) 234000 Total number of votes polled in
C) 276000 Faridabad = (2000/10) * 100 =
D) 211000 20,000
E) Cannot be determined Total number of votes polled in
Gurgaon = (11400/38) * 100 =
30,000
View Answer
Total number of votes polled in
Option E Hisar cannot be found because there
Solution: is no invalid vote here.
In Karnal, valid votes are 45+36 = Total number of votes polled in Sirsa
81%, so invalid votes are 100-81 = = (2700/9) * 100 = 30,000
19% Total number of votes polled in
Page 231 Follow us: Official Site, Telegram, Facebook, Instagram, Instamojo
Bhiwani = (150/1) * 100 = 15,000 View Answer
Total number of votes polled in
Option D
Ambala = (4200/14) * 100 = 30,000
Solution:
Total number of votes polled in
Total number of votes of winners
Rohtak = (360/2) * 100 = 18,000
from Panipat, Ambala and Karnal =
Total number of votes polled in
4000*81/100 + 20000*45/100 +
Panipat = (320/8) * 100 = 4,000
30000*47/100 = 3240 +9000+
Total number of votes polled in
14100 = 26340
Kurukshetra = (6800/17) * 100 =
40,000 4. In the last elections, there was a
total of 3,00,000 voters in these
2. What is the difference between
10 constituencies. If this time,
the number of votes received by
there is an increase by 10%,
the winning and the losing
thhen what is eth total number of
candidate from Sirsa?
voters from Hisar?
A) 5400
A) 1,28,000
B) 5700
B) 1,23,000
C) 6600
C) 1,32,000
D) 4500
D) 1,34,000
E) 5900
E) 1,25,000

View Answer
View Answer
Option B
Option B
Solution:
Solution:
Total number of votes from Sirsa =
Total number of votes this time =
30,000
110/100 * 300000 = 3,30,000
Difference between votes =
Total number of voters from other
30000/100 * (55-36) = 5700
constituencies this time = 2,07,000
3. What is the total number of valid So total number of votes from Hisar
votes received by winning = 3,30,000 – 2,07,000 = 1,23,000
candidates from Panipat, Ambala
5. What is the total number of valid
and Karnal together?
votes from Bhiwani and
A) 26540
Kurukshetra together?
B) 21760
A) 32,030
C) 23780
B) 38,400
D) 26340
C) 23,760
E) Cannot be determined
D) 45,550

Page 232 Follow us: Official Site, Telegram, Facebook, Instagram, Instamojo
E) 48,050 Bhiwani = 15000 – 150 = 14,850
Total number of valid from
Kurukshetra = 40,000 – 6800 =
View Answer
33,200
Option E So total number of valid votes =
Solution: 14850+33200 = 48,050
Total number of valid votes from

(LINE GRAPH)
Directions(1-5): Study the following graph carefully and answer the given questions.
Description of income and expenditure of a company in 7 months of the year 2015 (in
Rs. thousands)

Profit = Income – Expenditure , Loss = Expenditure – Income


Profit% = (Profit/Expenditure)*100
Loss% = (Loss/Expenditure )*100

1. What is the average expenditure of A) Rs.389


the company in the given months(in B) Rs.350
thousands)? C)Rs.420
Page 233 Follow us: Official Site, Telegram, Facebook, Instagram, Instamojo
D) Rs.310 Option D
E) Rs.440 Solution:
Total profit earned by the company
View Answer
in the months of February, April and
Option A July
Solution: = 200 + 280 + 140 = Rs. 620
Average expenditure of company = thousands
2720/7 = Rs. 389 thousands Profit earned in the months of
January, March and June = 340 + 180
2. What is the respective ratio between + 260 = Rs.780 thousands
the percentage profits earned by the Difference = 780 – 620 = Rs. 160
company in the months of February thousands
and May?
A) 21 :48 4. By what percent is the profit earned
B) 25 : 44 by the company in the month of
C) 23 : 42 February less than that earned in the
D) 23 : 50 month of January?
E) 20 : 45 A) 45%
B) 52%
View Answer C) 47%
Option D D) 41%
Solution: E) 50%
Profit percent of company View Answer
= February = [(780 – 580)/580]*100
= 34.5 Option D
May = [(560-320)/320]*100 = 75 Solution:
= 345 : 750 = 23 : 50 Profit of company :
February = 780 – 580 = Rs. 200
3. What is the difference between the thousand
total profit earned by the company January = 660 – 320 = Rs. 340
in the months of February, April and thousand
July and that earned in the months of Required % = [(340 –
January, March and June? 200)/340]*100 = 41%
A) Rs.150
B) Rs. 210 5. In how many months, the income of
C) Rs.200 company was more than the average
D) Rs.160 income during the given months?
E) Rs.110 A) 2
B) 1
View Answer C) 3

Page 234 Follow us: Official Site, Telegram, Facebook, Instagram, Instamojo
D) 4 Average income of company =
E) None 4360/7 = Rs. 623 thousand
Hence, the answer is 3 (Ian., Feb. and
View Answer
Apr.)
Option C
Solution:
(MISSING DATA INTERPRETATION)
Questions (1-5) : Study the following table carefully to answer the following
questions given below:

1) The average no of B.Ed students 2) If the ratio of no of B.Sc students


and B.Sc students in T was 520. to that of B.Ed students in P was 9:6,
What was the total no of students in what was the no of B.Ed students in
T? P?
A) 1500 A) 620
B) 1450 B) 588
C) 1600 C) 565
D) 1560 D) 482
E) None E) None

View Answer View Answer


Option C Option B
Solution: Solution:
B.Ed students in T = 100-75 = 25% P = 2100
65% = 2*520 = 1040 B.Sc students + B.Ed students in P =
100% = 1040*100/65 = 1600 100-30 = 70%
B.Ed students = 70*6/15 = 28% =
28% of 2100 = 588.

Page 235 Follow us: Official Site, Telegram, Facebook, Instagram, Instamojo
E) None
3) If the difference between the no
of B.Ed students in S and that in R
View Answer
was 300. What was the total no of
students in R? (Assume no of Option D
students in R and S are same) Solution:
A) 1110 Q = 700*120/100 = 840*80/100 =
B) 1200 672
C) 1000 B.Com = 672 * 29/100 = 195.
D) 1250
E) None 5)If the total no of students in S is
View Answer equal to twice the number of
students in R .What is the ratio of
Option C
number of B.Sc students in Q to the
Solution:
no of B.Ed students in S(Total no of
% of B.Ed students in R = 100-50 =
students in R is 1000) ?
50%
A) 217:400
Difference between the no of B.Ed
B) 215:206
students in S and that in R = 50-20 =
C) 145:132
30%
D) 120:133
X *30/100 = 200
E) None
X = 300*100/30 = 1000

4)The total no of students in Q is View Answer


increased by 20% and then
decreased by 20%, what is the total Option A
number of B.Com students in Q ? Solution:
A) 160 No of B.Sc students in Q =
B) 175 700*31/100 = 217
C) 180 No of B.Ed students in S = 2000*
D) 195 20/100 = 400
Ratio 217:400.
(PIE CHARTS)
Directions(1-5): In the following pie-chart , the percentage of students studying in 6
different schools has been shown. Study the pie-chart carefully to answer the

Page 236 Follow us: Official Site, Telegram, Facebook, Instagram, Instamojo
questions.

1. By what percent the number of C) 4 : 3


students studying in school D is less D) 2 : 1
than that of studying in school C? E) 3 : 2
A) 21(1/2)%
B) 42(1/3)%
View Answer
C) 33(1/3)%
D) 24(1/2)% Option B
E) 34(1/6)% Solution:
Required ratio = 14 : 12 = 7 : 6
View Answer
3. What is the average number of
Option C
students studying in schools B , D
Solution:
and E?
Percentage decrease = [(24 –
A) 7500
16)/24]*100 = 33(1/3)%
B) 13540
C) 12500
2. Ratio between the number of D) 11000
students studying in school B and E . E) 10500
A) 5 : 4
B) 7 : 6
View Answer
Page 237 Follow us: Official Site, Telegram, Facebook, Instagram, Instamojo
Option E 24%
Solution:
Number of students in schools B, D
5. In which two other schools the
and E
number of students is same as that
= 75000*(14+16+12)/100 =
of students is same as that of
75000*42/100 = 31500
studying in schools A and C ?
Required average = 31500/3 =
A) D and C
10500
B) C and B
C) A and E
4. In which school the number of D) E and F
students is the highest? E) B and D
A) 24%
B) 48%
View Answer
C) 20%
D) 25% Option D
E) 30% Solution:
Percentage of students in students
in schools A and C
View Answer
= 24 + 11 = 35%
Option A Percentage of students in students
Solution: in schools E and F
Clearly, the percentage of students in = 23 + 12 = 35%
school C in pie-chart is highest i.e.

(Caselet DI)
Direction (1-3): Study the following information carefully and answer the
questions given below.

Box A: The total number of balls is 36 and the ratio of the number of red, blue and green
balls in box A is 3:2:4.

Box B: The total number of balls is 25% more than that of the number of balls in box A.
The number of red balls in box B is 25% more than that of the number of red balls in A.
The ratio of the number of blue to green ball in B is 3:2

Page 238 Follow us: Official Site, Telegram, Facebook, Instagram, Instamojo
Box C: Total number of balls is C is 5 balls more than that of the total number of balls in
B. If the number of red balls in C is one less than that of the number of red balls in B.
The number of blue balls in C is sum of the red and blue balls in A.

1) What is the total number of green C) 14: 15


balls in all the boxes together?
D) 7: 9
A) 42
E) None of these
B) 44
3) The total number of blue balls in
C) 48 all the boxes together is what percent
of the total number of balls in box C?
D) 41
A) 95%
E) 44
B) 93%
2) What is the ratio of the total
number of red balls in all the boxes C) 90%
together to the total number of balls
in box B? D) 91%

A) 1: 5 E) 92%

B) 11: 15

SOLUTION (1-3):

Box A: Red = 125/100 * 12 = 15

Total = 36 Blue = (45 – 15) * 3/5 = 18

Red = 36 * 3/9 = 12 Green = 30 * 2/5 = 12

Blue = 36 * 2/9 = 8 Box C:

Green = 36 * 4/9 = 16 Total = 45 + 5 = 50

Box B: Red = 15 – 1 = 14

Total = 125/100 * 36 = 45 Blue = 12 + 8 = 20


Page 239 Follow us: Official Site, Telegram, Facebook, Instagram, Instamojo
Green = 50 – 14 – 20 = 16

1) Answer: B = 41: 45

Total = 16 + 12 + 16 = 44 3) Answer: E

2) Answer: E Required percentage = [(8 + 18 +


20)/50] * 100
Required ratio = (12 + 15 + 14):45
= 92%

MIXTURE & ALLIGATION


Mixture & Alligation Formula
1. Alligation:

It is the rule that enables us to find the ratio in which two or more ingredients at the given price must
be mixed to produce a mixture of desired price.
2. Mean Price:

The cost of a unit quantity of the mixture is called the mean price.
3. Rule of Alligation:

If two ingredients are mixed, then


Quantity of cheaper C.P. of dearer - Mean Price
=
Quantity of dearer Mean price - C.P. of cheaper

We present as under:
C.P. of a unit quantity C.P. of a unit quantity of dearer
of cheaper
(c) Mean Price (d)
(d - m) (m) (m - c)
(Cheaper quantity) : (Dearer quantity) = (d - m) : (m - c).
4. Suppose a container contains x of liquid from which y units are taken out and replaced by water.
After n operations, the quantity of pure liquid = x 1 - y n units.

Page 240 Follow us: Official Site, Telegram, Facebook, Instagram, Instamojo
x

Practice Problem
Question 1: 18 litres of pure water was ratio of milk and water in the resulting
added to a vessel containing 80 litres of mixture ?
pure milk. 49 litres of the resultant a) 59 : 21
mixture was then sold and some more b) 35 : 22
quantity of pure milk and pure water c) 64 : 23
was added to the vessel in the respective d) 65 : 21
ratio of 2:1. If the resultant respective e) None of these
ratio of milk and water in the vessel was Question 4: In Jar A, 180 litre milk was
4:1, what was the quantity of pure milk mix with 36 litre water. Some of this
added in the vessel ? (in litres) mixture was taken out from Jar A and
a) 4 put it in Jar B. If after adding 6 litres of
b) 8 water in the mixture, the respective
c) 10 ratio between milk and water in Jar B
d) 12 was 5 : 2 respectively, what was the
e) 2 amount of mixture that was taken out
Question 2: A vessel contains a mixture from Jar A ? (in litres)
of Grape, Pineapple and Banana juices in a) 24
the respective ratio of 4 : 6 : 5. 15 litres b) 54
of this mixture is taken out and 8 litres c) 30
of grape juice and 2 litres of pineapple d) 36
juice is added to the vessel. If the e) 42
resultant quantity of grape juice is 10 Question 5: In Jar A, 140 litre milk was
litres less than the resultant quantity of mixed with 40 litre water. Some of this
pineapple juice. what was the initial mixture was taken out from Jar A and
quantity of mixture in the vessel ? (in put in Jar B. If before the operation,
litres) there was 17 litres of milk in Jar B, and
a) 120 afterwards the resultant ratio between
b) 150 milk and water in jar B was 19 : 3
c) 105 respectively, what was the amount of
d) 135 mixture that was taken out from Jar A ? (
e) 90 in litre)
Question 3: A vessel contains 64 litres a) 21
of mixture of milk and water in the ratio b) 36
7 : 3 respectively. 8 litres of mixture is c) 46
replaced by 8 litres of milk. What is the d) 18
e) 27

Page 241 Follow us: Official Site, Telegram, Facebook, Instagram, Instamojo
Question 6: In a 90 litres mixture of Question 9: In a vessel there is 40 litres
milk and water, percentage of water is mixture of milk and water. There is 15%
only 30%. The milkman gave 18 litres of water in the mixture. The milkman sells
this mixture to a customer and then 10 litres of mixture to a customer and
added 18 litres of water to the thereafter adds 12.5 litres of water to
remaining mixture. What is the the remaining mixture. What is the
percentage of milk in the final mixture ? respective ratio of milk and water in the
a) 64 new mixture ?
b) 48 a) 2 : 3
c) 52 b) 3 : 2
d) 68 c) 3 : 4
e) 56 d) 4 : 3
Question 7: A vessel contains a mixture e) None of these
of milk and water in the respective ratio Question 10: Jar A contains ‘X’ litre of
of 10 : 3. Twenty-six litre of this mixture pure milk only. A 27 litre mixture of milk
was taken out and replaced with 8 litre and water in the respective ratio of 4 : 5,
of water. If the resultant respective ratio is added to jar A. The new mixture thus
of milk and water in the mixture was 5 : formed in jar A contains 70% milk, what
2, what was the initial quantity of is the value of X ?
mixture in the vessel ? (in litre) a) 23
a) 143 b) 30
b) 182 c) 27
c) 169 d) 48
d) 156 e) 28
e) 130 Question 11: Jar A contains 78 litres of
Question 8: 18 litres of pure water was milk and water in the respective ratio of
added to a vessel containing 80 litres of 6 : 7. 26 litres of the mixture was taken
pure milk. 49 litres of the resultant out from Jar A. What quantity of milk
mixture was then sold and some more should be added to jarA, so that water
quantity of pure milk and pure water constitutes 40% of the resultant mixture
was added to the vessel in the respective in jar A?
ratio of 2 : 1. If the resultant respective a) 8 litres
ratio of milk and water in the vessel was b) 36 litres
4 : 1, what was the quantity of pure milk c) 12 litres
added in the vessel ? (in litres) d) 14 litres
a) 4 e) 18 litres
b) 8 Question 12: Jar A has 36 litres of
c) 10 mixture of milk and water in the
d) 12 respective ratio of 5 : 4. Jar B which had
e) 2 20 litres of mixture of milk and water,
Page 242 Follow us: Official Site, Telegram, Facebook, Instagram, Instamojo
was emptied into jar A, and as a result in respective ratio of 14 : 3. 25.5 litres of
jar A, the respective ratio of milk and the mixture is taken out from the vessel
water becomes 5: 3. What was the and 2.5 litres of pure water and 5 litres
quantity of water in jar B? of pure milk is added to the mixture. If
a) 5 litres the resultant mixture contains 20%
b) 3 litres water, what was the initial quantity of
c) 8 litres mixture in the vessel before the
d) 2 litres replacement? (in litres)
e) 1 litre a) 51
Question 13: Jar A has 60 litres of b) 102
mixture of milk and water in the c) 68
respective ratio of 2 : 1. Jar B which had d) 85
40 litres of mixture of milk and water e) 34
was emptied into jar A, as a result in jar Question 16: In a mixture ratio of milk
A, the respective ratio of milk and water and water is 7:6. If 12 liters of water is
became 13 : 7. What was the quantity of added into the mixture, Then the new
water in jar B? ratio of milk and water will be 13:12.
a) 8 litres How much quantity of milk is available
b) 15 litres initially in the mixture?
c) 22 litres a) 182 liters
d) 7 litres b) 172 liters
e) 1 litre c) 194 liters
Question 14: A vessel contains 100 d) 164 liters
litres mixture of milk and water in the e) None of these.
respective ratio of 22 : 3. 40 litres of the Question 17: In a mixture ratio of milk
mixture is taken out from the vessel and and water is X:Y. If 20 liters milk and 10
4.8 litres of pure milk and pure water liters water is taken out from the
each is added to the mixture. By what mixture then both milk and water
percent is the quantity of water in the quantities are equal to each other. If in
final mixture less than the quantity of initial ratio 10% milk and 20% water is
milk? added, Then ratio between milk and
a) 7812 water is 11:10. Find out the value of Y?
b) 7916 a) 48
c) 7256 b) 40
d) 76 c) 55
e) 7712 d) 45
Question 15: A vessel contains a e) 50
mixture of milk and water in the

Answers & Solutions:

Page 243 Follow us: Official Site, Telegram, Facebook, Instagram, Instamojo
1) Answer (A) Now when we removed 15 ltr from
18 litres of pure water was added to 80 vessel the juice will be removed in their
litres of pure milk. given ratio i.e 4 ltr of grape juice will be
This we get a mixture where removed and 6 ltrs of pineapple will be
quantity of water = 18 litre removed and 5 ltrs of banana juice will
quantity of milk = 80 litre be removed and hence new quantities
Total quantity of the mixture = 18+80 = are
98 litre Grape juice = 4y-4
49 litres of the resultant mixture was Pineapple juuce = 6y- 6
then sold. Since half of the mixture is Banana juice = 5y- 5
removed and only the other half is Niw 8 ltrs of grape juuce is added and 2
remaining, ltrs of pineapple juice is added so new
quantity of water remaining = 18/2 = 9 quantities of Juices in vessel are
litre Grape juice = 4y+4
Quantity of milk remaining = 80/2 = 40 Pineapple juuce = 6y-4
litre It is given that grape juice amount is 10
Total quantity remaining = 49 litre ltrs less than pineapple juice quantity .
some more quantity of pure milk and So
pure water was added to the vessel in 6y-4 – 4y-4 = 10
the ratio 2:1 2y= 18
Let quantity of milk added =2x y= 9
quantity of water added =x Initial quantity in vessel = 15 y =
Now, 15×9=135 ltrs
quantity of water =9+x
quantity of milk =40+2x 3) Answer (A)
Given that ratio of milk and water in the Solution of milk and water in vessel = 64
vessel is now 4:1 litres
=> (40+2x):(9+x)=4:1 ration of Milk:Water = 7:3
=> 40+2x=4(9+x) using
=> 40+2x=36+4x waterconcentrationfinaltotal=initialwat
=> 2x=4 erconc.total(1−removedvolumetotal)n
=> x=2 waterconcentrationfinaltotal=310(1−86
quantity of pure milk added in the vessel 4)1
=2x = 4 litre waterconcentrationfinaltotal=310(1−18
)n
2) Answer (D) waterconcentrationfinaltotal=2180
let the amount of grape juice ,pineapple water : milk in new solution after
juice and banana juice in vessel be 4y replacement = 21: 59
,6y,5y respectively

Page 244 Follow us: Official Site, Telegram, Facebook, Instagram, Instamojo
4) Answer (D) Amount of water in the
The ratio of milk to water in Jar X solution=21.6+18=39.6 liters.
= 180 : 36 = 5:1 Hence, Percentage of milk in
Now, let 6x litres of mixture be taken out solution=50.450.4+39.6×100.
from Jar X and put in Jar Y. =56%.
Then, milk in Jar Y = 5x Hence, Option E is correct.
Water in Jar Y= x 7) Answer (E)
So, 5x/(x+6) = 5/2 Let quantity of Milk and water be M and
or, 10x=(5x + 30) W respectively.
or, 5x=30, M : W =10 : 3
:. x=6 3M=10W
Hence the mixture that was taken out In 26 litre of mixture
from Jar X = 6x =6 × 6 = 36 litres M =26(10/13) = 20 litre and
W =26(3/13) = 6 litre
5) Answer (E) 8 litre of water is added.
Milk to water ratio in Jar A is 140:40 = Resulting ratio of M and W is
7:2. Let the quantity of taken out M-20 : W-6+8 = 5 : 2
mixture from jar A = 9x litre. 2(M-20) = 5(W+2)
Hence, milk will be 7x and water will be 2M – 40 = 5W + 10
2x litres. Multiplyin all the terms by 2.
Therefore, (7x + 17) / 2x = 19/3 4M – 80 = 10 W + 20
=>x = 3 Replacing 10W with 3M.
Hence, amount taken out is 9*3 = 27 4M – 80 = 3M + 20
litres. M = 100
Hence W would be 30.
6) Answer (E) Total quantity = 100+30 = 130.
In 90 liters of mixture, Option E is the correect answer.
Amount of water=90×30100.
=27 liters. 8) Answer (A)
Amount of milk=90×70100. 18 litres of pure water was added to a
=63 liters. vessel containing 80 litres of pure milk.
Similarly, in 18 liters of mixture, Total mixture = 80 + 18 = 98 litres
Amount of water=18×30100. Now, 49 litres i.e., 12is removed, => Milk
=5.4 liters. left = 802 = 40 litres
Amount of milk=18×70100. Water left = 182 = 9 litres
=12.6 liters. Let milk added be 2x litres and water
After removing 18 liters of solution, added is x litres
Amount of water=27-5.4=21.6 liters. => 40+2x9+x=41
Amount of milk=63-12.6=50.4 liters. => 40+2x=36+4x
After adding 18 liters of water, => 2x=40–36=4
Page 245 Follow us: Official Site, Telegram, Facebook, Instagram, Instamojo
=> x=42=2 => 48+2x=84
∴ Quantity of milk added = 2×2=4 litres => 2x=84–48=36
=> x=362=18 litres
9) Answer (B)
Mixture remaining after selling 10 litres 12) Answer (A)
= 40 – 10 = 30 litres Jar A has 36 litres of mixture of milk and
Now, quantity of water in 30 litres of water in the respective ratio of 5 : 4
mixture = 15100∗30 = 4.5 litres => Quantity of milk in Jar A
Milk = 30 – 4.5 = 25.5 litres = 59×36=20 litres
After adding 12.5 litres of water, total Quantity of water in Jar A = 36–
quantity of water = 12.5 + 4.5 = 17 litres 20=16 itres
∴ Required ratio of milk and water = Let quantity of water in Jar B = x litres
25.5 : 17 => Quantity of milk in Jar B = (20–
= 1.5 : 1 = 3 : 2 x) litres
10) Answer (A) Acc. to ques, => 20+(20–x)16+x=53
Quantity of milk in 27 litre mixture => 120–3x=80+5x
= 44+5×27=12 litre => 5x+3x=120–80
Quantity of water = 27–12=15 litre => 8x=40
Ratio of milk and water in the new => x=408=5 litres
mixture = 70:30=7:3
Acc to ques, 13) Answer (B)
=> X+1215=73 Jar A has 60 litres of mixture of milk and
=> 3X+36=15×7=105 water in the respective ratio of 2 : 1
=> 3X=105–36=69 => Quantity of milk in Jar A
=> X=693=23 litre = 23×60=40 litres
Quantity of water in Jar A = 60–
11) Answer (E) 40=20 itres
Jar A has 78 litres of mixture of milk and Let quantity of water in Jar B = x litres
water in the respective ratio of 6 : 7 => Quantity of milk in Jar B = (40–
=> Quantity of milk in Jar A x) litres
= 613×78=36 litres Acc. to ques, => 40+(40–x)20+x=137
Quantity of water in Jar A = 78– => 560–7x=260+13x
36=42 litres => 13x+7x=560–260
26 litres of the mixture was taken out => 20x=300
from Jar A, i.e., 2678=(13)rd => x=30020=15 litres
=> Milk left = 36–13×36=24
Water left = 42–13×42=28 14) Answer (B)
Let milk added to jar A = x litres Quantity of milk in vessel
Acc. to ques, => 24+x28=6040 = 2225×100=88 litres
=> 24+x28=32 => Quantity of water = 100–88=12 litres
Page 246 Follow us: Official Site, Telegram, Facebook, Instagram, Instamojo
40 litres of the mixture is taken out, So 7Y6(Y+2)=1312
i.e., 40100=(25)th 7Y(Y+2)=132
=> Milk left = 88–25×88=52.8 litres 7Y×2=13×(Y+2)
Water left = 12–25×12=7.2 litres 14Y=13×Y+13×2
Now, 4.8 lires of milk and water are 14Y = 13Y + 26
added. 14Y – 13Y = 26
=> Quantity of milk in the vessel = 52.8 + Y = 26
4.8 = 57.6 litres Initially we have assumed quantity of
Quantity of water in the vessel = 7.2 + milk is 7Y.
4.8 = 12 litres So 7Y=7×26.
∴ Required % = 57.6–1257.6×100 = 182 liters
= 4756=7916% Hence, option a is the correct answer.

15) Answer (C) 17) Answer (E)


Let the total quantity of mixture in the In a mixture ratio of milk and water is
vessel initially = 17x litres X:Y. If 20 liters milk and 10 liters water
=> Quantity of milk is taken out from the mixture then both
= 1417×17x=14x litres milk and water quantities are equal to
Quantity of water = 17x–14x=3x litres each other.
Acc. to ques, X – 20 = Y – 10
=> 14x–(1417×25.5)+53x– So X – Y = 20 – 10.
(317×25.5)+2.5=8020 X – Y = 10. Eq.(1)
=> 14x–21+53x–4.5+2.5=41 If in initial ratio 10% milk and 20%
=> 14x–163x–2=41 water is added, Then ratio between milk
=> 14x–16=12x–8 and water is 11:10.
=> 14x–12x=16–8 X+(0.1X)Y+(0.2Y)=1110
=> x=82=4 1.1X1.2Y=1110
∴ Initial quantity of mixture in the vessel 11X12Y=1110
before the replacement = 17×4=68 litres X12Y=110
XY=1210
16) Answer (A) XY=65
Let’s assume that ratio of milk and water Y = 5X6 Eq.(2)
is 7Y : 6Y respectively. Put Eq.(2) in Eq.(1).
Now 12 liters of water is added into the X–5X6=10
mixture. X–5X6=10
So 7Y6Y+12 6X−5X6=10
As per question this ratio is equal to the X = 60. Eq.(3)
new ratio which is 13:12. Put Eq.(3) in Eq.(1).
So 7Y6Y+12=1312 60 – Y = 10
Now take 6 common from denominator. 60 – 10 = Y
Page 247 Follow us: Official Site, Telegram, Facebook, Instagram, Instamojo
Y = 50.
Hence, option e is the correct answer

Page 248 Follow us: Official Site, Telegram, Facebook, Instagram, Instamojo
ENGLISH LANGUAGE SECTION
It is very important to learn English Language if you are preparing for any competitive exam and this
language is used as an official language in most of the nations around the globe and has become a
prominent language for communication. Just like any other language, English too has its own semantics
and symbols. The symbols are the mere alphabets and it’s the semantics part that you need to master.

Grammar and Vocabulary are the key to master any language and same is the case with English. Unlike
mathematics, you need not learn and by heart one formula after another, you don’t have to perform any
calculation here, you just need to understand the grammar rules and build your English Vocabulary.

Topics Covered
Reading Comprehension
Cloze Test
Fill in the Blanks
Error Detection
Sentence Rearrangement / Parajumble
Phrase Replacement
Sentence Improvement
Words Replacement

Reading Comprehension
How to Master Reading Comprehension to Score Maximum Marks?
English Language is one section that scares most of the candidates as it doesn’t have a
defined syllabus and anything can be asked from any topic of grammar but one topic that
surely comes is Reading comprehensions and carries a lot of weighatge. Be it any exam
like: SBI, IBPS or any other banking exam reading Comprehension is one such topic that
majorly dominates the English Language section & you can easily expect approximately
10-15 marks questions from this section.
This topic not only dominates in prelims but in Mains as well we have seen that a major
chunk of 20-25 marks out of 40 are allocated to this topic. Students cannot afford to give
it a cold shoulder to this topic and sail through this recruitment process. The number of
marks given to makes it important from exam point of view and you can understand the
importance of this topic in clearing both the stages of prelims as well as mains of banking
sector recruitment.
Page 249 Follow us: Official Site, Telegram, Facebook, Instagram, Instamojo
Tips and Tricks To Solve Reading Comprehension For Bank And Other Competitive Exams

 Read the Questions First:


Time plays a very crucial role in the Examination. It is advised not to waste time reading
the whole passage, instead start from reading questions first. Once you get the needed
lines of the answer in the RC, then compare it with the options given in the question. If you
are unable to locate an answer then move to the next question, do not get stuck on any
question!! Through this approach, you will be able to find the theme and flow of passage in
the shortest span of time. Pick out the keywords and then go the passage and look for your
answer. This reverse approach saves your time.

 Solve Vocabulary Questions:


There are 1-2 Questions based on antonyms and synonyms in every RC. Try to solve these
questions as they consume less time. To tackle questions related to vocabulary you can get
idea about the meaning of the word by reading the line in which the word lies and also the
line preceding or succeeding it. This trick will help you in getting the right
synonym/antonym of the word asked.

 Improve your Focus and Grasp:


Reading lengthy passages becomes boring at times and one cannot understand what is
written by the author. Try to improve your focus and grasp and while reading the passage,
make sure that you understand the gist of the passage.

 Focus on Opening and Closing paragraphs:


Questions such as the main idea question are based on the overall subject of the passage,
and majority of the times the answer for this question type lies in the first and last
paragraph of the passage. Ensure you derive the maximum possible information from
these paragraphs.

 Improve reading Speed:


Do not move your lips while reading, it slows you down. Reading Comprehension requires
you to be smart and smartness comes from practice. Developing daily reading habit will
help you enhance your reading speed and comprehending the passage quickly.

Practice a lot because making excuses never brings success. Daily Reading and practicing
will help you score well and achieve your dream job this year itself.

Reading Comprehension Set-1 (Internet Based )

Direction (1-10): Read the following passage carefully and answer the questions
given below

Page 250 Follow us: Official Site, Telegram, Facebook, Instagram, Instamojo
Paragraph 1:

2018 has been the year of privacy. News of Facebook’s exposure of tens of millions of user
accounts to data firm Cambridge Analytica broke in March — a scandal that was only
compounded by recent news that the tech giant shared even more private data through
hidden agreements with other companies. Then in May, the European Union’s General
Data Protection Regulation, the world’s most stringent privacy law, came into effect. By
the end of the year, even Apple’s and Microsoft’s CEOs were calling for new national
privacy standards in the United States. It’s not just a coincidence that privacy issues
dominated 2018. These events are symptoms of larger, profound shifts in the world of
data privacy and security that have major implications for how organizations think about
and manage both.

Paragraph 2:

So what, exactly, is changing?

Put simply, privacy and security are converging, thanks to the rise of big data and machine
learning. What was once an abstract concept designed to protect expectations about our
own data is now becoming more concrete, and more critical on par with the threat of
adversaries accessing our data without authorization. More specifically, the threat
of unauthorized access to our data used to pose the biggest danger to our digital selves that
was a world in which we worried about intruders attempting to get at data we wanted
private. And it was a world in which privacy and security were largely separate functions,
where privacy took a backseat to the more tangible concerns over security. Today,
however, the biggest risk to our privacy and our security has become the threat
of unintended inferences, due to the power of increasingly widespread machine learning
techniques. Once we generate data, anyone who possesses enough of it can be a threat,
posing new dangers to both our privacy and our security. These inferences may, for
example, threaten our anonymity — like when a group of researchers used machine
learning techniques to identify authorship of written text based simply on patterns in
language. (Similar techniques have been used to identify software developers based
simply on the code they’ve written.)

Paragraph 3:

These inferences might reveal information about our political leanings like when
researchers used the prevalence of certain types of cars in Google’s Street View image
database to determine local political affiliations. Or these inferences might also indicate
intimate details about our health like when researchers used online search history

Page 251 Follow us: Official Site, Telegram, Facebook, Instagram, Instamojo
to detect neurodegenerative disorders such as Alzheimer’s. So what does a world look like
when privacy and security are focused on preventing the same harms? To start with,
privacy will no longer be the merely immaterial or political concept it once was. Instead,
privacy will begin to have substantial impacts on businesses’ bottom lines — something
we began to see in 2018. Facebook, for example, lost a whopping $119 billion in market
capitalization in the wake of the Cambridge Analytica scandal because of concerns over
privacy. Polls show that consumers are increasingly concerned about privacy issues. And
governments around the world are reacting with new privacy legislation of their own.

Paragraph4:

Within organizations, this convergence also means that the once clear line between
privacy and security teams is beginning to blur a trend that businesses in general, and
security and privacy practitioners in particular, should embrace. From a practical
perspective, this means that legal and privacy personnel will become more technical, and
technical personnel will become more familiar with legal and compliance mandates. The
idea of two distinct teams, operating independent of each other, will become a relic of the
past. Individuals and governments alike should no longer expect consent to play a
meaningful role in protecting our privacy. Because the threat of unintended inferences
reduces our ability to understand the value of our data, our expectations about our
privacy and therefore what we can meaningfully consent to are becoming less
consequential. Being surprised at the nature of the violation, in short, will become an
inherent feature of future privacy and security harms.

Paragraph5:

This is precisely why the recent string of massive data breaches, from the Marriott breach
that impacted 500 million guests to the Yahoo breach that affected 3 billion users, are so
troubling. ____________________________________________________the problem is all the unforeseen
uses and all the intimate inferences that this volume of data can generate going forward)
It is for this reason that legal scholars such as Oxford’s Sandra Watcher are now proposing
legal constraints around the ability to perform this type of pattern recognition at all. Once
described by Supreme Court Justice Louis Brandeis as “the right to be let alone,” privacy is
now best described as the ability to control data we cannot stop generating, giving rise to
inferences we can’t predict. And because we create more and more data every day an
estimated 2.5 quintillion bytes of it these issues will only become more pressing over
time. If we thought that 2018 was dominated by privacy concerns, just wait until 2019.

1) What is the tone of the passage? a) Sad

Page 252 Follow us: Official Site, Telegram, Facebook, Instagram, Instamojo
b) Analytical a) 2018 has showed good growth in
privacy issues when compared to before
c) Fearful years

d) Joy b) The number of cybercrimes reduced in


2018
e) Both b) & c)
c) The threat of privacy and personal data
2) What can be the correct theme of increased enormously in 2018 when
the passage? compared to before years

a) Facebook Scandal d) All of the above

b) Political allegations on privacy e) None of the above

c) Privacy and Cyber security 5) Facebook scandal of which firm


broke out in 2018?
d) All of the above
a) Cambridge Analytica
e) None of the above
b) Oxford Analytica
3) What is the world’s most stringent
Privacy law? c) Harvard Analytica

a) American Law d) All of the above

b) European Law e) None of the above

c) Dutch Law 6) Which of the following is closest


meaning of word “Prevalence”
d) French Law mentioned in the passage?

e) None of the above a) Dynamic

4) What did the author mean by saying b) Generality


2018 as the year of privacy?
c) Anxiety

Page 253 Follow us: Official Site, Telegram, Facebook, Instagram, Instamojo
d) All of these a) When leaders assume that their
meetings are going well, they are less apt
e) Courage to solicit feedback and seek opportunities
to improve
7) Which of the following is closest
meaning of word “Convergence” b) One recent study found that the effects
mentioned in the passage? of a bad meeting can linger for hours in
the form of attendee grousing and
a) Action complaining—a phenomenon dubbed
“meeting recovery syndrome.”
b) Punctuality
c) Better meeting leadership requires
c) Combining better self-observation. Take a few
minutes after each meeting you run to
d) Extraordinary reflect.

e) All of the above d) In addition to these routine scans,


check in periodically with people who
8) Which of the following is the closest attend your meetings.
meaning of the word “Precisely”
mentioned in the passage? e) The problem isn’t simply that
unauthorized intruders accessed these
a) Exact records at a single point in time.

b) Sharp 10) After Cambridge scandal Facebook


lost _____ in market capitalization
c) Definite
a) $120 billion
d) Absolute
b) $119 billion
e) All of the above
c) $300 billion
9) Which of the following will
replace________________________ in the d) $780 billion
passage?
e) $250 billion
Answers:

Page 254 Follow us: Official Site, Telegram, Facebook, Instagram, Instamojo
1) Answer: e) accounts to data firm Cambridge
Analytica broke in March
As the author explained cyber security
problem with few examples and their 6) Answer: b)
analysis and also represented some fear
regarding privacy issues option e) will be Prevalence: Commonness, Ubiquity,
appropriate tone for the passage. Regularity

2) Answer: c) 7) Answer: c)

As the author highlighted Privacy and Convergence: Unification,Connection,


cyber security as the main problems in Coupling
the passage option c) will be appropriate
theme for the passage 8) Answer: e)

3) Answer: b) Precisely: without vagueness which is


accurately defined
Refer 1st Paragraph: Then in May, the
European Union’s General Data 9) Answer: e)
Protection Regulation, the world’s most
stringent privacy law. When compared to remaining options;
option e) is explaining the exact theme of
4) Answer: c) the passage and can be linked with the
continuation sentence also.
The author means the threat of privacy
and personal data increased enormously 10) Answer: b)
in 2018 when compared to before years
Refer 3rd Paragraph: Facebook, for
5) Answer: a) example, lost a whopping $119 billion in
market capitalization in the wake of the
Refer 1st paragraph: News of Facebook’s Cambridge Analytical scandal because of
exposure of tens of millions of user concerns over privacy.
Reading Comprehension Set-2 (Political Based )

Direction (1-5): Read the passage carefully and answer the questions given below it.
There are some blanks given in the passage based on which some questions are
framed, and some words are highlighted as well to help you answer some of the
questions.
Page 255 Follow us: Official Site, Telegram, Facebook, Instagram, Instamojo
But international support for the Amazon has been tepid. This was clear last month in
Altamira, northern Brazil, at the aptly named Amazon: Centre of the World
gathering. (A) In the days before the meeting, rightwing agitators(1) called for farmers,
cattle ranchers(2), police and other “patriots” to mobilise
against sovereignty(3) communities, environmentalists and human rights groups taking
part, that they claimed were “eco-socialists” working for international interests against
Brazil’s traditional(4) and economic development. These messages were enough to
spook two foreign organisations – a huge environmental NGO and one of the world’s
biggest foundations – who pulled out of the event., rather than risk becoming embroiled in
a potentially tense stand-off. Other foreign groups were uncowed. For domestic activists,
threats are a fact of life – and they find their own way to deal with them. At the opening
session, a group of land grabbers – some wrapped in the Brazilian flag – shoved their way
to the front, jostled the speakers and disrupted proceedings. They were pushed back by
Kayapo warriors in war paint, while other activists formed a human barrier to enable the
speakers to continue.(B) It was symbolic:/ white farmers attempting to take over;/
Amazon dwellers defending their space,/ while prominent international supporters ran
scared. /

(C) This is a shame on our generation. The Amazon (along with the Congo and Papua New
Guinea, the oceans and other capitals of nature) should be as central
to ____________________________. Back then, working-class idealists joined public intellectuals in
the fight against fascism. George Orwell, Martha Gellhorn, WH Auden, Pablo Neruda,
Emma Goldman and Ernest Hemingway were among tens of thousands who risked their
lives reporting on the fighting or taking part in it as members of the International Brigade.
Some saw it as a civilisation-defining moment. Others described it in apocalyptic terms as
“the last great cause”. For Orwell– who was shot in the conflict – it was simply a fight for
“common decency”.

Unlike then, the threat to civilisation and decency is not a new ideology, but the
accumulated consequences of the old one. Bolsonaro, Donald Trump, Vladimir Putin,
Rodrigo Duterte, Recep Tayyip Erdoğan and other populists may often resemble fascists,
but actually they are arch-capitalists. Their main appeal to voters comes not from a
twisted vision of a future but from a promise to turn back the clock to a more stable age.
This is impossible, because manmade climate chaos is increasingly disrupting more lives
and economic activity. Until governments deal with that, all other battles will be futile.

That is why defence of the Amazon – and the broader struggle to restore nature – is
today’s “last great cause”. Move this issue from the periphery to the centre and everything
Page 256 Follow us: Official Site, Telegram, Facebook, Instagram, Instamojo
– global politics, economics and individual thinking – changes. (D) Ecology will
be seen(1) as more representation(2) than economy, long-term fertility(3) will come
ahead of destructive GDP growth, ecocide will be punishable in criminal courts, future
generations and other species will be given democratic fundamental(4), and school
curriculums will teach children how to maintain our home, planet Earth.

That may seem a distant prospect. The forces lined up against such a radical but necessary
shift in thinking have more political power and force of arms. The same was true in the
Spanish civil war. Then, the antifascists lost the war but, as the great historian Eric
Hobsbawm noted, they won the battle for ideas.(E) With so many writers, poets and
journalists/ on their side, the losers got to write the history for a change /– and these
shaped the debate for the bigger conflict /that was to come in the second world war.

Similarly, the battle for the Amazon cannot be won on the ground with guns and bombs,
but it can be shaped by opinions, money, consumer choices, street protests and
international pressure. It is no longer enough for today’s intellectuals, celebrities and
other opinion formers to declare support for the rainforest on social networks. More
people need to get out from behind their screens, to feel what nature provides and how it
is being lost.

1) The sentence given in (A) has four 2) In the above passage, sentence (B)
words given in bold. among the given may or may not have an error in one
bold words, which of the following part of the sentence, select the part
must replace each other to make the having error in it as your answer.
sentence grammatically correct and
meaningful? A) It was symbolic

A) 1-3 B) white farmers attempting to take over;

B) 1-4 C) Amazon dwellers defending their


space,
C) 2-3
D) while prominent international
D) 2-4 supporters ran scared.

E) 3-4 E) No error

Page 257 Follow us: Official Site, Telegram, Facebook, Instagram, Instamojo
3) Which of the following phrase A) 1-3
should fill the blank (C) to make it
contextually and grammatically B) 1-4
correct and meaningful?
C) 2-3
A) foreign governments need to step up
and help Brazil recognise the value of the D) 2-4
rainforest
E) 3-4
B) global debate and international
activism as the Spanish civil war was in 5) In the above passage, sentence (E)
the 1930s may or may not have an error in one
part of the sentence, select the part
C) rather than leaving the battle to having error in it as your answer.
courageous but outmuscled and
outmoneyed traditional communities A) With so many writers, poets and
journalists
D) nature should be pivotal in all
decision-making B) on their side, the losers got to write the
history for a change
E) None of these
C) and these shaped the debate for the
4) The sentence given in (D) has four bigger conflict
words given in bold. amongst the given
bold words which of the following D) that was to come in the second world
must replace each other to make the war
sentence contextually correct and
meaningful? E) No error

Answers :

1) Answer: E communities, environmentalists and


human rights groups taking part, that
After the replacement the thus formed they claimed were “eco-socialists”
sentence is “In the days before the working for international interests
meeting, rightwing agitators called for against Brazil’s sovereignty and economic
farmers, cattle ranchers, police and other development.”
“patriots” to mobilise against traditional

Page 258 Follow us: Official Site, Telegram, Facebook, Instagram, Instamojo
2) Answer: E After making the replacements, the thus
formed sentence is “Ecology will be seen
All parts of the sentence is correct. as more fundamental than economy, long-
term fertility will come ahead of
3) Answer: B destructive GDP growth, ecocide will be
punishable in criminal courts, future
This is a shame on our generation. The generations and other species will be
Amazon (along with the Congo and Papua given democratic representation, and
New Guinea, the oceans and other capitals school curriculums will teach children
of nature) should be as central to global how to maintain our home, planet Earth.”
debate and international activism as the
Spanish civil war was in the 1930s. 5) Answer: C

4) Answer: D ‘These’ should be replaced with ‘this’.


Thus, option C is the correct answer here.
Reading Comprehension Set-3 ( Science Based )

Direction (1-5): Read the passage carefully and answer the questions given below it.
There are some blanks given in the passage based on which some questions are
framed, and some words are highlighted as well to help you answer some of the
questions.

As many as four potentially hazardous asteroids went past the Earth late on Tuesday.
Three out of four of these ………………. (A)rocks were barely discovered a few hours before
zipping past the Earth-Moon system, the Daily Express reported. One asteroid called 2019
SM8 was seen by space researchers at the Mount Lemmon Observatory in Arizona on
Monday, which flew by our planet hours later, according to NASA. The asteroid 2019 SM8
at its closest was approximately 99,000 miles (159,000 kilometres) away from Earth,
which is a little less than half of the average distance between the Moon and our planet, a
report by Space.com said.

According to NASA’s estimate, this particular asteroid was about 16 feet (4.8 metres) in
diameter, which is approximately the size of an SUV. (B) Around an hour later,
another celestial (1) was discovered, however, this asteroid (2) rock named asteroid
2019 SE8 flew from a distance (3) of approximately 674,000 miles (1.1 million
kilometres) from the Earth and hence did not prove to be a threat (4). In terms of size,
this asteroid was comparatively bigger than the previous one. It was estimated to be
around 47 feet (14 metres) in size. (C) Apart from these, two more celestial rocks named
Page 259 Follow us: Official Site, Telegram, Facebook, Instagram, Instamojo
asteroid 2019 SD8 and asteroid 2018 FK5 were detected by the US space agency, which
passed approximately 331,000 miles (532,000 kilometres) and 3 million miles (5 million
kilometres) far away from the Earth respectively. Even though none of these asteroids had
much chance of hitting our planet, however, these rocks are classified by NASA as
potentially ……………….. (D)asteroids as they pose a threat in their future course when
their orbits intersect that of our planet. In the past, some key space researchers and
entrepreneurs have pointed out fears regarding asteroids hitting the Earth.

In August, SpaceX and Tesla CEO Elon Musk had pointed out on Twitter that the Earth
presently has no defence against ‘killer’ asteroids. Responding to a tweet about the
asteroid Apophis — dubbed as the ‘God of Chaos’ that is expected to scrape past Earth in
2029, Musk pointed out that there is, currently, no defence system to protect our
planet. …………………… (E), a famous astrophysicist Neil deGrasse Tyson had also warned
about the Apophis 99942 hitting the Earth, causing a major tsunami that can wipe out the
entire west coast of North America. Researchers have said that the asteroid which wiped
away dinosaurs is estimated to have been equivalent to 10 billion atomic bombs that were
used in World War II. The impact of that giant asteroid had triggered massive tsunamis
and let to wildfires that were thousands of miles away.

1) Which of the following word given C) alluvial


in the options should come at the place
marked as (A) in the above passage to D) terrestrial
make it grammatically correct and
meaningful? Also, the word should fill E) None of the above
in the two sentences given below to
make them contextually correct and 2) The sentence given in (B) has four
meaningful? words given in bold. Amongst the given
bold words which of the following
(I) There were light bulbs representing must replace each other to make the
the stars and line drawings of the sentence contextually correct and
………………….. figures. meaningful?

(II) The moon is a ……………… body. A) Both 2-1 and 3-4

A) platonic B) Both 1-4 and 2-3

B) celestial C) Both 2-4 and 1-3

Page 260 Follow us: Official Site, Telegram, Facebook, Instagram, Instamojo
D) 1-2 A) hazardous

E) 3-4 B) envious

3) In the above passage, sentence (C) C) contagious


may or may not have an error in one
part of the sentence, select the part D) voracious
having error in it as your answer.
E) None of the above.
A) Apart from these, two more celestial
rocks named asteroid 2019 SD8 and 5) Which of the following phrase
asteroid 2018 FK5 should fill the blank (F) to make it
contextually and grammatically
B) were detected by the US space agency, correct and meaningful?
which passed approximately 331,000
miles (532,000 kilometres) A) Musk’s priority

C) and 3 million miles (5 million B) Prioritising Musk


kilometres)
C) Prior of Musk
D) far away from the Earth respectively.
D) Prior to Musk
E) Both (a) and (b)
E) None of the above
4) Which of the following should fill
the blank given in (D) to make it
contextually correct and meaningful?

Answers :

1) Answer: (b) ‘platonic’ means (of love or friendship)


intimate and affectionate but not sexual.
‘celestial’ means positioned in or relating
to the sky, or outer space as observed in ‘alluvial’ means relating to or derived
astronomy.. It perfectly fits in the blank from alluvium.
(A) and in the given two statements as
well. Hence, option (A) is the correct ‘terrestrial’ means of, on, or relating to the
answer choice. earth.

Page 261 Follow us: Official Site, Telegram, Facebook, Instagram, Instamojo
2) Answer: (d) 4) Answer: (a)

After making the replacements, the thus The Blank (D) is suitably fit by
formed is ‘Around an hour later, ‘hazardous’ which means risky;
another asteroid was discovered, dangerous.
however, this celestial rock named
asteroid 2019 SE8 flew from a distance of ‘envious’ means feeling or showing envy.
approximately 674,000 miles (1.1 million
kilometres) from the Earth and hence did ‘contagious’ means (of a disease) spread
not prove to be a threat’. from one person or organism to another
by direct or indirect contact.
3) Answer: (d)
‘voracious’ means wanting or devouring
Here only ‘away’ should come in place of great quantities of food.
‘far away’. The error free sentence is
“Apart from these, two more celestial 5) Answer: (d)
rocks named asteroid 2019 SD8 and
asteroid 2018 FK5 were detected by the Option D is suitable to fill in the blank (F)
US space agency, which passed making the sentence both grammatically
approximately 331,000 miles (532,000 and contextually right.
kilometres) and 3 million miles (5 million
kilometres) away from the Earth
respectively. ”
Reading Comprehension Set-4 (Social Based )

Direction (1-5): Read the passage carefully and answer the questions given below it.
Certain words/ phrases have been given in bold to help you locate them while
answering some of the questions.

Anybody who has watched the quick succession of aircraft landings and takeoffs at Delhi
or Mumbai airports would be aware of the time precision required. An air-traffic-control
error of less than 30 seconds could be disastrous. Yet, the entire civil aviation sector,
including passengers, takes such clockwork operations for granted. The same cannot be
said about the railways, or other means of public transport in most of India. Nor can it be
said about our social behaviour as a nation, where being on time for a meeting, party or
wedding often puts you at risk of being ahead of the hosts themselves. Often, not being on

Page 262 Follow us: Official Site, Telegram, Facebook, Instagram, Instamojo
time is also seen a power statement—it’s a mark of status to have a roomful of people wait
for your arrival.

We, as citizens of India, seem to believe that we are “like that only”, culturally hardwired
to say “chaltahai” as a catch-all pardon for anything slipshod. As a visitor once wryly
observed of Indian timeliness, “two minutes” here could mean anything from “right now”
to an eternity. But habits and a “culture” of lateness are not encoded in anybody’s genes.
It’s not about nature, but nurture. And that could change. Difficult as it is to believe, the
Japanese were known for their ambivalence towards time just a century ago. In fact, the
big societal change that turned Japan into a nation always on time took place only after the
shock of World War II. In a paper titled Japanese Clocks and the History of Punctuality in
Modern Japan, Takehiko Hashimoto, a professor of history and technology, maps out its
transition from the tardy to the time-bound country it is today.

When the colonizing West encountered the Japanese East on factory shop floors, at train
stations and at shipping docks in mid- to late-19th century Japan, Westerners found that
“the Japanese worked with an apparent indifference to the clock”. Sensitivity increased
after being on time in schools, factories, docks and work places became mandatory. But in
their private spaces, people were still on the “variable hour system of seasonal time”. How
did that change? Cheap quartz watches on every wrist, a realization that time is money,
social movements, and a leadership push in 1960, when Japan’s then Prime
MinisterHayato Ikeda galvanized the country to double its national income in a decade. It
was largely over the 1960s that Japan transformed its consciousness of time and what it
meant for daily life. If the Japanese could throw off the sloth of the past without any
cultural loss, so can we. The conditions are ripe for a sharp habitual shift in India—a
prime minister who is leading citizens to break centuries-old habits like open defecation, a
buoyant middle class, 300 million aspirants to the mid-income bracket, and a national
desire to shed all that held us back, so that a quantum leap can be taken for prosperity.
Some signs of change are already visible. Metro trains run mostly on schedule, the
government issuance of passports, driving licences and other such documents are now
time-bound. A major digital push aims at removing the human interface that delays
services which citizens have a right to. Changes at the workplace may take a while to reach
the wedding pandal, but don’t write off the possibility. Success is not inevitable, but maybe
25 years later, the world will be saying, “If it’s on time, it’s Indian.”

1) Which of the following is true I) Japanese were known for their


regarding Japanese people the punctuality towards time just a century
passage? ago.
Page 263 Follow us: Official Site, Telegram, Facebook, Instagram, Instamojo
II) Westerners found that the Japanese D) A major digital push aims at removing
worked with an apparent indifference to the human interface that delays services
the clock. which citizens have a right to.

III) In a paper titled Japanese Clocks and E) None of the above.


the History of Punctuality in Modern
Japan, Takehiko Hashimoto, a professor of 3) Which of the following is/are
history and technology, maps out its synonymous with wryly observation?
transition from the tardy to the time-
bound country it is today. I) As a visitor once wryly observed of
Indian timeliness, “two minutes” here
A) I & II could mean anything from “right now” to
an eternity.
B) I & III
II) The big societal change turned Japan
C) II & III into a nation, always on time, took place
only after the shock of World War II.
D) All of the above
III) Japanese could throw off the sloth of
E) None of the above the past without any cultural loss.

2) Which of the following statements A) I & II


does not satisfy with the habitual shift
in India? B) I & III

A) Metro trains run mostly on schedule. C) Only I

B) In a country like India, there is no one D) All of the above


to usher people to break centuries-old
habits that held us back, so that a E) None of the above
quantum leap can be taken for prosperity.
4) Choose the word/group of words
C) The time bound issuance of passports, which is most opposite in meaning to
driving licences and other such the word/ group of words printed in
documents. bold as used in the passage.

Slipshod

Page 264 Follow us: Official Site, Telegram, Facebook, Instagram, Instamojo
A) Token Ambivalence

B) Slapdash A) Decisiveness

C) Painstaking B) Equivocation

D) Attestation C) Resolution

E) None of the above. D) Redundancy

5) Choose the word/group of words E) None of the above.


which is most similar in meaning to the
word/ group of words printed in bold
as used in the passage.

Answers (1-5)

1) Answer: (c) and work places became mandatory. But


in their private spaces, people were still
Japanese were known for their on the “variable hour system of seasonal
ambivalence towards time just a century time”.
ago. In fact, the big societal change that
turned Japan into a nation always on time 2) Answer: (b)
took place only after the shock of World
War II. In a paper titled Japanese Clocks The conditions are ripe for a sharp
And The History of Punctuality In Modern habitual shift in India—a prime minister
Japan, Takehiko Hashimoto, a professor of who is leading citizens to break centuries-
history and technology, maps out its old habits like open defecation, a buoyant
transition from the tardy to the time- middle class, 300 million aspirants to the
bound country it is today. When the mid-income bracket, and a national desire
colonizing West encountered the to shed all that held us back, so that a
Japanese East on factory shop floors, at quantum leap can be taken for prosperity.
train stations and at shipping docks in Some signs of change are already visible.
mid- to late-19th century Japan, Metro trains run mostly on schedule, the
Westerners found that “the Japanese government issuance of passports,
worked with an apparent indifference to driving licences and other such
the clock”. Sensitivity increased after documents are now time-bound. A major
being on time in schools, factories, docks digital push aims at removing the human

Page 265 Follow us: Official Site, Telegram, Facebook, Instagram, Instamojo
interface that delays services which ‘Slapdash’ means done too hurriedly and
citizens have a right to. Changes at the carelessly.
workplace may take a while to reach the
wedding pandal, but don’t write off the ‘Painstaking’ means done with or
possibility. Success is not inevitable, but employing great care and thoroughness.
maybe 25 years later, the world will be
saying, “If it’s on time, it’s Indian.” ‘Attestation’ means evidence or proof of
something.
3) Answer: (c)
5) Answer: (b)
As a visitor once wryly observed of Indian
timeliness, “two minutes” here could ‘Ambivalence’ means the state of having
mean anything from “right now” to an mixed feelings or contradictory ideas
eternity. But habits and a “culture” of about something or someone.
lateness are not encoded in anybody’s
genes. It’s not about nature, but nurture. ‘Decisiveness’ means the ability to make
And that could change. decisions quickly and effectively.

4) Answer: (c) ‘Equivocation’ means the use of


ambiguous language to conceal the truth
‘Slipshod’ means (typically of a person or or to avoid committing oneself;
method of work) characterized by a lack prevarication.
of care, thought, or organization.
‘Resolution’ means a firm decision to do
‘Token’ means a voucher that can be or not to do something.
exchanged for goods or services, typically
one given as a gift or offered as part of a ‘Redundancy’ means the state of being not
promotional offer. or no longer needed or useful.

Reading Compresension Set-5 (Technology Based)

Most money these days is electronic—a series of ones and zeros on a computer. So it is
rather neat that bitcoin, a privately created electronic currency, has lurched from $1,000
to above $10,000 this year, an epic journey to add an extra zero. On the way, the currency
has been controversial. Jamie Dimon, the boss of JPMorgan Chase, has called it a fraud.
Nouriel Roubini, an economist, plumped for “gigantic speculative bubble”. Ordinary
investors are being tempted into bitcoin by its rapid rise—a phenomenon dubbed FOMO
(fear of missing out). Both the Chicago Mercantile Exchange, America’s largest futures

Page 266 Follow us: Official Site, Telegram, Facebook, Instagram, Instamojo
market, and the NASDAQ stock exchange have seemingly added their imprimaturs by
planning to offer bitcoin-futures contracts. It is easy to muddle two separate issues. One is
whether the “blockchain” technology that underpins bitcoin becomes more widely
adopted. Blockchains, distributed ledgers that record transactions securely, may prove
very useful in some areas of finance, and beyond. The second is whether bitcoin will
become a widely adopted currency in everyday life. Here the evidence is weak. Bitcoin can
be used to buy a few things. But a currency has three main functions: store of value; means
of exchange; and unit of account. Bitcoin’s volatility, seen when it fell 20% within minutes
on November 29th before rebounding, makes it both a nerve-racking store of value and a
poor means of exchange. Imagine buying an iPhone X with bitcoin in January. You would
by now be cursing as the same coin could buy ten phones—Christmas gifts for the whole
family.

A currency is also a unit of account for debt. Paul Mortimer-Lee of BNP Paribas, a French
bank, tartly remarks: “Imagine if you had financed your house with a bitcoin mortgage.”
This year your debt would have risen tenfold. Your salary, paid in dollars, euros or
whatever, would not have kept pace. Put another way, had bitcoin been widely used, the
last year might have been massively deflationary. Such issues will be of minor concern to
those who managed to buy bitcoin earlier in the year. They will just be delighted with the
profits. But why has the price risen so fast? One justification for the existence of bitcoin is
that central banks, via quantitative easing (QE), are debasing fiat money and laying the
path to hyperinflation. But this seems a very odd moment for that view to gain adherents.
Inflation remains low and the Federal Reserve is pushing up interest rates and unwinding
QE.A more likely explanation is that as new and easier ways to trade in bitcoin become
available, more investors are willing to take the plunge. As the supply of bitcoin is limited
by design, that drives up the price. But it is worth remembering that the cost of using
bitcoin is going up. Each transaction has to be verified by “miners” who need a lot of
computing power to do so, and a lot of energy: 275kWh for every transaction, according to
Digiconomist, a website. In total, bitcoin uses as much electricity a year as Morocco, or
enough to power 2.8m American households. All this costs much than processing credit-
card transactions via Visa or MasterCard. The miners are rewarded for their efforts by
being paid in bitcoin; they are delighted by the rise in the currency’s price. But some are
finding ingenious ways to cut back on their energy costs; one even put computers in his
Tesla car so he could mine bitcoins using its free charging stations. Much mining is done in
parts of China where electricity is cheap. There are two ways of thinking about this. One is
that the eventual price of bitcoin will equal the marginal cost of mining, which may be
rising but is well below the current price. The second is that institutions will not want to

Page 267 Follow us: Official Site, Telegram, Facebook, Instagram, Instamojo
use the technology if it relies on such a “Wild West” process; banks are already looking at
cheaper forms of blockchain technology.

Whether the investors driving the price higher are pondering all this is open to doubt. It
looks like a re-run of the dotcom craze. Adverts for trading digital currencies are
appearing on the London tube and celebrities have piled onto the bandwagon. As seen
many times before, when lots of investors buy an illiquid asset, the price can rise
exponentially. The top is hard to call. At some point, the urge to turn all those digital zeros
into cars and iPhones will prove too great. Getting out of an illiquid asset—as this week,
when exchanges struggled to cope with trading volumes—can be harder than getting into
it. Some remember Nathan Rothschild’s remark about the secret of his wealth: “I always
sold too soon.”

1. Why the bitcoin has been


controversial? 3. Why bitcoins cannot be
(i) Because it was regularly criticized. adopted as currency of
(ii) Jamie Dimon called it a fraud. everyday life according to the
(iii) Economist Nouriel Roubini called passage?
it a gigantic bubble. (i) According to the passage, volatility
a) Only i of the bitcoin is the reason.
b) Only ii (ii) Bitcoin may be a bubble according
c) i & iii to economist, Nouriel Roubini.
d) i & ii (iii) Boss of JPMorgan Chase has called
e) All of the above bitcoin a fraud.
a) Only i
2. What is the meaning of FOMO b) Only ii
in the context of passage? c) i & iii
(i) People don’t want to miss the d) i & ii
chance to invest in Bitcoin. e) None of the above
(ii) It is the fear for investing in
Bitcoin. 4. What is the reason described
(iii) Fear of rapid rise in bitcoin’s price. in the passage for increase in
a) Only i the price of bitcoin?
b) Only ii (i) Because it is easier to trade now.
c) i & iii (ii) Supply of bitcoin is limited.
d) i & ii (iii) Number of investors of bitcoin
e) None of the above increased day by day.
a) Only i

Page 268 Follow us: Official Site, Telegram, Facebook, Instagram, Instamojo
b) Only ii 7) Which of the following can
c) i & iii be the nearest synonyms of
d) i & ii ‘lurched’?
e) All of the above a) Stumbled
b) Increased
5. What is the cons described in c) Risen
the passage for transacting d) Stabilized
bitcoin? e) None of the above
(i) It is highly volatile in nature.
(ii) Transaction cost of bitcoin is very 8. Which of the following can be
high. the nearest synonyms of
(iii) It is not safe for transaction as it is ‘imprimaturs’?
digital currency. a) Approval
a) Only i b) Veto
b) Only ii c) Prohibition
c) i & iii d) Protocol
d) i & ii e) None of the above
e) None of the above
9. Which of the following can be
6. What is true about bitcoin the nearest antonyms of
mining? ‘nerve-racking’?
(i) A lot of energy is used for bitcoin a) Distressing
mining. b) Annoying
(ii) Mining is a process in which bitcoin c) Stressful
transaction is verified. d) Aggravating
(iii) Miners are being paid in bitcoin. e) None of the above
a) Only i
b) Only ii 10)Which of the following can be the
c) i & iii nearest synonyms of ‘adherents’?
d) i & ii a) Popularity
e) All of the above b) Moment
c) Positivity
d) Supporter
e) None of the above
Answers:

1.Correct Answer is: e, all of the above are the reason for bitcoin
being controversial.

Page 269 Follow us: Official Site, Telegram, Facebook, Instagram, Instamojo
2. Correct Answer is: a, all of the above are true regarding bitcoin
mining.
FOMO means fear of missing out.
7. Correct Answer is: a,
3. Correct Answer is: a,
lurched means sudden move.
volatility is the main reason due to which
bitcoin cannot be adopted as currency of
everyday life.
8. Correct Answer is: a,
4. Correct Answer is: e,
imprimaturs means official approval.
all of the above are the reasons for
increasing price of bitcoin. 9. Correct Answer is: e,

5. Correct Answer is: d, nerve-racking means annoying or


stressful.
transaction cost and volatility are the
main cons for transacting bitcoin. 10. Correct Answer is: d,

6. Correct Answer is: e, adherents means supporter.

Reading Comprehension Set-6 (Language Based )

Innovations in language are never completely new. When the words used for familiar
things change, or words for new things enter the language, they are usually borrowed or
adapted from stock. Assuming new roles, they drag their old meanings along behind them
like flickering shadow. This seems especially true of the language of the contemporary
school of literary criticism that now prefers to describe its work simply and rather
presumptuously as theory but is still popularly referred to as post structuralism of
deconstruction.

The first neologisms adopted by this movement were signifier and signified, employed
to distinguish arbitrariness of the term we choose. The use of these particular terms
(rather than, respectively, words and thing) underlined the seriousness of the naming
process and its claim on our attention. Since in English “to signify” can also mean “to
portend,” these terms also suggest that words predict coming events.

Page 270 Follow us: Official Site, Telegram, Facebook, Instagram, Instamojo
With the use of the term deconstruction we move into another and more complex realm of
meaning. The most common use of the terms construction and deconstruction is in the
building trades, and their borrowing by literary theorists for a new type of criticism
cannot help but have certain overtones to the outsider. First, the usage suggests that the
creation and critical interpretation of literature are not organic but mechanical processes;
that the author of any piece of writing is not an inspired, intuitive artist, but merely a
labourer who cobbles existing materials (words) into more or less conventional
structures. The term deconstruction implies that the text has been put together like a
building or a piece of machinery, and that it is in need of being taken apart, not so much in
order to repair it as to demonstrate underlying inadequacies, false assumptions, and
inherent contradictions. This process can supposedly be repeated many times and by
many literary hard hats; it is expected that each deconstruction will reveal additional
flaws and expose the illusions or bad faith of the builder. The fact that deconstructionists
prefer to describe their activities as deconstruction rather than criticism is also
revealing. Criticism and critic derive from the Greek Kritikos, “skillful in judging,
decisive.”Deconstruction, on the other hand, has no overtones of skill or wisdom; it merely
suggests demolition of an existing building. In popular usage criticism suggests censure
but not change. If we find fault with a building, we may condemn it, but we do not carry
out the demolition ourselves. The deconstructionist, by implication, is both judge and
executioner who leaves a text totally dismantled, if not reduced to a pile of rubble.

Questions

1) Which one of the following best expresses the main idea of the passage?

(A) Implicit in the terminology of the school of criticism known as deconstruction are
meanings that reveal the true nature of the deconstructionist’s endeavor.

(B) The appearance of the terms signifier and signified in the field of literary theory
anticipated the appearance of an even more radical idea known as deconstruction.

(C) Innovations in language and relations between old and new meanings of terms are a
special concern of the new school of criticism known as deconstruction.

(D) Deconstructionists maintain that it is insufficient merely to judge a work: the critic
must actively dismantle it.

(E) Progress in the field of literary theory is best achieved by looking for new terms like
signifier and deconstruction that might suggest new critical approaches to a work.
Page 271 Follow us: Official Site, Telegram, Facebook, Instagram, Instamojo
2) Which one of the following is a claim that the author of the passage makes about
deconstructionists?

(A) Deconstructionists would not have been able to formulate their views adequately
without the terms signifier and signified.

(B) Deconstructionists had no particular purpose in mind in choosing to use neologisms.

(C) Deconstructionists do not recognize that their own theory contains inherent
contradictions.

(D) Deconstructionists find little interest in the relationship between words and their
referents.

(E) Deconstructionists use the terms signifier and signified to stress the importance of the
process of naming.

3) Which one of the following generalizations about inventions is most analogous to


the author’s point about innovation in language?

(A) A new invention usually consists of components that are specifically manufactured for
the new invention.

(B) A new invention is usually behind the times, never making as much use of all the
available modern technology as it could.

(C) A new invention usually consists of components that are already available but are
made to function in new ways.

(D) A new invention is most useful when it is created with attention to the historical
tradition established by implements previously used to do the same job.

(E) A new invention is rarely used to its full potential because it is surrounded by out-of-
date technology that hinder its application.

4) The author of the passage uses the word “criticism” primarily in order to ?

(A) give an example

(B) introduce a contrast


Page 272 Follow us: Official Site, Telegram, Facebook, Instagram, Instamojo
(C) undermine an argument

(D) codify a system

(E) dismiss an objection

5) Which one of the following best describes the function of the second paragraph
within the passage as a whole?

(A) It introduces a hypothesis that the author later expands upon.

(B) It qualifies a claim made earlier by the author.

(C) It develops an initial example of the author’s general thesis.

(D) It predicts a development.

(E) It presents a contrasting view.

6) The passage suggests that the author most probably holds the view that an
important characteristic of literary criticism is that it ?

(A) demonstrate false assumptions and inherent contradictions

(B) employ skill and insight

(C) be carried out by one critic rather than many

(D) reveal how a text is put together like a building

(E) point out the superiority of conventional text structures

7) The passage suggests that which one of the following most accurately describes
the author’s view of deconstructionist thought?

(A) The author is guardedly optimistic about the ability of deconstruction to reveal the
intentions and biases of a writer.

(B) The author endorses the utility of deconstruction for revealing the role of older
meanings of words.

Page 273 Follow us: Official Site, Telegram, Facebook, Instagram, Instamojo
(C) The author is enthusiastic about the significant neologisms that deconstruction has
introduced into literary criticism.

(D) The author regards deconstruction’s tendency to focus only on the problems and
faults of literary texts as too mechanical.

(E) The author condemns deconstruction’s attempts to define literary criticism as a


creative act.

8) The meaning of ‘flickering’ is ?

(A) fixed

(B) jammed

(C) unstable

(D) narrow

(E) none of the above

9) The meaning of ‘distinguish’ is ?

(A) similar

(B) unique

(C) differentiate

(D) photo copy

(E) none of the above

10) The antonym of ‘demolish’ ?

(A) destroy

(B) construct

(C) ruin

Page 274 Follow us: Official Site, Telegram, Facebook, Instagram, Instamojo
(D) wreck

(E) none of the above

Answers

1) Answer: A

From reading the whole passage, this question can be answered.

Especially , from the lines “Deconstruction, on the other hand, has no overtones of skill or
wisdom; it merely suggests demolition of an existing building. In popular usage criticism
suggests censure but not change. If we find fault with a building, we may condemn it, but
we do not carry out the demolition ourselves. The deconstructionist, by implication, is
both judge and executioner who leaves a text totally dismantled, if not reduced to a pile of
rubble..”

2) Answer: E

From reading the lines,

“The first neologisms adopted by this movement were signifier and signified, employed to
distinguish arbitrariness of the term we choose. The use of these particular terms (rather
than, respectively, words and thing) underlined the seriousness of the naming process and
its claim on our attention. Since in English “to signify” can also mean “to portend,” these
terms also suggest that words predict coming events.

With the use of the term deconstruction we move into another and more complex realm of
meaning. The most common use of the terms construction and deconstruction is in the
building trades, and their borrowing by literary theorists for a new type of criticism
cannot help but have certain overtones to the outsider.” this question can be answered.

3) Answer: C

From reading the lines,

“Innovations in language are never completely new. When the words used for familiar
things change, or words for new things enter the language, they are usually borrowed or
adapted from stock. Assuming new roles, they drag their old meanings along behind them

Page 275 Follow us: Official Site, Telegram, Facebook, Instagram, Instamojo
like flickering shadow. This seems especially true of the language of the contemporary
school of literary criticism that now prefers to describe its work simply and rather
presumptuously as theory but is still popularly referred to as post structuralism of
deconstruction.” this question can be answered.

4) Answer: B

From reading the lines,

“The fact that deconstructionists prefer to describe their activities as deconstruction


rather than criticism is also revealing. Criticism and critic derive from the Greek Kritikos,
“skillful in judging, decisive.”Deconstruction, on the other hand, has no overtones of skill
or wisdom; it merely suggests demolition of an existing building. In popular usage
criticism suggests censure but not change. If we find fault with a building, we may
condemn it, but we do not carry out the demolition ourselves.” this question can be
answered.

5) Answer: C

From reading the lines,

“The first neologisms adopted by this movement were signifier and signified, employed to
distinguish arbitrariness of the term we choose. The use of these particular terms (rather
than, respectively, words and thing) underlined the seriousness of the naming process and
its claim on our attention. Since in English “to signify” can also mean “to portend,” these
terms also suggest that words predict coming events.” this question can be answered.

6) Answer: B

From reading the whole passage, this question can be answered.

7) Answer: D

From reading the lines,

“With the use of the term deconstruction we move into another and more complex realm
of meaning. The most common use of the terms construction and deconstruction is in the
building trades, and their borrowing by literary theorists for a new type of criticism
cannot help but have certain overtones to the outsider. First, the usage suggests that the

Page 276 Follow us: Official Site, Telegram, Facebook, Instagram, Instamojo
creation and critical interpretation of literature are not organic but mechanical processes;
that the author of any piece of writing is not an inspired, intuitive artist, but merely a
labourer who cobbles existing materials (words) into more or less conventional
structures. The term deconstruction implies that the text has been put together like a
building or a piece of machinery, and that it is in need of being taken apart, not so much in
order to repair it as to demonstrate underlying inadequacies, false assumptions, and
inherent contradictions. This process can supposedly be repeated many times and by
many literary hard hats; it is expected that each deconstruction will reveal additional
flaws and expose the illusions or bad faith of the builder. The fact that deconstructionists
prefer to describe their activities as deconstruction rather than criticism is also revealing.”
this question can be answered.

8) Answer: C

flickering – burning or shining unsteadily; wavering.

9) Answer: C

distinguish –recognize or treat (someone or something) as different.

10) Answer: B

Demolish – pull or knock down (a building).

Reading Comprehension Set-7 (Globalisation Based )

India had the distinction of being the world’s largest economy in the beginning of the
Christian era, as it accounted for about 32.9% share of world GDP and about 17% of
the world population. The goods produced in India had long been exported to far off
destinations across the world. The concept of globalisation is hardly new to India.

Until the liberalisation of 1991, India was largely and intentionally isolated from the world
markets, to protect its fledgling economy and to achieve self-reliance.

Globalization is the trend of increasing interaction between people on a worldwide scale


due to advances in transportation and communication. With increasing interactions
between nation-states and individuals, the international trade, ideas, and culture have also

Page 277 Follow us: Official Site, Telegram, Facebook, Instagram, Instamojo
been expanding. Globalization is primarily an economic process of integration that has
social and cultural aspects.

The transport and communications have accelerated the pace of globalisation over the
past 30 years. The internet has enabled fast and 24/7 global communication.

More recently, the rise of social media means that national boundaries have, in many ways
become irrelevant as producers use new forms of communication and marketing,
including micro-marketing, to target international consumers. The widespread use of
smart-phones has also enabled global shoppers to have easy access to ‘virtual’ global
markets.

The rise of new electronic payments systems,, including e-Wallets, pre-pay and mobile
pay, e-Invoices and mobile pay apps, also facilitate increased global trade.

The emergence of footloose multinational and transnational companies (MNCs and TNCs)
and the rise in the significance of global brands such as Microsoft, Apple, Google, Sony, and
McDonalds, has been central to the emergence of globalisation. The drive to reduce tax
burdens and avoid regulation has also meant the establishment of complex international
business structures.

Most economists agree that globalization provides a net benefit to individual economies
around the world, by making markets more efficient, increasing
competition, limiting military conflicts, and spreading wealth more equally around the
world.

The impact of globalization has been tremendous. The greater competition among the
producers resulting from globalization is a great advantage to consumers as there is
greater choice before them. Consumers now enjoy improved quality and lower prices for
several products.

Due to the globalization many MNCs have increased their investment in India. It means
thousands of people getting highly paid jobs and enjoy much highest standards of living
than was possible earlier. Top Indian companies have benefit from increased competition.
They have invested in newer technologies and production methods and raised their
production standards.

The term globalization are being modified with changing time, in its newest form, has four
major parameters. These are free flow of goods and services between countries due to the
Page 278 Follow us: Official Site, Telegram, Facebook, Instagram, Instamojo
reduction in trade barriers, Creation of an environment for flow of capital and investment
among countries, Free flow of technology from one country to another, Free movement of
labor among countries.

True globalization is attainable only if all four components are present. However,
globalization has some advantages and some disadvantages.

Globalisation paves the way for redistribution of economic power at the world level
leading to domination by economically powerful nations over the poor nations. It can also
be said that globalization has contributed towards increasing the gap between the rich
and the poor. Rich and wealthy people are able to exercise more control over the national
resources through the application of science and technology.

Globalisation usually results greater increase in imports than increase in exports leading
to growing trade deficit and balance of payments problem.

Although globalisation promote the idea that technological change and increase in
productivity would lead to more jobs and higher wages but during the last few years, such
technological changes occurring in some developing countries have resulted more loss of
jobs than they have created leading to fall in employment growth rates.

Due to globalization and industrialization, various chemicals have been thrown into the
soil which has resulted into the growth of many noxious weeds and plants. This toxic
waste has caused a lot of damage to plants by interfering in their genetic makeup. It has
put pressure on the available land resources. In various parts of the world, mountains are
being cut to make way for a passing tunnel or a highway. Vast barren lands have been
encroached upon to pave way for new buildings. While humans may rejoice on the
glimmer with these innovations, these can have long-term effects on the environment.
Various studies over the years, have found that plastic is one of the major toxic pollutants,
as it is a non-biodegradable product. However, plastic is of immense use when it comes to
packaging and preserving goods that are to be exported.

Globalization has also led to an increase in the transportation of raw materials and food
from one place to another. Earlier, people used to consume locally-grown food, but with
globalization, people consume products that have been developed in foreign countries.
The amount of fuel that is consumed in transporting these products has led to an increase
in the pollution levels in the environment.

Page 279 Follow us: Official Site, Telegram, Facebook, Instagram, Instamojo
Even with so many disadvantages of Globalization, one must admit that no country can
afford to ignore the wave of Globalization. Globalization, thus, is a reality. However,
adequate care, caution, and measures should be taken to mitigate the ill effects of
Globalization

Questions

1) According to the passage, which of a) social media


the following will not be counted as the
outcome of globalization? b) electronic payment system

a) improved living standard c) reduce tax burden

b) technological advancement d) private financial institutions

c) financial assistance from world bank e) multinational companies

d) improved quality of product 4) According to the passage, why was


India isolated from the international
e) increased competition market?

2) According to the passage, as per the a) for increasing its production


newest form of globalization what all
can move freely from one country to b) for controlling poverty
another?
c) for increasing GDP
a) workforce
d) for achieving the economic goal
b) capital
e) for achieving self sufficiency
c) technologies
5) According to the passage, which of
d) goods and services the following will be considered as the
drawback of globalization?
e) all of the above
a) balance of payment problems
3) According to the passage, which of
the following does not support the b) reducing scope of small industries
international business structure or
globalization? c) trade deficit

Page 280 Follow us: Official Site, Telegram, Facebook, Instagram, Instamojo
d) economic inequality a) agitate

e) All except (b) b) intensify

6) According to the passage, which of the c) provoke


following does not support the statement
“Globalization is responsible d) relieve
for environmental degradation”?
e) aggravate
a) excessive use of fuel
9) Choose the word which as opposite
b) cutting of mountains meaning as the word “complex”

c) polluted water of rivers and lakes a) multifaceted

d) growth of noxious weeds b) intricate

e) immense use of plastic c) mosaic

7) Choose the word which as same d) discernible


meaning as the word “accelerated”
e) manifold
a) suspended
10) Choose the word which as opposite
b) adjourned meaning as the word “limiting”

c) shelved a) preventing

d) deferred b) restraining

e) hastened c) warning

8) Choose the word which as same d) elevating


meaning as the word “mitigate”
e) restricting
Answers:

1). Correct Answer is: c) According to the passage, financial


assistance from World Bank has not been
mentioned as a result of globalization.
Page 281 Follow us: Official Site, Telegram, Facebook, Instagram, Instamojo
2). Correct Answer is: e) 6). Correct Answer is: c)

It is mentioned in the passage that According to the passage, “polluted water


labours, capital and investment, of rivers and lakes” does not support the
technologies, and goods and services can given statement as it is not mentioned in
be moved freely from one country to the passage.
another according to the newest form of
globalization. 7). Correct Answer is: e)

3). Correct Answer is: d) The meaning of “accelerate” is “hasten /


quicken” and second and third form of
According to the passage “private verb is “hastened”.
financial institutions” does not support
the globalization. 8). Correct Answer is: d)

4). Correct Answer is: e) The meaning of “mitigate” is “alleviate /


relieve”.
It is clearly mentioned in the passage, that
India was intentionally isolated from 9). Correct Answer is: d)
world market in order to achieve self-
reliance. The meaning of “complex” is “multipart /
complicated” and its opposite is
5). Correct Answer is: e) “apparent /discernible”

According to the passage, “reducing scope 10). Correct Answer is: d)


of small industries” will not be considered
as the drawback of globalization as it is The meaning of “limiting” is “preventing/
not mentioned in the passage. restricting” and its opposite is
“encouraging /elevating”.

ClOZE TEST
Cloze Test is one of the most commonly asked topic in banking exams. Be it prelims or
mains, you will find this topic in both the recruitment stages. This topic alone can help you
to fetch some very good marks. It is very much similar to Reading comprehension except
the fact that questions asked in this are in the form of fill in the blanks because It tests a
candidates understanding of the passage, grammatical skills and vocabulary, and like RC.

Page 282 Follow us: Official Site, Telegram, Facebook, Instagram, Instamojo
Tips And Tricks To Solve Cloze Test For Bank Exams

How to Solve Cloze Test?

Step 1: Find the theme of the passage

One of the common mistake that students commit is that they read the first line only and
start solving the cloze test. First step should be to read the passage carefully to learn about
the theme of the passage. This theme will help you to find answers in a better way and will
also help you in substitution method.

Step 2: Understand the Author’s tone

The author of a passage writes it with certain though process and that thought process
further build the tone of the passage. Reading the passage carefully in first step will also
help you to find the tone of the passage like: is it sarcastic, funny, narrative etc. This will
help you to find the sentence in a more easy way. The passage is written in a chronological
order and identifying the pattern in the sentence will help you get an idea of the kind of
word that you need to fill; whether it is a
noun/pronoun/verb/preposition/conjunction/article, etc.

Let’s Understand it with an Example:

Use Of Articles: He opened ___ rack and took ___ bat out

Solution: He opened the rack and took the bat out

Step 4: Use Elimination Method

At times, the options given are so close that it becomes very difficult to to spot the
answers. In such cases, use the elimination method technique to discard answers that
are most definitely wrong and out of context.

Example:

Ram likes his Coffee steaming ____.

Options: (a). Cold (b) Hot (c). Lukewarm (d) None of the above

Page 283 Follow us: Official Site, Telegram, Facebook, Instagram, Instamojo
Coffee cannot steam when cold or lukeward and hence you can easily eliminate them and
is left with option b i.e. Hot which is the right option

Step 5: Find the right word

If you are unable to find the right word then the tone of the passage can help you a lot for
example- If the tone of the passage is negative then look for a word that conveys the
negative meaning out of the given option. In case all the given options convey negative
meaning then read the line before the blank to get an idea about what it is exactly asking.

Final step: Re-read the passage

Now, that you are done with all the steps given above, make sure that you read the
passage again to see if your chosen options are in coherent with the passage tone.

Cloze Test Set-1

Directions (1-5): In the following passage there are blanks each of which has been
numbered, these numbers are printed below the passage and against each five
words are suggested one of which tits the blank appropriately. Find out the
appropriate word in each case.

In the decade since reforms were introduced, India has achieved substantial success in the
sphere of macroeconomics. Overall growth rate has been (1) except for the last couple of
years. It bears pointing out that we have now come to view a 6 per cent growth rate as a
slowdown! This is a far cry from pre-reforms rate of growth of 3 per cent. The price level
has by and large remained (2) both as measured by the WPI and CPI. India’s (3) of
payments position has been comfortable. Exports, while exhibiting some sluggishness this
fiscal, have been growing. Imports, in spite of substantial liberalization, have not gone out
of hand. This is amply reflected in the comfortable current account deficits (CAD); the
CAD-to-GDP ratio has remained way below the crisis (4) that it had achieved in 1991. The
rupee has weathered external turbulence rather well even when East Asia was
experiencing severe difficulties.
However, the one unambiguous Achilles’ heel of the reforms has been the vulnerable state
of government finances. One of the two crises that India faced in 1990-91 was the
unsustainable imbalance between government revenues and (5).

Q1. (a) Pulsating


Page 284 Follow us: Official Site, Telegram, Facebook, Instagram, Instamojo
(b) shocked (d) intention
(c) commendable (e) idea
(d) promotable
(e) dipped Q4.
(a) Rationalization
Q2. (b) Handling
(a) Moderate (c) management
(b) crawling (d) proportions
(c) shaky (e) ration
(d) considered
(e) obstinate Q5.
(a) Surplus
Q3. (b) Measurement
(a) Ledger (c) thinking
(b) Balance (d) incomes
(c) equilibrium (e) expenditure

Answers
S1. Ans.(c) S4. Ans.(d)
S2. Ans.(a) S5. Ans.(e)
S3. Ans.(b)

Cloze Test Set-2

Directions (1-7): In the following passage, some of the words have been left out,
each of which is indicated by a number. Find the suitable word from the options
given against each number and fill up the blanks with appropriate words to make
the paragraph meaningfully complete.

The Asian crisis has taught us that economic dislocations can __(1)____from the private
sector rather than the government budget; maintaining fixed exchange rates in a world of
free __(2)___ flows is almost impossible; currency panics can be self-fulfilling, so capital
controls should be used in rare cases as an emergency tool; countries need to think deeply

Page 285 Follow us: Official Site, Telegram, Facebook, Instagram, Instamojo
about their economic development models, especially if they have become __(3)_____ as
they move up the value chain.
The Asian tigers eventually bounced back—but they have never been able to match the
performance of the years before the __(4)_____. The structural transformation of these
economies can perhaps best be __(5)____ in the story of the Korean chaebol, such as
Samsung or Hyundai, which reinvented themselves as engines of global innovation while
others, such as Daewoo, were allowed to die. China is in the middle of a similar transition
right now. It remains to be seen _(6)____ it can change its economic model without ___(7)___
disruption.

Q1. (a) happen (d) dilemma


(b) yield (e) events
(c) appear
(d) turn up Q5. (a) checked
(e) emanate (b) learned
(c) tested
Q2. (a) budget (d) seen
(b) funding (e) manifest
(c) capital
(d) goods Q6. (a) that
(e) loan (b) whether
(c) through
Q3. (a) ancient (d) by
(b) frumpish (e) over
(c) obsoletes
(d) outdated Q7. (a) such
(e) recent (b) severe
(c) sudden
Q4. (a) crises (d) genuine
(b) incident (e) much
(c) problem

Answers

S1. Ans. (e)


Sol. “emanate” is the correct word to fill S2. Ans. (c)
the gap as it means originate from; be Sol. “world of free capital” is the correct
produced by. phrase in context of the meaning of the
Page 286 Follow us: Official Site, Telegram, Facebook, Instagram, Instamojo
sentence. The word ‘capital’ means people
who possess wealth and use it to control a S5. Ans. (d)
society's economic activity, considered Sol. “can perhaps best be seen” is the
collectively. correct phrase in context of its meaning of
the sentence.
S3. Ans. (d)
Sol. “outdated” is the correct word to fill S6. Ans. (b)
the gap as it means out of date. Other Sol. “whether” is the correct word as it
words are almost similar but they are not means expressing a doubt or choice
the most appropriate in adding meaning between alternatives.
to the sentence.
S7. Ans. (b)
S4. Ans. (a) Sol. “severe” is the correct word as it
Sol. “crises” is the correct word to fill the means strict or harsh.
gap as it means a time of intense difficulty
or danger.

Cloze Test Set-3

Directions (1-8) : In the following passage there are blanks, each of which has been
numbered. These numbers are also printed below the passage and against each five
words are suggested, one of which fits the blank appropriately. Find out the
appropriate word in each case.
Nature, a perceptive thinker has said, is always ahead of the avant-garde. Which is to say
that even the ‘advance guard’ of artists and writers which thinks and expresses things
before others do is behind nature. Art can be daring, writing _________________ (1). For sheer
originality, however, nature is not to be overtaken.
Human nature, likewise, may be said to be ever ahead of those who try to understand it or
dare to shape it. M.K. Gandhi, who tried to do both, has, not surprisingly, been described as
one who was ‘ahead of his time’. I am not sure if anyone can, in the linear logic
of _____________ (2), ever be ahead of his time. But studying human behaviour as thoroughly
as he did, ‘in foul weather and fair’, Gandhi had acquired the ____________ (3) of anticipating
collective responses to events and then either encouraging or resisting those responses.
He knew in his bones that India’s struggle for freedom was a historical ____________ (4) and
did all he could to take it forward. He knew likewise in the pulse of his being that
partitioning India was wrong and did all he possibly could to avert it. He along with Khan
Abdul Ghaffar Khan knew that communal divisiveness would __________ (5) the drawing of
the Radcliffe lines. So much so that in the _______________ (6) of 1946, while the subcontinent
Page 287 Follow us: Official Site, Telegram, Facebook, Instagram, Instamojo
prepared itself for the transfer of power, he, with Ghaffar Khan, made sure that where the
fires would ____________ (7) — along the borders of the two coming-to-be dominions —
they would be. As the “one man boundary force” (Lord Mountbatten’s description of him)
that he became, Gandhi could not extinguish those fires but he did succeed in ____________
(8) innumerable flames — with the help of some incredible volunteers in the same cause,
like SachinMitra and SmritishBanerjea of Calcutta who died by standing between two
rioting mobs.

Q1. Q5.
(a) Spat
(b) Warble (a) Reticent
(c) Audacious (b) Propensity
(d) Mien (c) Chide
(e) Sate (d) Outlast
(e) Espoused
Q2.
Q6.
(a) Docile
(b) Patronizing (a)Cravens
(c) Teem (b) Embers
(d)Estranged (c) Ostensible
(e) Chronology (d) Vestige
(e)Plumb
Q3.
Q7.
(a) Fitful
(b) Erode (c)Knack (a)Innate
(d)Unaffected (b) Reverie
(e)Canto (c) Rage
(d) Wrangle
Q4. (e) Crevice

(a)Imperative Q8.
(b) Raiment
(c) Intrepid (a) Deign
(d) Seemly (b) Capricious
(e) Allay (c) Stupendous
(d) Chaffing
Page 288 Follow us: Official Site, Telegram, Facebook, Instagram, Instamojo
(e) Dousing

Answers

S1. Ans. (c) Hence, option (c) is the right answer


Sol. “Audacious” fits the blank choice.
appropriately making the sentence Knack: an acquired or natural skill at
contextually meaningful and doing something.
grammatically correct. Hence, option Fitful: occurring in spells and often
(c) is the right answer choice. abruptly
Audacious: showing a willingness to take Erode: become ground down or
surprisingly bold risks. deteriorate
Spat: a quarrel about petty points Canto: a major division of a long poem
Warble: sing or play with trills
Mien: a person's appearance, manner, or S4. Ans. (a)
demeanor Sol. “Imperative” fits the blank
Sate: fill to contentment appropriately making the sentence
contextually meaningful and
S2. Ans. (e) grammatically correct. Hence, option
Sol. “Chronology” fits the blank (a) is the right answer choice.
appropriately making the sentence Imperative: of vital importance; crucial.
contextually meaningful and Raiment: especially fine or decorative
grammatically correct. Hence, option clothing
(e) is the right answer choice. Intrepid: invulnerable to fear or
Chronology: the arrangement of events or intimidation
dates in the order of their occurrence. Seemly: according with custom or
Docile: easily handled or managed propriety
Patronize: treat condescendingly Allay: lessen the intensity of or calm
Teem: be full of or abuzz with
Estrange: arouse hostility or indifference S5. Ans. (d)
in Sol. “Outlast” fits the blank appropriately
making the sentence contextually
S3. Ans. (c) meaningful and grammatically correct.
Sol. “Knack” fits the blank appropriately Hence, option (d) is the right answer
making the sentence contextually choice.
meaningful and grammatically correct. Outlast: live or last longer than.

Page 289 Follow us: Official Site, Telegram, Facebook, Instagram, Instamojo
Reticent: temperamentally disinclined to meaningful and grammatically correct.
talk Hence, option (c) is the right answer
Chide: scold or reprimand severely or choice.
angrily Rage: violent uncontrollable anger.
Propensity: an inclination to do Innate: not established by conditioning
something or learning
Espouse: choose and follow a theory, idea, Reverie: an abstracted state of
policy, etc. absorption
Wrangle: quarrel noisily, angrily, or
S6. Ans. (b) disruptively
Sol. “Embers” fits the blank Crevice: a long narrow opening
appropriately making the sentence
contextually meaningful and S8. Ans. (e)
grammatically correct. Hence, option Sol. “Dousing” fits the blank
(b) is the right answer choice. appropriately making the sentence
Embers: a small piece of burning or contextually meaningful and
glowing coal or wood in a dying fire. grammatically correct. Hence, option
Craven: lacking even the rudiments of (e) is the right answer choice.
courage; abjectly fearful Dousing: extinguish (a fire or light).
Ostensible: appearing as such but not -pour a liquid over; drench.
necessarily so Deign: do something that one considers to
Vestige:an indication that something has be below one's dignity
been present Capricious: determined by chance or
Plumb: examine thoroughly and in great impulse rather than by necessity
depth Stupendous: so great in size, force, or
extent as to elicit awe
S7. Ans. (c) Chaffing: material consisting of seed
Sol. “Rage” fits the blank appropriately coverings and pieces of stem
making the sentence contextually

Page 290 Follow us: Official Site, Telegram, Facebook, Instagram, Instamojo
CLOZE Test Set-4

Direction (1-5): In the following passage, certain words which may be either
contextually or grammatically incorrect have been highlighted and numbered.
For each highlighted four alternatives are provided that could replace the
highlighted word to make the sentence both grammatically and contextually
correct. Option corresponding to that word will be your answer. If none of the
given options could replace the highlighted word then choose option (E), i.e.
‘None of these’ as your answer.

It has historically been a difficult topic to address — the 1962 Indo-China War,
India’s provocation(1) record in terms of the origins of the conflict, and its disastrous
performance during the war itself (the bravery of our unsupported, ill-equipped jawans
notwithstanding). Addressed it must be, if only to prevent a similarly ostrich-in-the-
sand mentality from taking over India’s utilize(2) on the China question in the 21st
century, more than 50 years since the two sides came to blows in the Himalayas.

China’s modern innings begins only with the end of the Chinese Civil War, which
followed World War II and ended in 1949. Mao’s communists
eventually envisage(3) Chiang Kai-Shek’s nationalists onto the island of Taiwan
(where their descendants thrive awkwardly today). In 1966, Mao overlook(4) his
“Cultural Revolution”, which is a polite way of describing a barbaric and seemingly
mindless defenestration of the flower of Chinese society. It was a genocide of
intellectuals, artists, authors and anyone with the mildest of political opinions. I admit
to being rather mystified by this back when I first studied it. It seemed like such an
own-goal, such a totally disastrous and delineation(5) period in Chinese history that it
barely survived logical examination. I think differently today but more on that later.

1) It has historically been a difficult B) dubious


topic to address — the 1962 Indo-China
War, India’s provocation(1) record in C) spawn
terms of the origins of the conflict, and
its disastrous performance during the D) bare
war itself (the bravery of our
unsupported, ill-equipped jawans E) None of these
notwithstanding).
2) Addressed it must be, if only to
A) portrayal prevent a similarly ostrich-in-the-sand
mentality from taking over

Page 291 Follow us: Official Site, Telegram, Facebook, Instagram, Instamojo
India’s utilize(2) on the China question 4) In 1966, Mao overlook(4) his
in the 21st century, more than 50 years “Cultural Revolution”, which is a polite
since the two sides came to blows in the way of describing a barbaric and
Himalayas. seemingly mindless defenestration of
the flower of Chinese society.
A) perspective
A) blessing
B) disenchantment
B) behest
C) depiction
C) slip into
D) strip
D) exhibit
E) None of these
E) None of these
3) Mao’s communists
eventually envisage(3) Chiang Kai- 5) It seemed like such an own-goal,
Shek’s nationalists onto the island of such a totally disastrous
Taiwan (where their descendants thrive and delineation(5) period in Chinese
awkwardly today). history that it barely survived logical
examination.
A) engender
A) flaunt
B) mantle
B) proposition
C) triumph
C) counterproductive
D) drove
D) refrain
E) None of these
E) None of these

Answers

1) Answer: B

In the given sentence, ‘dubious’ should replace ‘provocation’ to make the sentence both
grammatically and contextually correct.

Page 292 Follow us: Official Site, Telegram, Facebook, Instagram, Instamojo
Option A): is incorrect because portrayal means a depiction of someone or something in
a work of art or literature; a picture which does not fit here.

Option B): is the correct alternative among the following as dubious means hesitating
or doubting fit here both grammatically and contextually

Option C): is incorrect as spawn means (of a fish, frog, mollusc, crustacean, etc.) release
or deposit eggs

Option D): is incorrect as bare means (of a person or part of the body) not clothed or
covered which does not fit here contextually.

2) Answer: A

In the given sentence, ‘perspective’ should replace ‘utilize’ to make the sentence both
grammatically and contextually correct.

Option A): is the correct alternative among the following as ‘perspective’ means a
particular attitude towards or way of regarding something; a point of view which fit
here both grammatically and contextually

Option B): is incorrect as disenchantment means a feeling of disappointment about


someone or something you previously respected or admired; disillusionment

Option C): is incorrect as depiction means the action of depicting something, especially
in a work of art.

Option D): is incorrect as strip means remove all coverings from which does not fit here
contextually.

3) Answer: D

In the given sentence, ‘drove’ should replace ‘envisage’ to make the sentence both
grammatically and contextually correct.

Option A): is incorrect because engender means cause or give rise to (a feeling,
situation, or condition) which does not fit here.

Option B): is incorrect as mantle means a loose sleeveless cloak or shawl, worn
especially by women which does not make any sense here.

Page 293 Follow us: Official Site, Telegram, Facebook, Instagram, Instamojo
Option C): is incorrect as triumph means a great victory or achievement which does not
fit here.

Option D): is the correct alternative among the following as ‘drove’ fit here both
grammatically and contextually.

4) Answer: E

In the given sentence, ‘commenced’ should replace ‘overlook’ to make the sentence
both grammatically and contextually correct.

Option A): is incorrect because blessing does not fit here.

Option B): is incorrect as behest means a person’s orders or command which does not
make any sense here.

Option C): is incorrect as ‘slip into’ means to gradually start to be in a bad state or
situation which does not fit here.

Option D): is incorrect as exhibit does not fit here contextually.

5) Answer: C

In the given sentence, ‘counterproductive’ should replace ‘delineation’ to make the


sentence both grammatically and contextually correct.

Option A): is incorrect because flaunt means display (something) ostentatiously,


especially in order to provoke envy or admiration or to show defiance which does not
fit here.

Option B): is incorrect as proposition means a statement or assertion that expresses a


judgement or opinion.

Option C): is the correct alternative among the following as ‘counterproductive’ fit here
both grammatically and contextually.

Option D): is incorrect as refrain means stop oneself from doing something which does
not fit

Cloze Test – Set- 5

Page 294 Follow us: Official Site, Telegram, Facebook, Instagram, Instamojo
Directions (1-5): Read the following information carefully and answer the
questions given below:

US President Joe Biden is __A__ pleased with the outcome of the historic first summit-
level __B__ of the Quad last Friday, one of his most __C__ foreign policy initiatives since
taking office less than two months ago. Biden and his counterparts – Prime Ministers
Narendra Modi of India, Scott Morrison of Australia and Yoshihide Suga of Japan – made
__D__ by getting together virtually for the first summit of the four-nation __E__, which
they have described as a “flexible group of like-minded partners”..

1) A C.crucial

A.anticipated D.significant

B.awaited E.None of the above

C.expectedly
4) D
D.hoped
A.history
E.None of the above
B.archive

2) B C.record

A.assembly D.report

B.meeting E.None of the above

C.conference
5) E
D.conclave
A.assemble
E.None of the above
B.collect

3) C C.grouping

A.notable D.mass

B.serious E.None of the above

Page 295 Follow us: Official Site, Telegram, Facebook, Instagram, Instamojo
Answers

1) Answer: C

Option A “anticipated” means ‘regard as probable’ and can be ruled out as it makes the
sentence incoherent.

Option B “awaited” means ‘wait for’ and can be eliminated as it is not related to the
context of the passage.

Option D “hoped” means ‘want something to happen’ and though it seems to be in sync
with the context, its usage makes the sentence grammatically incorrect.

Option C “expectedly” means ‘In an expected way’ and it aptly fits the context and
makes the sentence a meaningful one.

Therefore, option C is the correct answer choice to this question.

2) Answer: B

Options A and C “assembly” and “conference” mean ‘the action of gathering together’
and can be eliminated as they have positive connotation and contradict the sentence.

Option D can be ruled out as “conclave” means ‘a private meeting’ and is vague in the
context of the sentence.

Option B “meeting” means ‘especially for formal discussion’ and is apt as it fits in the
blank perfectly.

Therefore, option B is the correct answer choice to this question.

3) Answer: D

Option A “notable” means ‘worthy of attention’ can be eliminated as its usage renders
the sentence incoherent.

Option B “serious” means ‘demanding’ and can be eliminated as it makes the sentence
grammatically incorrect.

Option C “crucial” means ‘extremely important’ and can be eliminated as it does not
lend any meaning to the sentence and is contextual misfit.
Page 296 Follow us: Official Site, Telegram, Facebook, Instagram, Instamojo
Option D “significant” means ‘sufficiently great’ and here it aptly shows the foreign
policy initiatives.

Therefore, option D is the correct answer choice to this question.

4) Answer: A

The sentence as a whole talk about getting together virtually for the first summit of the
four-nation. Thus, the word to be used in the blank should indicate strengthening these
thoughts.

Option A “history” means ‘study of the past’ and conforms itself to the explanation
above.

Option B “archive” means ‘a collection of historical documents’ and can be eliminated as


it is contradictory to the meaning expressed in the passage.

Options C and D “record” and “report” means ‘account kept in writing’ and are not
usually used with the preposition “first summit of the four-nation”.

Therefore, option A is the correct answer choice to this question.

5) Answer: C

The sentence hints at various clauses for which the social media cannot be held
responsible for governing the mental wellbeing of people.

Option C “grouping” means ‘a set of associated people’ and aptly fits the context and
makes the sentence a meaningful one.

Options A and D “assemble” and “mass” mean ‘gather together in one place’ and can be
ruled out as make the sentence logically incorrect as they have positive connotation.

Option B “collect” means ‘gather together’ and can be eliminated as it is nowhere


related to the context of the passage.

Therefore, option C is the correct answer choice to this question.

Cloze Test – Set- 6

Page 297 Follow us: Official Site, Telegram, Facebook, Instagram, Instamojo
Direction (1-5): In the following passage, some of the words have been made
bold, each of which is preceded by a letter. Find the suitable word from the
options given below that could replace the bold word so as to make the
paragraph meaningful.

In the case of human beings killed by carnivore, the doubt is often


_____A____ produced as to whether the animal responsible for the kill is the tiger or
leopard. As a general rule – to which I have seen no exceptions – tigers are responsible
for all kills that take place in daylight, and leopards are responsible for all kills that take
place in the dark. Both animals are semi-nocturnal forest-dwellers, have much the same
habits, ____B___ entertain similar methods of killing, and both are capable of carrying
their human victims for long distances. It would be natural, therefore, to expect them to
hunt at the same hours; and that they do not do so is due to the difference in courage of
the two animals. When a tiger becomes a man-eater it loses all fear of human beings
and, as human beings move about more freely in the day than they do at night, it is able
to ____C___ guarantee its victims during daylight hours and there is no necessity for it to
visit their habitations at night. A leopard, on the other hand, even after it has killed
scores of human beings, never loses its fear of man; and, as it is ____D____ ready to face
human beings in daylight, it secures its victims when they are moving about at night or
by breaking into their houses at night. Owing to these characteristics of the two
animals, namely that one loses its fear of human beings and kills in the daylight, while
the other ____E___ avoids its fear and kills in the dark, man-eating tigers are easier to
shoot than man-eating leopards.

1) Which of the following words fit in 3) Which of the following words fit in
best in the slot A? best in the slot C?

A. Clarified A. Lose
B. Questioned B. Secure
C. Expressed C. Harass
D. Debated D. Smash
E. No replacement required E. No replacementrequired

2) Which of the following words fit in 4) Which of the following words fit in
best in the slot B? best in the slot D?

A. Employ A. Apologetic
B. Elucidate B. Angry
C. Enforce C. Unwilling
D. Accept D. Unwell
E. No replacementrequired E. No replacementrequired

Page 298 Follow us: Official Site, Telegram, Facebook, Instagram, Instamojo
5) Which of the following words fit in C. Changes
best in the slot E? D. Retains
E. No replacementrequired
A. Banishes
B. Vacates
Answers :

Directions (1-5) :

1) Answer: C

The passage starts with discussing doubt regarding the identity of the killer in case of
humans killed by carnivorous animals. A doubt is not produced, – so we need to replace
this word. Clarifying a doubt means closing the doubt or query. It doesn’t fit in the
implied meaning here so we can rule out option A. Questioning and Debating are both
ways of raising doubts so options B and D are also not appropriate. One “expresses”
doubt with regard to something – so expressed is the correct choice.

Therefore option C is the correct answer to this question.

2) Answer: A

The term “entertain” is incorrect here as it means to provide amusement / enjoyment


or give attention/consideration, which is irrelevant here. To “elucidate” is to explain or
make clear – not fitting into the meaning here. So option B is ruled out. To “enforce” is
to implement something usually with force – again not appropriate thus ruling out
option C. To “accept” is to provide assent or approval – incorrect here hence option D is
also ruled out. To “employ” is to use something which fits in the meaning implied here.

Therefore option A is the correct answer to this question.

3) Answer: B

Option A, lose is contrary to the implied meaning here. Harass which means to bother or
annoy someone and Smash which means to demolish or destroy something are both ill-
fitting to this part of the passage. To secure is to make something safe or protected.
Here from the perspective of the predator, this term fits well as it implies that the
creature is able to bag its victim and have a meal.

Therefore option B is the right answer to this question.

4) Answer: C

Page 299 Follow us: Official Site, Telegram, Facebook, Instagram, Instamojo
The given term is opposite to the intended meaning since it appears that the leopard is
trying to avoid facing humans in daylight. So we need a term that is opposite of ready.
Of the given options, A. apologetic meaning remorseful, B. angry meaning annoyed or
irritated and D. unwell meaning sick or ill – do not work. The only one that works
is unwilling which means reluctant or disinclined and is thus opposite of ready.

Therefore option C is the correct answer to this question.

5) Answer: D

“Avoids its fear” is not a logical expression. From reading the passage, we can make out
that here, the second animal being referred to is the leopard which doesn’t lose its fear
of humans even after becoming a man-eater. Among the given options, the only term
that fits in is D. retains which means “continue to have” (something) or “keep
possession” (of something), thus fitting into the intended meaning.

Therefore option D is the correct answer to this question.

Cloze Test – Set- 7

Directions (1-5): In the following passage, some of the words have been left out,
each of which is indicated by a letter. Find the suitable word from the options
given against each letter and fill up the blanks with appropriate words to make
the paragraph meaningful.

A sunny midsummer day. There was such a thing sometimes, even in Coketown. Seen
from a distance in such weather, Coketown lay _____(A)______ in a haze of its own, which
appeared_____(B)_____ to the sun’s rays. You only knew the town was there, because you
knew there could have been no such sulky______(C)_______upon the prospect without a
town. A blur of soot and smoke, now confusedly tending this way, now that way, now
aspiring to the vault of Heaven, now____(D)____ creeping along the earth, as the wind
rose and fell, or changed its quarter: a dense formless jumble, with sheets of cross light
in it, that showed nothing but masses of darkness:- Coketown in the distance was
suggestive of itself, though not a brick of it could be seen.The wonder was, it was there
at all. It had been ruined so often, that it was amazing how it had____(E)_______ so many
shocks. Surely there never was such fragile china-ware as that of which the millers of
Coketown were made.

1) Which of the following words A. tranced


should fill in the blank (A) to make a
B. translucent
contextually correct and meaningful
sentence? C. silhouette
Page 300 Follow us: Official Site, Telegram, Facebook, Instagram, Instamojo
D. crowded E. monument
E. shrouded

4) Which of the following words


should fill in the blank (D) to make a
2) Which of the following words
contextually correct and meaningful
should fill in the blank (B) to make a
sentence?
contextually correct and meaningful
sentence? A. insanely
A. impervious B. murkily
B. unburned C. aimlessly
C. pacified D. beautifully
D. dazzling E. eerily
E. scintillating

5) Which of the following words


should fill in the blank (E) to make a
3) Which of the following words
contextually correct and meaningful
should fill in the blank (C) to make a
sentence?
contextually correct and meaningful
sentence? A. withstand
A. pasture B. engraved
B. slum C. born
C. blotch D. borne
D. greenery E. taken
Answers :

1) Answer: E

The word ‘shrouded’ means to be covered in something or enveloped. The question can
be answered by reading the sentence carefully. The author says that even in such a
sunny midsummer day, Coketown remains covered in a haze of its own. The word
suitable for this blank will be ‘shrouded’. Tranced means to be in a state of hypnosis or
hallucination. Translucent means something that cannot be seen from outside. Now this
may be relevant to the context, but the haze mentioned in the passage prevents it from
being used. Silhouette means the dark outline of something. This can be eliminated

Page 301 Follow us: Official Site, Telegram, Facebook, Instagram, Instamojo
because it is in present tense and also, it does not make any sense. Option D is absurd
and irrelevant to the context. This makes option E the correct answer.

2) Answer: A

The question can be easily answered by reading he sentence carefully and


understanding what the author is trying to tell us. The author implies that though it is a
sunny midsummer day, Coketown remains covered in a haze of its own such that sun
cannot penetrate it. The blank generally explains the sun being not able to penetrate.
Option A seems to be perfect fit. Impervious means not allowing anything to go inside,
here referring to sunlight. Unburned is a little aggressive and is absurd to the context of
the sentence. Pacified means to quell the anger of something. This is also irrelevant in
the context of the sentence. Option D and E are both words almost opposite to what
should be used in the blank. Scintillating means dazzling and are synonyms. This makes
option A the correct answer.

3) Answer: C

The blank is a little confusing. The author is using a metaphor to describe the town.
Sulky in the sentence something that is bad, angered or tempered, always complaining.
Now, the first part of the paragraph suggests an irregularity of the Coketown which was
covered in haze even though it was a sunny day. Now, pasture cannot be sulky and is
not irregular in metaphor, so is eliminated. Slum is absurd to use and there is no
mention of it. Blotch means an irregular surface. Yes, this can be used as it is a
metaphor describing the irregularity of the town. Option D and E are also absurd and
out of context. This makes option C the correct answer.

4) Answer: B

The trick to answering such questions is to reading the sentence carefully and try to fill
the blank with our own word and the choose from the options. This causes less
confusion while choosing the answers. The creeping is referred to the flowing of soot
and smoke mentioned earlier, also known as a smog. Now, the blank should be
something dark and gloomy. Option A is vague and related to madness, is irrelevant to
the context here. Option C is also dull and vague and if used does not tell us about the
meaning of the sentence or stresses it. Option D, if used will give the exact opposite vibe
and make the sentence cheerful which is not the intention of the author. Option B and E
can be the probable correct answers. Now option E can be used as a metaphor for dark,
but option B is much more fitting because ‘murkily’ means to be dark and gloomy
because of being covered in thick fog, which is the exact meaning described in the
sentence. This makes option B the correct answer.

Page 302 Follow us: Official Site, Telegram, Facebook, Instagram, Instamojo
5) Answer: D

The sentence implies that it was surprising as to how many shocks the city has endured
or carried in spite of everything. Now, option A is correct in context of the sentence but,
cannot be used as it would be grammatically wrong. Option B and C are absurd in the
context of the sentence. Option D can also be thought of as the correct answer but when
compared with option D, it stands vague. Borne means to carry with it and is perfect in
the context of the sentence. This makes option D the correct answer.

Cloze Test – Set- 8

Directions (1-10) : In the passage given below there are 10 blanks, each followed
by a word given in bold. Even blank has four alternative words given in options
(A),(B),(C) and (D). You have to tell which word will not suit the respective blank.
Mark (E) as your answer if all words are suitable.

Last year, when American officials visited Beijing for trade negotiations, they spent
more time fighting among themselves than against China. They could not agree on who
should lead the talks or what their goal should be. Seeing such amateurism, their
Chinese interlocutors _ (1) _ [think] that they had little to worry about. America has set
a deadline of March 1st for an agreement. If it is missed, tariffs on $200bn-worth of
imports from China are due to rise from 10% to 25%, _ (2) _ [applying] more pain on a
slowing Chinese economy. That would invite a sharper _ (3) _ [reactions] from China.
Its ability to direct firms to shift purchases to other countries has already hurt
American exporters. Chinese officials resent the deadline but it has focused minds. The
latest talks, which began on February 11th and were due to end on February 15th, are
the third round this year. All this has fed expectations that the two sides could soon
make peace or at least extend their _ (4) _ [terms]. Mr Trump told reporters that he
might let the deadline slide if a good deal is within reach. Another meeting between Mr
Trump and Xi Jinping, China’s president, is under discussion. Investors have taken
heart. American and Chinese stockmarkets have both risen by about 10% this year. The
outlines of a deal have been in view for a while. China would probably promise to buy
lots of goods from America, from soyabeans to natural gas, and allow foreign
companies more access to its economy. America would cut tariffs and perhaps promise
to remain open to Chinese investors, as long as they are not part of a state-backed _ (5)
_ [attack] on sensitive technology. Yet when negotiators get into the details, problems
surface. After many frustrating years waiting for China to open its markets, the
Americans suspect that its _ (6) _ [promises] will be empty. The Chinese _ (7) _
[suspect] that America is motivated by a desire not for fair trade but for _ (8) _
[defeating] a new rival. And if it is judged that China is not keeping its promises? One
option is to submit _ (9) _ [quarrels] to neutral arbitration. Mr Lighthizer is said to

Page 303 Follow us: Official Site, Telegram, Facebook, Instagram, Instamojo
dislike this idea. Another is to give America the right to slap tariffs unilaterally on
Chinese goods which China is understandably _ (10) _ [disliking] to accept. Even as the
trade war seems to be cooling, a chasm still lies between the combatants.

1) ? B) Hostility

A) Reckoned C) Treaty

B) Surmised D) Truce

C) Considered E) All Are Correct

D) Evaded 5) ?

E) All Are Correct A) Evacuation

2) ? B) Onslaught

A) Inflicting C) Incursion

B) Foisting D) Assault

C) Enduring E) All Are Correct

D) Imposing 6) ?

E) All Are Correct A) Vows

3) ? B) Assurances

A) Backlash C) Pledges

B) Counteraction D) Barratries

C) Repercussion E) All Are Correct

D) Response 7) ?

E) All Are Correct A) Abstain

4) ? B) Suspect

A) Agreement C) Speculate

Page 304 Follow us: Official Site, Telegram, Facebook, Instagram, Instamojo
D) Presume B) Concords

E) All Are Correct C) Squabbles

8) ? D) Conflicts

A) Dodging E) All Are Correct

B) Thwarting 10) ?

C) Expediting A) Loth

D) Countering B) Disinclined

E) All Are Correct C) Reluctant

9) ? D) Averse

A) Disputes E) All Are Correct

Answers:

Directions (1-10): Evacuation- Removal/Clearing

1) Answer : D) 6) Answer :D)

Evaded- Get Away From Barratries- Violation Of A Law

2) Answer : C) 7) Answer – A)

Enduring- Remain In Existence Or To Abstain- Stop Oneself From Doing Or


Continue Enjoying Something.

3) Answer : E) 8) Answer : C)

Backlash- A Strong Negative Reaction By Expediting- To Help Or Promote


A Large Number Of People, Especially To
A Social Or Political Development. 9) Answer : B)

4) Answer : B) Concords- Unity Or Harmony

Hostility- Hatred Or Bitterness 10) Answer : E)

5) Answer : A) Loth- Unwilling


Page 305 Follow us: Official Site, Telegram, Facebook, Instagram, Instamojo
Fill in the Blanks
General tips to Solve Fillers:

1. Make sure that you read the sentence thoroughly and understand the message
that the sentence is trying to communicate. On doing that, you will surely find a
clue to identify the correct option.
2. You might be tested on knowledge and understanding of idioms and phrases.
Hence, you should look to have a good skill set of such phrases
3. Do not alter the meaning of the sentence. Further, once you have thought about
the word, then do read the entire sentence a couple of times to make sure that the
meaning of the sentence is intact.
4. Apply trial and error method only when you are confused between two or more
words or are not sure which word will fit the best in the given sentence.
5. Questions on fillers can be solved quickly and so do not spend much time on it. In
case you find yourself spending too much time then simply move to the next
question and come back to it later when you have time.
6. Finally, make sure you are practicing questions on fillers by taking mock tests in a
proctored environment

Single Fillers Questions

Directions (1-5): A sentence with one blank is given, indicating that something
has been omitted. Choose the word that best fits the blank.

1) The Colonel, normally a fast A. Jettison


thinking man was left perfectly B. Swindling
_________ by the situation and seemed C. Apathy
at a loss for what to say. D. Careened
E. None of these
A. Damaged
B. Nonplussed 3) Even though everyone knows the
C. Rotund ________ story of the shipwreck and the
D. Conceited lost treasure is not true, still they tell
E. None of these it over and over again.

2) How many foreign visitors, fleeced A. Apocryphal


by mock auctioneers, have been B. Dazzling
surprised that such obvious _________ C. Sensational
could be conducted in such brazen D. Famous
and open manner? E. None of the above
Page 306 Follow us: Official Site, Telegram, Facebook, Instagram, Instamojo
4) Despite his best efforts and use of 5) Jim’s totally __________ attitude and
every possible ___________ that he could antagonized co-workers everywhere
think of, the salesman failed to sell a and that’s why he failed to hold up a
single product on that day. job for too long.

A. Bolster A. Congruity
B. Calumny B. Despotic
C. Blandishment C. Recalcitrant
D. Compunction D. Ebullient
E. None of the above E. None of the above

Answers

1) Answer: B

Let us look at the given statement and understand the situation described. It is being
said that a man, normally fast thinking was left tongue-tied and unsure. It implies we
need a word which means a state of confusion.

‘Damaged’ means wounded or hurt. ‘Rotund’ means large and plump (person).
‘Conceited’ means being excessively proud of one one. None of these words fit the
required meaning.

‘Nonplussed’ means being “so surprised and confused that one is unsure how to react.”
It fits the given blank perfectly.

Therefore, option B is the correct answer to this question.

2) Answer: B

The meaning that can be gathered from the sentence implies that a fraudulent or
immoral activity was committed on foreign visitors. The word in the blank should help
propagate that theme.

‘Jettison’ is the act of throwing or dropping off something (usually from a ship or
aircraft). ‘Apathy’ means lack of interest or concern. ‘Careened’ is the past form of the
verb ‘careen’ meaning the turning over an object (usually an automobile or a
boat/ship). None of them make any logical sense in the given blank.

Page 307 Follow us: Official Site, Telegram, Facebook, Instagram, Instamojo
‘Swindling’ means using deception to deprive (someone) of money or possessions. It
fits perfectly with the theme of the sentence.

Therefore, option B is the correct answer to this question.

3) Answer: A

The statement clearly says that the story being referred to is not true – therefore the
adjective in the given blank must convey a similar meaning. The only word among the
given options that fits the requirement is ‘apocryphal’ which means ‘(something) of
doubtful authenticity, although widely circulated as being true. ’

Therefore, option A is the correct answer to this question.

4) Answer: C

The salesman was trying to sell his wares and the word that will go into the blank
should be something that implies an act of persuasion or convincing.‘Bolster’ means to
support or strengthen (something), usually a structure; ‘Calumny’ is false and
defamatory statements about someone in order to damage their reputation;
‘Compunction’ is the feeling of guilt or moral scruple that prevents or follows the act of
doing something bad. None of these words convey the desired meaning.

‘Blandishment’ is a flattering or pleasing statement or action used as means of gently


persuading someone to do something – it fits the blank perfectly.

Therefore, option C is the correct answer to this question.

5) Answer: C

The word for the blank has to convey a negative feel as that is what the statement is
implying. There are only two words among the given options which have a negative
connotation – despotic and recalcitrant.

‘Despotic’ means a tyrannical attitude – generally used to describe rulers of older times
or governments. It doesn’t fit in well in this case. ‘Recalcitrant’ is ‘having an obstinately
uncooperative attitude towards authority or discipline’ – a perfect fit for the given
blank.

Therefore, option C is the correct answer to this question.

Page 308 Follow us: Official Site, Telegram, Facebook, Instagram, Instamojo
New Pattern Fillers (Sentence Fillers)

Directions (1-5): In each of the following questions, a paragraph with a blank is


given. From the five choices given below, select the sentence which can go into
the blank to make the paragraph logically coherent.

1) SpiceJet on Wednesday announced 11 new flights from March 29. Among


these, the Mumbai-Leh, Leh-Srinagar and Srinagar-Mumbai routes are a first on
the airline’s network. Passengers travelling between Leh and Mumbai will fly via
Srinagar. The airline will also enhance its operations with additional frequencies
on Mumbai-Rajkot. In a statement, the airline, which is already the largest
operator under UDAN, said it was further strengthening its regional network
with the launch of two new flights on the Jalandhar-Jaipur-Jalandhar route under
the regional connectivity scheme.____________________________.

A) SpiceJet also plans to enhance its operations with additional frequencies on Delhi-
Dharamshala and Bengaluru-Vijayawada sectors. It had recently announced the launch
of 20 new domestic flights

B) All the new flights introduced will be operational daily except the Bengaluru-
Vijayawada-Bengaluru route which will not operate on Wednesdays

C) The airline will deploy a mix of its Boeing 737 and Bombardier Q400 aircraft on all
these routes

D) With this, the airline will operate a total of 54 flights connecting 12 cities under the
regional connectivity scheme

E) None of these

2) When late theoretical physicist, Stephen Hawking, wrote A Brief History of


Time in 1988, he was urging lay people to question the workings of our solar
system that went beyond blind faith. “There are children who want to know what
a black hole looks like; what is the smallest piece of matter; why we remember
the past and not the future; and why there is a universe,” writes cosmologist, Carl
Sagan in his forward as part of Hawking’s book.____________________________, a gifted
few are unpacking the scientific threads that bind us together. Now, in a two-day
workshop titled, ‘Space for Everyone’, astrology enthusiasts with no formal
education in the field, are invited to engage with astronauts, and scientists from
NASA, the Mars Society Australia, Amity University, and Open University to do
just that.

Page 309 Follow us: Official Site, Telegram, Facebook, Instagram, Instamojo
A) The team of instructors includes Dr. Jennifer Blank, from NASA Ames Research
Centre, Dr. Jonathan Clarke, president of the Mars Society Australia

B) The discussions will be astrobiological, and allow participants to touch and feel
fossils

C) As we existentially ponder upon human existence and its place in an infinite abyss

D) We intend to discuss concepts of time and space

E) None of these

3) It seemed as I knew the tune, but couldn’t quite place it. It was very much in
the 18th century baroque style, and could have been by Bach, Handel or Vivaldi.
It lingered as an earworm, and it was only after checking Spotify later that I
discovered it was ‘The Belt Of Faith’, an original tune by Jung Jaeil, who’s
composed the film’s score. Based largely on western classical technique, Parasite
has, one of the most powerful soundtracks in recent times. Barring a portion
from Handel’s opera seriaRodelinda, most of it is original. Strangely enough,
while the film got all the big Oscars, Jaeil’s music wasn’t even
nominated.___________________________, was less brilliant in any way. Her cello-driven
‘Bathroom Dance’ and pathos-filled ‘Defeated Clown’ had the same stunning
effect.

A) Often before, some pieces of instrumental music in a film have succeeded in

B) A few years ago, Steven Price’s Gravity title tune did it with its sudden burst of
screams

C) That’s not to say that HildurGuonadottir, who bagged a clutch of awards for Joker

D) Among popular and older fare, there are tunes from Love Story

E) None of these

4) The Legislative Council on Wednesday unanimously passed a Bill making


teaching and learning Marathi compulsory in schools across Maharashtra. The
Maharashtra Compulsory Teaching and Learning of Marathi Language in Schools
Bill, 2020,____________________________, will apply to all schools irrespective of the
board they are affiliated to, and calls for phase-wise introduction of Marathi
language as a compulsory subject from Class I to Class X starting from the
academic year 2020-21.

Page 310 Follow us: Official Site, Telegram, Facebook, Instagram, Instamojo
A) will be introduced in Class I and Class VI from this academic year and extended to
further classes

B) introduced by Minister for Marathi Language Subhash Desai

C) will be compulsory for schools seeking recognition or NOC from government

D) in this case as an exception, the Council chairman allowed Mr. Desai to introduce the
Bill in the Upper House

E) None of these

5) Punjab Chief Minister Amarinder Singh said on Wednesday that his


government’s campaign against drugs had broken the back of the drug mafia in
the State. He added that the campaign continues unabated in line with the party’s
poll promise. “The State government had adopted a comprehensive EDP –
Enforcement, De-addiction and Prevention -____________________________,” the Chief
Minister said in the Assembly during the ongoing budget session.

A) strategy to meet the requirements of its promise to the people of Punjab

B) the allegation levelled by Shiromani Akali Dal chief Parkash Singh Badal

C) with the arrest of 42,571 persons and seizure of 974.15 kg of heroin by the Punjab
police

D) at present, 3.70 lakh persons were undergoing treatment for drug addiction in the
State

E) None of these

Answers

1) Answer: (d)

SpiceJet on Wednesday announced 11 new flights from March 29. Among these, the
Mumbai-Leh, Leh-Srinagar and Srinagar-Mumbai routes are a first on the airline’s
network. Passengers travelling between Leh and Mumbai will fly via Srinagar. The
airline will also enhance its operations with additional frequencies on Mumbai-Rajkot.
In a statement, the airline, which is already the largest operator under UDAN, said it
was further strengthening its regional network with the launch of two new flights on
the Jalandhar-Jaipur-Jalandhar route under the regional connectivity scheme. With

Page 311 Follow us: Official Site, Telegram, Facebook, Instagram, Instamojo
this, the airline will operate a total of 54 flights connecting 12 cities under the
regional connectivity scheme.

2) Answer: (c)

When late theoretical physicist, Stephen Hawking, wrote A Brief History of Time in
1988, he was urging lay people to question the workings of our solar system that went
beyond blind faith. “There are children who want to know what a black hole looks like;
what is the smallest piece of matter; why we remember the past and not the future; and
why there is a universe,” writes cosmologist, Carl Sagan in his forward as part of
Hawking’s book. As we existentially ponder upon human existence and its place in
an infinite abyss, a gifted few are unpacking the scientific threads that bind us
together. Now, in a two-day workshop titled, ‘Space for Everyone’, astrology
enthusiasts with no formal education in the field, are invited to engage with astronauts,
and scientists from NASA, the Mars Society Australia, Amity University, and Open
University to do just that.

3) Answer: (c)

It seemed as I knew the tune, but couldn’t quite place it. It was very much in the 18th
century baroque style, and could have been by Bach, Handel or Vivaldi. It lingered as an
earworm, and it was only after checking Spotify later that I discovered it was ‘The Belt
Of Faith’, an original tune by Jung Jaeil, who’s composed the film’s score. Based largely
on western classical technique, Parasite has, one of the most powerful soundtracks in
recent times. Barring a portion from Handel’s opera seriaRodelinda, most of it is
original. Strangely enough, while the film got all the big Oscars, Jaeil’s music wasn’t
even nominated. That’s not to say that HildurGuonadottir, who bagged a clutch of
awards for Joker, was less brilliant in any way. Her cello-driven ‘Bathroom Dance’ and
pathos-filled ‘Defeated Clown’ had the same stunning effect.

4) Answer: (b)

The Legislative Council on Wednesday unanimously passed a Bill making teaching and
learning Marathi compulsory in schools across Maharashtra. The Maharashtra
Compulsory Teaching and Learning of Marathi Language in Schools Bill,
2020, introduced by Minister for Marathi Language Subhash Desai, will apply to all
schools irrespective of the board they are affiliated to, and calls for phase-wise
introduction of Marathi language as a compulsory subject from Class I to Class X
starting from the academic year 2020-21.

5) Answer: (a)

Page 312 Follow us: Official Site, Telegram, Facebook, Instagram, Instamojo
Punjab Chief Minister Amarinder Singh said on Wednesday that his government’s
campaign against drugs had broken the back of the drug mafia in the State. He added
that the campaign continues unabated in line with the party’s poll promise. “The State
government had adopted a comprehensive EDP – Enforcement, De-addiction and
Prevention – strategy to meet the requirements of its promise to the people of
Punjab,” the Chief Minister said in the Assembly during the ongoing budget session.

New Pattern Fillers Based on Error

Direction (1-5): A sentence divided D. they are relooking


into 5 parts is given, each of which is E. None of these
numbered. One of the parts of the
sentence is missing and is marked by 3) US secretary of state Mike Pompeo
a blank. Choose the option as your comes out forcefully (1)/in support of
answer, which when filled in the India (2)/ in its border standoff with
blank will render the first part of the China (3)/ ______________________________
sentence incorrect. (4)/directly on Chinese President Xi
Jinping.(5)
1) Citizens had approached human
rights forum (1)/in such large A. laid the blame
numbers (2)/ _______________________(3)/ B. laying the blame
political representatives and the C. to lay the blame
justice delivery system(4)/weren’t D. to laying the blame
helping them.(5) E. None of these

A. would be a sure sign 4) Private labs in Pune (1)/


B. could be a sure sign ___________________________(2)/ from
C. was a sure sign conducting covid-19 tests(3)/
D. is a sure sign following some discrepancy(4) / in
E. None of these reports.(5)

2) European countries like the UK A. did stop


have announced (1)/ B. had stopped
___________________(2)/ at the 5G C. have been stopped
contracts (3)/with Chinese D. has been stopped
equipment vendors(4)/ given E. None of these
security vulnerabilities.(5)
5) In March the ICMR plans to
A. they is relooking launch(1)/ a new pan-India
B. they relooked serological survey (2)/ as a follow up
C. they were relooking to the one(3)/ _____________________(4)/

Page 313 Follow us: Official Site, Telegram, Facebook, Instagram, Instamojo
across 21 states the final results of B. conducted in february
which are yet to come (5). C. conducted in january
D. conducted in april
A. conducted earlier E. None of these
Answers :

1) Answer: D

Note that in the given question we are required to find a segment which when put in
the blank will render the first part of the sentence incorrect. So we can either look for
the segment which will make a contextual error or one which introduces a grammatical
error in the asked part. So let’s first read the sentence and the options thoroughly.

Here we can have a grammatical error. Since the first part is in the past perfect tense,
the rest of the sentence should be structurally similar and thus the same tense should
be used throughout. However we can see that part D clearly mentions ‘present tense’.
As a result the entire sentence changes into simple present sense. This in turn
introduces an error in part 1. The first part should have been in present tense too.
‘Approaching’ should have been used in place of ‘had approached’.

Therefore, option D is the correct answer choice for this question.\

2) Answer: C

Note that in the given question we are required to find a segment which when put in
the blank will render the first part of the sentence incorrect. So we can either look for
the segment which will make a contextual error or one which causes a grammatical
error in the asked part. So let’s first read the sentence and the options thoroughly.
Here we can have a grammatical error. Since the first part is in the present perfect
tense, the rest of the sentence should be structurally similar and thus the same tense
should be used throughout. However we can see that part 3 clearly mentions ‘past
tense’. As a result the entire sentence changes into past sense. This in turn introduces
an error in part 1. The first part should have been in past tense too. ‘Had announced’
should have been used in place of ‘have announced’.

Therefore, option C is the correct answer choice for this question.

3) Answer: B

Note that in the given question we are required to find a segment which when put in
the blank will render the first part of the sentence incorrect. So we can either look for
Page 314 Follow us: Official Site, Telegram, Facebook, Instagram, Instamojo
the segment which will make a contextual error or one which causes a grammatical
error in the asked part. So let’s first read the sentence and the options thoroughly.
Here we can have a grammatical error. Since the first part is in the simple present
tense, the rest of the sentence should be structurally similar and thus the same tense
should be used throughout. However we can see that part 2 clearly mentions ‘present
continuous tense’. This in turn introduces an error in part 1. The first part should be in
present perfect to make it grammatically correct. ‘has come’ should have been used in
place of ‘comes’.

Therefore, option B is the correct answer choice for this question.

4) Answer: D

Note that in the given question we are required to find a segment which when put in
the blank will render the first part of the sentence incorrect. So we can either look for
the segment which will make a contextual error or one which causes a grammatical
error in the asked part. So let’s first read the sentence and the options thoroughly.

We must note that ‘labs’ as mentioned in the first part of the sentence, is a plural
subject, which would take a plural verb in part 2. But here in option D singular verb is
being used. This undoubtedly introduces an error in the first part of the sentence. In the
first part a singular entity needs to be used.

Therefore, option D is the correct answer choice for this question.

5) Answer: D

Note that in the given question we are required to find a segment which when put in
the blank will render the first part of the sentence incorrect. So we can either look for
the segment which will make a contextual error or one which causes a grammatical
error in the asked part. So let’s first read the sentence and the options thoroughly.

There seems to be an error in the time frame when we try to understand the time frame
after putting option D in part 4. If there has been a plan in march mentioning a month
later than March seems meaningless here. In fact, any month before March can make
the first part correct and meaningful.

Therefore, option D is the correct answer choice for this question.

Double Fillers

Page 315 Follow us: Official Site, Telegram, Facebook, Instagram, Instamojo
Direction (1-5): In the Questions B) The southerners are _______except
given below, there are two when roused, and socialist doctrines
statements, each statement consists find their chief adherents in the
of a blank. You have to choose the north.
option which provides the correct set
of words that fits in the blanks in both A. acquainted
the statements appropriately making B. adhesive
them meaningful and grammatically C. apathetic
correct: D. accede
1) E. none of the above
A) In this paper, I shall argue that 4)
Process Buddhism is indeed a A) From that time Conselheiro was a
liberating ______ . victim of _______ , and to expiate his sin
B) Through this critical appropriation became a missionary in the sertao or
critical _______becomes a possibility. interior of Brazil among the wild
Jagunco people.
A. praxis B) The demon lord strode into a
B. curtail portal, leaving her angry and no
C. prosperous closer to alleviating her _______ .
D. propel
E. none of the above A. reluctance
2) B. remnant
A) The wounded are getting to be C. reimburse
common, and people grow ________ . D. remorse
B) Kiera looked at Evelyn, astonished E. none of the above
by her _________ 5)
A) His youthful ____________ always
A. catastrophe manages to keep the family on their
B. callous toes.
C. combat B) Many people lead very passive
D. corrode lives and such ___________ actually
E. none of the above scares them.
3)
A) He had shown himself so incapable A. exuberance
and _______that his followers were sick B. emancipation
of fighting for such a despicable C. eloquent
master. D. extravagant
E. none of the above
Answers

1) Answer: A

Page 316 Follow us: Official Site, Telegram, Facebook, Instagram, Instamojo
Option B: curtail means to cut short or reduce. This option is not apt.
Option C: prosperous mean successful; thriving; having or characterized by financial
success or good fortune. This option is out of context.
Option D: propel means drive forward; cause to move forward or onward; push. This is
incorrect option.
Option E: this cannot be the correct option as the question is fill in the blanks.
Option A: Praxis means as an accepted practice or custom, or an idea translated into
action, or something in reality rather than something in
theory. This fits well in both the blanks.
Therefore, option A is the correct answer choice to this question.

2) Answer: B
Option A: catastrophe means calamity; disaster; state of extreme ruin and misfortune.
This is incorrect option.
Option C:combat means struggle as with an opposing force; fight with; this option is not
apt.
Option D: corrode means destroy metal or alloy gradually, especially by chemical
action; be eaten or worn away. This option is incorrect.
Option E: this cannot be the correct option as the question is fill in the blanks.
Option B: callous means as cruel or insensitive, which fits perfectly in both the
sentences
Therefore, option B is the correct answer choice to this question.

3) Answer: C
Option A:acquainted means to know someone or to be familiar with something.It does
not fit in hence it is incorrect.
Option B: adhesive means substance that unites or bonds surfaces together. This is
incorrect option.
Option D: accede means agree; give consent, often at insistence of another;This option
is not apt
Option E: this cannot be the correct option as the question is fill in the blanks.
Option C: apathetic means someone who is not interested or doesn’t show any feeling
or emotion. This option is apt for the given blanks.
Therefore, option C is the correct answer choice to this question.

4) Answer: D
Option A: reluctance means unwillingness; hesitancy in taking some action. This option
does not fit in correctly.
Option B: remnant means remainder; small part or portion that remains after the main
part no longer exists. This option is incorrect.

Page 317 Follow us: Official Site, Telegram, Facebook, Instagram, Instamojo
Option C: reimburse means pay back for some expense incurred. This is contextually
incorrect.
Option E: this cannot be the correct option as the question is fill in the blanks.
Option D: remorse means a feeling of regret or sadness. This option fits in
grammatically and contextually.
Therefore, option D is the correct answer choice to this question.

5) Answer: A
Option B: emancipation: freeing someone from the control of another. This option
is not apt.
Option C: eloquent: vividly or movingly expressive; persuasive. This is incorrect option.
Option D: extravagant means wandering beyond one’s bounds; exceeding due bounds;
wild; excessive; unrestrained. This option is out of context.
Option E: this cannot be the correct option as the question is fill in the blanks.
OptionA : exuberance means as the state of being overly excited, joyful and happy.
Therefore, option A is the correct answer choice to this question.

Double Fillers

Directions (1-5): In the questions given below a sentence is given with two blanks
in each. Corresponding to each question two columns are given with three words
in each column. Which combinations of words from two columns will perfectly fit
into the blanks to make the sentence contextually correct and meaningful.

1) Living in Delhi proved to be _______in providing opportunities—a human rights


conference here, a film festival there—and he lapped them all up. “Delhi changed
my ideas about gender and sexuality,” he says about his growing _______with the
LGBTQ+ community.

A) A-D

B) B-F

Page 318 Follow us: Official Site, Telegram, Facebook, Instagram, Instamojo
C) C-F

D) A-F

E) None of these

2) The cast is _______. Thomas Middleditch is a Richard so awkward you can


believe he’s actually shrinking due to stress, Kumail Nanjiani is perfect as the
wannabe Dinesh, who needs to own the fastest car in office, Martin Starr slays
every single line as the smug and mostly all-knowing Gilfoyle, and Zach Woods
turns Jared into the show’s true leading man, with his horrifying backstories and
_______optimism.

A) A-D

B) C-D

C) B-E

D) A-F

E) None of these

3) “There is a unique food _______not to be missed if you are in Calcutta over a


weekend: Terreti Bazar on Sun Yat Sen Street. It is _______, with Chinese and Indian
stalls selling breakfast.”

Page 319 Follow us: Official Site, Telegram, Facebook, Instagram, Instamojo
A) A-D

B) B-F

C) C-F

D) A-F

E) None of these

4) This was the beginning of an email from Dan Bolton, tea _______and editor of the
digital magazine Tea Journey, in response to a question on how to best _______a
tea.

A) A-D

B) B-F

C) B-E

D) A-F

Page 320 Follow us: Official Site, Telegram, Facebook, Instagram, Instamojo
E) None of these

5) The ________ ______ reflects Shah’s personal fascination with thrift stores.

A) A-D

B) B-F

C) C-F

D) A-F

E) None of these

Answers

1) Answer: (a)

From the given options only A-D is matched and make a meaningful sentence. The
correct answer after matching will be

(A-D): Living in Delhi proved to be transformative in providing opportunities—a


human rights conference here, a film festival there—and he lapped them all up. “Delhi
changed my ideas about gender and sexuality,” he says about his
growing involvement with the LGBTQ+ community.

2) Answer: (c)

From the given options only B-E is matched and make a meaningful sentence. The
correct answer after matching will be

(B-E): The cast is phenomenal. Thomas Middleditch is a Richard so awkward you can
believe he’s actually shrinking due to stress, Kumail Nanjiani is perfect as the wannabe

Page 321 Follow us: Official Site, Telegram, Facebook, Instagram, Instamojo
Dinesh, who needs to own the fastest car in office, Martin Starr slays every single line as
the smug and mostly all-knowing Gilfoyle, and Zach Woods turns Jared into the show’s
true leading man, with his horrifying backstories and perennial optimism.

3) Answer: (b)

From the given options only B-F is matched and make a meaningful sentence. The
correct answer after matching will be

(B-F): “There is a unique food experience not to be missed if you are in Calcutta over a
weekend: Terreti Bazar on Sun Yat Sen Street. It is fascinating, with Chinese and Indian
stalls selling breakfast.”

4) Answer: (d)

From the given options only A-F is matched and make a meaningful sentence. The
correct answer after matching will be

(A-F): This was the beginning of an email from Dan Bolton, tea veteran and editor of
the digital magazine Tea Journey, in response to a question on how to
best appreciate a tea.

5) Answer: (c)

From the given options only C-F is matched and make a meaningful sentence. The
correct answer after matching will be

(C-F): The underlyingethos reflects Shah’s personal fascination with thrift stores.

Double Fillers

Directions (1-5): In the following Pacific countries ____ acute shortage of


question a sentence is given with two vaccines.
blanks. You have to choose the
correct pair of words that fill the A.advantage,cladding
blanks correctly in terms of grammar
and context. B.initiative, facing

C.edge,veneer
1) They also announced a major
D.lead,skin
vaccine ____ to battle Covid-19 in Indo-

Page 322 Follow us: Official Site, Telegram, Facebook, Instagram, Instamojo
E.gambit,facade they are not an exclusive club and
that they will “work with all of those
who share” their goals, ____ , a free
2) The day after, they lent their ____ to and open Indo-Pacific.
a joint op-ed in The Washington Post
____ their partnership as a “spark of A.certain,mainly
hope”.
B.sure,primarily
A.bylines, portraying
C.secure,mostly
B.secondary,paint
D.assured, chiefly
C.sideline,draw
E.reliable,especially
D.begining,picture

E.nameline,present 5) Work began on the summit early


February, according to people ____
with its planning and execution, and
3) They explained in ____ what they continued at a ____ pace since even as
did last Friday and what they ____ to their respective nations fought the
do going forward. deadly Covid-19.
A.specific,plan A.close,frantic
B.detail, intend B.dear,wild
C.fact,mean C.familiar,frenetic
D.point,aim D.near,frenzied
E.section,aspire E.bosom,hectic

4) They ____ the rest of the world that


Answers

1) Answer: B Option B- Both the words suit the


Option A-advantage,cladding do not fit context absolutely and are
into the context as they also announced grammatically also correct.
a major vaccine to battle Covid-19 in Therefore, option B is the correct
Indo-Pacific countries, then how answer choice to this question.
come advantage,claddingnot known.
Page 323 Follow us: Official Site, Telegram, Facebook, Instagram, Instamojo
who share” mentioning all the goals
2) Answer: A ‘especially’ again are contrary to each
Option A- Both the words suit the other.
context absolutely and are Option D – Both the words fit the blank
grammatically also correct. absolutely well, as assured and chieflyare
Therefore, option A is the correct within the contextual meaning of the
answer choice to this question. sentence.
Therefore, option D is the correct
3) Answer: B answer choice to this question.
Option A –specific and plan is not going
well withportraying their partnership as 5) Answer: C
a “spark of hope”, because by order it Option A – close and frantic is
will grant a relief but not partnership. contradictory with respect to the
Option B – Both the words are correct context. As if Work began on the summit
as detail by the explanation and what early February, according to people
they intend to do going forward. without its planning and execution.
Therefore, option B is the correct Option B – dear won’t ensure the use
answer choice to this question. “according to people” thus both the
words near and bosom in option are
4) Answer: D absurd.
Option A – Referring to “rest of the Option C – With regard to “according to
world that they are not an exclusive people familiar”and “continued at
club” certain and mainlyare contrary to a frenetic pace”, both the words fit the
each other. blank contextually and grammatically in
Option B – Referring to “rest of the a correct manner.
world that they are not an exclusive Therefore, option C is the correct
club” sure and primarily or mostly durin answer choice to this question.
g the briefing “work with all of those

Double Fillers

Directions (1-5): In each of the 1) The Scots singing sensation made


questions below, a sentence is given headlines last night after______ from a
with two blanks that indicate that bottle of Buck fast Tonic Wine
some parts are missing. Identify the on stage at the annual______ in
correct pair of words that fit in the London’s O2 Arena.
sentence to make it grammatically A) Idea, paralyze
and contextually correct. B) swigging, event
C) swig, jamboree
D) swigged, bash
Page 324 Follow us: Official Site, Telegram, Facebook, Instagram, Instamojo
E) None of these
4) He advised them that an interview
2) Home Secretary Priti Patel called with the Chancellor in the next day’s
the rapper’s additional verses to his Times should be____ to mean though
number one single Black highly_____ Brown didn’t_____ say this–that Britain
after they blasted the Government’s would not be joining the EU single
response to the Grenfell_______ and currency.
Wind rush scandal. A) interpreted, quite
A) Satisfaction, ingrown B) Undivided, bawdy
B) Tasteless, bummer C) crack, altogether
C) inappropriate, tragedy D) decode, utterly
D) out of order, agony E) None of these
E) None of these
5) In a short social media __________a
3) Scotland will be allowed to father and son are playing football in
administer these functions, a local park when they watch a video
but____ will be set by the UK the of SNP leader Nicola Sturgeon
better to_____ trade deals with ___________ Scotland being taken out of
countries like America. the EU against their will.
A) Swamp, hide A) clip, discussing
B) worth, aid B) link, canvass
C) grade, promote C) coupler, moot
D) standards, facilitate D) cylinder, argue
E) None of these E) None of these

Answers

1) Answer: (b)

In the given sentence, only ‘swigging, event’ make it grammatically as well as


contextually correct.

Option A): is incorrect because ‘Idea’ means a thought or suggestion as to a possible


course of action which does not fit here.

Option B): is the correct alternative among the following as ‘swigging’ means drink in
large draughts and it’s a present participle tense and ‘event’ means a thing that
happens or takes place, especially one of importance; both of which fit here both
grammatically and contextually.

Option C): is incorrect as ‘swig’ which does not fit here grammatically.
Page 325 Follow us: Official Site, Telegram, Facebook, Instagram, Instamojo
Option D): is incorrect as ‘swigged, bash’ does not fit here contextually and
grammatically.

2) Answer: (c)

In the given sentence only ‘inappropriate, tragedy’ make it grammatically as well as


contextually correct.

Option A): is incorrect because ‘Satisfaction’ means fulfillment of one’s wishes,


expectations, or needs, or the pleasure derived from this which does not fit here
contextually.

Option B): is incorrect ‘tasteless’ means lacking flavor which does not make any sense
here.

Option C): is the correct alternative among the following as ‘inappropriate’ means not
suitable or proper in the circumstances and ‘tragedy’ means an event causing great
suffering, destruction, and distress, such as a serious accident, crime, or natural
catastrophe both of which fit here both grammatically and contextually.

Option D): is incorrect as‘out of order, agony’ does not fit here contextually and
grammatically.

3) Answer: (d)

In the given sentence, only ‘standards, facilitate’ make it grammatically as well as


contextually correct.

Option A): is incorrect because ‘Swamp’ means an area of low-lying, uncultivated


ground where water collects; a bog or marsh which does not fit here contextually.

Option B): is incorrect as ‘worth’ means equivalent in value to the sum or item
specified. which does not make any sense here.

Option C): is incorrect as‘grade, promote’ does not fit here contextually and
grammatically.

Option D): is the correct alternative among the following as ‘standards’ means a level of
quality or attainment and ‘facilitate’ means make easy or easier which fit here both
grammatically and contextually.

4) Answer: (a)

Page 326 Follow us: Official Site, Telegram, Facebook, Instagram, Instamojo
In the given sentence, only ‘interpreted, quite’ make it grammatically as well as
contextually correct.

Option A): is correct because ‘interpreted’ means explain the meaning of and ‘quite’
means to the utmost or most absolute extent or degree which fit here contextually and
grammatically.

Option B): is incorrect as ‘Undivided’ means devoted completely to one object which
does not make any sense here.

Option C): is incorrect as‘crack, altogether’ which does not fit here contextually and
grammatically.

Option D): is the incorrect as ‘decode’ means convert (a coded message) into intelligible
language which does not fit here grammatically and contextually.

5) Answer: (a)

In the given sentence, only ‘clip, discussing’ make it grammatically as well as


contextually correct.

Option A): is the correct alternative among the following as ‘Clip’ means a flexible or
spring-loaded device for holding an object or objects together or in place and
‘discussing’ means talk or write about (a topic) in detail, taking into account different
issues or ideas both of which fit here both grammatically and contextually.

Option B): is incorrect as ‘link’ means a relationship between two things or situations,
especially where one affects the other does not make any sense here.

Option C): is incorrect because ‘coupler’ means a thing that connects two things,
especially mechanical components or systems which does not fit here contextually.

Option D): is incorrect as‘cylinder, argue’ does not fit here contextually and
grammatically.

Triple Fillers

Direction (1-5): Each question below contains a statement with three blanks
followed by four options. Choose the option that can fill the three blanks and
mark it as your answer.

Page 327 Follow us: Official Site, Telegram, Facebook, Instagram, Instamojo
1) So going forward, the model will move to how do you _______both people and
bots to deliver work. Also how do you leverage automation to actually disrupt
business processes for a customer, how do you help them _______better with legal
contracts, how do you help them with anomaly detection, how do you help with
anti-money laundering so business specific use cases that you can also devise.
There is second thing around _______and automation and then leveraging data,
intelligence that comes with it.

A) leverage, comply, balk

B) surveillance, impede, balk

C) surveillance, impede, cognition

D) leverage, comply, cognition

E) None of these

2) According to Taparia, in order to stay up to speed with your financial health,


you can create simple lists that clearly _______your monthly cash flows and give
you a holistic picture of your income, expenses, liabilities, savings and
investments. “This can be _______for optimal investment as it can alert you
whenever you are going off track or you have more _______income to invest,” he
said.

A) articulate, potent, disposable

B) scrutiny, thwart, disposable

C) scrutiny, thwart, foil

D) articulate, potent, foil

E) None of these

3) The _______about Cengiz suggests that, far from reining in a Saudi campaign to
silence critics at home and abroad, the Saudi government is _______what one
former Obama administration official called the state’s “posture” to monitor
_______and critics.

A) revelation, reinforcing, inhibit

Page 328 Follow us: Official Site, Telegram, Facebook, Instagram, Instamojo
B) assist, nurture, inhibit

C) assist, nurture, dissidents

D) revelation, reinforcing, dissidents

E) None of these

4) In December, the Polish supreme court, which is still independent, ruled that
the National Council of the Judiciary and the new _______chamber were unlawfully
_______. The government responded with legislative _______to make it illegal for any
judge to question the legality of its appointments – in effect banning judges from
complying with the supreme court ruling.

A) disciplinary, constituted, adjourn

B) discourage, defer, adjourn

C) disciplinary, constituted, proposals

D) discourage, defer, proposals

E) None of these

5) Since coming to power in a military coup in 2013, President Abdel-Fatah al-Sisi


has _______a broad crackdown on political activity and free speech,
_______journalists, NGO workers and even the mildest critics. Egypt has built at
least 19 new prisons since 2011, housing a swath of ordinary citizens such as
Mohamed Nassif Mohamed Ghoneim, a tax office employee who was jailed in
2018 for a Facebook comment considered “offensive to the ruling _______, Sisi and
the tax authority’s leaders.”

A) overseen, detaining, uphold

B) overseen, detaining, regime

C) hurry, offset, uphold

D) hurry, offset, regime

E) None of these

Page 329 Follow us: Official Site, Telegram, Facebook, Instagram, Instamojo
Answers

1) Answer: (d)

In the given sentence, only ‘leverage, comply, cognition’ make it grammatically as well
as contextually correct.

Option A): is incorrect because balk means hesitate or be unwilling to accept an idea or
undertaking which does not fit here.

Option B): is incorrect as ‘surveillance’ does not make any sense here.

Option C): is incorrect as impede means delay or prevent (someone or something) by


obstructing them; hinder which does not fit here.

Option D): is the correct alternative among the following as ‘leverage, comply,
cognition’ fit here both grammatically and contextually.

2) Answer: (a)

In the given sentence, only ‘articulate, potent, disposable’ make it grammatically as well
as contextually correct.

Option A): is the correct alternative among the following as ‘articulate, potent,
disposable’ fit here both grammatically and contextually.

Option B): is incorrect as scrutiny does not make any sense here.

Option C): is incorrect as thwart means prevent (someone) from accomplishing


something which does not make any sense here.

Option D): is incorrect as foil does not fit here contextually

3) Answer: (d)

In the given sentence, only ‘revelation, reinforcing, dissidents’ make it grammatically as


well as contextually correct.

Option A): is incorrect because inhibit does not fit here.

Option B): is incorrect as ‘assist’ does not make any sense here.

Page 330 Follow us: Official Site, Telegram, Facebook, Instagram, Instamojo
Option C): is incorrect as ‘nurture’ does not fit here.

Option D): is the correct alternative among the following as ‘revelation, reinforcing,
dissidents’ fit here both grammatically and contextually.

4) Answer: (c)

In the given sentence, only ‘disciplinary, constituted, proposals’ make it grammatically


as well as contextually correct.

Option A): is incorrect because adjourn means break off (a meeting, legal case, or game)
with the intention of resuming it later which does not fit here.

Option B): is incorrect as ‘discourage’ does not fit here.

Option C): is the correct alternative among the following as ‘disciplinary, constituted,
proposals’ fit here both grammatically and contextually.

Option D): is incorrect as defer does not fit here contextually

5) Answer: (b)

In the given sentence, only ‘overseen, detaining, regime’ make it grammatically as well
as contextually correct.

Option A): is incorrect as ‘uphold’ does not make any sense here.

Option B): is the correct alternative among the following as ‘overseen, detaining,
regime’ fit here both grammatically and contextually.

Option C): is incorrect as ‘hurry’ does not fit here.

Option D): is incorrect as ‘offset’ does not make any sense here.

Triple Fillers

Direction (1-5): In each questions the options, which can fit in the
below, a paragraph is given, blanks both grammatically and
containing three blanks. Six words meaningfully.
are given after the paragraph. Choose
correct combination of words from 1) Without that, the science becomes
_______, global issues seem too huge to
Page 331 Follow us: Official Site, Telegram, Facebook, Instagram, Instamojo
grasp, and it becomes difficult to F. impotence
relate to far off places and other
species. Without that, the A) A, C & E
“environment” slips too easily into an
elite _______for academics, B) B, D & F
policymakers and middle-class white
people, when it should be recognised C) A, D & F
as the main driver of inequality,
_______and injustice. D) B, C & F

A. certitude E) None of these


B. abstract
C. baleful 3) “We think there’s a way to get to a
D. pigeonhole point where it’s going to be less and
E. salutary less _______over time,” he said. “It will
F. conflict take them some time to
_______someone to lead the
A) A, C & E organisation and during that period
of time, their actions may be a little
B) B, D & F bit _______.”

C) A, D & E A. effective
B. fruition
D) B, C & E C. inception
D. re-establish
E) None of these E. rail
F. disjointed
2) One, _______state and district-level
initiatives in both the ADP and non- A) A, C & E
ADP districts in areas prioritised
under the programme. Two, _______by B) B, D & F
competition on outcomes, local
governments target their efforts and C) A, D & F
improve programme _______and
design. D) B, C & E

A. pioneering E) None of these


B. bandwidth
C. spurred 4) On Thursday, the House of
D. inability Representatives will vote on whether
E. implementation to _______a series of guardrails of its
own, for the possible _______of the
Page 332 Follow us: Official Site, Telegram, Facebook, Instagram, Instamojo
president. The resolution now before name of cow protection; it was also
Congress avoids past missteps by because it contained all the
allowing extended questioning of ingredients of a hate crime:
witnesses by staff lawyers before unplanned violence, ideological
_______lawmakers take the stage, and it motivation, _______towards sections of
sets fair rules that respect precedent. society and interference with dietary
choice. The manner in which the
A. inveigh murder case was allowed to collapse
B. erect in court, possibly due to planned
C. impeachment lapses, shows how the system was
D. reflex _______to favour the mob.
E. preening
F. plight A. emblematic
B. serenity
A) A, C & E C. decimate
D. intolerance
B) B, D & F E. subvert
F. rigged
C) A, D & F
A) A, C & E
D) B, C & E
B) B, D & F
E) None of these
C) A, D & F
5) The Pehlu Khan murder case
attained _______significance not only D) B, C & E
because it was one of the earliest
instances of the wave of vigilante E) None of these
attacks across the country in the

Answers :

1) Answer: (b)

In the given sentence, only ‘abstract, pigeonhole, conflict’ make it grammatically as well
as contextually correct.

Option A): is incorrect because certitude means absolute certainty or conviction that
something is the case which does not fit here.

Page 333 Follow us: Official Site, Telegram, Facebook, Instagram, Instamojo
Option B): is the correct alternative among the following as ‘abstract, pigeonhole,
conflict’ fit here both grammatically and contextually.

Option C): is incorrect because salutary means (especially with reference to something
unwelcome or unpleasant) producing good effects; beneficial which does not fit here.

Option D): is incorrect as baleful means threatening harm; menacing which does not fit
here contextually

2) Answer: (a)

In the given sentence, only ‘pioneering, spurred, implementation’ make it


grammatically as well as contextually correct.

Option A): is the correct alternative among the following as ‘pioneering, spurred,
implementation’ fit here contextually.

Option B): is incorrect as bandwidth means a range of frequencies within a given band,
in particular that used for transmitting a signal.

Option C): is incorrect as ‘inability’ does not make any sense here.

Option D): is incorrect as ‘impotence’ does not fit here contextually.

3) Answer: (c)

In the given sentence, only ‘effective, re-establish, disjointed’ make it grammatically as


well as contextually correct.

Option A): is incorrect because inception does not fit here.

Option B): is incorrect as fruition means the realization or fulfilment of a plan or


project.

Option C): is the correct alternative among the following as ‘effective, re-establish,
disjointed’ fit here both grammatically and contextually.

Option D): is incorrect as rail does not fit here contextually

4) Answer: (d)

Page 334 Follow us: Official Site, Telegram, Facebook, Instagram, Instamojo
In the given sentence, only ‘erect, impeachment, preening’ make it grammatically as
well as contextually correct.

Option A): is incorrect because inveigh means speak or write about (something) with
great hostility which does not fit here.

Option B): is incorrect as reflex means an action that is performed without conscious
thought as a response to a stimulus.

Option C): is the incorrect as plight means a dangerous, difficult, or otherwise


unfortunate situation

Option D): is the correct alternative among the following as ‘erect, impeachment,
preening’ fit here both grammatically and contextually.

5) Answer: (c)

In the given sentence, only ‘emblematic, intolerance, rigged’ makes it grammatically as


well as contextually correct.

Option A): is incorrect because decimate means kill, destroy, or remove a large
proportion of which does not fit here.

Option B): is incorrect as ‘serenity’ means the state of being calm, peaceful, and
untroubled.

Option C): is the correct alternative among the following as ‘emblematic, intolerance,
rigged’ fit here both grammatically and contextually.

Option D): is incorrect as ‘subvert’ does not fit here contextually.

Phrase Replacement
Phrase Replacement: Tips And Tricks

What is Phrase Replacement ?

Before we dive in further into this topic, let’s understand what is phrase
replacement? Phrase replacement refers to the type of question in which a certain

Page 335 Follow us: Official Site, Telegram, Facebook, Instagram, Instamojo
sentence is given and some part of it is highlighted, which is incorrect and need to
be replace. Now we usually come across to two types of phrase replacement. One
which is based on grammar and another which is based on phrases. We will be
discussing both of them one by One

How to attempt phrase replacement?

 To find the error in the sentence you must have a good knowledge of Grammar
Rules. Always remember to check the correct use of the verb and the tense.
Many times, the tense is written wrongly or the verb is misplaced. This will
help you to solve the questions easily and quickly.
 To solve these questions, read the sentence or statement carefully
Then, analyze the meaning and more importantly the tone of the given
sentence i.e., positive, negative, sarcastic, critical or analytical. After that, find
the correct option that will make the sentence grammatically correct and will
not change the meaning of the statement.
 You can also use the elimination approach. Try to eliminate the inappropriate
options. The correct option or answer will be grammatically correct based on
that eliminate all grammatically incorrect options. This approach will help you
to find the answers quickly
 Replace the phrase written in the correct option with the bold or underline
part of the question and read the whole sentence once again. While
changing/replacing the underlined or bold part, make sure that the meaning of
the sentence doesn’t change.
 It is not necessary that there should always be errors present in the sentence.
Often, the sentence is grammatically correct and there are no errors present in
the statement. Try to avoid the habit of always finding the errors in a sentence.
Sometimes no correction is needed in a sentence and in that situation choose
“No correction required” option.

Phrase Replacement - Set-1


Directions (1-5): In this question, a 1) This proceeding was fated to shine a
part of the sentence is made bold. light on the Court’s conduct ratheron
Below are given alternatives to the the actions of the contemnor.
bold part at (A), (B), (C) and (D)
which may improve the sentence. A. rather than on the actions of the
Choose the correct alternative. In contemnor
case no replacement is needed, mark B. rather than in the actions of the
(E) as your answer. contemnor
Page 336 Follow us: Official Site, Telegram, Facebook, Instagram, Instamojo
C. rather at the actions of the 4) In a State that resisted multiple
contemnor attempts in impose Hindi since 1937,
D. rather than on top of actions of political parties are understandably
the contemnor wary of any mandate to impart an
E. No replacement additional language in schools.

2) The Court, in its order, did take A. multiple attempts for impose
cognisance of the multiple threats in Hindi since 1937
the industry’s well-being. B. multiple attempts on imposing
Hindi since 1937
A. multiple threats of the C. multiple attempts to impose
industry’s well-being. Hindi since 1937
B. multiple threats with the D. multiple attempts imposing
industry’s well-being. Hindi since 1937
C. multiple threats at the E. No replacements
industry’s well-being.
D. multiple threats of the 5) India’s federal nature and
industry’s well-being. diversity demands in regional
E. No replacement languageis given supremacy over
another.
3) In the euphoria surrounding the great
Indian victory, some grim facts should A. demand that regional language
be overlooked. B. demands in no regional
language
A. some grim fact should always C. demanded that no regional
be overlooked. language
B. some grim facts should not be D. demand that no regional
overlooked. language
C. some grim facts should be E. No replacement.
never overlooked.
D. some grim facts should be
always overlooked.
E. No replacement.
Answers
1) Answer: A The preposition ‘in’ is contextually
incorrect and should be replaced by ‘of’.
Use of ‘than’ after ‘rather’ is Therefore, option A is the correct option.
grammatically correct. Therefore,
option A is the correct answer. 3) Answer: B

2) Answer: A
Page 337 Follow us: Official Site, Telegram, Facebook, Instagram, Instamojo
The sentence implies a concern amidst Therefore, option C is the correct
of happiness. Only option B is answer.
grammatically and contextually correct
in positioning of the word ‘not’. 5) Answer: D
Therefore, option B is the correct
answer. The replacement “demand that no
regional language” serves both
4) Answer: C contextuallyand grammatically correct
with context meaning of the sentence.
The use of ‘to’ between ‘attempts’ and Therefore, option D is the correct
‘impose’ is grammatically meaningful answer.
and hence, contextually correct.
Phrase Replacement - Set-2
Direction (1-5): In the following environmental advocacy groups. The
sentences, a part is bold and proposal have already won state and
underlined. You have to choose which federal approval.
phrase given in options should
replace the phrase printed in bold to A. had already won
make the sentence grammatically B. already won
correct. Mark (E) as your answer if no C. is winning
correction is required. D. won
E. No correction required
1) Nearly a third of Delhi’s population
has been exposed from Covid-19 and 3) In the months since Covid-19 swept
has developed antibodies, the city the globe, leaders the world over was
government said today, giving details of been accused of exploiting the
the second serological survey or pandemic for political gain while laying
serosurvey in the capital. waste to democratic norms. Few, if any,
have gone as far as to reveal those plans
A. Exposed for publicly
B. Exposed to
C. Exposes with A. is being accused
D. Expose from B. was accused
E. No correction required C. has been accused
D. have been accused
2) A plan to release over 750 million E. No correction required
genetically modified mosquitoes into the
Florida Keys in 2021 and 2022 received 4) In the last one month, the number of
final approval from local authorities, confirmed cases has grown by almost
against the objection of many local four times, at a rate which are
residents and a coalition of among the fastest in India.
Page 338 Follow us: Official Site, Telegram, Facebook, Instagram, Instamojo
A. which were among remain hostage to one singular issue as
B. which was among this is an all-encompassing relation.
C. which is among
D. is among A. for the outcome
E. No correction required B. of the outcome
C. to the outcome
5) But irrespective on the outcome of D. off the outcome
the CPN tussle, Indo-Nepal ties cannot E. No correction required
Answers

1) Answer: B the correct phrase will be “have been


accused”.
When it comes to a disease/infection,
the correct usage is “exposed to” (the Therefore option D is the right answer
disease). The phrase “expose from” is to this question.
incorrect in this case.
Therefore option B is the correct answer 4) Answer: C
to this question.
The form of the expression is singular so
2) Answer: A the given expression – “which are” is
incorrect as it is in the plural form. The
The form of the sentence is singular as correct expression is singular – so we
only one proposal is being referred to. need to use the phrase “which is among”
Therefore, the verb form is incorrect the fastest.
since it is in plural. Since the tense is
past (i.e. approval from authorities Therefore option C is the correct answer
received aprior), hence the correct to this question.
expression to use will be “had already
won” 5) Answer: B

Therefore option A is the correct answer The preposition used at the start of the
to this question. phrase is incorrect. The correct
preposition to be used here is “of”. The
3) Answer: D phrase “on the outcome” could be used
in situation like follows – the
We never use the auxiliary “was” before organization of the IPL abroad is
the verb “been”. The auxiliaries to use dependenton the outcome of the
before been are “has”, “have” or “had”. In meeting between the BCCI and the
this case, the form is plural and the sports ministry officials. Here though,
sentence is in present perfect tense. So we need to use the phrase – “of the
outcome”.
Page 339 Follow us: Official Site, Telegram, Facebook, Instagram, Instamojo
Phrase Replacement - Set-3
Direction (1-5): The following E) No correction required
question consists of a sentence which
is divided into three parts which 2) He had been reading Gottlob Frege
contain grammatical errors in one or and Bertrand Russell in his spare
more than one part of the sentence, time, (a)/and believed that
as specified in bold in each part. If their inquiries into the foundations of
there is an error in any part of the logic heralded a revolution(b)/ even
sentence, find the correct alternative more exciting than the invention of
to replace those parts from the three powered flight.(c)/
options given below each question to
make the sentence grammatically a) had being reading
correct. If the given sentence is
grammatically correct or does not b) inquiries to
require any correction, choose (E), i.e
“No correction required” as your c) even more excited
answer.
A) only a
1) Several posters asked Manne to
provide better evidence; one suggested B) only c
she try cognitive behavioral
therapy, (a)/and then there were the C) both a and c
cruder responses, the ones that women
who think in public have learned to D) both b and c
expect: (b)/the knocks on her
intelligence, the speculate about her E) No correction required
love life, the slur “soulless bitch. (c)/
3) Russell’s friend G.E. Moore had
a) Several poster already wowed the gilded youth of
Cambridge with his doctrine that
b) has learned to expect ‘goodness’ and ‘beauty’
are supernatural entity (a)/accessible
c) the speculation about only to first-class minds, and Russell
was trying to do something similar for
A) only a logic, (b)/maintaining that anyone
clever enough to understand it gain
B) only c access to a realm of intellectual
perfection beyond the hurly-burly of
C) both a and c earthly existence. (c)/

D) both b and c a) supernatural entities


Page 340 Follow us: Official Site, Telegram, Facebook, Instagram, Instamojo
b) trying to doing B) only c

c) it gains access C) both a and c

A) only a D) both b and c

B) only c E) No correction required

C) both a and c 5) He was the great innovator of


middle-class domestic
D) both b and c tragedy, (a)/ endowing ordinary
life with the moral significance (b)/
E) No correction required previously reserved of princes and
legendary heroes .(c)/
4) They lost touch but continued to
work separately in the a) the best innovator
problems (a)/they had clashed
over before the war: the nature of logic b) endowing ordinary lives
and its relation to (b)/eternal truths on
the one hand and human experience on c) reserved for princes
the other.(c)/
A) only a
a) on the problems
B) only c
b) clashed on
C) both a and c
c) eternal truth
D) both b and c
A) only a
E) No correction required
Answers:

1) Answer: B 3) Answer: C

Part c is incorrect as ‘speculate’ should Part a is incorrect as ‘entity’ should be


be replaced with ‘speculation’. replaced with ‘entities’.

2) Answer: E Part c is incorrect as ‘gain’ should be


replaced with ‘gains’.
All bold parts are correct.
4) Answer: A
Page 341 Follow us: Official Site, Telegram, Facebook, Instagram, Instamojo
Part a is incorrect as ‘in’ should be Part b is incorrect as ‘life’ should be
replaced with ‘on’. replaced with ‘lives’.

5) Answer: D Part c is incorrect as ‘of’ should be


replaced with ‘for’.
Phrase Replacement - Set-4

Direction (1-5): The following B) both a and b


question consists of a sentence which
is divided into three parts which C) only c
contain grammatical errors in one or
more than one part of the sentence, D) both b and c
as specified in bold in each part. If
there is an error in any part of the E) No correction required
sentence, find the correct alternative
to replace those parts from the three 2) If we can assure them that these
options given below each question to risks can be cheaply hedged, capital
make the sentence grammatically will flow in—(a)/not just in technology
correct. If the given sentence is startups and infrastructure, and also
grammatically correct or does not into companies marked (b)/for
require any correction, choose (E), i.e resolute or liquidation at the
“No correction required” as your country’s insolvency and bankruptcy
answer. courts(c)/

1) It is also evident that any a) If we can assure them that this risks
rejuvenation of spirits will can be cheaply hedged
need (a)/the government to
demonstratively work on lifting the b) but also into companies marked
pall of trepidation (b)/that hang heavy
over the financial sector (c)/ c) for resolution or liquidation at the
country’s insolvency
a) It is also evidence that any
rejuvenation A) only a

b) the government to demonstrate work B) both a and b


on
C) only c
c) that hangs heavy over
D) both b and c
A) only a
E) No correction required
Page 342 Follow us: Official Site, Telegram, Facebook, Instagram, Instamojo
3) As the only two major developing b) but the drafting process highlights
countries with a population of over one the complexities inherent in
billion (a)/and important
representatives of emerging c) welfare scheme and points out the
economies, China-India relations enormous challenges
transcend the bilateral
dimension (b)/and assume global A) only a
and strategic significance.(c)/
B) both a and b
a) As the only two major developing
country with C) only c

b) important representative of emerging D) both b and c


economies
E) No correction required
c) strategic signify
5) It talks about insurance, provident
A) only a fund and other benefits to gig
workers,(a)/ but fails to clarify who is
B) both a and b responsible for providing them—
(b)/the state or the new-age
C) only c entrepreneurs who are
promoting platform economy for lakhs
D) both b and c of people.(c)/

E) No correction required a) provident fund and other benefits for


gig workers
5) The central government have
released a revised draft Code on Social b) but fail to clarify who is responsible
Security (CSS) (a)/and the drafting for
process highlights the complexities
inherent in combining a diverse set of c) the state or the new-age
labour laws (b)/and welfare schemes entrepreneurs who is promoting
and points out the enormous
challenges for implementing it after its A) only a
enactment, which appears to be a far cry
at this stage.(c)/ B) both a and b

a) The central government has released C) only c

D) both b and c

Page 343 Follow us: Official Site, Telegram, Facebook, Instagram, Instamojo
E) No correction required

Answer

1) Answer: (c) All statements are in correct form.

Statement c is incorrect here. ‘hang’ 4) Answer: (b)


should be replaced with ‘hangs’.
Statement a is incorrect here. ‘have’
2) Answer: (d) should be replaced with ‘has’.

Statement b is incorrect here. ‘and also’ Statement b is incorrect here. ‘and’


should be replaced with ‘but also’. should be replaced with ‘but’.

Statement c is incorrect here. ‘resolute’ 5) Answer: (a)


should be replaced with ‘resolution’.
Statement a is incorrect here. ‘To gig
3) Answer: (e) workers’ should be replaced with ‘for gig
workers’.

Phrase Replacement – Set-5

Directions (1-5): In each of the and emerged as an inspiration for


questions given below a part of the society at large.
sentence is given in bold. It is then
followed by three options. Find the I) to great minds
alternative that can replace the given
bold part to make the sentence II) upto great minds
grammatically and contextually
III) onto great minds
correct. If none of the alternatives is
correct and the sentence is correct as A.both II and III
it is then select option (E) as your
choice. B.both I and III

C.both I and II
1) HT Trailblazers awards, presented by
Ambience, is a token of gratitude to the D.only III
great minds who did not let their spirit
go down even in the darkest of times E.no correction required

Page 344 Follow us: Official Site, Telegram, Facebook, Instagram, Instamojo
E.no correction required
2) The awards was a token of
appreciation to these great minds who
came from different walks of life and use 4) Here are the eight winners who left a
innovation to create all-new point in their respective fields, which
experiences. earned them the HT Trailblazer badge.

I) being a token of appreciation I) who left a mark

II) are a token of appreciation II) who left a point

III) been a token of appreciation III) who left a token

A.both I and II A.both I and III

B.only II B.both II and III

C.only III C.both I and II

D.both I and III D.only III

E.no correction required E.no correction required

3) Congratulations to all our winners for 5) I hope Ambience – Hindustan Times


dreaming big and scripting inspiring Trailblazers will continue to serve as
stories during the majority testing one of the most prestigious and credible
times of the pandemic. platforms for acknowledging
and honouring real life heroes.
I) during the majority testing times
I) esteeming real life heroes
II) during the most testing times
II) respecting real life heroes
III) during the utmost testing times
III) keeping real life heroes
A.both I and II
A.both I and II
B.both II and III
B.both II and III
C.both I and III
C.both I and III
D.only II
D.only III

Page 345 Follow us: Official Site, Telegram, Facebook, Instagram, Instamojo
E.no correction required

Answers

1) Answer: E According to the context of the sentence


it is regarding the fact that
The sentence is correct in the present scripting inspiring stories during the
form and no correction is required to be testing times of the pandemic. The
carried out in the sentence. correct adverb used for that purpose
is most. Therefore it should have
Therefore, option E is the correct been during the most testing times.
answer choice to this question. The correct sentence would be:
Congratulations to all our winners for
dreaming big and scripting inspiring
2) Answer: B
stories during the majority testing
There is the error in the subject-verb times of the pandemic.
agreement here because the subject
Therefore, option D is the correct
here is in plural form whereas the
answer choice to this question.
corresponding verb has been used in the
singular form i.e. was a token of
appreciation. The subject awards is 4) Answer: C
plural and is followed by plural verbs
only. Therefore it should have been are a Here the context is that the eight
token of appreciation. winners which earned them the
Trailblazer badge. Both I and II will be
The correct sentence would have correct in this context if placed instead
been: of the bold phrase in the given sentence.
The awards are a token of The correct sentence would be:
appreciation to these great minds who
came from different walks of life and use Here are the eight winners who left a
innovation to create all-new point in their respective fields, which
experiences. earned them the HT Trailblazer badge.
Therefore, option B is the correct Therefore, option C is the correct
answer choice to this question. answer choice to this question.

3) Answer: D 5) Answer: E

Page 346 Follow us: Official Site, Telegram, Facebook, Instagram, Instamojo
There is no error in the highlighted part Therefore, option E is the correct
of the given sentence and therefore no answer choice to this question.
replacement is required here.

Phrase Replacement – Set-6

Directions(1-5) : In this question, a


part of the sentence is made bold. 3) The entry of the Sars-CoV-2 virus on
Below are given alternatives to the the person body is a two-step
bold part at (A), (B), (C) and (D) procedure.
which may improve the sentence.
Choose the correct alternative. In A. into the person
case no replacement is needed, mark B. onto the person
(E) as your answer.
C. within the human
1) It was not the age profile or weather D. in the human
conditions, but their deficiency of a
especial protein that led to the rapid E. no correction required
spread of coronavirus disease.
4) As a result of its rapid spread in
A. their deficiency of a especial
various countries around the world,
B. the deficiency of a particular WHO issued an announcement of the
change in coronavirus status from
C. this deficiency of a peculiar
pandemic to epidemic disease.
D. a deficiency of an estranged
A. of pandemic to epidemic
E. no correction required
disease
B. about pandemic to epidemic
2) The study was recently published in
disease
the journal Infections, Genetics and
Evolution. C. which is pandemic to epidemic
disease
A. is recently published
D. that of pandemic to epidemic
B. were recently published disease
C. which was recently published E. no correction required
D. that was recently published
E. no correction required 5) China called on the United States on
Wednesday onto invite the World

Page 347 Follow us: Official Site, Telegram, Facebook, Instagram, Instamojo
Health Corporation to investigate C. to invite the World Health
origins of the Covid-19. Organisation

A. onto invite the World Health D. wasto invite the World Health
Organisation Organisation

B. isto invite the World Health E. no correction required


Corporation
Answers

1) Answer: B

The original sentence is incorrect.

Reason: There are two errors in the bold phrase.

1st. Instead of ‘their deficiency’, an article ‘the deficiency’ should be used here.

2nd. In the sentence, the writer refers to the subject ‘especial protein’ and not the
adjective ‘particular’. Therefore, ‘especial’

should be replaced by ‘particular’ to make it a meaningful sentence.

Therefore, among the given choices option B replaces the bold part most appropriately.

The sentence after replacement becomes:

It was not the age profile or weather conditions, but the deficiency of a particular
protein that led to the rapid spread of coronavirus disease.

Therefore, option B is the correct answer choice to this question.

2) Answer: E

The original sentence is absolutely correct and needs no correction.

Therefore, option E is the correct answer choice to this question.

3) Answer: D

The original sentence is incorrect.

Page 348 Follow us: Official Site, Telegram, Facebook, Instagram, Instamojo
Reason:

1st. As we can observe that the sentence is made in present tense, usage of the
preposition ‘on’ is erroneous here. Instead of ‘on’, ‘in’ (because the meaning conveyed is
`into’ the human body) should be used here.

2nd. ‘person’ means ‘a human being regarded as an individual’ and the word doesn’t
make any sense in the context of the sentence. Instead of it, the noun ‘human’ that
means ‘characteristic of humankind’ should be used here.

Therefore, among the given choices option D replaces the bold part most appropriately.

The sentence after replacement becomes:

The entry of the Sars-CoV-2 virus in the human body is a two-step procedure.

Therefore, option D is the correct answer choice to this question.

4) Answer: C

The original sentence is incorrect.

Reason:

1st. The phrase ‘epidemic’ and not ‘pandemic’ is used to denote an outbreak of disease
that spreads quickly. Therefore, ‘pandemic’ must be replaced by ‘epidemic’ here.

2nd. As the preposition ‘from’ is used in its plural form, it’s clear that the subject has to
be plural too. Therefore, the noun ‘disease’ must be replaced by ‘diseases’ here.

Therefore, among the given choices option C replaces the bold part most appropriately.

The sentence after replacement becomes:

As a result of its rapid spread in various countries around the world, WHO issued an
announcement of the change in coronavirus status from pandemic to epidemic
diseases.

Therefore, option C is the correct answer choice to this question.

5) Answer: C

Page 349 Follow us: Official Site, Telegram, Facebook, Instagram, Instamojo
The original sentence is incorrect.

Reason:

1st. The phrase ‘to invite’ and not ‘onto invite’ is used to denote `approaching or
reaching (a particular condition)’.

2nd. As the preposition ‘to invite’ is used in its plural form, it’s clear that the subject has
to be plural too. Therefore, ‘Corporation’ must be replaced by ‘Organisation’ here.

Therefore, among the given choices option C replaces the bold part most appropriately.

The sentence after replacement becomes:

China called on the United States on Wednesday to invite the World Health
Organisation to investigate origins of the Covid-19.

Therefore, option C is the correct answer choice to this question.

Phrase Replacement – Set-7

Directions (1-5): In each of the III) Respective states to be reopen


questions given below a part of the
sentence is given in bold. It is then A. Both II and III
followed by three options. Find the B. Both I and III
alternative that can replace the given
bold part to make the sentence C. Both I and II
grammatically and contextually D. Only III
correct. If none of the alternatives is
correct and the sentence is correct as E. No Correction Required
it is then select option (E) as your 2) Parents had been also askedabout,
choice. whether they would consider giving he
Covid-19 vaccine to their child after it is
1) Nearly 70% parents who responded rolled out for school children.
to a nationwide survey said they prefer
schools in their respective states to I) Parents have also asked
reopen only post April in light of Covid-
19 and the resultant lockdown. II) Parents were also asked

I) Respective states will reopen III) Parents were also being asked

II) Respective states should reopen A. Both I and II

Page 350 Follow us: Official Site, Telegram, Facebook, Instagram, Instamojo
B. Only II III) Were arrested on suspicion
C. Only III A. Both I and III
D. Both I and III B. Both II and III
E. No Correction Required C. Both I and II
3) The 10 members of the module flew D. Only III
out of the country in a plane arranged
by the Chinese government. E. No Correction Required
5) Ant Group was also under heavy
I) Module flew out of
scrutiny by the Chinese government
II) Module were flown out of after Jack Ma’s speech that criticised
the regulation system of the country
III) Module had been flown out and called the banks “pawnshops”.

A. Both I and II I) Whose criticism in the regulation


system
B. Both II and III
C. Both I and III II) Who hadcriticised the regulation
system
D. Only II
E. No Correction Required III) Whose critics in the regulation
system
4) A number of them (Chinese
National)were arrested on A. Both I and II
suspicion of being involved in
B. Both II and III
kidnapping and assassination…,
C. Both I and III
I) Have been arrested on suspicion
D. Only III
II) Had been arrested on suspicion E. No Correction Required
Answers

1) Answer: E

The sentence is correct in the present form and no correction is required to be carried
out in the sentence.

Therefore, option E is the correct answer choice to this question.

2) Answer: B

Page 351 Follow us: Official Site, Telegram, Facebook, Instagram, Instamojo
There is the error in the subject-verb agreement here because the subject here is in
plural form whereas the corresponding verb has been used in an auxillarysingular form
i.e. `had been also asked’. The subject parents is in plural form and is to be followed by
anauxillary verb in plural form only. Therefore it should have been `were also asked’.

The correct sentence would have been:


Parents were also asked about, whether they would consider giving he Covid-19
vaccine to their child after it is rolled out for school children.

Therefore, option B is the correct answer choice to this question.

3) Answer: D

According to the context of the sentence it is regarding the fact that `10 members of the
module flew out of the country ‘. The correct intransitive verb used for that purpose is `
were flown’ in plural form, adjective, decorated with colors that have been fluidly
blended: `flown ceramic ware’.Therefore it should have been `were flown out of `.

The correct sentence would be:


The 10 members of the module were flown out of the country in a plane arranged by
the Chinese government.

Therefore, option D is the correct answer choice to this question.

4) Answer: C

Here the context is that `A number of them (Chinese national) were arrested on
suspicion……’subject in plural form,followed by `being involved’ verb. Involvement,
`noun’, is the act of participating in something. Verb, The police arrested (transitive
verb with inflections) him on drug charges. Both I and II will be correct in this context if
placed instead of the bold phrase in the given sentence.
The correct sentence would be:
A number of them (Chinese National)have/had been arrested on suspicion of being
involved in kidnapping and assassination…,

Therefore, option C is the correct answer choice to this question.

5) Answer: E

There is no error in the highlighted part of the given sentence and therefore no
replacement is required here.

Therefore, option E is the correct answer choice to this question.


Page 352 Follow us: Official Site, Telegram, Facebook, Instagram, Instamojo
Phrase Replacement – Set-8

Directions : (1-5): In the given E. No Correction Required


questions a part is highlighted. It may
3) The HC observed that even though
or may not need improvement. If it
the festive season has come to an end,
has any issue then select the best
people have repeatedly started
choice as a replacement.
hitting the streets to celebrate the
1) France’s Academy of Fine Arts says New Year.
Pierre Cardin, the French
I) people have started again
designer whose Space Age style was
among the iconic looks of 20th-century II) people have again started
fashion, has died at 98.
III) people have reportedly started
I) Who had a space age style
A. Both I and II
II) while his space age style
B. Both II and III
III) where his space age style C. Both I and III
A. Both II and III D. Only II
B. Both I and III E. No Correction Required
C. Both I and II 4) The total number of people testing
positive for the new UK variable
D. Only III
of genome of Sars-CoV-2 virus has
E. No Correction Required reached 20, the Union ministry of health
2) They were on the backfoot after the and family welfare announced on
debacle in Adelaide and did not have Wednesday.
the service of his captain Virat Kohli.
I) UK variant of genome for
I) never had the service
II) UK variant genome
II) have the services of their
III) UK variable of genome
III) hasn’t had the services
A. Both I and III
A. Both Iand II B. Both II and III
B. Only II C. Both I and II
C. Only III D. Only III
D. Both I and III E. No Correction Required
Page 353 Follow us: Official Site, Telegram, Facebook, Instagram, Instamojo
5) Union defence minister Rajnath Singh III) no “meaningful aerosol” has
on Wednesday said that no
“meaningful solution” has come out of A. Both I and II
diplomatic and military level B. Both II and III
talks with China to resolve the standoff
on the Line of Actual Control (LAC). C. Both I and III
D. Only III
I) no “meaningful outcome” has
E. No replacement required
II) no “meaningful infusion” has
Answers

1) Answer: E

The sentence is correct in the present form and no correction is required to be carried
out in the sentence.

Therefore, option E is the correct answer choice to this question.

2) Answer: B

There is the error in the subject-verb agreement here because the subject here is in
plural form whereas the corresponding verb

has been used in the singular form i.e.`did not have….’. The subject `service’ is plural
and is followed by plural verbs only.Therefore it should have been `services of
their’because`Their’ is a plural possessive pronoun, and means “belonging to

them”; it’s the plural form of “his” or “her” The correct sentence would have been:

They were on the backfoot after the debacle in Adelaide and did not have the
servicesof their captain Virat Kohli.

Therefore, option B is the correct answer choice to this question.

3) Answer: D

According to the context of the sentence it is regarding the fact that `people have
repeatedly started hitting the streets to celebrate the New Year’. The correct
(adverb) born –`again’ to be used for that purpose is `again’ Therefore it should have
been `people have again started’.
The correct sentence would be:

Page 354 Follow us: Official Site, Telegram, Facebook, Instagram, Instamojo
The HC observed that even though the festive season has come to an end, people have
again started hitting the streets to celebrate the New Year.

Therefore, option D is the correct answer choice to this question.

4) Answer: C

Here the context is that due to `UK variant genome of Sars-CoV-2’ which has been
inherited from UK,it is expected that the figure of twenty, testing positive, will soon
cross the mark. This is going to be another bad sign for the economy which will again
entail for a vaccine to be developed with the same variant of genome detected from UK.
Both I and II will be correct in this context if placed instead of the bold phrase in the
given sentence.
The correct sentence would be:

The total number of people testing positive for the new UK variant genome of Sars-
CoV-2 virus has reached 20, the Union ministry of health and family welfare
announced on Wednesday.

Therefore, option C is the correct answer choice to this question.

5) Answer: E

There is no error in the highlighted part of the given sentence and therefore no
replacement is required here.

Therefore, option E is the correct answer choice to this question.

Error Detection / Spotting

Error Spotting - Set-1

Directions (1-5): In this question, a If the sentence is absolutely correct,


sentence has been divided into four mark (E) as your answer.
parts marked as I, II, III and IV. You
need to find which part/parts does 1)
not/do not have an error in terms of
its grammatical or contextual usage. I) After images of bleed captured

Page 355 Follow us: Official Site, Telegram, Facebook, Instagram, Instamojo
II) Indian Air Force (IAF) fighter pilot II) from his aged MiG-21

III) Wing Commander Abhinandan III) hit in the aerial dogfight


Varthaman were put out by Pakistan
IV) over the skies of Jammu
IV) on this day in 2019
A.only II and III
A.only I and III
B.only II and IV
B.only II and IV
C.I,III and IV
C.only IV
D.I, II and III
D.only III
E.No error
E.No error

4)
2)
I) he shot down a fourth-generation F-
I) MiG-21 Bison was shot down 16 fighter

II) after successfully intercepted a posse II) he was captured by villagers


of Pakistani fighters
III) in Pakistan-occupied Kashmir where
III) over the Line of Control he landed

IV) in the Rajouri-Mendhar sector in 27 IV) and handed over to the Pakistan
February military.

A.only I and IV A.only II

B.only I and III B.only II and IV

C.I, III and IV C.I, III and IV

D.only I D.only III and IV

E.No error E.No error

3) 5)

I) Before the IAF pilot ejected


Page 356 Follow us: Official Site, Telegram, Facebook, Instagram, Instamojo
I) Hindustan Times, after talked to A.only I, II and III
former air force
B.only I and III
II) intelligence and politicians has been
able to reconstruct C.only II and III

III) some of the behind-the-scenes D.only III


action in New Delhi
E.No error
IV) Islamabad that led Pakistan Prime
Minister Imran Khan to released the
Indian fighter pilot

Answers :

1) Answer: B

Fragment I: After images of bleed captured

Here, the preposition ‘of’ must follow the verb ‘bleeding’ to make it correct.

Ex. After images of bleeding captured

Fragment II is absolutely correct.

Fragment III: Abhinandan Varthaman were put out. This fragment has an error
of parallelism. Here ‘were’ must be replaced by ‘was’ to make it grammatically correct.

Fragment IV is correct and has no error in it.

Clearly, only fragments II and IV are error free.

Therefore, option B is the correct answer choice to this question.

2) Answer: B

Fragment I is absolutely error free.

Fragment II: There is an error of tense here. As the sentence talks about a past event,
the verb ‘intercepted’ has to be in its past form; ‘intercepting’.

Fragment III does not have an error.

Page 357 Follow us: Official Site, Telegram, Facebook, Instagram, Instamojo
Fragment IV: Here the phrase ‘Rajouri-Mendhar sector in 27 February’ must be
changed to ‘Rajouri-Mendhar sector on 27 February’ to make it a grammatically correct
sentence.

Clearly, fragments I and III are absolutely error free.

Therefore, option B is the correct answer choice to this question.

3) Answer: C

Fragment I is correct and has no error in it.

Fragment II: There is an error of subject-verb agreement here. The main subject of the
sentence ‘from his aged MiG-21’ is in singular number and the verb following it has to
be in agreement. ‘ageing’ must be used here in place of ‘aged’ to make it a
grammatically correct sentence.

Fragment III is error free.

Fragment IV is error free as well.

Clearly, fragments I, III and IV do not have any error.

Therefore, option C is the correct answer choice to this question.

4) Answer: E

All the fragments are correct and error-free.

Option E is hence the correct answer.

Therefore, option E is the correct answer choice to this question.

5) Answer: C

The errors are in part I and part IV.

Reason:

Page 358 Follow us: Official Site, Telegram, Facebook, Instagram, Instamojo
In part I, “after talked to former” is erroneous. The correct word in place of “talked”
should have been “talking”.

In part IV, the word “released” is erroneous. The correct word in place of “released”
should have been “release”. [“release” and “liberate” are both verbs.]

The rest of the parts are correct.

Therefore, option C is the correct answer choice to this question.

Error spotting - Set-2 ( NEW Pattern )


Error counting

Directions (1-5): In the paragraphs given below count the number of grammatical
errors and mark as your answer:

1) a) The Andaman and Nicobar Islands became first Union Territory to be free of

b) the COVID-19. The Union Territory reported no active cases of the noble
Coronavirus

c) and the last four infected people tested negative. The information was share by the

d) Union Health Ministry on February 2, 2021. On the other hand, Kerala has the

e) highest number of active COVID cases.

A) if the sentence has four or less than four errors

B) if the sentence has six errors

C) if the sentence has no error

D) if the sentence has seven errors

E) if the sentence has eight or more than eight errors

2) a) Nguyen Phu Trong, the Chief of ruling Communist Party of Vietnam, was re-
elected

b) as the leader of country for a third term. The 76 years old have become the longest-

Page 359 Follow us: Official Site, Telegram, Facebook, Instagram, Instamojo
c) serving leaders of Vietnam of decades. The develop was reported by the Vietnam

d) News Agency (VNA) at January 31, 2021.

A) if the sentence has four errors

B) if the sentence has six errors

C) if the sentence has no error

D) if the sentence has seven errors

E) if the sentence has five errors

3) The facility for instant allotment of Permanent Account Number (PAN) has
been formally launched by the Union Minister for Finance & Corporate Affairs
Nirmala Sitharaman. This facility rolled out on near to real time basis. The PAN
applicants who has a validity Aadhaar number and have a mobile number
registered with Aadhaar will be able to avail the recently launched facility. After
the launch of this new facility, the allotment process is paperless and an
electronic PAN (e-PAN) will be issued to the applicants free of cost. Launching
this facility will create further ease of compliance to the taxpayers.

A) if the sentence has four errors

B) if the sentence has six errors

C) if the sentence has two errors

D) if the sentence has seven errors

E) if the sentence has five errors

4) a) Karur Vysya Bank has launch Indias first pre-paid card Enkasu (My cash at Tamil)

b) in Karur, Tamil Nadu under its Quit cash movement. The card functions based on

c) Near Field Communication Technology (NFC) in a semi closed-loop and the

d) customers buy this card can make ‘tap & go’ payments for even small purchases to

Page 360 Follow us: Official Site, Telegram, Facebook, Instagram, Instamojo
e) merchants from an amount of Rs. 1 to doing away with the need for cash and small
change.

A) if the sentence has four errors

B) if the sentence has six errors

C) if the sentence has no error

D) if the sentence has seven errors

E) if the sentence has five errors

5) a) The Chauri Chaura centenary celebrations will be inaugurated by Prime Minister

b) Narendra Modi on February 4, 2021 true video conferencing. PM Modi had

c) release a postal stamp on the Chauri Chaura incident to mark the occasion.

A) if the sentence has four errors

B) if the sentence has six errors

C) if the sentence has three error

D) if the sentence has seven errors

E) if the sentence has five errors

Answers :

1) Answer: A
The sentence has a total of four errors.
The first error is ‘the’, here article is missing. It should be added before the word ‘first’
to make it contextually and grammatically correct.
The second error is ‘of’. It should be replaced with ‘from’ after the word ‘free’. Incorrect
preposition is used here.
The third error is ‘noble’. It should be replaced with ‘novel’. As adjectives the difference
between novel and noble is that novel is new, original, especially in an interesting way
while noble is having honourable qualities;

Page 361 Follow us: Official Site, Telegram, Facebook, Instagram, Instamojo
The fourth error is ‘share’. It should be replaced with ‘shared’. Here the sentence is in
past tense hence the verb should also be in past tense to make it grammatically correct.
Hence the correct passage:
‘The Andaman and Nicobar Islands became the first Union Territory to be free from the
COVID-19. The UT reported no active cases of the novel Coronavirus and the last four
infected people tested negative. The information was shared by the Union Health
Ministry on February 2, 2021. On the other hand, Kerala has the highest number of
active COVID cases.’
Therefore, option A is the correct answer choice to this question.

2) Answer: E
The sentence has a total of five errors.
The first error is ‘a’. It should be replaced with ‘the’ before the word ‘third’. Here the
article is incorrect.
The second error is ‘have’. It should be replaced with ‘has’. Auxiliary word ‘have’ is used
with plural form and ‘has’ is used with singular form.
The third error is ‘of’. It should be replaced with ‘for’. Incorrect preposition is used
here.
The fourth error is ‘develop’ a verb. It should be replaced with ‘development’ a noun.
The fifth error is ‘at’. It should be replaced with ‘on’. Incorrect preposition is used here.
Here the verb does not fit grammatically well in the sentence is in past tense hence the
verb should also be in past tense to make it grammatically correct.
Hence the correct passage:
‘Nguyen Phu Trong, the Chief of ruling Communist Party of Vietnam, was re-elected
as the leader of country for the third term. The 76 years old has become the longest-
serving leaders of Vietnam for decades. The development was reported by the
Vietnam News Agency (VNA) on January 31, 2021.’
Therefore, option E is the correct answer choice to this question.

3) Answer: C
The sentence has a total of two errors.
The first error is that the word ‘has been’ is missing, this should be added to make the
statement contextually and grammatically correct. Here article is incorrect.
The second error is ‘validity’. It should be replaced with ‘valid’ to make it contextually
and grammatically correct.
Hence the correct passage:
‘The facility for instant allotment of Permanent Account Number (PAN) has been
formally launched by the Union Minister for Finance & Corporate Affairs Nirmala
Sitharaman. This facility has been rolled out on near to real time basis. The PAN
applicants who has a valid Aadhaar number and have a mobile number registered with
Aadhaar will be able to avail the recently launched facility. After the launch of this new
Page 362 Follow us: Official Site, Telegram, Facebook, Instagram, Instamojo
facility, the allotment process is paperless and an electronic PAN (e-PAN) will be issued
to the applicants free of cost. Launching this facility will create further ease of
compliance to the taxpayers.’
Therefore, option C is the correct answer choice to this question.

4) Answer: E
The sentence has a total of five errors.
The first error is ‘Indias’. It should be replaced with ‘India’s’. Here article is incorrect.
The second error is ‘have’. It should be replaced with ‘has’. Auxiliary word ‘have is used
with plural form and ‘has’ is used with singular form.
The third error is ‘of’. It should be replaced with ‘for’. Incorrect preposition is used
here.
The fourth error is ‘develop’ a verb. It should be replaced with ‘development’ a noun.
The fifth error is ‘at’. It should be replaced with ‘on’. Incorrect preposition is used here.
Here the verb does not fit grammatically well in the sentence is in past tense hence the
verb should also be in past tense to make it grammatically correct.
Hence the correct passage:
‘Karur Vysya Bank has launched India’s 1st pre-paid card Enkasu (My cash in Tamil)
in Karur, Tamil Nadu under its Quit cash movement. The card functions based on
Near Field Communication Technology (NFC) in a semi closed-loop & the customers
buying this card can make ‘tap & go’ payments for even small purchases to merchants
from an amount of Rs. 1 to doing away with the need for cash & small change.’
Therefore, option E is the correct answer choice to this question.

5) Answer: C
The sentence has a total of four errors.
The first error is ‘will be’. It should be replaced with ‘was’ as the sentence is in past
tense.
The second error is ‘true’. It should be replaced with ‘through’. Here the homophone of
the word ‘true’ should be used to make it contextually and grammatically correct.
The third error is ‘release’. It should be replaced with ‘released’. The sentence is in past
tense hence the verb used here should be in the past tense
Hence the correct passage:
‘The Chauri Chaura centenary celebrations will be inaugurated by Prime Minister
Narendra Modi on February 4, 2021 through video conferencing. PM Modi will
release a postal stamp on the Chauri Chaura incident to mark the occasion.’
Therefore, option C is the correct answer choice to this question.

Error spotting - Set-3

Page 363 Follow us: Official Site, Telegram, Facebook, Instagram, Instamojo
Directions (1-10): In the sentences d) a, b, d
given below, the parts marked a), b),
c) and d) may contain errors. Choose e) no error
the part which is error-free. If the
sentence is error free, choose e) no 3) The CBI have sought his custodial
error. interrogation and (a)/ claimed that he
responded either evasive (b)/ or
1) Last time when Abu Bakr al-Baghdadi arrogantly during an apex court-ordered
appeared in a video, (a)/ the Islamic questioning(c)/ in Shillong recently. It
State (IS) was at the pinnacle of its said his arrest was being required. (d)
glory. (b)/ He was seen in the pulpit of
the Great Mosque of al-Nuri in a) a and b
Mosul, (c)/ Iraq’s second largest city,
b) only b
after the IS declared a new
“Caliphate.(d) c) only c
a) a and b d) a and d
b) only b e) no error
c) only c 4) Meera Selva, Director of the
journalism fellowship (a)/ program at
d) a and d
the Reuters Institute for the Study of
e) no error Journalism, University of Oxford,
explains (b)/ that Sri Lanka has a “long
2) Almost five years later, he reappeared history of censoring the press, by killing
in another video,(a)/ released by al- journalists,(c)/ blocking websites and
Furqan, part of the IS media cell, on using draconian laws to fine and
April 29 at a time (b)/ when the IS was imprison reporters”. (d)
practically on the run. In the 18-minute
video, he was seeing sitting cross- a) a and b
legged (c)/ on the floor of a room talking
b) only b
to a few followers, with a Kalashnikov
leaning against the wall in the c) only c
background.(d)
d) a and d
a) a and b
e) no error
b) only b
5) Mr. Mnuchin expressed hope on April
c) only c 29 (a)/ the latest talks will make enough
Page 364 Follow us: Official Site, Telegram, Facebook, Instagram, Instamojo
progress so that (b)/ U.S. officials can a) a, b, and c
recommend to President (c)/ Donald
Trump whether to making a deal with b) only b
Beijing.(d)
c) only a
a) a and b
d) a and d
b) a, b, c
e) no error
c) only c
8) She draws our attention to the fact
d) a and d that the country’s (a)/ media remains
largely divided by language and
e) no error geography. She points out that (b)/
there are no outlets that are used and
6) This is not the first time the IS trusted equally by the Sinhala-speaking
had (a)/ using the crusade imagery. In majority(c)/ in the south and west, and
the IS (b)/ literature, the Westerner the Tamil-speaking minority in the
have (c)/ always being called north and east.(d)
crusaders.(d)
a) a and b
a) a and b
b) only b
b) only b
c) only c
c) only c
d) a and d
d) a and d
e) no error
e) no error
9) U.S. Treasury Secretary Steven
7) In the five years since his Mosul Mnuchin says he(a)/ hopes for
appearance, the IS has lost (a)/ almost “substantial progress” in talks with(b)/
all territories it had captured. The Chinese officials aimed of ending a tariff
remains of the al-Nuri war over(c)/ Beijing’s technology
Mosque,(b)/ from where Baghdadi gave ambitions.(d)
the sermons, are now with the Iraqi
government. Raqqa, the de facto capital a) a and b
of the IS, is (c)/ controlled by Kurdish
militias with help from the U.S. A month b) only b
ago, the IS lost Baghouz, the last slicing
of territory it had in Syria, to Kurdish c) only c
militants. (d)
Page 365 Follow us: Official Site, Telegram, Facebook, Instagram, Instamojo
d) a , b and d a) a and b

e) no error b) only b

10) Banning social media is a easy c) only c


option,(a)/ indeed a lazy one, that leave
no (b)/ room for innovative(c)/ d) a and d
approaches on crisis management.(d)
e) no error

Answers :

1) Answer: e) Use ‘is’ instead of ‘had’ as it is a present


continuous tense.
The sentence is error-free and
grammatically correct. Use ‘westerners’ since you are referring
to crusaders.
2) Answer: d)
Use ‘been’ instead of ‘being’.
Use ‘seen’ instead of ‘seeing’ in part c of
the sentence. 7) Answer: a)

3) Answer: c) It should be– last slice, and not last


slicing. So only part d contains error.
Use ‘has’ for a single entity.
8) Answer: e)
Use ‘evasively’ instead of ‘evasive’.
The sentence is error-free and
Use only ‘was ‘ instead of ‘was being’. grammatically correct.
4) Answer: e) 9) Answer: d)
The sentence is error-free and Use ‘at’ instead of ‘of’ as the suitable
grammatically correct. preposition.

5) Answer: b) 10) Answer: c)


Use ‘to make’ instead of ‘to making’ here. Use ‘an’ instead of ‘a’.

6) Answer: b) Use ‘leaves’ for a single entity.

Page 366 Follow us: Official Site, Telegram, Facebook, Instagram, Instamojo
Use ‘in’ as the suitable preposition instead of ‘on’.

Error spotting - Set-4


Direction (1-5): In the given question, thumbs down to social distancing norms
a sentence is divided into four parts. prescribed (C)/ during the upgoing
There are errors in three parts. nationwide lockdown period. (D).
Choose the part which doesn’t have
an error. If all the four parts are A. A
correct, mark E, i.e. ‘All are correct’ as B. B
the answer. C. C
D. D
1) More than 400 people with E. All are correct
symptoms(A)/ were admit to different
hospitals(B)/ and over 1,100 shifted to 4) He said the Delhi government
government-run quarantine facility(C)/ prepared(A)/ an list of the people
in Delhi alone in Monday and Tuesday, evacuated,(B)/ along with there phone
officials said(D). numbers, (C)/and the police were
looking in it(D).
A. A
B. B A. A
C. C B. B
D. D C. C
E. All are correct D. D
E. All are correct
2) We are not see(A)/ no variation
from(B)/ what is being saw on(C)/ how 5) An US government’s emergency
this strain is behaving around the stockpile(A)/ of protective equipment
world(D). is(B)/ near exhausted because of the
extraordinary demands (C)/of a
A. A coronavirus pandemic(D).
B. B
C. C A. A
D. D B. B
E. All are correct C. C
D. D
3) Among chants of ‘Jai Sri Ram’, E. E. All are correct
hundreds of devotees assembled (A)/ in
the temples on the occasion of Ram
Navami on Thursday, (B)/ giving a

Page 367 Follow us: Official Site, Telegram, Facebook, Instagram, Instamojo
Answers

1) Answer: A

Out of the given parts, part A is error free.

In part B, ‘were admit’ is incorrect here. After ‘were’ we can use either continuous form
or participle form of the verb. As the sentence is in passive voice so we should use past
participle form of the verb. So ‘were admitted’ will be the correct here.

In part C, ‘quarantine facility’ is incorrect here. Instead we should use ‘quarantine


facilities’ as there are more than one quarantine facilities available.

In part D, ‘in Monday and Tuesday’ is incorrect here. ‘Over’ will be the correct
preposition here. Here it refers to the period of time of Monday and Tuesday.

So, the correct choice will be option A.

2) Answer: D

Out of the given parts, part D is error free.

In part A, ‘see’ is incorrect here. As the sentence is n present continuous tense ‘seeing’
will be the correct choice.

In part B, ‘no variation’ is incorrect here. We cannot use double negative in the same
sentence. ‘Any variation’ will be the correct choice.

In part C, ‘is being saw’ is incorrect here. As the sentence is in passive voice so we
should use past participle form of the verb. So ‘is being seen’ will be the correct choice.

So, the correct choice will be option D.

3) Answer: C

Out of the given parts, part C is error free.

In part A, ‘amid’ should be used in place of ‘among’. Amid is used in uncountable things.
Among is used in more than two countable things or people.

In part B, ‘the’ should not be used before temples. We should not use ‘the’ before places
for which they are being made, that is, when their primary purpose is being served. But
Page 368 Follow us: Official Site, Telegram, Facebook, Instagram, Instamojo
when they are being used for a secondary purpose, then we use ‘the’. Eg:- The temple is
beside the mosque.

In part D, ‘ongoing’ should be used. There is no such term as ‘upgoing’. Ongoing means
continuing or still in progress.

So, the correct choice will be option C.

4) Answer: A

Out of the given parts, part A is error free.

In part B, ‘an list’ is incorrect here. Article is incorrect here. We should use ‘a’ instead.

In part C, ‘there phone numbers’ is incorrect here, as ‘there’ does not make any sense.
‘Their’ should be the correct choice here.

In part D, ‘looking in it’ is incorrect here. Here incorrect preposition is used. ‘Into’ will
be the correct choice here.

So, the correct choice will be option A.

5) Answer: B

Out of the given parts, part B is error free.

In part A, ‘An US government’ is incorrect here. Before the country name we should use
‘the’. We always use ‘the’ before USA, USSR, Hague etc.

In part C, ‘near exhausted’ is incorrect here. Before the verb ‘exhausted’ we should use
the adverb, i.e ‘nearly’.

In part D, ‘a coronavirus pandemic’ is incorrect here. Before name of any pandemic we


should use ‘the’.

So, the correct choice will be option B.

Error spotting New Pattern - Set-5

Direction (1-5): From the options given below, select the option which states the
correct combination of incorrect/correct sentences.

Page 369 Follow us: Official Site, Telegram, Facebook, Instagram, Instamojo
1)

a) Today, Xi Jinping faces a similarly fraught internal situation with an economy


hammered by the COVID-19 pandemic and the long-aspired for goal of a ‘moderately
well-off society’ by the CPC’s 100th anniversary next year, looking ever more distant.

b) Meanwhile, Xi’s policies since his ascension to power in 2012 – his centralization of
power and his hard-selling of the ‘Chinese dream’ of nationalism, and of being a natural
challenger to the United States –has had the implication of painting other countries as
weaker than or less importance than China.

c) Thus, when a Vietnam or Indonesia or India decides to challenge China’s bad


behaviour, the Chinese tendency has been to view such reactions with surprise as if the
smaller country were being both unreasonable and arrogant.

d) While the incidents – whether in the South China Sea or on the LAC might be
localized and the tactics used by military commanders shaped by local factors, they are
sanctioned by the larger set of goal and objectives under Xi’s plans for ‘national
rejuvenation’.

A) both a and d are correct

B) both a and c are incorrect

C) all sentences are correct

D) both b and d are incorrect

E) both a and b are incorrect

2)

a) If essential service providers are not paid their dues immediately, then this
hypothetical scenario could soon become a reality.

b) It has not only made the world pause, reflect and rearrange priorities in life, but has
made many of us aware of our privileges and shown us a mirror to how we react to
human sufferings as a society.

Page 370 Follow us: Official Site, Telegram, Facebook, Instagram, Instamojo
c) Aksai Chin, formerly part of Ladakh before the Chinese occupation following the
1962 war, connects the Xinjiang Province of China to Tibet, which China annexed in
1950 and remains a site of political contestation.

d) It is not surprising that India’s activities near the Line of Actual Control (LAC) in the
past decade have been a source of concern for China.

A) both a and d are correct

B) both a and c are incorrect

C) all sentences are correct

D) both b and d are incorrect

E) both a and b are incorrect

3)

a) Given these escalating pressures on felid species in Asia, an in-depth understanding


of their population status and habitat use would be fundamental for designing effectual
effective conservation and management policies.

b) Rusty-spotted cat, the smallest cat in the world (average body weight 1.6 kg), is also
reported to use the arboreal habitat alongside the terrestrial domain.

c) As both species have overlapping distribution ranges, we aimed at predicting fine-


scale segregation in a habitat that enable the coexistence of these sympatric small cats.

d) Also, the population estimation would fill the existing knowledge gap of these
species using camera-trap captures as our study is the first attempt to estimate the
population density of these individually unidentifiable small cats.

A) both a and d are correct

B) both a and c are incorrect

C) all sentences are correct

D) both b and d are incorrect

E) both a and b are incorrect


Page 371 Follow us: Official Site, Telegram, Facebook, Instagram, Instamojo
4)

a) The Scottish National party’s Ian Blackford, a former member of the committee, has
accused the government of “repeatedly and intentionally failing to establish
parliament’s intelligence and security committee to escape scrutiny on important
security matters”.

b) Ministers were told earlier this year they can not longer say there have been “no
successful examples” of Russian disinformation affecting UK elections, after the
apparent hacking of an NHS dossier seized on by Labour during the last campaign.

c) Ahead of the election in December, fresh evidence emerged of attempts by a senior


Russian diplomat, Sergey Nalobin – which once described the future prime minister as
“our good friend” – to cultivate leading Tories during his five years in London .

d) The Investigatory Powers Act allows the UK to maintain one of the most stringent
scrutiny regimes in the world through the investigatory powers commissioner, the
investigatory powers tribunal and both executive and judicial oversight.

A) both a and d are correct

B) both a and c are incorrect

C) all sentences are correct

D) both b and d are incorrect

E) both a and b are incorrect

5)

a) In an extraordinary case that will have profound economic, political and social
consequences for the impoverished South American country, the Banco Central de
Venezuela (BCV) is seeking an order that would forced the BoE to release $1bn of its
gold.

b) In a submission obtained by the Observer, they claim that “the most basic
information such as whether the Maduro board has a buyer lined up for the gold has
yet to be clarify. The timeframe to resolve the operational issues remains unclear.”

Page 372 Follow us: Official Site, Telegram, Facebook, Instagram, Instamojo
c) The BCV’s representatives will argue that the UK government has not formally
recognised Guaidó as either head of state or head of the government in Venezuela

d) The high court ruling, expected before the end of July, could help determine who
controls an estimated $5bn of Venezuelan funds frozen in foreign bank accounts.

A) both a and d are correct

B) both a and c are incorrect

C) all sentences are correct

D) both b and d are incorrect

E) both a and b are incorrect

Answers :

1) Answer: D

Statement b: is incorrect as ‘less importance’ should be replaced with ‘less important’


as after more or less we need an adjective. But ‘importance’ is a noun.

Statement d: is incorrect as ‘larger set of goal’ should be replaced with ‘larger set of
goals’ as set of represents plural quantity of noun.

Hence option D is the correct answer.

2) Answer: C

All sentences are correct. So option C should be the correct option here.

3) Answer: B

Statement a: is incorrect as ‘effectual effective’ should be replaced with ‘ effective’ as


effectual and effective are synonymous. So this is a superfluous error.

Statement c: is incorrect as ‘enable’ should be replaced with ‘enables’ as that represents


‘a habitat’ which is singular in nature.

Hence option B is the correct answer.

Page 373 Follow us: Official Site, Telegram, Facebook, Instagram, Instamojo
4) Answer: A

Statement b: is incorrect as ‘can not longer say’ should be replaced with ‘can no longer
say’ as We use no before a noun phrase and not with any other phrase or clause. so the
sentence is contextually incorrect.

Statement c: is incorrect as ‘which’ should be replaced with ‘who’ as we are denoting


here a senior Russian diplomat, Sergey Nalobin .

Hence option A is the correct answer.

5) Answer: E

Statement a: is incorrect as ‘would force’ should be replaced with ‘would forced’ as


after modals we need the verb in present form.

Statement b: is incorrect as ‘yet to be clarify’ should be replaced with ‘yet to be clarified’


as the sentence is in passive voice.

Hence option E is the correct answer choice.

Error Spotting New Pattern - Set-6

Direction (1-5): Given below are three sentences I,II and III. each sentence is
divided into five parts (a), (b), (c), (d) and (e). Read each sentence to find out
whether there is any grammatical error in it. The error, if any, will be one part of
the sentence where part (e) denotes no error. You are given five options
suggesting which part of each of the sentences together can have a grammatical
error. Identify the correct option.

1)

(I). More than two dozen firefighters (a)/ and police officers in Kirkland which have
responded (b)/ to calls at the Life Care Center over the last few weeks have been
placed (c)/ under quarantine, according to the city government.(d)/

(II). A group of about 16 nursing students (a)/ and one faculty member from the Lake
Washington Institute of Technology (b)/ visited the Life Care Center late last
week, (c)/Dr Amy Morrison, president of the college, said in a statement.(d)/

Page 374 Follow us: Official Site, Telegram, Facebook, Instagram, Instamojo
(III). Health officials said on Saturday (a)/ they did not see any connection
between (b)/ the man who died but the nursing facility, (c)/ but they were still
investigating how those patients contracted the virus.(d)/

A) add

B) cbc

C) dab

D) bec

E) bbc

2)

(I). Prof Chris Whitty, the chief medical officer for England,(a)/ said the patient “had no
relevant travel”(b)/ and an investigation was under way to determine (c)/whether the
patient contracted the virus directly or indirectly from someone who had recent
returned from overseas.(d)/

(II). The case suggest the country is edging closer (a)/ to the point where containment
becomes impossible (b)/ because the virus is being spread by people who are (c)/
either unaware they are infected, or are diagnosed too late to prevent onward
transmission. (d)/

(III). Nicola Sturgeon, Scotland’s first minister,(a)/ chaired an meeting of the Scottish
government resilience committee (b)/on Sunday evening and will be taking part
in (c)/a similar meeting chaired by Boris Johnson on Monday morning.(d)/

1. A) add
2. B) cbc
3. C) dab
4. D) bec
5. E) bbc

3)

(I). The 38-year-old former mayor of South Bend has never hold (a)/ statewide or
national office but made a strong run in the Democratic primary,(b)/ winning the Iowa

Page 375 Follow us: Official Site, Telegram, Facebook, Instagram, Instamojo
caucuses narrowly from Bernie Sanders, now the national frontrunner, (c)/ to whom
he placed second in New Hampshire.(d)/

(II). Speaking to CNN on Sunday morning, Biden said he had not had (a)/ any
conversations with other candidates about whether they should drop out (b)/ and back
him but added: “I think everyone knows it’s going to be much more difficult (c)/ to win
back the Senate and keeping control of the House if Bernie’s at the top of the
ticket.” (d)/

(III). “He obviously has work to do on some things that (a)/ – some weaknesses we’ve
seen in this election (b)/ – but whenever there is a conversation again about
Democratic candidates, (c)/he’ll be in that conversation and that’s a remarkably
achievement, given where he started a year ago.”(d)/

A) add

B) cbc

C) dab

D) bec

E) bbc

4)

(I). The prime minister made similar appeals (a)/ before the previous votes on April,
he said (b)/he would annex all current settlements, and in September (c)/ he promised
to go further, by claiming sovereignty over a third of the entire West Bank.(d)/

(II). In preparation for election day,(a)/ the Israeli military closed crossings on the
occupied West Bank,(b)/ where more than 2.5 million Palestinians live under Israeli
rule(c)/ but cannot vote, and Gaza, where 2 million more live under a blockade(d)/.

(III). Political parties fear a low turnout on Monday (a)/ as voters become apathetic
about (b)/the politically stalemate and fearful of the spread of coronavirus, (c)/issues
that could drastically affect the result in such a small country.(d)/

A) add

B) cbc

Page 376 Follow us: Official Site, Telegram, Facebook, Instagram, Instamojo
C) dab

D) bec

E) bbc

5)

(I). At her weekly news conference in Wellington on Monday,(a)/ Ardern was scathing
about Dutton’s criticism of her recent meeting with Scott Morrison, (b)/saying it was
not her plain-talking that was to blame for increase tensions between the
neighbours (c)/ – but Australia’s policy decisions on immigration matters which were
hurting Kiwis.(d)/

(II). Prisoners and members of the Roma community,(a)/ along with elderly people
and a poorest in society,(b)/ will be the focus of a new EU push to help Europeans (c)/
“outside the mainstream” to remain in the UK after Brexit.(d)/

(III). EU nationals need to apply before 31 June 2021 (a)/ to remain in the UK but there
is uncertainty (b)/about the number of citizen affected and the level of success (c)/ in
information getting through to the most cut-off groups.(d)/

A) add

B) cbc

C) dab

D) bec

Answers

1) Answer: (d)

Part b is incorrect as ‘which have’ is incorrect here. It should be replaced with ‘who
have’.

Part (II) does not contain any error.

Part c is incorrect as ‘but’ is incorrect here. It should be replaced with ‘and’.

2) Answer: (c)
Page 377 Follow us: Official Site, Telegram, Facebook, Instagram, Instamojo
Part d is incorrect as ‘had recent returned’ is incorrect here. It should be replaced with
‘had recently returned’.

Part a is incorrect as ‘The case suggest’ is incorrect here. It should be replaced with
‘The case suggests’.

Part b is incorrect as ‘an meeting’ is incorrect here. It should be replaced with ‘a


meeting’.

3) Answer: (a)

Part a is incorrect as ‘has never hold’ is incorrect here. It should be replaced with ‘has
never held’.

Part d is incorrect as ‘and keeping’ is incorrect here. It should be replaced with ‘and
keep’.

Part d is incorrect as ‘remarkably’ is incorrect here. It should be replaced with


‘remarkable’.

4) Answer: (e)

Part b is incorrect as ‘on April’ is incorrect here. It should be replaced with ‘in April’.

Part b is incorrect as ‘closed crossings on the occupied West Bank’ is incorrect here. It
should be replaced with ‘closed crossings with the occupied West Bank’.

Part c is incorrect as ‘the politically stalemate’ is incorrect here. It should be replaced


with ‘the political stalemate’.

5) Answer: (b)

Part c is incorrect as ‘for increase tensions’ is incorrect here. It should be replaced with
‘for increasing tensions’.

Part b is incorrect as ‘a poorest’ is incorrect here. It should be replaced with ‘the


poorest’.

Part c is incorrect as ‘the number of citizen’ is incorrect here. It should be replaced with
‘the number of citizens’.

Page 378 Follow us: Official Site, Telegram, Facebook, Instagram, Instamojo
Error Spotting - Set-7

Direction (1-5): The following E) No correction required


question consists of a sentence which
is divided into three parts which 2) Trump is face re-election in
contain grammatical errors in one or November (a)/and would be happy to
more than one part of the sentence, see his warmth towards Modi and India
as specified in bold in each part. If translate (b)/ into valuable
there is an error in any part of the support from the Indian American
sentence, find the correct alternative community.(c)/
to replace those parts from the three
options given below each question to a) is facing re-election
make the sentence grammatically
correct. If the given sentence is b) to see his warm
grammatically correct or does not
require any correction, choose (E), i.e c) in valuable support
“No correction required” as your
answer. A) only a

1) It is true that several states B) both a and b


have enacted laws purportedly (a)/in
comply with the Supreme Court’s C) only c
orders, but these acts, (b)/as there
critical examination reveals, violate D) both b and c
the letter and spirit of the judicial
directions.(c)/ E) No correction required

a) several states has 3) EPS showed signs of having a spine


when his government (a)/made the
b) in compliance with mayoral election indirect,
checkmating the BJP that was toying
c) their critical examination reveals with (b)/the idea of getting a couple of
mayor posts for its representatives.(c)/
A) only a
a) showed signs of have
B) both a and b
b) make the mayoral election indirect
C) only c
c) a couple of mayor post
D) both b and c
A) only a

Page 379 Follow us: Official Site, Telegram, Facebook, Instagram, Instamojo
B) both a and b D) both b and c

C) only c E) No correction required

D) both b and c 5) Given the key functions that the lands


perform, in modulating
E) No correction required climate,(a)/ fostering
biodiversity and providing sustenance,
4) From just two cases on February 20, new research findings (b)/ arguing that
the numbers in Iran increased to 245 the Ghats face a serious threat from
in February 27;(a)/ there have been 26 climate change, and temperature
deaths too, a highest outside China and variation are a cause for worry.(c)/
from three countries (b)/ in the WHO
East Mediterranean region on February a) in modulate climate
21, it has increased to nine now.(c)/
b) foster biodiversity
a) in Iran increased to 245 on February
27 c) temperature variations are

b) the highest outside China A) only a

c) it have increased to B) both a and b

A) only a C) only c

B) both a and b D) both b and c

C) only c E) No correction required

Answers :

Directions (1-5) : 2) Answer: (a)

1) Answer: (d) Parta is incorrect as ‘face’ should be


replaced with ‘facing’.Option A is the
Partb is incorrect as ‘compliance’ should correct choice here.
be replaced with ‘comply’.
3) Answer: (e)
Partc is incorrect as ‘there’ should be
replaced with ‘their’. Option D is the All bold parts are correct. Option E is the
correct choice here. correct choice here.

Page 380 Follow us: Official Site, Telegram, Facebook, Instagram, Instamojo
4) Answer: (b) 5) Answer: (c)

Parta is incorrect as ‘in February’ should Partc is incorrect as ‘temperature


be replaced with ‘on February’. variation are’ should be replaced with
‘temperature variations are’. Option C is
Partb is incorrect as ‘a highest’ should the correct choice here.
be replaced with ‘the highest’. Option B
is the correct choice here.

Error Spotting - Set-8

Direction (1-5): The following A) only a


question consists of a sentence which
is divided into three parts which B) both a and b
contain grammatical errors in one or
more than one part of the sentence, C) only c
as specified in bold in each part. If
there is an error in any part of the D) both b and c
sentence, find the correct alternative
to replace those parts from the three E) No correction required
options given below each question to
make the sentence grammatically 2) Prime Minister Narendra Modi’s
correct. If the given sentence is government has accelerated these
grammatically correct or does not trends by implement(a)/a set of far-
require any correction, choose (E), i.e reaching reforms, which have led to
“No correction required” as your macro-economic stability,(b)/a robust
answer. safety net for all and strong growth.(c)/

1) In mid-February, the European a) these trends by implementing


Parliament approved (a)/a wide-
ranging free trade agreement b) a set of far-reaching reform
between the European Union and
Vietnam, (b)/paving the way of the c) a robusting safety net
deal to take effect later this year.(c)/
A) only a
a) an European Parliament approved
B) both a and b
b) free trade agreement among
C) only c
c) paving the way for
D) both b and c

Page 381 Follow us: Official Site, Telegram, Facebook, Instagram, Instamojo
E) No correction required b) to rely more on investigations

3) Even without her calling me grandpa c) drive care beyond


in Mandarin, I would have
guessed that (a)/ the little angel was A) only a
from China as her sprightly gait was
unmistakably that of the hundreds of B) both a and b
little empresses (b)/ I had seen
during my stints in the Indian C) only c
diplomatic missions in that country.(c)/
D) both b and c
a) I would had guessed
E) No correction required
b) the hundreds of little empress
5) It took both sides eight years to
c) I had saw during conclude negotiating (a)/this historic
pact, that would deepen and broaden
A) only a commercial ties between (b)/the fast-
growing Southeast Asian nation and the
B) both a and b world’s larger trading bloc.(c)/

C) only c a) eight year to conclude negotiating

D) both b and c b) which would deepen

E) No correction required c) the world’s largest

4) The evident lack from trust in A) only a


clinical medicine is driving many
clinicians (a)/to rely more in B) both a and b
investigations and imaging than on
their professional judgment, (b)/ C) only c
sounding the death knell for clinical
medicine and driving care beyond the D) both b and c
reach of most. (c)/
E) No correction required
a) evident lack of trust
Answers:

1) Answer: C

Page 382 Follow us: Official Site, Telegram, Facebook, Instagram, Instamojo
Part c is incorrect as ‘of’ should be as ‘rely more in’ should be replaced with
replaced with ‘for’. So option C will be ‘rely more on’. So option B will be the
the correct choice here. correct choice here.

2) Answer: A 5) Answer: D

Part a is incorrect as ‘implement’ should Part b is incorrect as ‘that’ should be


be replaced with ‘implementing’. So replaced with ‘which’, Part c is incorrect
option A will be the correct choice here. as ‘larger’ should be replaced with
‘largest’. So option D will be the correct
3) Answer: E choice here.

All bold parts are correct. So option E 6) Answer: C


will be the correct choice here.
From the given options only C-F-G is
4) Answer: B matched and make a meaningful
sentence. The correct answer after
Part a is incorrect as ‘from’ should be matching will be
replaced with ‘of’ and Part b is incorrect

Page 383 Follow us: Official Site, Telegram, Facebook, Instagram, Instamojo
Sentence Rearrangements/ Parajumble

Sentence Rearrangements – Set 1 (With fillers)

Directions (1-5): In the given questions the parts of the sentences are jumbled with
a blank. Read and understand the correct sequence of the parts and the word that
can fit the blank to make the sentences meaningful.

1) The Ease of ____ Index (EoLI) (A)/ in planning and regional disparities.(B)/2020 for
India’s cities reveals gaps (C)/ evolve a holistic strategy to improve(D)/ service delivery,
but also make them equitable(E)/

A.ACBD- living

B.CBAD- income

C.BDAC- upkeep

D.ACDB- means

E.No arrangement required

2) The Centre, on Thursday, (A)/ of more than a ____ (B)/ released the rankings of the
Ease of Living Index (EoLI) (C)/ 2020 for cities with a population (D)/ and those with less
than a million people(E)/

A.ACBD- jillion

B.CBAD- billion

C.BDAC- thousand

D.ACDB- million

E.No arrangement required

Page 384 Follow us: Official Site, Telegram, Facebook, Instagram, Instamojo
3) For development and provides an understanding(A)/ the quality of life and the impact
of initiatives (B)/EoLI is an assessment tool that ____ (C)/ of cities based on the quality of
life,(D)/ economic-ability and sustainability(E)/

A.ACBD- judge

B.CDAB- guess

C.BDAC- assess

D.CBAD- evaluates

E.No arrangement required

4) One, Article 21 of the Constitution (A)/ it is important to measure (B)/ guarantees


every citizen a certain standard (C)/ the performance of cities for three ____ (D)/ of life
that ensures dignity and personal growth(E)/

A.ACBD- cause

B.CBAD- ground

C.BDAC- reasons

D.ACDB- basis

E.No arrangement required

5) Economic growth is ____ intricately linked with urbanisation, (A)/but, in India, this has
not kept pace with the rate of economic growth (B)/ the rise in urban population (40% of
India’s total population is expected to live in urban areas by 2030) (C)/ has vastly
outpaced the capacity of Urban Local Bodies (ULBs) (D)/ No arrangement required (E)/

A.ACBD- tactfully

B.CBAD- sensitively

Page 385 Follow us: Official Site, Telegram, Facebook, Instagram, Instamojo
C.BDAC- carefully

D.ACDB- delicately

E.No arrangement required

Answers :

1) Answer: A

Here, if we look closely we can see that neither part B nor part C can be used as the

starting of the sentence. Also, we can understand that after part A part C will be there.
Again we can

understand by the linking keywords that after part D part E will come.

Now if we check option A we can understand that it makes proper sense. On the other
hand only

‘Living’ will fit the given blank.

Therefore option A is the correct answer choice to this question.

2) Answer: D

Here, if we look closely we can see that neither part B nor part C can be used as the

starting of the sentence. Also, we can understand that after part A part C will be there.
Again we can

understand by the linking keywords that after part D part B will come.

Now if we check option D we can understand that it makes proper sense. On the other
hand only

‘million’ will fit the given blank.

Page 386 Follow us: Official Site, Telegram, Facebook, Instagram, Instamojo
Therefore option D is the correct answer choice to this question.

3) Answer: D

Here, if we look closely we can see that neither part A nor part B can be used as the

starting of the sentence. Also, we can understand that after part A part D will be there.
Again we can

understand by the linking keywords that after part D part E will come.

Now if we check option D we can understand that it makes proper sense. On the other
hand only

‘evaluates’ will fit the given blank.

Therefore option D is the correct answer choice to this question.

4) Answer: C

Here, if we look closely we can see that neither part A nor part C can be used as the

starting of the sentence. Also, we can understand that after part A part C will be there.
Again we can

understand by the linking keywords that after part D part A will come.Now if we check
option C we can understand that it makes proper sense. On the other hand only‘reasons’
will fit the given blank.

Therefore option C is the correct answer choice to this question.

5) Answer: E

Here, if we look closely we can see that neither part A nor part B can be changed from its
position of the sentence. Also, we can understand that after part A part B will be there.
Again we can

Page 387 Follow us: Official Site, Telegram, Facebook, Instagram, Instamojo
understand by the linking keywords that after part D part E will come normally.

Now if we check any of the options with the given word arrangement we understand that
it does not make a proper sense. On the other hand the blank has been shown as proxy to
the arrangement.

Therefore option E is the correct answer choice to this question.

Sentence Rearrangements - Set-2


Directions (1-5): In the questions given The Public Investment Fund of Saudi
below, a sentence has been broken Arabia will invest (B)/ Rs 9,555 crore in
down into four fragments labeled (A), the company for an equity stake of
(B), (C) and (D) and arranged, not 2.04% (C)/ MukeshAmbani’s Reliance
necessarily in the correct order. You Retail Ventures Limited (RRVL)(D).
have to find the correct order of
arrangement from the options given A. ACBD
below. In case, the sentence is correct B. DABC
in its original form, please select (E) as
your answer. C. BACD
D. ADCB
1) The retail unit of MukeshAmbani’s RIL
empire (A)/ for Rs 37,710 crore. PIF had E. No changes required
earlier this year invested Rs 11,400 3) On Wednesday, a proposal has been
crore (B)/ has so far sold 8.22% equity approved by Prime Minister Narendra
stake to eight other investors, excluding Modi (A)/ chaired Cabinet Committee on
PIF (C)/, in the digital services arm of RIL, Economic Affairs (CCEA) to invest an (B)/
Jio Platforms for a 2.32% equity on river Satluj, located in the districts of
stake (D). Shimla and Kullu in the state of Himachal
Pradesh (C)/ amount of Rs 1,810 crore
A. ACBD
for 210 MW Luhri Stage-I hydropower
B. BDCA project (D).
C. ACBD A. ABDC
D. DACB B. BADC
E. No changes required C. ACBD
2) A subsidiary of Reliance Industries D. ADCB
Limited (RIL), today announced that (A)/
Page 388 Follow us: Official Site, Telegram, Facebook, Instagram, Instamojo
E. No changes required E. No changes required
4) More revenue streams for service 5) In other words, for certain types of
providers, (A)/ industry think tank BIF properties, (A)/ which was not possible
has said (B)/ will offer scope for sharing even a year before, (B)/ the rental
of resources, including networks, leading amount can pay for the interest part of
to its optimum utilisation and (C)/ the loan, (C)/in addition to that some
unbundling of various layers in telecom state government provided discounts on
like infrastructure, network and stamp duty and registration. (D)
services (D)
A. ACDB
A. ACDB
B. BCAD
B. BDAC
C. DCAB
C. DCAB
D. ACBD
D. CDAB
E. No changes required

Answers :

Directions (1-5) :

1) Answer: C

Let understand the sentence fragmentally

A-C

A: The retail unit of MukeshAmbani’s RIL empire

C: has so far sold 8.22% equity stake to eight other investors, excluding PIF

C-B

C: has so far sold 8.22% equity stake to eight other investors, excluding PIF

B: for Rs 37,710 crore. PIF had earlier this year invested Rs 11,400 crore

Page 389 Follow us: Official Site, Telegram, Facebook, Instagram, Instamojo
B-D

B: for Rs 37,710 crore. PIF had earlier this year invested Rs 11,400 crore

D: in the digital services arm of RIL, Jio Platforms for a 2.32% equity stake

Only logical and meaningful sentence is made from these combinations.

Therefore, option C is the correct answer.

2) Answer: B

Let understand the sentence fragmentally

D-A

D: MukeshAmbani’s Reliance Retail Ventures Limited (RRVL)

A: a subsidiary of Reliance Industries Limited (RIL), today announced that

A-B

A: a subsidiary of Reliance Industries Limited (RIL), today announced that

B: The Public Investment Fund of Saudi Arabia will invest

B-C

B: The Public Investment Fund of Saudi Arabia will invest

C: Rs 9,555 crore in the company for an equity stake of 2.04%

Only logical and meaningful sentence is made from these combinations.

Therefore, option B is the correct answer.

3) Answer: A
Page 390 Follow us: Official Site, Telegram, Facebook, Instagram, Instamojo
Let understand the sentence fragmentally

A: On Wednesday, a proposal has been approved by Prime Minister Narendra Modi

B: chaired Cabinet Committee on Economic Affairs (CCEA) to invest an

B-D

B: chaired Cabinet Committee on Economic Affairs (CCEA) to invest an

D: amount of Rs 1,810 crore for 210 MW Luhri Stage-I hydropower project

D-C

D: amount of Rs 1,810 crore for 210 MW Luhri Stage-I hydropower project

C: on river Satluj, located in the districts of Shimla and Kullu in the state of Himachal
Pradesh

Only logical and meaningful sentence is made from these combinations.

Therefore, option A is the correct answer.

4) Answer: C

Let understand the sentence fragmentally

D-C

D: Unbundling of various layers in telecom like infrastructure, network and services

C: will offer scope for sharing of resources, including networks, leading to its optimum
utilisation and

C-A

C: will offer scope for sharing of resources, including networks, leading to its optimum
utilisation and

Page 391 Follow us: Official Site, Telegram, Facebook, Instagram, Instamojo
A: more revenue streams for service providers,

A-B

A: more revenue streams for service providers,

B: industry think tank BIF has said.

Only logical and meaningful sentence is made from these combinations.

Therefore, option C is the correct answer.

5) Answer: D

Let understand the sentence fragmentally

A-C

A: In other words, for certain types of properties,

C: the rental amount can pay for the interest part of the loan,

C-B

C: the rental amount can pay for the interest part of the loan,

B: which was not possible even a year before,

B-D

B: which was not possible even a year before

D: in addition to that some state government provided discounts on stamp duty and
registration.

Only logical and meaningful sentence is made from these combinations.

Therefore, option D is the correct answer.

Page 392 Follow us: Official Site, Telegram, Facebook, Instagram, Instamojo
Sentence Rearrangements - Set-3
Directions (1-5): In the questions given below, a sentence has been broken down
into four fragments labelled (A), (B), (C) and (D) and arranged, not necessarily in the
correct order. You have to find the correct order of arrangement from the options
given below. In case, the sentence is correct in its original form, please select (E) as
your answer.

1) About why the Estonia sank and (A)/while most of the dead were women, children and
the elderly (B)/ however, there were an unpleasant number of questions (C) /why so
many survivors were men in the prime of life, (D)

A) DCBA

B) ACBD

C) CBDA

D) CADB

E) No rearrangement necessary

2) For the suppressing of vice among our young people, which gave so great
offense, (A)/and so far, as I know my own heart, it was from hence (B)/ and by which I
became so obnoxious (C)/ that I formerly led this church to some measures (D)

A) BDAC

B) BACD

C) CBAD

D) DABC

E) No rearrangement required

Page 393 Follow us: Official Site, Telegram, Facebook, Instagram, Instamojo
3) Must be counted among the instinctive functions, (A)/having read between their lines
long enough, I now say to myself that the greater part of conscious thinking(B)/and it is so
even in the case of philosophical thinking(C)/ having kept a sharp eye on philosophers
and(D)

A) CBAD

B) ACBD

C) DBAC

D) CABC

E) No rearrangement required

4) Respite from this unrelenting pandemic(A)/ with bated breath for anything can
provide some(B)/ and while there are various treatments being tested(C)/ in labs all over
the world, we are all waiting(D)

A) BCDA

B) CDBA

C) CABD

D) ACDB

E) No rearrangement required

5) They would seek over the period, (A)/ but said the National Development and Reform
Commission(B)/ would work on guidelines to be submitted to country’s parliament in
March(C)/ Officials didn’t specify the pace of growth(D)

A) BDCA

B) ABDC

Page 394 Follow us: Official Site, Telegram, Facebook, Instagram, Instamojo
C) ABDB

D) DABC

E) No rearrangement required

Answers
1) Answer: D.

On looking at the fragments, we only find C as the suitable opening fragment. Now, we
know that the sentence will describe the unpleasant questions mentioned in the first
fragment. Fragment A starts with ‘about why…’, so logically A will follow as the second
fragment of the sentence. By this time, we can connect the sentence fragments and find the
placement of he other two. The third fragment should be D as it also describes the
unpleasant question and the concluding fragment is B which is an extension of it. The
correct sequence is CABD. The sentence after rearrangement will be: “However, there
were an unpleasant number of questions about why the Estonia sank and why so many
survivors were men in the prime of life, while most of the dead were women, children and
the elderly.” This makes option D the correct answer.

2) Answer: A.

The first fragment is a little difficult to find out as it starts with a connector. Option B is the
suitable fragment for opening the statement. D can be tempting but since the statement
contains which, this means that there was a fragment before explaining something. The
second fragment is easy to decipher because the last word of B fragment is hence and it
would only take D as the following fragment and no other. Now fragment B explains some
measures which are to be explained in the next fragment. This makes A and C follow the
second fragment. The correct sequence of the fragments will be BDAC. The sentence after
rearrangement will be: “And so far, as I know my own heart, it was from hence that I
formerly led this church to some measures, for the suppressing of vice among our young
people, which gave so great offense, and by which I became so obnoxious.”. This makes
option A the correct answer.

Page 395 Follow us: Official Site, Telegram, Facebook, Instagram, Instamojo
3) Answer: C

Upon looking as the fragments B and D may qualify as the opening statements. But, if we
put them together, we can understand properly about their placements. The opening
statement will be D followed by B as the second fragment. From, this we can easily find the
placement of the A and C. C is starting with an ‘and’, so logically it must follow A. The
correct sequence is DBAC. The sentence after rearrangement will be: “Having kept a sharp
eye on philosophers and having read between their lines long enough, I now say to myself
that the greater part of conscious thinking must be counted among the instinctive
functions, and it is so even in the case of philosophical thinking.” This makes option C the
correct answer.

4) Answer: B

From the sentence fragments, only C qualifies as the opening fragment. C gives the hint for
the second fragment. C hints of something being tested and D starts with ‘in labs…’. This
immediately gives the green light for D to be the second fragment of the sentence.
Fragment D also gives the hint for next fragment. It says that we are waiting and B starts
with ‘with bated breath…’. These connect together properly and the third fragment
becomes B followed by A. The correct sequence is CDBA. The sentence after
rearrangement will be: “And while there are various treatments being tested in labs all
over the world, we are all waiting with bated breath for anything can provide some respite
from this unrelenting pandemic.” This makes option B the correct answer.

5) Answer: D

Only D qualifies as the opening fragment. Fragment D says that officials did not specify the
pace of growth. This immediately hints us of the second fragment A which mentions the
phrase ‘over the period’ because ‘pace of growth’ can be related to the phrase ‘over the
period’ easily. Now B and C are fragment left. If we observe carefully, B will come first
before C as the third fragment as C is its extension of something said. The correct sequence
is DABC. The sentence after rearrangement will be: “Officials didn’t specify the pace of
growth they would seek over the period, but said the National Development and Reform
Commission would work on guidelines to be submitted to country’s parliament in March.”
This makes option D the correct answer.
Page 396 Follow us: Official Site, Telegram, Facebook, Instagram, Instamojo
Sentence Rearrangements - Set-4 (Jumbled Sentences)

Direction (1-5): Given below are six statements A, B, C, D, E and F, which when
arranged in the correct order, form a coherent and meaningful paragraph. The
sentence marked as D is fixed and would fit in the fourth position. Rearrange the
other statements in a proper sequence to form a meaningful paragraph, then
answer the questions below.

A) Turkey dispatched gunboats last year to prevent the Italian oil company ENI from
participating in a search for underwater gas deposits commissioned by the Greek Cypriot
government.

B) Ankara has recently upped the ante by disputing Greece’s maritime borders and the
continental shelves of its islands and opposing moves by Cyprus to drill for offshore
natural gas reserves.

C) The tensions come against a backdrop of deepening hostility between the two
neighbours and Nato allies. In echoes of confrontations that have pushed them to the
brink of war five times since 1967, Greece and Turkey have sparred increasingly over
offshore energy exploration in the eastern Mediterranean as well as over refugees.

D) Greek government officials say Erdoğan’s deployment of elite forces to Evros further
escalated what has become an extremely worrying situation.

E) In meetings with the German and Austrian chancellors this week, the Greek prime
minister, Kyriakos Mitsotakis, was praised for responding so swiftly to the crisis and
deploying extra forces to defend the bloc’s external borders.

F) The unprecedented sight of Turkish fighter jets flying over the region late Wednesday
prompted speculation that the strongman leader was actively spoiling for a fight with
Athens.

1) Which of the following will C) F


be LASTsentence after rearrangement?
D) E
A) D
E) None of these
B) A

Page 397 Follow us: Official Site, Telegram, Facebook, Instagram, Instamojo
2) Which of the following pairs form two 4) Which of the following will
consecutive statements after be SECONDsentence after
rearrangement? rearrangement?

A) A-D A) D

B) B-C B) C

C) A-E C) B

D) C-D D) A

E) None of these E) None of these

3) Which of the following will 5) Which of the following will


be FIRSTsentence after rearrangement? be FIFTHsentence after rearrangement?

A) D A) D

B) C B) E

C) B C) F

D) A D) A

E) None of these E) None of these

Answers
1) Answer: (d) 2) Answer: (a) 3) Answer: (b) 4) Answer: (c) 5) Answer: (c)

Sentence Rearrangements - Set-5


Direction (1-5): Given below are six statements A, B, C, D, E and F, which when
arranged in the correct order, form a coherent and meaningful paragraph. The
sentence marked as D is fixed and would fit in the fourth position. Rearrange the
other statements in a proper sequence to form a meaningful paragraph, then
answer the questions below.
Page 398 Follow us: Official Site, Telegram, Facebook, Instagram, Instamojo
A) Turkey dispatched gunboats last year to prevent the Italian oil company ENI from
participating in a search for underwater gas deposits commissioned by the Greek Cypriot
government.

B) Ankara has recently upped the ante by disputing Greece’s maritime borders and the
continental shelves of its islands and opposing moves by Cyprus to drill for offshore
natural gas reserves.

C) The tensions come against a backdrop of deepening hostility between the two
neighbours and Nato allies. In echoes of confrontations that have pushed them to the
brink of war five times since 1967, Greece and Turkey have sparred increasingly over
offshore energy exploration in the eastern Mediterranean as well as over refugees.

D) Greek government officials say Erdoğan’s deployment of elite forces to Evros further
escalated what has become an extremely worrying situation.

E) In meetings with the German and Austrian chancellors this week, the Greek prime
minister, Kyriakos Mitsotakis, was praised for responding so swiftly to the crisis and
deploying extra forces to defend the bloc’s external borders.

F) The unprecedented sight of Turkish fighter jets flying over the region late Wednesday
prompted speculation that the strongman leader was actively spoiling for a fight with
Athens.

1) Which of the following will be LAST 2) Which of the following pairs form two
sentence after rearrangement? consecutive statements after
rearrangement?
A) D
A) A-D
B) A
B) B-C
C) F
C) A-E
D) E
D) C-D
E) None of these
E) None of these

Page 399 Follow us: Official Site, Telegram, Facebook, Instagram, Instamojo
3) Which of the following will be FIRST C) B
sentence after rearrangement?
D) A
A) D
E) None of these
B) C
5) Which of the following will be FIFTH
C) B sentence after rearrangement?

D) A A) D

E) None of these B) E

4) Which of the following will be SECOND C) F


sentence after rearrangement?
D) A
A) D
E) None of these
B) C

Answers

The correct sequence of the paragraph should be CBADFE

C) The tensions come against a backdrop of deepening hostility between the two
neighbours and Nato allies. In echoes of confrontations that have pushed them to the
brink of war five times since 1967, Greece and Turkey have sparred increasingly over
offshore energy exploration in the eastern Mediterranean as well as over refugees.

B) Ankara has recently upped the ante by disputing Greece’s maritime borders and the
continental shelves of its islands and opposing moves by Cyprus to drill for offshore
natural gas reserves.

A) Turkey dispatched gunboats last year to prevent the Italian oil company ENI from
participating in a search for underwater gas deposits commissioned by the Greek Cypriot
government.

Page 400 Follow us: Official Site, Telegram, Facebook, Instagram, Instamojo
D) Greek government officials say Erdoğan’s deployment of elite forces to Evros further
escalated what has become an extremely worrying situation.

F) The unprecedented sight of Turkish fighter jets flying over the region late Wednesday
prompted speculation that the strongman leader was actively spoiling for a fight with
Athens.

E) In meetings with the German and Austrian chancellors this week, the Greek prime
minister, Kyriakos Mitsotakis, was praised for responding so swiftly to the crisis and
deploying extra forces to defend the bloc’s external borders.

1) Answer: (d) 2) Answer: (a) 3) Answer: (b) 4) Answer: (c) 5) Answer: (c)

Sentence Rearrangements - Set-6


Directions (1-5):In the questions given below a sentence is divided into five parts
are jumbled and one of the part may have a misspelt word or grammatically
incorrect word or group of words. Arrange the divided parts into
a coherent sentence and identify the part which has an error in it.

1) More than seven hours after the 2) Emerged out of a talk with the police,
protest against the (A)/Temple was BJP leader (A)/ C.G. Rajagopal said that
called off, the protesters resume (B)/ the activists were not willing (B)/ to take
women activists’ propose visit to the return flight ticket despite the police
Sabarimala Ayyappa (C)/it in front of the attempt (C)/ to send them back by a flight
city Police Commissionerate around 6.20 to the Kochi airport shortly after
p.m.(D) midnight (D)

A) CADB, (A) and (B) A) ABCD, (A) and (D)

B) ACBD,(B) and (C) B) BDCA, (D)and (B)

C) DBAC, (C) and (D) C) ACDB, (B) and (C)

D) CBDA, (A) and (C) D) BADC, No error

E) None of these E) None of these

Page 401 Follow us: Official Site, Telegram, Facebook, Instagram, Instamojo
3) The Bench, also comprise Justices A) CBDA, No error
Ashok Bhushan and Sanjiv Khanna, (A)/
said there were need to order a floor test B) ADCB, No error
as an interim measure (B)/ also because
“oath has not been administered to the C) CADB, No error
elected members (C)/even though a
month has elapsed since the declaration D) BACD, No error
of election results”(D)
E) None of these
A) BDCA, (A) and (C)
5) asked the Kerala government to come
B) CADB, (A) and (C) clean on what he allege was
a (A)/conspirasy behind the sudden visit
C) ABCD, (A) and (B) of activists led (B)/ Bhumata Brigade
activist Trupti Desai for Sabarimala
D) CBDA, (A) and (B) pilgrimage (C)/R.V. Babu, State general
secretary of the Hindu AikyaVedi, has (D)
E) None of these
A) DCAB, (A) and (C)
4) Objective of offering ‘darshan’ at the
hilltop Ayyappa Temple at B) DABC, (A) and (B)
Sabarimala (A)/by Pune-based Bhumata
Brigade leader Trupti Desai (B)/High C) BCDA, No error
drama unfolded in the city when six
women activists led (C)/descended here D) CBAD, (B) and (D)
early on Tuesday with the (D)
E) None of these

Answers:

1) Answer: (b)

More than seven hours after the protest against the(A)/ women activists’ proposed visit to
Sabarimala Ayyappa (C)/Temple was called off, the protesters resumed (B)/it in front of
the city Police Commissionerate around 6.20 p.m.(D)

Part (B) has an error, “resumed” in the place if “resume”.


Page 402 Follow us: Official Site, Telegram, Facebook, Instagram, Instamojo
Part (C)has an error, “proposed” in the place of “propose”.

2) Answer: (a)

Emerging out of a talk with the police, BJP leader(A)/ C.G. Rajagopal said that the activists
were not willing (B)/ to take the return flight ticket despite the police attempt(C)/ to send
them back by a flight from the Kochi airport shortly after midnight. (D)

Part (A) has an error, “emerging” in the place of “emerged”.

Part (D) has an error, “from” in the place if “to”.

3) Answer: (c)

The Bench, also comprising Justices Ashok Bhushan and Sanjiv Khanna,(A)/ said there
was need to order a floor test as an interim measure(B)/ also because “oath has not been
administered to the elected members(C)/ even though a month has elapsed since the
declaration of election results”.(D)

Part (A) has an error, “comprised” in the place of “comprise”.

Part (B) has an error, “was” in the place of “were”.

4) Answer: (a)

High drama unfolded in the city when six women activists led(C)/ by Pune-based Bhumata
Brigade leader Trupti Desai(B)/ descended here early on Tuesday with the (D)/ objective
of offering ‘darshan’ at the hilltop Ayyappa Temple at Sabarimala.(A)

No error.

5) Answer: (b)

R.V. Babu, State general secretary of the Hindu AikyaVedi, has (D)/asked the Kerala
government to come clean on what he alleged was a(A)/ conspiracy behind the sudden
visit of activists led(B)/ by Bhumata Brigade activist Trupti Desai for Sabarimala
pilgrimage.(C)

Page 403 Follow us: Official Site, Telegram, Facebook, Instagram, Instamojo
Part (A) has an error, “alleged”in the place of “allege”.

Part (B) has an error, “conspiracy”” in the place of “conspirasy”.

Sentence Rearrangements - Set-7


Directions (1-5):In the questions given below a sentence is divided into five parts
are jumbled and one of the part may have a misspelt word or grammatically
incorrect word or group of words. Arrange the divided parts into a coherent
sentence and identify the part which has an error in it.

1) which will bring down the air pollution B) BCDA, No error


levels to the very poor
category (A)/November 15, as the MeT C) CADB, (A) and (D)
department had said strong winds are
expect from Friday, (B)/ On Wednesday, D) DABC, (A) and (C)
the EPCA had extended the ristrictions on
dirty-fuel based industries (C)/in Delhi- E) None of these
NCR, hot mix plants and stone crushers
till the morning of (D) 3) Jihad commander, with 34 Palestinians
kill in exchanges of fire (A)/ in Gaza take
A) BCDA, (A) and (C) hold on Thursday after two days of
fighting (B)/ A ceasefire between Israel
B) CDBA, (B) and (C) and Palestinian militants (C)/triggered by
an Israeli strike on an Islamic (D)
C) ACDB, (B) and (D)
A) ABCD, (B) and (C)
D) ACBD, No error
B) BCDA, (C) and (D)
E) None of these
C) BDCA, (A) and (D)
2) The attackers stormed the home of
Lasantha (A)/ Development and D) CBDA, (A) and (B)
Corruption last week and
gave (B)/Wijeratne, who released his E) None of these
book Wasteful (C) /a copy to ruling party
candidate SajithPremadasa (D) 4) Five rockets was fired at Israel from
Gaza after the ceasefire (A)/ of them, the
A) ACBD,No error army said, but the incident did not
Page 404 Follow us: Official Site, Telegram, Facebook, Instagram, Instamojo
appear (B)/ came into effect and air The unrest in Hong Kong has “seriously
defences intersepted two (C)/likely to set challenged” (B)/ the “one country, two
off another severe round of fighting (D) systems” principle governing (C)/ the
semi-autonomous city, China’s President
A) CDBA, (A) and (C) Xi Jinping (D)

B) BCDA, No error A) ACBD, No error

C) ACBD, (A) and (D) B) BCDA, No error

D) ABCD, (C) and (D) C) DCBA, (A)

E) None of these D) CBDA, (D)

5) said on Thursday in comments E) None of these


reported by the Chinese state media (A)/
Answers

1) Answer: (b)

On Wednesday, the EPCA had extended the restrictions on dirty-fuel based


industries (C)/ in Delhi-NCR, hot mix plants and stone crushers till the morning of (D) /
November 15, as the MeT department had said strong winds are expected from
Friday, (B)/ which will bring down the air pollution levels to the very poor category.(A)

Part (B) has an error, use “expected”in the place of “expect”.

Part (C) has an error, use “restrictions” in the place of “ristrictions”.

2) Answer: (a)

The attackers stormed the home of Lasantha (A)/Wijeratne, who released his
book Wasteful (C)/ Development and Corruption last week and gave (B)/a copy to ruling
party candidate SajithPremadasa. (D)

No error.

3) Answer: (d)

Page 405 Follow us: Official Site, Telegram, Facebook, Instagram, Instamojo
A ceasefire between Israel and Palestinian militants (C)/ in Gaza took hold on Thursday
after two days of fighting (B)/ triggered by an Israeli strike on an Islamic (D)/ Jihad
commander, with 34 Palestinians killed in exchanges of fire. (A)

Part (A) has an error, use “killed” in the place of “kill”.

Part (B) has an error, use “took” in the place of “take”.

4) Answer: (c)

Five rockets were fired at Israel from Gaza after the ceasefire (A)/ came into effect and air
defences intercepted two (C)/ of them, the army said, but the incident did not
appear (B)/ likely to set off another severe round of fighting. (D)

Part (A) has an error, use “were” in the place of “was”.

Part (D) has an error, use “intercepted” in the place of “intersepted”.

5) Answer: (b)

The unrest in Hong Kong has “seriously challenged” (B)/the “one country, two systems”
principle governing (C)/ the semi-autonomous city, China’s President Xi Jinping (D)/ said
on Thursday in comments reported by the Chinese state media. (A)

No error.

Sentence Rearrangements - Set-8


Direction (1-5): Given below are six statements A, B, C, D, E and F, which when
arranged in the correct order, form a coherent and meaningful paragraph. The
sentence marked as D is fixed and would fit in the fourth position. Rearrange the
other statements in a proper sequence to form a meaningful paragraph, and then
answer the questions below.

A) The course was founded by the speculative-fiction writers Nisi Shawl, who is black, and
Cynthia Ward, who is white, nearly twenty years ago.

B) “My immediate thought was, ‘well that’s taking the easy way out!’” recalled Shawl.
While imagining the lives of people who are different from you is virtually a prerequisite
Page 406 Follow us: Official Site, Telegram, Facebook, Instagram, Instamojo
of most successful fiction writing, the consequences of doing it poorly have grown more
serious since the pre-Twitter, pre-woke ’90s, as the conversation about who gets to tell
whose stories has moved from the fringes of publishing into the mainstream.

C) She’d tried to write from one of the kid’s perspectives, but Saigal, who is Indian-
American, wasn’t sure that she had the skill or knowledge to write from the point of view
of a black child. She decided to sign up for an online creative writing course called
“Writing the Other.”

D) They’d met a decade or so earlier, at a fantasy and science-fiction workshop, and were
inspired to design their own writing class after a conversation with another classmate, a
white friend who’d declared that she’d never write a character who didn’t share her
background or identity because she’d be sure to get it wrong.

E) A few years ago, a writer named AshimaSaigal from Grand Rapids, Michigan, witnessed
an incident on a bus in which a group of black kids were mistreated by the police. She was
disturbed, and soon after, she wrote about it. Later, reading over what she’d written, she
realized the story wasn’t working.

F)K. Rowling, Lionel Shriver, and Kathryn Stockett have all caught heat for botching the
job. In the young-adult fiction world, a number of books have been pulled in advance of
their releases for clichéd and problematic portrayals of minorities. The conversation is
often depicted in the media as a binary: On one side are those who argue that only writers
from marginalized backgrounds should tell stories about people who share their cultural
histories — a course correction for an industry that is overwhelmingly white — while on
the other are those who say this wish amounts to censorship.

1) Which of the following will be LAST E) None of these


sentence after rearrangement?
2) Which of the following pairs form two
A) D consecutive statements after
rearrangement?
B) A
A) A-D
C) F
B) B-C
D) E
C) A-E

Page 407 Follow us: Official Site, Telegram, Facebook, Instagram, Instamojo
D) C-D A) D

E) None of these B) C

3) Which of the following will be FIRST C) B


sentence after rearrangement?
D) A
A) D
E) None of these
B) C
5) Which of the following will be FIFTH
C) B sentence after rearrangement?

D) A A) D

E) None of these B) E

4) Which of the following will C) F


be SECONDsentence after
rearrangement? D) B

E) None of these
Answers

The correct sequence of the paragraph should be ECADBF

E) A few years ago, a writer named AshimaSaigal from Grand Rapids, Michigan, witnessed
an incident on a bus in which a group of black kids were mistreated by the police. She was
disturbed, and soon after, she wrote about it. Later, reading over what she’d written, she
realized the story wasn’t working.

C) She’d tried to write from one of the kid’s perspectives, but Saigal, who is Indian-
American, wasn’t sure that she had the skill or knowledge to write from the point of view
of a black child. She decided to sign up for an online creative writing course called
“Writing the Other.”

A) The course was founded by the speculative-fiction writers Nisi Shawl, who is black, and
Cynthia Ward, who is white, nearly twenty years ago.

Page 408 Follow us: Official Site, Telegram, Facebook, Instagram, Instamojo
D) They’d met a decade or so earlier, at a fantasy and science-fiction workshop, and were
inspired to design their own writing class after a conversation with another classmate, a
white friend who’d declared that she’d never write a character who didn’t share her
background or identity because she’d be sure to get it wrong.

B) “My immediate thought was, ‘well that’s taking the easy way out!’” recalled Shawl.
While imagining the lives of people who are different from you is virtually a prerequisite
of most successful fiction writing, the consequences of doing it poorly have grown more
serious since the pre-Twitter, pre-woke ’90s, as the conversation about who gets to tell
whose stories has moved from the fringes of publishing into the mainstream.

F) K. Rowling, Lionel Shriver, and Kathryn Stockett have all caught heat for botching the
job. In the young-adult fiction world, a number of books have been pulled in advance of
their releases for clichéd and problematic portrayals of minorities. The conversation is
often depicted in the media as a binary: On one side are those who argue that only writers
from marginalized backgrounds should tell stories about people who share their cultural
histories — a course correction for an industry that is overwhelmingly white — while on
the other are those who say this wish amounts to censorship.

Answers

1) Answer: C 2) Answer: A 3) Answer: E 4) Answer: B 5) Answer: D

Words Replacement
Words Replacement Set 1
Directions (1-5): In the following passage, some of the words have been highlighted
in bold. First read the passage and try to understand what it is about. Then replace
the highlighted words with the correct option in order to make grammatical and
contextual sense. In case the highlighted word is correct as it is, mark ‘No
Improvement’ as the right answer.

Australian Prime Minister Scott Morrison has reached out to his


Indian (A) complement Narendra Modi for support in his fight against Facebook over
a (B) proposed law that aims to make the social media giant pay for sharing content
from (C) medium organisations. Morrison said in a tweet on Friday that he and his “good

Page 409 Follow us: Official Site, Telegram, Facebook, Instagram, Instamojo
friend” Modi had “discussed (D) advance of our [Australia’s] media platform bill” but
didn’t give details. He said India and Australia, as comprehensive (E) strategic partners,
can work together on common challenges such as Covid-19, the circular economy and an
open, secure and prosperous Indo-Pacific.

1) A C.television

A.addition D.journal

B.counterpart E.no improvement

C.supplement
4) D
D.companion
A.development
E.no improvement
B.headway

2) B C.progress

A.suggested D.move

B.advance E.no improvement

C.offered
5) E
D.presented
A.tactical
E.no improvement
B.political

3) C C.judicious

A.radio D.decisive

B.media E.no improvement

Answers :

1) Answer: B

Page 410 Follow us: Official Site, Telegram, Facebook, Instagram, Instamojo
According to the given context, we are talking about Australian Prime Minister Scott
Morrison talking to Narendra Modi for support in his fight against Facebook. This new
concern will take into account various existing laws and one law will be there for
controlling the Facebook issue in the whole country. The word complement used in this
context is not correct since we are saying that Narendra Modi for support in his fight
against Facebook implies that to make the social media giant pay for sharing content and it
is not going in the context. Among the given words, counterpart will be the correct word
for the given context. Other words can be eliminated since they do not go with the given
context.

Therefore, option B is the correct answer choice to this question.

2) Answer: E

As per the given context, we can understand that the new law that aims to make the social
media giant pay for sharing content take into account the grievances of the people and
media organisation familiar with developments and will also redress such grievances.
According to the Australian government, the new system has understood the point that
the general mass is the most important beings in the whole system and that is why
because of concerns of the people Australian Prime Minister Scott Morrison has reached
out to his Indian counterpart . Now the given word proposed implies that put forward a
plan or suggestion and this is the perfect fit for the given context. Hence, no improvement
is required to be carried out in this case.

Therefore, option E is the correct answer choice to this question.

3) Answer: B

In the given context, we are talking about Morrison he and his “good friend” Modi had
“discussed [Australia’s] media platform bill” introduced in the Parliament, it is said that
the bill was introduced last time also but since the Parliament was dissolved, the bill also
could not be taken up as a continuation. Ahead of Australia’s Senate debating the
proposed law on Monday, Facebook chief Mark Zuckerberg held talks with treasurer Josh
Frydenberg on Friday and the negotiations were set to continue over the weekend. Among
the given words, we have media and that will be the correct pick once the context is
understood.

Page 411 Follow us: Official Site, Telegram, Facebook, Instagram, Instamojo
Therefore, option B is the correct answer choice to this question.

4) Answer: C

According to the given context, we are saying that his “good friend” Modi had
“discussed progress of our [Australia’s] media platform bill etc. for sharing content
from media organisations in the country. Also the grievances will be addressed in this new
legislation with sincerity so that they do not remain there for long. Now among the given
word we can say that progress will be the correct word here since all the laws regarding
Facebook will come together in order to put forward support in his fight issue. Other
words do not fit in the context and that is why they can be eliminated.

Therefore, option C is the correct answer choice to this question.

5) Answer: E

From the given context, it is said that there were considerable discussions within the
government to solve the common challenges and their issues, such as to make Facebook
pay for news content and is seeking allies in the campaign against tech giants etc. which
was possible to be decided by the Australian Prime Minister Scott Morrison. Therefore it is
clear that the power will be provided to these authorities only and strategic is the correct
word for this context. So no improvement is required here.

Therefore, option E is the correct answer choice to this question.

Words Replacement Set 2


Directions: (1-5) In the following passage, some of the words have been highlighted
in bold. First read the passage and try to understand what it is about. Then replace
the highlighted words with the correct option in order to make grammatical and
contextual sense. In case the highlighted word is correct as it is, mark ‘No
Improvement’ as the right answer.

Sri Lanka has cancelled a planned address to its Parliament by Pakistan Prime Minister
Imran Khan next week, (A) obviously because of concerns that he could rake up the
Kashmir issue. Officials cited Covid-19-related (B) restrictions as the reason for the
move, though people familiar with developments in Colombo said on condition

Page 412 Follow us: Official Site, Telegram, Facebook, Instagram, Instamojo
of (C) obscurity that the Sri Lankan government had done a re-think following
discussions on the (D) insinuation of Khan raising the Kashmir issue in his speech. The
planned address to Parliament doesn’t appear to have been well thought through. There
were considerable discussions within the government at which the (E) possibility of
Prime Minister Khan referring to the Kashmir issue came up,” said one of the people cited
above.
1) A C. objectivity

A. clearly D. knowledge

B. apparently E. no improvement

C. evidently
4) D
D. seemingly
A. insinuation
E. no improvement
B. suggestion
2) B C. implications

A. reduction D. inference

B. limitation E. no improvement

C. diminution
5) E
D. cutback
A. chance
E. no improvement
B. likelihood
3) C C. prospect

A. namelessness D. risk

B. anonymity E. no improvement

Answers :

1) Answer: B

According to the given context, we are talking about Sri Lanka has cancelled a planned
address to its Parliament by Pakistan Prime Minister Imran Khan. This new concern will
Page 413 Follow us: Official Site, Telegram, Facebook, Instagram, Instamojo
take into account various existing laws and one law will be there for controlling the
Kashmir issue in the country. The word used in this context is not correct since we are
saying that the new concern will rake up the Kashmir issue. Obviously implies that in a
way that is easily perceived and it is not going in the context. Among the given
words, apparently will be the correct word for the given context. Other words can be
eliminated since they do not go with the given context.

Therefore, option B is the correct answer choice to this question.

2) Answer: E

As per the given context, we can understand that the new concern will take into account
the grievances of the people familiar with developments and will also redress such
grievances. According to the Pakistan government, the new system has understood the
point that the general mass is the most important beings in the whole system and that is
why because of concerns of the people he will be able to rake up Kashmir. Now the given
word restrictions implies that a limiting condition and this is the perfect fit for the given
context. Hence, no improvement is required to be carried out in this case.

Therefore, option E is the correct answer choice to this question.

3) Answer: B

In the given context, we are talking about the history of the present Sri Lankan
government and its rethinking on discussions about Kashmir introduced in the
Parliament, it is said that the bill was introduced last time also but since the Parliament
was dissolved, the bill also could not be taken up as a continuation. As a result of that we
can say that the bill must have lapsed at the end of the period of the last session. Among
the given words, we have anonymity and that will be the correct pick once the context is
understood.

Therefore, option B is the correct answer choice to this question.

4) Answer: C

Page 414 Follow us: Official Site, Telegram, Facebook, Instagram, Instamojo
According to the given context, we are saying that Khan raising the Kashmir issue in his
speech where certain laws will be put together in order to make the new law and it will
take care of a lot of thing such as uniform pay structure, a separate land and identity etc.
for Kashmiris in the country. Also the grievances will be addressed in this new legislation
with sincerity so that they do not remain there for long. Now among the given word we
can say that implications will be the correct word here since all the laws regarding
Kashmiris will come together in order to put forward Kashmir issue. Other words do not
fit in the context and that is why they can be eliminated.

Therefore, option C is the correct answer choice to this question.

5) Answer: E

From the given context, it is said that there were considerable discussions within the
government to solve the problems with the Kashmiris and their issues, such as to fix the
minimum wages, statehood, equality etc. which was possible to be decided by the central
government and the state governments. Therefore it is clear that the power will be
provided to these authorities only and possibility is the correct word for this context. So no
improvement is required here.

Therefore, option E is the correct answer choice to this question.

Words Replacement Set 3


Directions (1-5): Which of the following words/ phrases should replace words/
phrases given in bold in the following sentences to make it meaningfully and
grammatically correct. If the bold words do not require any replacement choose as
E as your answer.

1) Early glitch of countrywide preventive measures may have also played an important
role. While European countries surge with restrictions and closures, it remains a question
if these measures were taken too late. India closed its doors to foreign travellers early on
in the outbreak and has not seen the downswing that could potentially have been
expected for a population of 1.3 billion people.

A) endeavour, extinction, languishing

B) perplex, gather, thriving

Page 415 Follow us: Official Site, Telegram, Facebook, Instagram, Instamojo
C) ineptness, tenets, vehemence

D) adoption, reacted, surge

E) No correction required

2) Many exigencies have tested the foundations of our federal democracy, but none as
harshly as this pandemic. And when India’s success in defeating COVID-19 actively rests
upon Centre-State collaboration, it is indeed its commitment to federalism that is under
the most strain. Federalism, K.C. Wheare notes, traditionally signifies the independence of
the Union and State governments of a country, in their own spheres. However, there was
nothing traditional about the circumstances in which India’s Constituent Assembly met.

A) zeal, forthcoming, paraphernalia

B) venture, fora, bloomy

C) passivity, streamline, upturn

D) flourish, novel, proficiency

E) No correction required

3) The pandemic brought to the fore at once the limits and antithesis of globalisation.
Countries such as the U.S. that concerted on others for the supply of essential medicines
and medical equipment were suddenly vulnerable. China’s unmatched sole in global
supply chains and concerns that it may weaponise trade have prompted a renewed global
discussion on the components of national security and how to protect them.

A) inevitability, relied, leverage

B) facilitate, unconventional, prowess

C) consolidate, prompt, dexterity

D) fervour, apathy, impedimenta

4) From an overall perspective, the first phased of announcements made by Finance


Minister Nirmala Sitharaman under the Atmanirbhar Bharat Abhiyan on Wednesday is
impressive indeed. There are, and will be, many issues in the details but taken as a whole,
Page 416 Follow us: Official Site, Telegram, Facebook, Instagram, Instamojo
the measures announced will go a long way in lifting the concomitant of the two key and
troubled sectors of MSMEs and non-banking finance companies. While for the former it is
an existential crisis, for the latter it is one of liquidity. The massive ₹3-lakh crore
collateral-free assistance handed out to MSMEs will help them shrewed their operations.

A) appurtenance, bandwagon, meagre

B) immediate, resurgence, dogma

C) tranche, spirits, crank up

D) skepticism, viable, imminent

E) No correction required

5) But the post-lockdown world will be different for several reasons. First, millions of
migrant workers have ascertain immense hardships to trudge back home, and are
unlikely to return to towns in the segregate future. Employment has to be provided to
them where they are, for which the Mahatma Gandhi National Rural Employment
Guarantee Scheme (MGNREGS) must be expanded greatly and curtail with wage arrears
paid immediately.

A) craze, vogue, latch

B) endured, foreseeable, revamped

C) revival, credo, impending

D) grasp, deploy, latch

E) No correction required

Answers:

1) Answer: D

In the given sentence, ‘adoption, reacted, surge’ should replace the bold words to make the
sentence both grammatically and contextually correct.

Page 417 Follow us: Official Site, Telegram, Facebook, Instagram, Instamojo
Option A): is incorrect because languishing means failing to make progress or be
successful which does not fit here.

Option B): is incorrect as ‘perplex, gather, thriving’ does not make any sense here.

Option C): is incorrect as ‘ineptness’ means showing a lack of skill or competence which
does not fit here.

Option D): is the correct alternative among the following as ‘adoption, reacted, surge’ fit
here both grammatically and contextually.

2) Answer: E

In the given sentence, ‘exigencies, collaboration, spheres’ is in both grammatically and


contextually correct position.

Option A): is incorrect because zeal means great energy or enthusiasm in pursuit of a
cause or an objective which does not fit here.

Option B): is incorrect as fora means a meeting or medium where ideas and views on a
particular issue can be exchanged which does not make any sense here.

Option C): is incorrect as ‘streamline’ does not fit here.

Option D): is incorrect as proficiency does not fit here contextually.

3) Answer: A

In the given sentence, ‘inevitability, relied, leverage’ should replace the bold words to
make the sentence both grammatically and contextually correct.

Option A): is the correct alternative among the following as ‘inevitability, relied, leverage’
fit here both grammatically and contextually

Option B): is incorrect as prowess means skill or expertise in a particular activity or field
which does not make any sense here.

Option C): is incorrect as dexterity means skill in performing tasks, especially with the
hands.

Page 418 Follow us: Official Site, Telegram, Facebook, Instagram, Instamojo
Option D): is incorrect as fervour does not fit here contextually.

4) Answer: C

In the given sentence, ‘tranche, spirits, crank up’ should replace the bold words to make
the sentence both grammatically and contextually correct.

Option A): is incorrect because bandwagon means used in reference to an activity, cause,
etc. that is currently fashionable or popular and attracting increasing support which does
not fit here.

Option B): is incorrect as dogma means a principle or set of principles laid down by an
authority as incontrovertibly true.

Option C): is the correct alternative among the following as ‘tranche, spirits, crank up’ fit
here both grammatically and contextually.

Option D): is incorrect as imminent means about to happen which does not fit here
contextually

5) Answer: B

In the given sentence, ‘endured, foreseeable, revamped’ should replace the bold words to
make the sentence both grammatically and contextually correct.

Option A): is incorrect because vogue means the prevailing fashion or style at a particular
time which does not fit here.

Option B): is the correct alternative among the following as ‘endured, foreseeable,
revamped’ fit here both grammatically and contextually

Option C): is incorrect as credo means a statement of the beliefs or aims which guide
someone’s actions.

Option D): is incorrect as latch means a metal bar with a catch and lever used for fastening
a door or gate which does not fit here contextually.

Words Replacement - Set 4

Page 419 Follow us: Official Site, Telegram, Facebook, Instagram, Instamojo
Direction(1-5): In each of the question below, a sentence has been broken into parts
with one word highlighted in bold in each fragment. From the given options,
identify the word which expresses similar meaning to the given sentences with the
highlighted word replaced by a word of the similar meaning.

1) On the way down I accidentally 3) Iran’s foreign minister flew into the
touched the dwarf woman and French resort hosting/a G7 summit on
she screeched/‘don’t touch me’ as if I August 25, an unexpected twist to a
had molested her, when I got downstairs, /meeting already troubled by differences
/I saw that there were around 30 people between U.S. President Donald Trump
in the raiding/ party andtheir first /and Western allies over a raft of issues,
purpose seemed to be to intimidate me. including Iran.

A) Encourage A) Crumple

B) Squealed B) Satisfied

C) Terrify C) Organising

D) Assaulted D) Disquiet

E) All are correct E) None of these

2) The liberation of Paris was both joyous 4) The drones crashed amid heightened
and chaotic, /it was faster and easier for tensions between neighbouring/ Israel
the Allies than their/protracted battle and Iran, which backs Hezbollah, and
through Normandy/ and its gun- shortly after Israeli warplanes attacked/
filled hedgerows. targets near the Syrian capital, Damascus,
Israeli aircraft buzzed over Beirut on
A) Disorganise /Sunday, hours after the drones crashed,
raising fears of a wider conflict.
B) Simple
A) Tipsy
C) Extended
B) Lambasted
D) Systematic
C) Harmonious
E) None of these
D) Strike

Page 420 Follow us: Official Site, Telegram, Facebook, Instagram, Instamojo
E) None of these A) Insignificant

5) The U.S. and the Taliban met in Doha B) Reduction


on Saturday, an American source/ close
to the negotiations/ said, resuming C) Purposeful
potentially decisive talks to allow/
Washington to drawdown militarily in D) Conference
Afghanistan.
E) None of these
Answers
1) Answer: (a)

New sentence-On the way down I accidentally touched the dwarf woman and she
screeched ‘don’t touch me’ as if I had molested her. When I got downstairs, I saw that
there were around 30 people in the raiding party. Their first purpose seemed to be
to encourage me.

In all other options except (A), the word given in the options are synonyms of those bold
ones, only “encourage” (give support or confidence) is the one that is opposite of
“intimidate” which means ‘frighten or overawe’.

Hence option (A) is correct.

2) Answer: (d)

New sentence- The liberation of Paris was both joyous and systematic, it was faster and
easier for the Allies than their protracted battle through Normandy and its gun-filled
hedgerows.

In all other options except (D), the word given in the options are synonyms of those bold
ones, only “systematic”is the one that is opposite of “chaotic” which means ‘in a state of
complete confusion and disorder’.

Hence option (D) is correct.

3) Answer: (b)

Page 421 Follow us: Official Site, Telegram, Facebook, Instagram, Instamojo
New sentence-Iran’s foreign minister flew into the French resort hosting a G7 summit on
August 25, an unexpected twist to a meeting already satisfied by differences between U.S.
President Donald Trump and Western allies over a raft of issues, including Iran.

In all other options except (B), the word given in the options are synonyms of those bold
ones, only “satisfied”(contented, pleased) is the one that is opposite of “troubled” , which
means ‘difficulties or problems’.

Hence option B is correct.

4) Answer: (c)

New sentence- The drones crashed amid heightened tensions between neighboring Israel
and Iran, which backs Hezbollah, and shortly after Israeli warplanes attacked targets near
the Syrian capital, Damascus, Israeli aircraft buzzed over Beirut on Sunday, hours after the
drones crashed, raising fears of a wider harmonious.

In all other options except (C), the word given in the options are synonyms of those bold
ones, only “harmonious” (marked by agreement in feeling, attitude or action) is the one
that is opposite of “conflict” which means ‘a serious disagreement or argument’.

Hence option C is correct.

5) Answer: (a)

New sentence- The U.S. and the Taliban met in Doha on Saturday, an American source
close to the negotiations said, resuming potentially insignificant talks to allow Washington
to drawdown militarily in Afghanistan.

In all other options except (A), the word given in the options are synonyms of those bold
ones, only “insignificant” (too small or unimportant to be worth consideration) is the one
that is opposite of “decisive” which means ‘producing a definite result’.

Hence option A is correct.

Words Replacement - Set 5


Directions (1-5): In each question below, four words printed in bold type are given.
These are numbered (A), (B), (C) and (D). One these words printed in bold might
either be wrongly spelt or is inappropriate in the context of the sentence. Find out
Page 422 Follow us: Official Site, Telegram, Facebook, Instagram, Instamojo
the word that is inappropriate or wrongly spelt, if any. The number of the word is
your answer. If the words printed in bold are correctly spelt and appropriate in the
context of the sentence then mark (E), i.e. ‘All Correct’, as your answer.

1) National Film award-winning director 3) A day after 17 political


Vetrimaaran has reported (A)/ cast parties jointly wrote to Vice-President
Tamil comedian Soori as the lead in his and Rajya Shabha Chairman(A)/ M.
next film, that is expected to be(B)/a big- Venkaiah Naidu complaining about
budget laugh riot and entertainer. This is “hurried legislation” that were being
a far departure for the filmmaker rushed through without
(C)/who is lauded for his intense work in due(B)/parliamentary procedures, Mr.
films like Aadukalam, Visaranai and the Naidu said on Saturday that
recent hit Vada Chennai.(D) the ruling and Opposition(C)/ parties
needed to function as “joint stakeholders”
A) Reported and not look at each other as “enemies
or adversary”.(D)
B) Expected
A) Jointly
C) Entertainer
B) Legislation
D) Intense
C) Ruling
E) All are correct
D) Adversary
2) It all went wrong when it mattered for
the sport’s(A)/ most successful E) All are correct
and glamorous team, with home
favourite(B)/Sebastian Vettel the 4) In a word of caution for the
first casual when(C)/ he failed to set a Opposition, Mr. Naidu said
time in the opening phase.(D) the(A)/mandatory of the people should
be respected. At the same time, the
A) Mattered Opposition parties(B)/ did have the right
and responsibility of getting
B) Glamorous governments(C)/ to deliver on
the promises made by them during the
C) Casual
polls, he said.(D)
D) Failed
A) Caution
E) All are correct
Page 423 Follow us: Official Site, Telegram, Facebook, Instagram, Instamojo
B) Mandatory

C) Responsibility

D) Promises

E) All are correct

5) Australia had submitted a draft MLSA soon after India signed the LEMOA, but(A)/ New
Delhi said at that time it would take up more logistics pacts after the first one(B)/
was operationalized. In March, before the bilateral naval exercise
AUSINDEX,(C)/Australian diplomatic sources made a strong pitch for a pact saying the
argument for one is “compelling”. (D)

A) submitted

B) logistics

C) operationalized

D) diplomatic

E) All are correct

Page 424 Follow us: Official Site, Telegram, Facebook, Instagram, Instamojo
Answers

1) Answer: (a)

National Film award-winning director Vetrimaaran has reportedly cast Tamil


comedian Soori as the lead in his next film that is expected to be a big-budget laugh riot
and entertainer. This is a far departure for the filmmaker who is lauded for his intense
work in films like Aadukalam, Visaranai and the recent hit Vada Chennai.

2) Answer: (c)

It all went wrong when it mattered for the sport’s most successful and glamorous team,
with home favourite Sebastian Vettel the first casualty when he failed to set a time in
the opening phase.

3) Answer: (d)

A day after 17 political parties jointly wrote to Vice-President and Rajya Shabha
Chairman M. Venkaiah Naidu complaining about “hurried legislation” that was being
rushed through without due parliamentary procedures, Mr. Naidu said on Saturday that
the ruling and Opposition parties needed to function as “joint stakeholders” and not
look at each other as “enemies or adversaries”.

4) Answer: (b)

In a word of caution for the Opposition, Mr. Naidu said the mandate of the people
should be respected. At the same time, the Opposition parties did have the right and
responsibility of getting governments to deliver on the promises made by them during
the polls, he said.

5) Answer: (e)

Australia had submitted a draft MLSA soon after India signed the LEMOA, but New
Delhi said at that time it would take up more logistics pacts after the first one was
operationalized. In March, before the bilateral naval exercise AUSINDEX, Australian
diplomatic sources made a strong pitch for a pact saying the argument for one is
“compelling”.

Page 425 Follow us: Official Site, Telegram, Facebook, Instagram, Instamojo
SBI CLERK PREVIOUS YEAR QUESTION PAPERS

Page 426 Follow us: Official Site, Telegram, Facebook, Instagram, Instamojo
SBI Clerk Prelims 2020 – English Language
Directions (71-77) Read the given passage carefully and answer Meanwhile, the impact of mosquitoes carrying these new genes
the following questions. Certain parts have been highlighted to remains largely unknown. One significant worry is that a new
help answer the questions. breed of mosquito might emerge that is more difficult to control.
These new genes could also potentially alter evolutionary
Every year, around one million people die of mosquito- borne
pressures on viruses carried by mosquitoes, like dengue fever,
diseases according to the World Health Organization (WHO).
in unpredictable ways. This includes potentially increasing
This is why mosquitoes are considered one of the deadliest
their virulence or changing their host-insect interactions.
living creatures on the planet — not because they are lethal
These are hypothetical risks that have been raised by scientists,
themselves, but because many of the viruses and parasites they
and reflect the need for further study.
transmit are.
In the absence of an effective vaccine for dengue fever, Zika 71. Why are mosquitoes considered as one of the deadliest
fever, chikungunya and other mosquito-borne diseases, living creatures on earth?
researchers have developed genetic strategies to reduce mosquito (a) Capability to transmit life threatening diseases.
populations. One such strategy involves the release into the (b) Mosquito bites cause fatal reactions in human beings
wild of genetically modified (GM) mosquitoes that express a (c) Mosquitoes cannot be killed easily
lethal gene — a strategy believed to have little impact on the (d) Both (a) and (c)
overall DNA of wild populations of mosquitoes. (e) None of the above
The transfer of new genes from GM organisms to wild or 72. What is one of the methods being developed to control
domesticated non-GM populations is a key criticism of GM crops population of mosquitoes?
like soybean and corn. There are concerns that the introduction (a) Spraying of pesticides using established
of GM genes into non-target species could have negative procedures.
consequences for both human and environmental health. (b) Introduction of a lethal gene into wild populations of
Oxitec, a company that spun out of research at Oxford mosquitoes using genetically modified ones.
University in the early 2000s, developed and trademarked GM (c) Not letting any stagnant water build-up near homes
Friendly™ mosquitoes (also known as strain OX513A of Aedes (d) Both (a) and (b)
aegypti). These male GM mosquitoes have what the company (e) None of the above
describes as a “self-limiting” gene, which means that when
73. What can be the potential side effect(s) of genetically
these so-called friendly mosquitoes‟ mate, their offspring
modified mosquito offspring?
inherit the self-limiting gene which is supposed to prevent them
(a) Evolution of a breed of mosquitoes which are more
surviving into adulthood.
difficult to control
In theory, when these mosquitoes are released in high
(b) Increase in the severity of diseases being currently
numbers, a dramatic reduction in the mosquito population
transmitted by mosquitoes.
should follow. According to research published by Oxitec
(c) Change in host-insect interactions.
researchers in 2015, field trials involving recurring releases of
(d) Loss of natural populations of mosquitoes
Friendly™ mosquitoes demonstrated a reduction of nearly 95
(e) (a), (b) and(c)
per cent of target populations in Brazil. In these field trials,
experiments were not performed to assess whether GM 74. Which of the statements can be considered as true with
mosquitoes might persist in the wild. respect to the passage given?
A recent study from the Powell lab at Yale University has since (a) Field trials on Genetically modified mosquitoes
confirmed that some of the offspring of the GM mosquitoes showed reduction of 96% target populations.
didn‟t succumb to the self-limiting lethal gene and survived to (b) Oxitec trademarked the GM Friendly mosquitoes in
adulthood. They were able to breed with native mosquitoes and 2015.
thereby introduce some of their genes into the wild population. (c) The female GM mosquito has a self-limiting gene
(d) Both (a) and (b)
1 (e) None of the above
75. What is the desired effect of releasing GM mosquitoes 80. ESCALATE
containing the „Self-limiting Gene‟? (i) The pedestrian was jaywalking when he escalate the
(a) A substantial reduction in the population of busy street, walking straight into the path of an
mosquitoes. oncoming vehicle.
(b) Birth of mosquitoes without a biting mechanism (ii) We've tried to escalate the most likely problems, but
(c) Decrease in the number of diseases caused due to it's impossible to be prepared for every eventuality.
mosquitoes (iii) The decision to escalate UN involvement has been
(d) Increase in the number of male mosquitoes. made in the hopes of a swift end to the hostilities.
(e) Both (b) and (c) (a) Only (i)
(b) Both (i) and (ii)
76. Which of the following word is similar to deadliest as (c) Both (ii) and (iii)
mentioned in the passage given? (d) Only (iii)
(a) fatal (b) demeaning (c) fulfil (e) All of these
(d) harmless (e) None of the above
Directions (81-85): In the following question, four sentences
77. Which of the statements can be considered as true with are given which may be grammatically and contextually
respect to the passage given?? incorrect. You need to find the one which has no error and mark
(a) GM mosquitoes are able to introduce some of their that as your answer. If all the given sentences are incorrect
genes into the wild population. then mark option (e) i.e. „all are incorrect‟ as your answer.
(b) Mosquitoes having GM genes have been
81. (a) India‟s chocolate market is pegged at Rs.11,000 crore,
thoroughly researched upon.
of which premium chocolates can be sized anywhere
(c) Oxitec released its friendly mosquitoes in Brazil
among 10% and 15%.
(d) (a) and (c)
(b) South Korea reported 52 new case of coronavirus.
(e) All of the above (c) ITC has plans to work with cocoa farmers directly in
Directions (78-80): A word has been given in each question future.
and has been used in the sentences given below. Identify the (d) The company initially invested Rs. 100 crore to set up
statements where the word has been used in a contextually and a greenfield chocolate manufacture facility in Haridwar.
grammatically correct manner. If the word has been used (e) All areincorrect
correctly in all the statements, mark (e), “All of these”, as your 82. (a) Ratnakar Bank was rename as RBL Bank by the
answer. government.
78. HOSTILE (b) The government expects all the mergers of public sector
(i) He made hostile efficiency a key part of his budget banks, which were announced in 2019, to become
plan. effective from April 1.
(ii) There has been a hostile reaction to the (c) Analysts believe that lower gas prices would be bad
government's proposed tax increase. with exploration
(iii) The security forces exercised great restraint by not (d) The Central Drugs Standard Control Organisation will
responding to hostile attacks and threats. be regulate the medical devices now.
(a) Only (i) (e) All are incorrect
(b) Both (i) and(ii) 83. (a) Mahindra Renewables will sell its entire stake in 3
(c) Both (ii) and (iii) subsidiaries to CLP India, a part of Hong Kong-
(d) Only (iii) based CLP Group, as nearly ₹340 crore.
(e) All of these (b) The agreement will be signed to Doha city between
Taliban representatives and U.S. special envoy Zalmay
79. VAGUE
Khalilzad.
(i) The judges determined that the law was too
(c) The U.S. intelligence community publicly concluded
vague to be fairly enforced.
that Russia intervened in there security matters.
(ii) The merger of these two companies would vague
(d) The injured animal was nursed back to health by the
the world's biggest accounting firm
zookeeper and then released back into the wild
(iii) The president had a vague reception in Ohio this
(e) All are incorrect
morning.
(a) Only (i)
(b) Both (i) and(ii)
(c) Both (ii) and (iii)
(d) Only (iii)
(e) All of these
84. (a) English language have the potential to connect the world Directions (91-95): In the following questions, a
without any barriers grammatically correct and meaningful sentence is given which
(b) P.K. Sinha was appointed in the PMO when is divided into five parts, where the first part is fixed and
Nripendra Misra, the then Principal Secretary, highlighted , in the remaining parts of the sentence namely
resigns. (A),(B),(C) and (D) You have to arrange the four parts to make a
(c) She drink the caramel liquid too fast and was soon contextually and grammatically meaningful sentence. If no such
rearrangement is possible mark (e) as your answer i.e. 'No
too dizzy to stand.
rearrangement required'.
(d) You‟re coming to the party, isn‟t you?
(e) All are incorrect 91. (A) the merger of Bharti Infratel
(B) the world‟s second largest
85. (a) Tamil Nadu is likely to get three textile parks and a (C) and Indus Towers will create
technology research centre under schemes that the (D) tower company
Union Ministry of Textiles is formulating. (a) BCDA (b) CBDA (c) ACBD
(b) What is the uses of a house if you haven't got a (d) DCAB (e) No rearrangement is required
tolerable planet to put it on?
92. (A) upcoming visit to India
(c) You have brushed your teeth today? (B) the long-awaited trade agreement
(d) Rahul had barely nothing to help the poor children. (C) during US President Donald Trump's
(e) All are incorrect (D) appears to be off the table
Directions (86-90): In the following questions, a sentence is (a) DABC (b) BDCA (C) CDAB
divided into four parts consisting of a highlighted word in (d) ADCB (e) No rearrangement required
each part. Choose the option reflecting the word which is 93. (A) Leaders of the Railway Employees Organization
either misspelt or grammatically incorrect. If all the highlighted (B) alleged that the government wants to
words are correct, choose option (e) i.e. “all are correct” as (C) hand over the rail infrastructure
your answer choice. (D) to the private operators for financial benefit
(a) ADCB (b) BACD (c) CDBA
86. The manager balanced (A) the strength (B) of his team (d) DABC (e) No rearrangement required
against that of their opponent (C) and sighed. (D)
(a) balanced (b) strength (c) opponent 94. (A) China has not yet given
(B) India the go-ahead to send an
(d) sighed (e) All are correct
(C) to coronavirus hit Wuhan
87. One evening Rohan pripared (A) a nice supper (B) and (D) aircraft with medical supplies
put it on low heat in the oven to keep it tepid (C) while his (a) DACB (b) ABDC (c) CBDA
wife dressed. (D) (d) BCDA (e) No rearrangement is required
(a) prepared (b) supper (c) tepid 95. (A) mammals and birds move from
(d) dressed (e) All are correct (B) with a change in season, many
88. She was able to breathe (A) easier when she stood (C) one country to another in search of food
outside the massive(B) fortress (C) that sat on a clearing (D) and shelter, and for breeding
(a) ACBD (b) CADB (c) DABC
the size of two footbal (D) fields.
(d) BACD (e) No rearrangement is required
(a) breathe (b) massive (c) fortress
(d) football (e) All are correct Directions (96-100): In the following passage there are blanks,
each of which has been numbered. These numbers are printed
89. From time to time Jack would looking over(A) Mungo's below the passage and against each, five options are given.
shoulder, suggesting(B) tactics (C) which invariably Find out the appropriate word which fits the blank
proved disastrous.(D) appropriately.
(a) looking over (b) suggesting (c)tactics
Bumblebees, among the most important pollinators, are in
(d) disastrous (e) All are correct
(96) ________________. Fuzzy and buzzy, they excel at
90. Horrified (A) passengers saw Olive stumble (B) and spreading pollen and fertilizing many types of wild flora, as well
fall of (C) a platform as an express roared (D) in. as crucial agricultural crops like tomatoes, blueberries, and
(a) Horrified (b) stumble (c) fall of squash. But their numbers are (97)
(d) roared (e) All are correct ________________. New research using a massive dataset
found that the insects are far less common than they used to be; in
North America, you are nearly 50 percent less likely to see a
3 bumblebee in any given area than you were prior to 1974.
Moreover, several once-common species
have (98) __________________ from many areas they were 96. (a) extinct (b) trouble (c) dropped
once found, becoming locally extinct in those places. For
(d) growth (e) difficult
example, the rusty patched bumblebee, which used to flourish
in Ontario, is no longer found in all of Canada—in the U.S., it‟s 97. (a) increasing (b) dripped (c) removing
endangered. In a new paper published this week in the journal
Science, researchers used a complex modeling process to (99) (d) dropping (e) generating
________________ that their (100)___________________ is
98. (a) multiplied (b) concerning (c) disappeared
driven in large part by climate change. Specifically, the
scientists found that in areas that have become hotter in the last (d) certain (e) vanish
generation, or have experienced more extreme temperature
swings, bumblebees are less abundant. In Europe, they are 17 99. (a) proposed (b) recommending
percent less plentiful than they were in the early 20th century. (c) implying (d) expresses (e) suggest
The scientists examined the abundance of 66 species across
the two continents. 100. (a) decline (b) deteriorate (c) rejection
(d) lessen (e) reduced
SBI Clerk Prelims 2020 – Reasoning Ability
Direction (1-5): Study the following information carefully 6. Statements: Some Banana are Orange.
and answer the questions given below: All Orange are Apple.
Conclusions: I. Some Banana are not Apple.
There are eight employees of a company and all of them are II. Some Orange are Banana.
working on eight different designation of a bank viz.
Chairman, CFO, GM, DGM, AGM, Manager, Junior 7. Statements: Only a few East are North.
Manager and Clerk. All the designations given are to be Few North are South.
considered in a given order (as Chairman is considered as All South are West.
Conclusions: I. All East being North is a possibility.
Senior- most and Clerk is considered as the Junior-most).
II. Some East are West.
Only two persons are senior to B. One designation lies
8. Statements: Only a few Song are Melody.
between B and G. The number of persons junior to G is same Only a few Melody are Film.
as the number of persons senior to C. H is just senior to E, but No Film is Award.
junior to C. More than four designations lie between H and F. Conclusions: I. All Song being Film is a possibility.
D is junior to A. II. Some Melody are not Award.
1. How many persons are junior to H? Direction (9-12): Study the following information
(a) None (b) One (c) More than four carefully and answer the questions given below:
(d) Four (e) Three In a certain code language:
“Club house near located” is coded as “ol gp ox ot”
2. Four of the following five are alike in a certain way and “both club view near” is coded as “mt ox sq ot” “make
hence they form a group. Which one of the following located house view” is coded as “nk ol gp sq” “near
does not belong to that group? club view area” is coded as “ot ox sq tm”
(a) A-F (b) B-A (c) H-D
(d) G-A (e) E-H 9. What is the code for “both” in the given code
language?
3. How many designation gaps are between A and D? (a) ox (b) sq (c) mt
(a) More than three (b) Two (c) Three (d) ot (e) Either (b) or (c)
(d) One (e) None 10. What is the code for “house” in the given code
4. Who among the following is just senior to B? language?
(a) A (b) D (c) C (a) ol (b) ox (c) gp
(d) E (e) None of these (d) Either (a) or (c) (e) None of these
11. The code “nk” is stands for?
5. Who among the following is AGM?
(a) make (b) both (c) area
(a) F (b) B (c) G
(d) club (e) None of these
(d) D (e) None of these
12. What may be the possible code for “both house” in the
Direction (6-8): In each of the questions below are given given code language?
some statements followed by some conclusions. You have to (a) gp mt (b) mt nk (c) mt sq
take the given statements to be true even if they seem to be at (d) ox mt (e) gp sq
variance with commonly known facts. Read all the
Directions (13-17): Study the following information
conclusions and then decide which of the given conclusions
carefully and answer the questions given below:
logically follows from the given statements disregarding
There are eight persons A, B, C, D, E, F, G and H sitting
commonly known facts.
around a circular table facing towards the centre of the table
(a) If only conclusion I follows. but not necessarily in the same order.
(b) If only conclusion II follows. B sits third to the right of A. Only one person sits between B
(c) If either conclusion I or II follows. and D. E faces C who is not an immediate neighbour of
(d) If neither conclusion I nor II follows. B. G sits third to the left of F. A is not an immediate
(e) If both conclusions I and II follow. neighbour of F.
13. Who among the following sits to the immediate left of C? 21. What is the sum of the 3rd digit of second number from
(a) G (b) D (c) F left and 2nd digit of third number from right?
(d) H (e) None of these (a) 10 (b) 15 (c) 14
(d) 12 (e) None of these
14. Who among the following sits to the opposite of A?
(a) H (b) G (c) F 22. If the position of first and third digits of each of the
(d) D (e) None of these numbers are interchanged, then which among the
15. How many persons sit between H and G when counted following is the highest number?
from the left of G? (a) 258 (b) 469 (c) 189
(a) Two (b) One (c) Three (d) 427 (e) 625
(d) Four (e) None of these 23. If all the digits in the number are arranged in the
16. If all the persons sit around the circle in clockwise descending order within the number from left to right,
direction by their names according to English then which among the following will be the lowest
alphabetical order starting from A, then how many number after rearrangement?
persons remain unchanged except A? (a) 427 (b) 189 (c) 258
(a) Four (b) One (c) Three (d) 625 (e) 469
(d) Two (e) None of these 24. What is the product of 3rd digit of 2nd lowest number
17. What is the position of B with respect to H? and 1st digit of 2nd highest number?
(a) Fourth to the left (a) 32 (b) 72 (c) 24
(b) Fourth to the right (d) 36 (e) None of these
(c) Third to the right
25. If 2 is subtracted from each number then how many
(d) Both (a) and (c)
numbers thus formed are odd numbers?
(e) Both (a) and (b)
(a) One (b) Two (c) Three
Direction (18-20): Study the following information carefully (d) More than three (e) None
and answer the questions given below:
26. How many such numerals are there in the number
Point E is in 15m north of Point D. Point F is in 20m north of „645903287‟ which will remain at the same position
Point C. Point A is in 35m east of Point F. Point P is in 25m when arranged in ascending order from left to right?
south of Point A. Point E is in 20m east of Point P. (a) Three (b) Two (c) One
18. What is the direction of point F with respect to point P? (d) Four (e) None of these
(a) North-west (b) North-east (c) South-west 27. If it is possible to make only one meaningful word with
(d) South-east (e) North the 1st, 3rd, 5th and 11th letters of the word
19. Four of the following five are alike in a certain way and „INHERITENCE‟, which would be the third letter of the
hence they form a group. Which one of the following word from the left? If more than one such word can be
does not belong to that group? formed give „Y‟ as the answer. If no such word can be
(a) C-P (b) A-E (c) A-C formed, give „Z‟ as your answer.
(d) P-D (e) F-E (a) Y (b) R (c) I
(d) E (e) Z
20. If point X is 20m south of point C, then what is the total
distance between point X and point D? Direction (28-32): Study the following information carefully
(a) 40m (b) 35m (c) 55m and answer the questions given below:
(d) 65m (e) 85m Eight persons A, B, C, D, E, F, G and H are buying some
Directions (21-25): Following questions are based on the products one after another but not necessarily in the same
five numbers given below, Study the given information and order.
answer the following questions. At most two persons are buying products before F. Only one
625 427 189 258 469 person is buying between D and F. C is buying just before H.
One person is buying between C and D. A is buying just
before E. B is buying before G and after E.
2
Adda247 | No. 1 APP for Banking & SSC Preparation
28. How many persons are buying their products after D? 32. How many persons are buying between E and C?
(a) None (a) Five (b) Four (c) Three
(b) Two (d) None (e) Two
(c) More than three
(d) One Directions (33-35): In each of the question, relationships
(e) None of these between some elements are shown in the statements. These
statements are followed by conclusions numbered I and II.
29. Who among the following is buying just after G? Read the statements and give the answer.
(a) E (b) F (c) B (a) If only conclusion I follows.
(d) C (e) None of these (b) If only conclusion II follows.
30. If all the persons are arranged in alphabetical order from (c) If either conclusion I or II follows.
left to right starting from A, then find how many persons (d) If neither conclusion I nor II follows.
remains at the same position (excluding A)? (e) If both conclusion I and II follows.
(a) One (b) None (c) Two 33. Statements: P > Q ≥ R = S < T = U
(d) Four (e) More than Four Conclusions: I. Q > S II. S = Q
31. Who among the following is buying exactly between D 34. Statements: J < D = L ≤ K ≤ Q ≥ R
and F? Conclusions: I. J < Q II. L ≤ R
(a) E (b) B (c) A
(d) H (e) None of these 35. Statements: O > P = G > B ≥ X = M ≤ H
Conclusions: I. G > X II. O > M
SBI Clerk Prelims 2020 – Quantitative Aptitude
Directions (36-40): Table given below shows the number of 44. 6, 8, 14, 26, 46, ?
male and female participated in an event from five different (a) 72 (b) 84 (c) 96
schools (A, B, C, D & E). Study the table carefully and (d) 80 (e) 76
answer the following questions. 45. 72000, 36000, 12000, 3000, 600, ?
Schools Male Female (a) 120 (b) 200 (c) 300
A 650 450 (d) 150 (e) 100
B 540 420 46. 12 men can do a work in 10 days while 10 women can do
C 720 500 the same work in 18 days. In how many days 4 men & 6
D 560 450 women together can do the same work?
120
E 680 320 (a) days (b) 24 days (c) 180 days
7 13
(d) 15 days (e) 18 days
36. Find average number of female participated from school
– A, B & D. 47. A car can cover a distance in 4 hour at speed 60 kmph
then by what percent should the speed of car be increased
(a) 400 (b) 380 (c) 350
to cover the same distance in 2.5 hr?
(d) 440 (e) 450
(a) 60% (b) 40% (c) 50%
37. Total male participated from school – B & D together are (d) 100% (e) 75%
how much more or less than total female participated 48. The ratio of the ages of Ram and Rahim 10 years ago
from school – A & C together? was 1 : 3. The ratio of their ages five years hence will be
(a) 150 (b) 110 (c) 170 2 : 3. Then, the ratio of their present ages is :
(d) 120 (e) 240 (a) 1 : 2 (b) 3 : 5 (c) 3 : 4
38. Total male participated from school – B & C together are (d) 2 : 5 (e) None of these
what percent more or less than total female participated 49. Two trains of length 140m & 120m are running in same
from school – A & D together? direction on parallel tracks with speeds 132 kmph & 80
(a) 20% (b) 60% (c) 50% kmph respectively. How much time will they take to
(d) 40% (e) 30% cross each other?
(a) 7.09 sec (b) 18 sec (c) 11.7 sec
39. If total male participated from school – F are 40%
(d) 4.42 sec (e) Cannot be determined
more than that of from school – A and ratio of female
participated from school – B to that of from school – F is 50. A person sold a book at 20% profit. If he had bought it at
21:32, then find total students participated from school 10% less cost and sold for Rs 90 more then he would
– F. have gained 40% profit. Find cost price of book.
(a) 1420 (b) 1550 (c) 1580 (a) Rs 800 (b) Rs 1600 (c) Rs 1500
(d) 1460 (e) 1490 (d) None of these (e) Rs 1200
40. Find total number of male students participated from all Direction (51–55): In each question two equations numbered
the five schools together. (I) and (II) are given. You have to solve both the equations
(a) 2860 (b) 3150 (c) 2940 and mark appropriate answer.
(d) 3200 (e) 3020 (a) If x = y or no relation can be established
Direction (41–45): What will come in the place of question (b) If x > y
(?) mark in following number series: (c) If x < y
(d) If x ≥ y
41. ?, 100, 150, 375, 1312.5 (e) If x ≤ y
(a) 100 (b) 200 (c) 150
(d) 400 (e) 50 51. I. � = √25 II. y3 = 125
42. 104, ?, 96, 120, 88, 128 52. I. x2 + 2x – 35 = 0 II. y2 + 15y + 56 = 0
(a) 112 (b) 110 (c) 114
53. I. x2 = 81 II. y2 = 64
(d) 118 (e) 108
54. I. 17x2 – 14x – 83 = - 80 II. y2 = 2y + 35
43. 15, 8, 9, 15, 32, ?
(a) 66 (b) 99 (c) 80 55. I. x2 + 4x – 45 = 0 II. y2 − 13� + 40 = 0
(d) 82.5 (e) 80.5

1
56. A container contains mixture of milk & water in ratio 5 : Direction (61–70): What will come in the place of (?) mark in
3 respectively. If 8 lit milk is added in it then ratio of following question.
milk to water becomes 11 : 5. Find difference between 61. 280 ÷ 4 ÷ 2 = 170 – ?
initial quantity of milk & that of water. (a) 105 (b) 115 (c) 125
(a) 5 lit (b) 38 lit (c) 18 lit (d) 135 (e) 145
(d) 30 lit (e) 10 lit ?
62. )×3=
(√144 + √
57. Rs 6000 when invested at a certain rate at SI for 2 years, (b) 325 (c) 350
(a) 375
it fetches Rs 1200. If same sum is invested at same rate (d) 275 (e) 475
for a year compounded half – yearly then find compound 63.
interest. (a) 100 (b) 140 (c) 120
(a) Rs 615 (b) Rs 600 (c) Rs 1200 (d) 80 (e) 90
(d) Rs 585 (e) Rs 1260 64. (120% of 750) ÷ ? = 25
(a) 30 (b) 36 (c) 24
58. A boat can cover 28 km downstream in 42 min. ratio (d) 18 (e) 48
of speed of boat in still water to speed of stream is 7 : 1 5 7
65. 8 2 − 4 6 =? −3 12
3. Find difference between time taken by boat to cover 1 5 7
(a) 3 (b) 3 (c) 2
60 km downstream & 40 km upstream. 4
1
12 12
(a) 2.25 hr (b) 1 hr (c) 1.5 hr (d) 7 (e) 5 2
4 3
(d) 0.4 hr (e) 0.9 hr 66. 275 + 64% of 750 = 750 + ?
59. A & B entered into a business by investing total capital (a) 25 (b) 8 (c) 10
(d) 15 (e) 5
of Rs 17000. B withdraws Rs 1500 after 6 months and
gets Rs 8100 as profit out of total profit of Rs 19500 at 67. √225 + √81 + 122 =?
(a) 168 (b) 164 (c) 162
the end of year. Find capital of B after 6 months from
(d) 172 (e) 182
starting.
(a) Rs 7000 (b) Rs 9500 (c) Rs 7500 68. 510 = √324 + 3.25
(d) Rs 6000 (e) Rs 6500 (a) 12 (b) 48 (c) 24
(d) 6 (e) 18
60. If length of a rectangle increases by 40% while keeping
69. 12.5% of (120 + ?) = 45
breadth constant then area of rectangle increased by 24
(a) 160 (b) 180 (c) 360
m2 and perimeter of original rectangle is 32 m. find (d) 240 (e) 120
breadth of rectangle.
70. 572 ÷ 13 × 12 – 16 = (8)?
(a) 8.4 m (b) 10 m (c) 6 m (a) 4 (b) 2 (c) 3
(d) 14 m (e) 8 m (d) 5 (e) None of these
SBI Clerk Prelims 2020 – English Language (Solutions)
71. (a); Reading the first paragraph of the passage it can be 76. (a); Among the given words, „fatal‟ is the synonym of
deduced that the correct answer is option(a). The „deadliest‟. Hence, option (a) is the
relevant sentences of the mentioned most suitable answer choice.
sentences have been quoted below: Fatal: causing death.
“This is why mosquitoes are considered one of the Demeaning: causing someone to lose their dignity
deadliest living creatures on the planet — not and the respect of others.
because they are lethal themselves, but because
77. (d); Reading the above passage it can be seen that the
many of the viruses and parasites they transmit are.”
correct statements are (a) and (c). The sentences
72. (b); Reading the second paragraph of the passage it can have been quoted below for your reference-
be deduced that the correct answer is option(b). The “According to research published by Oxitec
sentences of the passage which substantiate this are researchers in 2015, field trials involving recurring
given below: releases of Friendly™ mosquitoes demonstrated a
“One such strategy involves the release into the wild reduction of nearly 95 per cent of target
of genetically modified (GM) mosquitoes that populations in Brazil. A recent study from the
express a lethal gene — a strategy believed to have Powell lab at Yale University has since confirmed
little impact on the overall DNA of wild populations that some of the offspring of the GM mosquitoes
of mosquitoes.” didn‟t succumb to the self- limiting lethal gene and
survived to adulthood. They were able to breed
73. (e); Reading the last paragraph of the passage it can be
with native mosquitoes and thereby introduce
deduced that the correct answer is
some of their genes into the wild population.”
option(e). The relevant sentences of the
mentioned sentences have been quoted below: “One 78. (c); Hostile means showing or feeling opposition or
significant worry is that a new breed of mosquito dislike; unfriendly. Therefore, sentence (ii) and
might emerge that is more difficult to control. These (iii) depict the correct usage of the given word.
new genes could also potentially alter evolutionary Hence, option (c) is the most suitable answer choice.
pressures on viruses carried by mosquitoes,
79. (a); Vague means thinking or communicating in an
like dengue fever, in unpredictable
unfocused or imprecise way. Therefore, only
ways. This includes potentially increasing their
sentence (i) depicts the correct usage of the given
virulence or changing their host- insect
word. Hence, option (a) is the most suitable answer
interactions.”
choice.
74. (e); Reading the passage carefully we can see that none
of the sentences given above are factually correct 80. (d); Escalate means increase rapidly. Therefore, only
hence the correct answer will be option (e). sentence (iii) depicts the correct usage of the given
word. Hence, option (d) is the most suitable answer
75. (a); Reading the fourth and fifth paras of the passage it choice.
can be deduced that the correct answer is option(a).
The relevant sentences of the 81. (c); As we go through the analysis of the sentence, we
mentioned sentences have been quoted below: find that the sentence of option (c) is correct.
“These male GM mosquitoes have what the (a) replace „among‟ with „between‟. Among is
company describes as a “self-limiting” gene, which used for a greater number.
means that when these so-called friendly (b) replace „case‟ with „cases‟.
mosquitoes‟ mate, their offspring inherit the self- (d) replace‟ manufacture‟ with „manufacturing‟.
limiting gene which is supposed to prevent them 82. (b); As we go through the analysis of the sentence, we
surviving into adulthood In theory, when these find that the sentence of option (b) is correct.
mosquitoes are released in high numbers, a dramatic (a) replace „rename‟ with „renamed‟.
reduction in the mosquito population should (c) replace „with‟ with „for‟.
follow.” (d) replace „regulate‟ with „regulating‟.

1
83. (d); As we go through the analysis of the sentence, we “The long-awaited trade agreement appears to be off
find that the sentence of option (d) is correct. the table during US President Donald Trump‟s
(a) replace „as‟ with „for‟. upcoming visit to India.”
(b) replace „to‟ with „in‟.
(c) replace „there‟ with „their‟. 93. (e); In the given sentence the sentence elaborates on
allegations made against the government by leaders
84. (e); As we go through the sentences, we find that all of railway employee organisations with respect to
sentences are incorrect. their stand on handing over railway infrastructure to
(a) replace „have‟ with „has‟. private companies. Hence, option (e) is the most
(b) replace „resigns‟ with „resigned‟. suitable answer choice which means the sentence
(c) replace „drink‟ with „drank‟. requires no rearrangement. The statement thus
(d) replace „isn‟t‟ with „aren‟t‟.
formed will be:
85. (a); As we go through the analysis of the sentence, we “Leaders of the Railway Employees Organization
find that the sentence in option (a) is correct. alleged that the government wants to hand over the
(b) replace „uses‟ with „use‟. rail infrastructure to the private operators for
(c) interchange „have‟ with „you‟ to maintain the financial benefit.”
interrogative form of the sentence.
(d) replace „nothing‟ with „anything‟ because 94. (b); The correct rearrangement of the parts of statement
„barely‟ will not take „nothing‟ after it. will be “ABDC” which will make a grammatically
correct and contextually meaningful statement. The
86. (a); Except “balenced” which is spelt as “balanced” all given statement talks about India not being granted
other words are correctly spelt so our answer choice
the permission to send an aircraft carrying medical
will be option (a).
supplies to the coronavirus hit Wuhan region. The
87. (a) ; Except “pripared” which is spelt as “prepared” all correct statement thus formed will be:
other words are correctly spelt so our answer choice “China has not yet given India the go-ahead to send
will be option (a). an aircraft with medical supplies to coronavirus hit
88. (d); Except “footbal” which is spelt as “football” all Wuhan.”
other words are correctly spelt so our answer choice 95. (d); The correct rearrangement of the parts of statement
will be option (d). will be “BACD” which will make a grammatically
89. (a); Except “looking over” which is grammatically correct and contextually meaningful statement. The
wrong all other words are correctly spelt and are given sentence talks about migration in animals with
grammatically flawless so our answer choice will be changing seasons in search of their basic needs.
option (a). Replace “looking over” with “look over”. Hence, option (d) is the most suitable answer choice.
The correct statement thus formed will be:
90. (c); Except “fall of” which is spelt as “fall off” all other
words are correctly spelt so our answer choice will “With a change in season, many mammals and birds
be option (c). move from one country to another in search of food
and shelter, and for breeding.”
91. (c); The correct rearrangement of the parts of statement
will be “ACBD” which will make a grammatically 96. (b); The given paragraph is describing the alarming
correct and contextually meaningful statement. situation of Bumblebees where their numbers are
Hence, option (c) is the most suitable answer choice. continuously declining. Thus, the most suitable
The correct statement thus formed will be: word for the given blank is “trouble”. Hence, option
“The merger of Bharti Infratel and Indus Towers (b) is the correct answer choice.
will create the world‟s second largest tower
company.” 97. (d); The given paragraph is describing the alarming
situation of Bumblebees where their numbers are
92. (b); The correct rearrangement of the parts of statement continuously declining. Thus, the most suitable
will be “BDCA” which will make a grammatically word for the given blank is “dropping”. All the other
correct and contextually meaningful statement. words are contextually incorrect. Hence, option (d)
Hence, option (b) is the most suitable answer choice. is the correct answer choice.
The correct statement thus formed will be:

2
98. (c); The given paragraph is describing the alarming 99. (e); The most suitable word for the given blank is
situation of Bumblebees where their numbers are “suggest”. All the other words are either
continuously declining. Thus, the most suitable grammatically or contextually incorrect. Hence,
word for the given blank is option (e) is the correct answer choice.
“disappeared”. All the other words are either
grammatically or contextually incorrect. Hence, 100.(a); The given paragraph is describing the alarming
option (c) is the correct answer choice. Disappeared situation of Bumblebees where their numbers are
means cease to exist or be in use. Vanish means continuously declining. Thus, the most suitable
disappear suddenly and word for the given blank is “decline”. All the other
completely. words are either grammatically or contextually
Certain means able to be firmly relied on to happen incorrect. Hence, option (a) is the correct answer
or be the case. choice.
SBI Clerk Prelims 2020 – Reasoning Ability (Solutions)
Solutions (1-5):
Solutions (13-17):
Designation Persons
Chairman F
CFO A
GM B
DGM C
AGM G
Manager D
Junior manager H
Clerk E
13. (b); 14. (c); 15. (b);
16. (d); 17. (e);
1. (b) 2. (d) 3. (c) Solutions (18-20):
4. (a) 5. (c)
6. (b);

18. (a);
19. (c);
20. (c);
7. (d);

8. (e);
21. (b); 22. (c); 23. (d)
24. (a); 25. (d);
Solutions (9-12): 26. (c); 6 4 5 9 0 3 2 8 7
Word Code 0 2 3 4 5 6 7 8 9
Club Ox 27. (a); Hire, Heir
Near Ot
House/Located Ol/gp Solutions (28-32):
Both Mt
Make Nk
Area Tm 28. (e); 29. (d); 30. (a);
View Sq 31. (b); 32. (b);
9. (c) 33. (c); I. Q > S (False) II. S = Q (False)
10. (d)
11. (a) 34. (a); I. J < Q (True) II. L ≤ R (False)
12. (a) 35. (e); I. G > X (True) II. O > M(True)
SBI Clerk Prelims 2020 – Quantitative Aptitude (Solutions)
36. (d); Required average = 450+420+450 = 440 48. (b); Let 10 years ago, ages of Ram and Rahim were x
3
years and 3x years, respectively.
37. (a); Total male participated from school – B & D Then, present age of Ram = (x + 10)
together = 540 + 560 = 1100 and present age of Rahim = (3x + 10)
Total female participated from school – A & C According to the question,
x+10+5 2
together = 450 + 500 = 950 =
3x+10+5 3
Required difference = 1100 − 950 = 150 ⇒ 3x + 45 = 6x + 30
38. (d); Total male participated from school – B & C ⇒ 3x = 15
together = 540 + 720 = 1260 ∴ x=5
5+10
Total female participated from school – A & D Hence, required ratio =
3×5+10
15
together = 450 + 450 = 900 = =3∶ 5
Required % = 1260−900 × 100 = 40% 25
900 140+120
49. (b); required time = 5
39. (b); Total students participated from school F = (132−80)×
18
260×18
140
× 650 + 420 ×
32 = = 18 sec
52×5
100 21
= 910 + 640 = 1550 50. (c); let CP of book be Rs x
SP = Rs 1.2x
40. (b); Total number of male students participated from all
New CP = Rs 0.9x
the five schools New SP = Rs 1.2x + 90
= (650 + 540 + 720 + 560 + 680) = 3150 140
ATQ, 0.9� × = 1.2� + 90
100
41. (b); Pattern of series – 1.26x = 1.2x + 90
x = Rs 1500
51. (a); I. x = 5
42. (a); Pattern of series - II. y =
5 So,
x=y
52. (d); I. x2 + 7x – 5x – 35 =0
43. (d); Pattern of series -
x (x + 7) – 5 (x + 7) =0
(x + 7) (x – 5) = 0
x = −7, 5
44. (e); Pattern of series - II. y2 + 7y + 8y + 56 = 0
y(y + 7) + 8(y + 7) = 0
(y + 7) (y + 8) = 0
y = − 8, −7
So, x≥y
45. (e); Pattern of series -
53. (a) I. x = ± 9
II. y = ± 8
So, no relation can be established
46. (d); let total work be 360 units
54. (a); I. 17x2 – 14x – 3 = 0
Efficiency of 1 man = 360 17x2 – 17x + 3x – 3 = 0
= 3 units/day 17x (x – 1) + 3(x – 1) = 0
12×10
360
Efficiency of 1 woman = = 2 units/day (17x + 3) (x – 1) = 0
10×18 3
Required time = 360
= 15 days x = − 17, 1
4×3+6×2
1
47. (a); distance = 240 kms II. y2 – 2y – 35 = 0
Required speed = 240 = 96 kmph y2 – 7y + 5y – 35 = 0
2.5
y(y −7) + 5(�− 7) = 0
Required % = 96−60 × 100 = 60%
60 y = 7, - 5
So, no relation can be established
55. (e); I. x2 + 9x – 5x – 45 = 0 60. (c); let length & breadth of rectangle be x & y m
x(x + 9) – 5(x + 9) = 0 respectively
(x – 5) (x + 9) = 0 ATQ, 1.4xy – xy = 24
x = 5, - 9 xy = 60 .................. (i)
II. y2 – 5y – 8y + 40 = 0 also, 2(x + y) = 32
y(y – 5) – 8(y – 5) = 0 x + y = 16 .............(ii)
(y – 5) (y – 8) = 0 from (i) & (ii)
y = 5, 8 x = 10 m, y = 6 m
So, x≤y breadth of rectangle = 6 m

56. (e); let initial quantity of milk & water be 5x & 3x lit 61. (d); ? = 170 – 35
respectively ? = 135
5�+8 11 ?
ATQ, = 62. (a); (12 + 13) × 3 =
3� 5 5
25x + 40 = 33x ⇒ x = 5 ? = 375
required difference = 5x – 3x = 2x = 10 lit
63. (c); ? = (3 × 5) × 8
57. (a); let rate of interest be R% ? = 120
6000×�×2
ATQ, 1200 = 120
R = 10% 100 64. (b); ( × 750 ) ÷? = 25
100
? = 900 ÷ 25
Since compounding is done half-yearly, rate of ? = 36
interest = 5%
Effective rate of interest = 5 + 5 + 5×5 = 10.25% 65. (d); ? = (8 – 4 + 3)+ 6−10+7
100 12
Required interest =
6000×10.25×1
= Rs 615 ?=71
4
100
64
58. (b); let speed of boat in still water & speed of stream 66. (e); 275 + × 750 = 750 + ?
100
be 7x & 3x kmph respectively 275 + 480 = 750 + ?
42
ATQ, 28 = ?=5
7�+3� 60
x=4 67. (a); ? = 15 + 9 + 144
60
Required difference = 40 − = 4 = 1 hour ? = 168
7�−3� 7�+3� �

59. (d); let amount invested by A be Rs x 68. (c); 510 = 18 + 3.25


?
Profit ratio; A : B = (x × 12) : (17000 – x) × 6 + ? = 24
(15500 – x) × 6 69. (d); 12.5 × (120+? ) = 45
= 2x : (32500 – 2x) 100
19500 120 + ? = 360
ATQ, 32500−2�+2� × (32500 − 2�) = 8100 ? = 240
32500 – 2x = 13500
?
x = Rs 9500 70. (c); 44 × 12 − 16 = (8)
Required capital of B after 6 months 528 – 16 = (8)
?

= 15500 – x = Rs 6000 ?=3

You might also like